Exit Hesi First Attempt Practice

¡Supera tus tareas y exámenes ahora con Quizwiz!

A pt on a trachea/endotrach with gurgling sound

always pick suctioning

Apgar scale

-Performed at 1 min and 5 min -7-10 is good -4-6 needs moderate resuscitative efforts -0-3 is severe need for resuscitation -5 Components: heart rate, resp effort, muscle tone, reflex irritability, color

Which statement by the nurse is appropriate when asking an unlicensed assistive personnel (UAP) to assist a 69 year-old surgical client to ambulate for the first time? A) "Have the client sit on the side of the bed for at least 2 minutes before helping him stand." B) "If the client is dizzy on standing, ask him to take some deep breaths." C) "Assist the client to the bathroom at least twice on this shift." D) "After you assist him to the chair, let me know how he feels."

A) "Have the client sit on the side of the bed for at least 2 minutes before helping him stand."

An increased number of elderly persons are electing to undergo a new surgical procedure which cures glaucoma. Which effect is the nurse likely to note as a result of this increase in glaucoma surgeries? A. Decreased prevalence of glaucoma in the population B. Increased incidence of glaucoma in the population C. Decreased morbidity in the elderly population D. Increased mortality in the elderly population

A. Decreased prevalence of glaucoma in the population

The nurse is providing care for a client with schizophrenia who receives haloperidol decanoate 75mg IM every 4 weeks. The client begins developing a puckering and smacking of the lips and facial grimacing. Which intervention should the nurse implement? A. Monitor lying, sitting, and standing blood pressures B. Provide coaching in relaxation techniques C. Complete abnormal involuntary movement scale (AIMS) D. Discontinue all medications immediately

C. Complete abnormal involuntary movement scale (AIMS)

What is the most important consideration when teaching parents how to reduce risks in the home? A) Age and knowledge level of the parents B) Proximity to emergency services C) Number of children in the home D) Age of children in the home

D) Age of children in the home

A client is diagnosed with methicillin resistant staphylococcus aureus pneumonia. What type of isolation is most appropriate for this client? A) Reverse B) Airborne C) Standard precautions D) Contact

D) Contact

When assessing a recently delivered, multigravida client, the nurse finds that her vaginal bleeding is more than expected. Which factor in this client's history is related to this finding? A. The second stage of labor lasted 10 minutes B. She received butorphanol 2mg IVP during labor C. She is over 35 years of age D. She is a gravida 6, para 5

D. She is a gravida 6, para 5

A client fell in the bathroom when left unattended by the unlicensed assistive personnel (UAP). Which information should the nurse include in the client's health record? A. The UAP left the client to assist another client B. The last time client was assisted to the bathroom C. The unit was understaffed when the client fell D. The client fell sustaining a fracture to the left hip

D. The client fell sustaining a fracture to the left hip

When administering an immunization in an adult client, the nurse palpates and administer the injection one inch below the acromion process into the center of the muscle mass. The nurse should document that the vaccine was administered at what site? a. Rectus femenis b. Ventrogluteous c. Vastus lateralis d. Deltoid

Deltoid

A 12 year old client who had an appendectomy two days ago is receiving 0.9% normal saline at 50 ml/hour. The client's urine specific gravity is 1.035. What action should the nurse implement? a. Evaluate postural blood pressure measurements b. Obtain specimen for uranalysis c. Encourage popsicles and fluids of choice d. Assess bowel sounds in all quadrants

Encourage popsicles and fluids of choice Rationale: specific gravity of urine is a measurement of hydration status (normal range of 1.010 to 1.025) which is indicative of fluid volume deficit when Sp Gr increases as urine becomes more concentrated.

After administering an antipyretic medication. Which intervention should the nurse implement? A. Encouraging liberal fluid intake

Encouraging liberal fluid intake

In monitoring tissue perfusion in a client following an above the knee amputation (aka), which action should the nurse include in the plan of care? a. Evaluate closet proximal pulse. b. Asses skin elasticity of the stump. c. Observe for swelling around the stump. d. Note amount color of wound drainage.

Evaluate closet proximal pulse. Rationale: A primary focus of care for a client with an AKA is monitoring for signs of adequate tissue perfusion, which include evaluating skin color and ongoing assessment of pulse strength.

The nurse discovers that an elderly client with no history of cardiac or renal disease has an elevated serum magnesium level. To further investigate the cause of this electrolyte imbalance, what information is most important for the nurse to obtain from the client's medical history? a. Genetically inherited disorders of family members b. Length and frequency of the client's tobacco use. c. Ingestion of selfish or fish oil capsules daily. d. Frequency of laxative use for chronic constipation

Frequency of laxative use for chronic constipation Rationale: Elderly clients are at risk of developing hypermagnesemia as a result of chronic laxative abuse.

Which action should the school nurse take first when conducting a screening for scoliosis? a. Compare dorsal measurement of trunk b. Extend arms over head for visualization c. Inspect for symmetrical shoulder height. d. Observe weight-bearing on each leg.

Inspect for symmetrical shoulder height. Rationale: Children between 9 and 15 years old should be screening for scoliosis, which is exhibited.... Vertebral column. Screening for scoliosis should begin with inspection of shoulder height

A client is receiving continuous bladder irrigation via a triple-lumen suprapubic catheter that was placed during prostatectomy. Which report by the unlicensed assistive personnel (UAP) requires intervention by the nurse? a. Leakage around catheter insertion site

Leakage around catheter insertion site

Client has a Lithium Rx level of 0.54 (know Lithium ranges

Low, so the answer is to ask client if they have been taking their medication everyday

A client is admitted to isolation with the diagnosis of active tuberculosis. Which infection control measures should the nurse implement? Negative pressure environment

Negative pressure environment

During the infusion of a second unit of packed red blood cells, the client's temperature increases from 99 to 101.6 f. which intervention should the nurse implement? a. Stop the transfusion start a saline

Stop the transfusion start a saline

The nurse assesses a 78-year-old male client who has left sided heart failure. Which symptoms would the nurse expect this client to exhibit? a- Dyspnea, cough, and fatigue. b- Hepatomegaly and distended neck veins c- Pain over the pericardium and friction rub. d- Narrowing pulse pressure and distant heart sounds.

a. Dyspnea, cough, and fatigue.

A male Korean-American client looks away when asked by the nurse to describe his problem. What is the best initial nursing action? a- Ask social services to dins a Korean interpreter b- Establish direct eye contact with the client. c- Allow several minutes for the client to respond. d- Repeat the question slowly and distinctly.

c. Allow several minutes for the client to respond

The nurse manager has been using a decentralized block scheduling plan to staff the nursing unit. However, staff have asked for many changes and exceptions to the schedule over the past few months. The manager considers self scheduling knowing that this method will A) Improve the quality of care B) Decrease staff turnover C) Minimize the amount of overtime payouts D) Improve team morale

is D: Improve team morale 3

A nurse who works in the nursery is attending the vaginal delivery of a term infant. What action should the nurse complete prior to leaving the delivery room? a. Place the id bands on the infant and mother

place the id bands on the infant and mother

An infant is placed in a radiant warmer immediately after birth. At one hour of age, the nurse finds the infant tachypneic, and hypotonic. What is the first action that the nurse should take? a. Notify the healthcare provider immediately b. Increase the temperature of the radiant warmer c. Assess the infant's heart rate. d. Determine the infant's blood sugar level.

d. Determine the infant's blood sugar level

Metabolic syndrome - high cholesterol &

high cholesterol & high BP (cardiac strip)

DVT assessment

-Pain -Warmth -Increased size in leg: sometimes they measure the calf

Which of these statements by the nurse reflects the best use of therapeutic interaction techniques? A) "You look upset. Would you like to talk about it? "B) "I'd like to know more about your family. Tell me about them." C) "I understand that you lost your partner. I don't think I could go on if that happened to me." D) "You look very sad. How long have you been this way?"

A) "You look upset. Would you like to talk about it?"

To prevent unnecessary hypoxia during suctioning of a tracheostomy, the nurse must A) Apply suction for no more than 10 seconds B) Maintain sterile technique C) Lubricate 3 to 4 inches of the catheter tip D) Withdraw catheter in a circular motion

A) Apply suction for no more than 10 seconds

An 85 year-old client complains of generalized muscle aches and pains. The first action by the nurse should be A) Assess the severity and location of the pain B) Obtain an order for an analgesic C) Reassure him that this is not unusual for his age D) Encourage him to increase his activity

A) Assess the severity and location of the pain

The nurse is caring for a client 2 hours after a right lower lobectomy. During the evaluation of the water-seal chest drainage system, it is noted that the fluid level bubbles constantly in the water seal chamber. On inspection of the chest dressing and tubing, the nurse does not find any air leaks in the system. The next best action for the nurse is to A) Check for subcutaneous emphysema in the upper torso B) Reposition the client to a position of comfort C) Call the health care provider as soon as possible D) Check for any increase in the amount of thoracic drainage

A) Check for subcutaneous emphysema in the upper torso

The nurse is teaching parents about accidental poisoning in children. Which point should be emphasized? A) Call the Poison Control Center once the situation is identified B) Empty the child's mouth in any case of possible poisoning C) Have the child move minimally if a toxic substance was inhaled D) Do not induce vomiting if the poison is a hydrocarbon

is B: Empty the child''s mouth in any case of possible poisoning 9

A nurse observes a family member administer a rectal suppository by having the client lie on the left side for the administration. The family member pushed the suppository until the finger went up to the second knuckle. After 10 minutes the client was told by the family member to turn to the right side and the client did this. What is the appropriate comment for the nurse to make? A) Why don't we now have the client turn back to the left side. B) That was done correctly. Did you have any problems with the insertion? C) Let's check to see if the suppository is in far enough. D) Did you feel any stool in the intestinal tract?

is B: That was done correctly. Did you have any problems with the insertion? 1

A 42 year-old male client refuses to take propranolol hydrochloride (Inderal) as prescribed. Which client statement s from the assessment data is likely to explain his noncompliance? A) "I have problems with diarrhea." B) "I have difficulty falling asleep." C) "I have diminished sexual function." D) "I often feel jittery."

is C: "I have diminished sexual function." 2

An 8 year-old client is admitted to the hospital for surgery. The child's parent reports the following allergies. Of these allergies which one should all health care personnel be aware of? A) Shellfish B) Molds C) Balloons D) Perfumed soap

is C: Balloons 9

The nurse is preparing the teaching plan for a group of parents about risks to toddlers. The nurse plans to explain proper communication in the event of accidental poisoning. The nurse should plan to tell the parents to first state what substance was ingested and then what information should be the priority for the parents to communicate? A) The parents' name and telephone number B) The currency of the immunization and allergy history of the child C) The estimated time of the accidental poisoning and a confirmation that the parents will bring the containers of the ingested substance D) The affected child's age and weight

is D: The affected child''s age and weight 7

Late decelerations

nurse should turn off the Pitocin infusion first

The nurse on a pediatric unit observes a distraught mother in the hallway scolding her 3 year old son for wetting his pants. What initial action should the nurse take? A. Provide disposable training pants while calming the mother B. Refer the mother to a community parent education program C. Inform the mother that toilet training is slower for boys D. Suggest that the mother consult a pediatric nephrologist

C. Inform the mother that toilet training is slower for boys

Teaching for mom breast feeding

-If engorged, feed the baby or pump

When assessing a client who has just undergone a cardioversion, the nurse finds the respirations are 12. Which action should the nurse take first? A) Try to vigorously stimulate normal breathing B) Ask the RN to assess the vital signs C) Measure the pulse oximetry D) Continue to monitor respirations

D) Continue to monitor respirations

A client with a prescription for "do not resuscitate" (DNR) begins to manifest signs of impending death. After notifying the family of the client's status, what priority action should the nurse implement? A. The impending signs of death should be documented B. The client's status should be conveyed to the chaplain C. The client's need for pain medication should be determined D. The nurse manager should be updated on the client's status

C. The client's need for pain medication should be determined

A preschooler with constipation needs to increase fiber intake. Which snack suggestion should the nurse provide? a. oatmeal cookies

oatmeal cookies

Risk Osteoarthritis

seen in long distance runner since high school client

Prednisone

-Class: Adrenal corticosteroid -Inhibits accumulation of inflammatory cells at inflammation site -Prevents/suppresses immune reactions -Frequent side effects: puffy round face, insomnia, nervousness, mood swings, increased appetite, delayed wound healing, increased susceptibility to infection -Adverse effects: muscle wasting, cushings syndrome, rebound inflammation, abrupt withdrawal, could be fatal -Nursing implications: watch for mania, monitor BP, serum electrolytes, glucose, bone mineral density test, ht/wt in children, increased risk for candida, be alert to infection, avoid alcohol, minimize use of caffeine

Wernicke's syndrome

-Common in alcoholics -Sx: confusion, memory loss, ataxia, nystagmus, ptosis, abnormal gait

The nurse assess a client being treated for Herpes zoster (shingles). Which assessments should the nurse include when evaluating the effectiveness of the the treatment? (Select all that apply) A. Functional ability B. Skin integrity C. Pain scale D. Bowel sounds E. heart sounds

A. Functional ability B. Skin integrity

Prior to obtaining a trapeze bar for a client with limited mobility, which client assessment is most important for the nurse to obtain? A. Upper body muscle strength B. Balance and posture C. Risk for disuse syndrome D. Pressure sore risk

A. Upper body muscle strength

A male client with cancer, who is receiving antineoplastic drugs, is admitted to the... what findings is most often manifest this condition? a. Ecchymosis and hematemesis b. Weight loss and alopecia c. Weakness and activity intolerance d. Sore throat and fever

a. Ecchymosis and hematemesis

A gravida 2 para 1, at 38-weeks gestation, scheduled for a repeat cesarean section in one week, is brought to the labor and delivery unit complaining of contractions every 10 minutes. While assessing the client, the client's mothers enter the labor suite and says in a loud voice, "I've had 8 children and I know she's in labor. I want her to have her cesarean section right now!" what action should the nurse take? a) Tell the mother to stop speaking for the client b) Notify the charge nurse of the situation c) Request that the mother leave the room d) Request security to remove her from the room

c) Request that the mother leave the room

The nurse is assessing the emotional status of a client with Parkinson's disease. Which client finding is most helpful in planning goals to meet the client's emotional needs? a- Stares straight ahead without blinking b- Face does not convey any emotion c- Cries frequently during the interview d- Uses a monotone when speaking

c. Cries frequently during the interview

A client with a history of chronic pain requests a nonopioid analgesic. The client is alert but has difficulty describing the exact nature and location of the pain to the nurse. What action should the nurse implement next? a. Assess the client's vital signs b. Observe the client's pupils for dilation c. Document the client's drug tolerance d. Administer the analgesic as requested

d. Administer the analgesic as requested Rationale: Chronic pain may be difficult to describe, but should be treated with analgesics as indicated

The nurse who works in labor and delivery is reassigned to the cardiac care unit for the day because of a low census in labor and delivery. Which assignments is best for the nurse to give this nurse? a. Transfer a client to another unit b. Monitor the central telemetry c. Perform the admission d. Assist cardiac nurses with their assignments

d. Assist cardiac nurses with their assignments

In caring for a client with a PCA infusion of morphine sulfate through the right cephalic vein, The nurse assesses that the client is lethargic with a blood pressure of 90/60, pulse rate of 118 beats per minute, and respiratory rate of 8 breaths per minutes. What assessment should the nurse perform next? a. Note the appearance and patency of the client's peripheral IV site. b. Palpate the volume of the client's right radial pulse c. Auscultate the client's breath sounds bilaterally. d. Observe the amount and dose of morphine in the PCA pump syringe.

d. Observe the amount and dose of morphine in the PCA pump syringe

The nurse is caring for a client with sickle cell disease who is scheduled to receive a unit of packed red blood cells. Which of the following is an appropriate action for the nurse when administering the infusion? A) Storing the packed red cells in the medicine refrigerator while starting IV B) Slow the rate of infusion if the client develops fever or chills C) Limit the infusion time of each of the unit to a maximum of 4 hours D) Assess vital signs every 15 minutes throughout the entire infusion

is C: Limit the infusion time of each of the unit to a maximum of four hours 8

A nurse is to present information about Chinese folk medicine to a group of student nurses. Based on this cultural belief, the nurse would explain that illness is attributed to the A) Yang, the positive force that represents light, warmth, and fullness B) Yin, the negative force that represents darkness, cold, and emptiness C) Use of improper hot foods, herbs and plants D) A failure to keep life in balance with nature and others

B) Yin, the negative force that represents darkness, cold, and emptiness

While teaching a young male adult to use an inhaler for his newly diagnosed asthma, the client stares into the distance and appears to be concentrating on something other than the lesson the nurse is presenting. What action should the nurse take a. Remind the client that a rescue inhaler might save his life b. Leave the client alone so that he can grieve his illness c. Ask the client what he is thinking about at his time. d. Gently touch the client then continue with teaching.

Ask the client what he is thinking about at his time.

A preschool-aged boy is admitted to the pediatric unit following successful resuscitation from a near-drowning incident. While providing care to child, the nurse begins talking with his preadolescent brother who rescued the child from the swimming pool and initiated resuscitation. The nurse notices the older boy becomes withdrawn when asked about what happened. What action should the nurse take a. Ask the older brother how he felt during the incident. b. Commend the older brother for his heroic actions c. Tell the older brother that he seems depressed d. Develop a water safety teaching plan for the family.

Ask the older brother how he felt during the incident Rationale: The brother's change in demeanor may indicate that he is experiencing post-traumatic stress that warrants further investigation, so the nurse should address the older brother's feeling.

The nurse is caring for 2 children who have had surgical repair of congenital heart defects. For which defect is it a priority to assess for findings of heart conduction disturbance? A) Arterial septal defect B) Patent ductus arteriosus C) Aortic stenosis D) Ventricular septal defect

D) Ventricular septal defect

What might the nurse suggest to a client with fibrocystic breasts in an attempt to help relieve her symptoms? A. "Eliminate caffeine from your diet" B. "Avoid vigorous physical exercise immediately after your menstrual periods" C. "Eat a low-carbohydrate, high-protein diet" D. "Increase high-calcium foods in your diet"

D. "Increase high-calcium foods in your diet"

The nurse leading a care team on a medical surgical unit is assigning client care to a practical nurse (PN) and an unlicensed assistive personnel (UAP). Which task should the nurse delegate to the UAP? A. Evaluate a client's mobility progress toward the plan of care B. Assess for side effects of administered pain medications C. Turn and reposition a client with a total hip replacement D. Monitor an intravenous infusion rate on an established schedule

C. Turn and reposition a client with a total hip replacement

A client with rapid respirations and audible rhonchi is admitted to the intensive care unit because of a pulmonary embolism (PE). Low-flow oxygen by nasal cannula and weight based heparin protocol is initiated. Which intervention is most important for the nurse to include in this client's plan of care? a. Evaluate daily blood clotting factors.

Evaluate daily blood clotting factors.

Chest tube disconnect

( Put the end of the tube in a sterile bottle fill with Normal saline and connect back to patient)

A client with a chest tube develops sepsis and dyspnea. Based on findings in the client's medical record, which prescription should the nurse implement first?

(Click on each chart tab for additional Information. Please be sure to scroll to the bottom right corner of each tab to view all information contained in the clients medical record)

Incidence

-# of new cases at a certain time

Describe Lupus and pt education

-*Autoimmune disorder -Could have a decreased WBC count; increased risk for infection -Avoid crowds, avoid sick people, avoid sunlight (trigger)

DNR

-Do not resuscitate -Need a document stating they're DNR, otherwise you're "failing to rescue" -Can't just take family's word for the pts code status

Chrones Diet

-Low fat -High-calorie: dried fruit increases calories and nutrients -High nutrient foods -Ex: eggs w/o yolk, lean meat, fish, seafood, chicken

DVT plan of care

-Maintain on bed rest -No SCDs -Concern after surgery -Tx: heparin; d/c daily dose of aspirin

Safety concerns with mental health problems

-Make sure they're not harming self -Suicidal/homicidal intentions -Always ask questions (if they're suicidal, their plan, etc) -Clarify what their answers imply

Hearing

-Maleus (hammer) is first small bone hit from the eardrum vibrations -Sound waves go to tiny hair cells in cochlea and communicate with the brain to perceive sound

Lithium Carbonate

-Mood stabilizer, antimanic, antidepressive -For bipolar disorder: affects storage, relsease, and reuptake of neurotransmitters -Adverse reactions/side effects: nausea, fatigue, hand tremors, weight gain, hypothyroid, renal impairment -Toxicity: diarrhea, muscle weakness, unsteady gait, slurred speech -Nursing considerations: maintain adequate serum levels (0.6-1.2), toxicity over 1.5, check renal, cardiac, thyroid levels, Na, do not use with diuretics

Teaching for mom bottle feeding

-No stimulation -Tight, supportive bra

Regression

-Reverting to an earlier level of development -Ex: Pedi pt wetting the bed at the hospital

Symptoms of transfusion reaction

-Sx: fever, chills, flank pain, change in VS, nausea, headache, urticaria, dyspnea, bronchospasm -Stop infusion immediately

Von Willebrand's disease signs/symptoms

-Sx: recurrent/prolonged nose bleeds, bleeding gums, increased menstrual flow, excessive bleeding from cut, blood in stool/urine

Describe post-surgical nursing assessment

-VITALS!! -Take VS first to give you "the lay of the land" and plan further treatment -Watch for shock and hemorrhaging -Make sure BP is above 90 -Give O2 if decreased sat -Look/ask about pain if HR increased

How the nurse provides continuity of care

-Verbal reports -Pt handoff: verbal and continuity of care form -Communication ensures continuity of care

Neurogenic Diabetes Insipidus assessment

-Very high urine output, very dilute -Monitor sodium *Monitor osmolarity and osmolality: abnormal means its dilute urine -Mostly from trauma to hypothalamus

Apgar score for muscle tone

0 = flaccid 1 = some flexion 2 = total flexion

Apgar score for resp effort

0 = no cry 1 = weak cry 2 = vigorous cry

A school-age child who weighs 42 pounds receives a post-tonsillectomy prescription for promethazine (Phenergan) 0.5 mg/kg IM to prevent postoperative nausea. The medication is available in 25 mg/ml ampules. How many ml should the nurse administer? (Enter numeric value only. If rounding is required, round to the nearest tenth). 0.4

0.4 Rationale: Convert pounds to kg 42lbs = 19.09 kg Next calculate to prescribed dose, 0.5 mg x 1909 kg = 9.545 Then use the desired dose/ dose on hand x volume on hand (9.545/25x1ml =0.3818=0.4 ml) Or use ratio proportion (9.545 mg: x ml = 25 mg: 1ml 25x = 9.545 X= 0.3818 = 0.4)

The nurse notes the client receiving heparin infusion labeled, Heparin Na 25,000 Units in 5% Dextrose injection 500 ml at 50ml/hr. What dose of Heparin is the client receiving per hour? 2,500

2500

An adult client experiences a gasoline tank fire when riding a motorcycle and is admitted to the emergency department (ED) with full thickness burns to all surfaces of both lower extremities. What percentage of body surface area should the nurse document in the electronic medical record (EMR)? 36 (1 total leg front/back = 18, 1 total arm front/back = 9, torso = 18, back = 18, head = 9, pubic = 1 = 100%)

36% Rational: according to the rule of nines, the anterior and posterior surfaces of one lower extremity is designated as 18 %of total body surface area (TBSA), so both extremities equal 36% TBSA, other options are incorrect.

The healthcare provider prescribes a fluid challenge of 0.9% sodium chloride 1,000 mL to be infused intravenously over 4 hours. The IV administration set delivers 10gtt/mL. How many gtt/minute should the nurse regulate the infusion? (Round to the nearest whole number)

42 gtt/min

The nurse mixes 250 mg of debutamine in 250 ml of D5W and plans to administer the solution at rate client weighing 110 pounds. The nurse should set the infusion pump to administer how many ml per hour only. If rounding is required, round the nearest whole number.) 45

45

Four clients arrive at the labor and delivery nurses' station at the same time. Which pt should the nurse assess first?

A 38-week multigravida with biophysical profile score of 4 out of 8

Prevalence

-# of new and existing cases at a certain time

Fat embolism risk factors

-Breaking a long bone (usually femur)

A client is receiving an IV solution labeled Heparin Sodium 20,000 Units in 5% dextrose injection 500 ml at 25 ml/hour. How many units of heparin is the client receiving each hour? 1000

1000 units/hour Rationale:20000/500=40x25=1000

A client is receiving an IV solution of nitroglycerin 100mg/500ml D5W at 10 mcg/ minute. The nurse should program the infusion pump to deliver how many ml/hour? ( Enter numeric value only) 3

3 ml/hour Rationale : 0.01 x 500 x 60 / 100 = 3

The nurse suspect a patient may be hemorrhaging internally. Which findings of an orthostatic test may indicate signs to the nurse of a major bleed? a. A decrease in the systolic b/p of 10mm/hg with a corresponding increase of heart rate of 20.

A decrease in the systolic b/p of 10mm/hg with a corresponding increase of heart rate of 20.

The nurse in the emergency department is using the simple triage and rapid transport (START) system to assess victims of a hurricane. Which statement correctly describes a yellow disaster tag?

A yellow disaster tag means critical injuries and require immediate intervention.

A client develops volume overload from an IV that has infused too rapidly. What assessment would the nurse expect to find? A) S3 heart sound B) Thready pulse C) Flattened neck veins D) Hypoventilation

A) S3 heart sound

The nurse is performing an assessment on a client in congestive heart failure. Auscultation of the heart is most likely to reveal A) S3 ventricular gallop B) Apical click C) Systolic murmur D) Split S2

A) S3 ventricular gallop

The nurse is managing the care of a client with Cushing's syndrome. Which interventions should the nurse delegate to the unlicensed assistive personnel (UAP)? (Select all that apply) A. Report any client complaint of pain or discomfort B. Evaluate the client for sleep disturbances C. Assess the client for weakness and fatigue D. Weigh the client and report any weight gain E. Note and report the client's food and liquid intake during meals and snacks

A. Report any client complaint of pain or discomfort D. Weigh the client and report any weight gain E. Note and report the client's food and liquid intake during meals and snacks

A new nurse preparing to irrigate an intravenous catheter is attaching a 24-gauge needle. Which action should the charge nurse implement? A. Suggest the nurse use a 20-gauge needle B. Instruct the nurse to remove the needle C. Direct the nurse to change the IV tubing D. Prompt the nurse to apply povidone to the site

A. Suggest the nurse use a 20-gauge needle

A client with carpal tunnel syndrome is in the out-patient surgical unit after an endoscopic carpal tunnel release. What instructions should the nurse provide the client regarding postoperative care?

After carpel tunnel release, measure to promote circulation, comfort, and to prevent complications should include positioning the hand above the heart to reduce edema.

Septic shock-best class of medicine to give are

Anti infectives

The healthcare provider performs peritoneal dialysis on a client, after which 2 liters of fluid is drained. What action should the nurse complete first?

Assess vital signs.

A mother brings her 15-month old son and 6 year-old daughter to the clinic for immunizations. Both children are fretful and obviously have upper respiratory infections. The mother tells the nurse that the younger child ran a fever of 100.2oF following his last immunization. What plan is best for the nurse to implement?

Administer the boosters and instruct the mother to call the clinic if either child's temperature exceeds 101oF.

A client arrives in the emergency center with a blood alcohol level of 500 mg/dl. When transferred to the observation unit, the client becomes demanding, aggressive, and shouts at the staff. Which assessments finding is most important for the nurse to identify in the first 24 hours? a. Agitation and threatens to harm the staff

Agitation and threats to harms staff

The father of an 8 month-old infant asks the nurse if his infant's vocalizations are normal for his age. Which of the following would the nurse expect at this age? A) Cooing B) Imitation of sounds C) Throaty sounds D) Laughter

B) Imitation of sounds

After receiving the Braden scale findings of residents at a long-term facility, the charge nurse should to tell the unlicensed assistive personnel (UAP) to prioritize the skin care for which client? a. An older adult who is unable to communicate elimination needs. b. An older man whose sheets are damped each time he is turned. c. A woman with osteoporosis who is unable to bear weight. d. A poorly nourished client who requires liquid supplement.

An older man whose sheets are damped each time he is turned. Rational: a Braden score of less than 18 indicates a risk for skin breakdown, and clients with such score require intensive nursing care. Constant moisture places the client at a high risk for skin breakdown, and interventions should be implemented to pull moisture away from the client's skin. Other options may be risk factors but do not have as high a risk as constant exposure to moisture.

Hypertension Diet Teaching- presented with a list of menu choices, the best choice was

Baked pork chops, applesauce,etc

A client has a history of chronic atrial fibrillation. Which instruction should the nurse include in the teaching plan for this client?

Be sure to take the prescribed daily aspirin.

In planning care for a 6 month-old infant, what must the nurse provide to assist in the development of trust? A) Food B) Warmth C) Security D) Comfort

C) Security

A male client who was admitted with an acute myocardial infarction receives a cardiac diet with sodium restriction and complains that his hamburger is flavorless. Which condiment should the nurse offer? A. Pickle relish. B. Steak sauce. C. Fresh horseradish. D. Tomato ketchup.

C. Fresh horseradish. Rationale: A cardiac diet restrict sodium intake. Horseradish (C) should be recommended because it is low in sodium. (A, B, and D) are high in salt content and should not be offered.

The healthcare provider prescribes heparin protocol at 18 units/kg/hr for a client with a possible pulmonary embolism. This client weighs 144 pounds. The available solution is labeled, heparin sodium 25,000 units in 5% dextrose 250 ml. the nurse should program the pump to deliver how many ml/hr? (Enter numeric value only. If rounding is require round to the nearest whole number.) 12

Answer 12 Rationale: 144/2.2= 65kg 18units/kg/hr 65 kg x 18units/kg/hr= 1170 units/hr 25000 units heparin/250 ml of D5W = 100 units heparin per ml of solution

The nurse assesses a client with new onset diarrhea. It is most important for the nurse to question the client about recent use of which type of medication? a. Antibiotics b. Anticoagulants c. Antihypertensive d. Anticholinergics

Antibiotics Rationale: Antibiotic use may be altering the normal flora in the GI tract, resulting in the onset of diarrhea, and several classes of antibiotics result in the overgrowth of Clostridium difficile, resulting in severe diarrhea.

An elderly male client is experiencing urinary incontinence. What is the best initial nursing action?

Apply an external condom catheter

The nurse should withhold which medication if the client's serum potassium level is 6.2 mEq/L? A. Metolazone B. Furosemide C. Spironolactone D. Hydrochlorothiazide

C. Spironolactone

After a spider bite on the lower extremity, a client is admitted for treatment of an infection that is spreading up the leg. Which admission assessment findings should the nurse report to the healthcare provider? (Select all that apply) A. Location of the initial IV site B. Red blood cell count (RBC) C. Swollen lymph nodes in the groin D. White blood cell count (WBC) E. Core body temperature

C. Swollen lymph nodes in the groin D. White blood cell count (WBC) E. Core body temperature

A client with hyperthyroidism is admitted to the postoperative unit after a subtotal thyroidectomy. Which of the client's serum laboratory values requires intervention by the nurse? A. T3- uptake at 50% B. Glucose 150 mg/dL C. Total calcium 5.0 mg/dL D. Thyroxine 12 mcg/dL

C. Total calcium 5.0 mg/dL

A male client with cancer is admitted to the oncology unit and tells the nurse that he is in the hospital for palliative care measures. The nurse notes that the client's admission prescription include radiation therapy. What action should the nurse implement? a. Ask the client about his expected goals for the hospitalization b. Explain the palliative care measures can be provided at home c. Notify do radiation department to withhold the treatment for now d. Determine if the client wishes to cancel further radiation treatment

Ask the client about his expected goals for the hospitalization Rationale: Palliative care measures provide relief or control of symptoms, so it is important for the nurse to determine the client's goals for symptom control while receiving treatment in the hospital. Although home care is available the client may not be legible for palliative care at home. Radiation therapy is an effective positive care measure used to manage symptoms and would be appropriate unless the radiation conflicts with the client goals.

A young adult male who is being seen at the employee health care clinic for an annual assessment tell the nurse that his mother was diagnosed with schizophrenia when she was his age and that life with a schizophrenic mother was difficulty indeed. Which response is best for the nurse to provide? a. Ask the client if he is worried about becoming schizophrenic at the age his mother was diagnosed. b. Encourage the client to seek genetic counseling to determine his risk for mental illness. c. Informed the client that his mother schizophrenic has affected his psychological development. d. Tell the client that mental illness has a familial predisposition, so he should see a psychiatrist.

Ask the client if he is worried about becoming schizophrenic at the age his mother was diagnosed.

Teaching

Carafate given before and after meal

A female client is admitted with end stage pulmonary disease is alert, oriented, and complaining of shortness of breath. The client tells the nurse that she wants "no heroic measures" taken if she stops breathing, and she asks the nurse to document this in her medical record. What action should the nurse implement? Ask the client to discuss "do not resuscitate" with her healthcare provider

Ask the client to discuss "do not resuscitate" with her healthcare provider

Leg cast Assessment

Compartment syndrome

Translator is working with the nurse who is giving discharge instructions to a non-English client. When the translator restates what the nurse is saying it appears that he is saying much more than what the nurse said. What action should the nurse take?

Ask the translator if there is a reason for the lengthiness of the translation

A male client with rheumatoid arthritis is schedule for a procedure in the morning. The... unable to complete the procedure because of early morning stiffness. Which intervention... implement? a. Assign a UAP to assist the client with a warm shower early in the morning

Assign a UAP to assist the client with a warm shower early in the morning

An unlicensed assistive personnel (UAP) assigned to obtain client vital signs reports to the charge nurse that a client has a weak pulse with a rate of 44 beat/ minutes. What action should the charge nurse implement? a- Instruct the UAP to count the client apical pulse rate for sixty seconds b- Determine if the UAP also measured the client's capillary refill time. c- Assign a practical nurse (LPN) to determine if an apical radial deficit is present. d- Notify the health care provider of the abnormal pulse rate and pulse volume.

Assign a practical nurse (LPN) to determine if an apical radial deficit is present

A confused, older client with Alzheimer's disease becomes incontinent of urine when attempting to find the bathroom. Which action should the nurse implement? a. Assist the client's to a bedside commode every two hours

Assist the client's to a bedside commode every two hours

A client with atrial fibrillation receives a new prescription for dabigatran. What instruction should the nurse include in this client's teaching plan? a. Keep an antidote available in the event of hemorrhage b. Continue obtaining scheduled laboratory bleeding test c. Eliminate spinach and other green vegetable in the diet. d. Avoid use of nonsteroidal ant-inflammatory drugs (NSAID).

Avoid use of nonsteroidal ant-inflammatory drugs (NSAID). Rationale: Dabigatran, a directed reversible thrombin inhibitor, is prescribe to reduce the risk of stroke in client with atrial fibrillation. The risk of bleeding and GI event can be significant and the concomitant use of NSAID and other anticoagulants should be avoided.

In a child with suspected coarctation of the aorta, the nurse would expect to find A) Strong pedal pulses B) Diminishing carotid pulses C) Normal femoral pulses D) Bounding pulses in the arms

D) Bounding pulses in the arms

A client is prescribed Zolpidem (Ambien). Which of the following side effects should be monitored while the client is taking this medication?

Daytime drowsiness

Which finding should the nurse expect a client to exhibit who is newly diagnosed with fibromyalgia?

Disrupt in sleep patherns.

The nurse is preparing a discharge plan for an older client who was recently diagnosed with Alzheimer's disease. Which intervention should the nurse suggest to the spouse if the client becomes uncooperative at home?

Ensure a calm and predictable environment.

What action should the nurse implement first when delegating nursing activities to an unlicensed assistive personnel (UAP)?

Evaluate the experience of the UAP.

Decerebrate Positioning

Extension of the upper and lower extremity.

A client with dyspnea is being admitted to the medical unit. To best prepare for the client's arrival, the nurse should ensure that the client's bed is in which position? A. Supine B. supine; feet elevated higher than head C. supine; head elevated higher than feet D. Fowlers

Fowlers

Teenager attempted suicide and during group session became upset and left group

Go to client room and ask what happened.

Nurse writes on the chart of Bob Miller that his nurse, Nurse Smith did not take great care of Mr. Miller. Nurse Jones has committed which of the following?

Libel

The nurse is planning care for a client admitted with a diagnosis of pheochromocytoma. Which intervention has the highest priority for inclusion in this client's plan of care? a. Record urine output every hour b. Monitor blood pressure frequently c. Evaluate neurological status d. Maintain seizure precautions

Monitor blood pressure frequently Rationale: A pheochromocytoma is a rare, catecholamine-secreting tumor that may precipitate life-threatening hypertension

A client with a history of using illicit drugs intravenously is admitted with Kaposi's sarcoma. Which intervention should the nurse include in this client's admission plan of care? a. Monitor for secondary infections.

Monitor for secondary infections.

The nurse is auscultating a client's heart sounds. Which description should the nurse use to document this sound? (Please listen to the audio first to select the option that applies)

Murmur

39 to 48 Glucose

Notify healthcare provider of client's status

Medical Surgical/Laxative abuses

Patient -taking Milk of Magnesium

A 17-year -old male is brought to the emergency department by his parents because he has been coughing and running a fever with flu-like symptoms for the past 24 hours. Which intervention should the nurse implement first? a. Place a mask on the client's face.

Place a mask on the client's face.

Endotrach has slipped into right main bronchus of no breath sounds are heard (Audio/Visual)

Pleaurvac, know how to set vac at 850

A client with diabetic peripheral neuropathy has been taking pregabalin (Lyrica) for 4 days. Which finding indicates to the nurse that the medication is effective? a. Reduced level of pain b. Full volume of pedal pulses c. Granulating tissue in foot ulcer d. Improved visual acuity.

Reduced level of pain Rationale: Pregabalin is prescribed to decrease the pain associated with diabetic peripheral neuropathy. A, C and D are not expected outcomes of this medication's effectiveness.

Which diagnosis tests are most important for the nurse to monitor when providing care for a client with a bowel movement?

Serum electrolytes.

The nurse notes that a postoperative adult client's respiratory rate is 10 breaths/minute. Which factor in the client's history is the most likely explanation for this finding?

The PCA pump containing morphine sulfate was discontinued 15 minutes before vital signs were taken.

A 68-year-old male has been admitted to the hospital with abdominal pain, anemia and bloody stools. He complains of feeling weak and dizzy. He has rectal pressure and needs to urinate and move his bowels.

The nurse should help him onto the bedpan/with the urinal

What explanation is best for the nurse to provide a client who asks the purpose of using the log-rolling technique for turning? a. working together can decrease the risk for back injury b. The technique is intended to maintain straight spinal alignment. c. Using two or three people increases client safety. d. turning instead of pulling reduces the likelihood of skin damage

The technique is intended to maintain straight spinal alignment. Rationale: The main rationale for use of the long-rolling technique is to maintain the client's spine straight alignment.

A 2-year-old is bleeding from a laceration on the right lower extremity that occurred as the result of a motor vehicle collision. The nurse is selecting supplies to start an IV access. Which assessment finding is most significant in the nurse's selection of catheter size? a. Thready brachial pulse. b. Respirations of 24/minute c. Right foot cool to touch. d. Swelling at the site of injury

Thready brachial pulse.

What is manntiol drug used for?

To promote diuresis in prevention and treatment of oliguric phase of renal failure, ICP, IOP, edema in CHF.

Which client should the nurse assess frequently because of the risk for overflow incontinence? A client a. Who is bedfast, with increased serum BUN and creatinine levels b. Who is confused and frequently forgets to go to the bathroom c. With hematuria and decreasing hemoglobin and hematocrit levels d. Who has a history of frequent urinary tract infections.

Who is confused and frequently forgets to go to the bathroom Rationale: Overflow incontinence occurs when the bladder becomes overly distended, which is common in the confused client (B) who does not remember to empty his/her bladder.

The nurse is teaching a group of clients with rheumatoid arthritis about the need to modify daily activities. Which goal should the nurse emphasize? a. Protect joint function b. Improve circulation c. Control tremors d. Increase weight bearing.

a. Protect joint function Rationale: Primary goal in the management of rheumatoid arthritis is to protect and maintain joint function.

Client has rapid speech, flight of ideas, 3hrs of sleep

client experiencing high levels Depakote

Anterior Cruciate Ligament injury

crutches

What makes a baby frontal bulge or rises

crying

Therapeutic nurse-client interaction occurs when the nurse A) Assists the client to clarify the meaning of what the client has said B) Interprets the client's covert communication C) Praises the client for appropriate feelings and behavior D) Advises the client on ways to resolve problems

is A: Assists the client to clarify the meaning of what the client has said 12

An 80 year-old client on digitalis (Lanoxin) reports nausea, vomiting, abdominal cramps and halo vision. Which of the following laboratory results should the nurse analyze first? A) Potassium levels B) Blood pH C) Magnesium levels D) Blood urea nitrogen

is A: Potassium levels 11

The nurse is planning a meal plan that would provide the most iron for a child with anemia. Which dinner menu would be best? A) Fish sticks, french fries, banana, cookies, milk B) Ground beef patty, lima beans, wheat roll, raisins, milk C) Chicken nuggets, macaroni, peas, cantaloupe, milk D) Peanut butter and jelly sandwich, apple slices, milk

is B: Ground beef patty, lima beans, wheat roll, raisins, milk 6

What is the best way for the nurse to accomplish a health history on a 14 year-old client? A) Have the mother present to verify information B) Allow an opportunity for the teen to express feelings C) Use the same type of language as the adolescent D) Focus the discussion of risk factors in the peer group

B) Allow an opportunity for the teen to express feelings

The nurse is caring for a 4 year-old 2 hours after tonsillectomy and adenoidectomy. Which of the following assessments must be reported immediately? A) Vomiting of dark emesis B) Complaints of throat pain C) Apical heart rate of 110 D) Increased restlessness

is D: Increased restlessness 10

Adolescents and drug abuse at 13yrs old-

mostly likely to use, Paint thinners

Acute Pancreatitis

withhold food and fluid intake

A client who experienced a cerebrovascular accident (CVA) is aphasic and has left sided paralysis. Which nurse should be responsible for coordinating the progression of this client's care? A. Nurse case manager B. Adult nurse practitioner C. Neurology unit supervisor D. Risk management nurse

B. Adult nurse practitioner

The nurse is teaching a group of women about osteoporosis and exercise. The nurse should emphasize the need for which type of regular activity? A. Core strengthening B. Aerobic exercise C. Weight-bearing exercise D. Muscle stretching and toning

B. Aerobic exercise

The nurse is teaching the client about home care after surgery for an ileal conduit placement. When reviewing the information, which statement should the nurse recognize as needing additional education? A. report presence of mucus in the urine B. Empty pouch when it is half full C. Look at the stoma when replacing appliance D. Anticipate shrinking of the stoma

B. Empty pouch when it is half full

When assessing an IV site that is used for fluid replacement and medication administration, the client complains of tenderness when the arm is touched above the site. Which additional assessment finding warrants immediate intervention by the nurse? A. Client uses the arm cautiously B. Red streak tracking the vein C. A sluggish blood return D. Spot of dried blood at insertion site

B. Red streaks tracking the vein

A mother calls the nurse to report that at 0900 she administered a PO dose of digoxin to her 4-month-old infant, but at 0920 the baby vomited the medicine, what instruction should the nurse provide to this mother? A. Give another dose. B. Withhold this dose. C. Administer a half dose now. D. Mix the next dose with food.

B. Withhold this dose. Rationale: This dose should be withheld (B) because the amount absorbed by the infant is unknown. (A and C) pose safety concerns due to the unknown absorption. (D) is not recommended because all of the mixture (food and medicine) may not be eaten.

Which breakfast selection indicates that the client understands the nurse's instructions about the dietary management of osteoporosis? a. Egg whites, toast and coffee. b. Bran muffin, mixed fruits, and orange juice. c. Granola and grapefruit juice d. Bagel with jelly and skim milk.

Bagel with jelly and skim milk Rationale: D includes dairy products which contain calcium and does not include any foods that inhibit calcium absorption. The primary dietary implication of osteoporosis is the need for increased calcium and reduction in foods that decrease calcium absorption, such as caffeine and excessive fiber.

An adult male reports the last time he received penicillin he developed a severe maculopapular rash all over his chest. What information should the nurse provide the client about future antibiotic prescriptions? a. Be alert for possible cross-sensitivity to cephalosporin agents. b. Monitor peak ad trough levels whenever taking any antibiotic c. Watch daily urine output and weight gain while taking antibiotics d. Wear sun block and protective clothing to avoid exposure to sun.

Be alert for possible cross-sensitivity to cephalosporin agents Rationale: Cross-sensitivity with cephalosporin can occur in those who are allergic to penicillin, so the nurse should provide this warning.

When the nurse auscultates the anterior chest just above the right nipple moderate, pitched breath sounds are heard that are equal on inspiration and expiration. Which statement best describes this finding?

Bronchovesicular breath sounds that are normal in that location

While teaching a client about their medications, the client asks how long it will take before the effects of lithium take place. What is the best response of the nurse? A) Immediately B) Several days C) 2 weeks D) 1 month

C) 2 weeks

The nurse is caring for a client who has developed cardiac tamponade. Which finding would the nurse anticipate? A) Widening pulse pressure B) Pleural friction rub C) Distended neck veins D) Bradycardia

C) Distended neck veins

The nurse is caring for a client undergoing the placement of a central venous catheter line. Which of the following would require the nurse's immediate attention? A) Pallor B) Increased temperature C) Dyspnea D) Involuntary muscle spasms

C) Dyspnea

At the geriatric day care program a client is crying and repeating "I want to go home. Call my daddy to come for me." The nurse should A) Invite the client to join the exercise group B) Tell the client you will call someone to come for her C) Give the client simple information about what she will be doing D) Firmly direct the client to her assigned group activity

C) Give the client simple information about what she will be doing

A client has returned from a cardiac catheterization. Which one of the following assessments would indicate the client is experiencing a complication from the procedure? A) Increased blood pressure B) Increased heart rate C) Loss of pulse in the extremity D) Decreased urine output

C) Loss of pulse in the extremity

A client asks the nurse to call the police and states: "I need to report that I am being abused by a nurse." The nurse should first A) Focus on reality orientation to place and person B) Assist with the report of the client's complaint to the police C) Obtain more details of the client's claim of abuse D) Document the statement on the client's chart with a report to the manager

C) Obtain more details of the client's claim of abuse

A client was just taken off the ventilator after surgery and has a nasogastric tube draining bile colored liquids. Which nursing measure will provide the most comfort to the client?A) Allow the client to melt ice chips in the mouth B) Provide mints to freshen the breath C) Perform frequent oral care with a tooth sponge D) Swab the mouth with glycerin swabs

C) Perform frequent oral care with a tooth sponge

A child is admitted to the pediatric unit with a diagnosis of suspected meningococcal meningitis. Which admission orders should the nurse do first? A) Institute seizure precautions B) Monitor neurologic status every hour C) Place in respiratory/secretion precautions D) Cefotaxime IV 50 mg/kg/day divided q6h

C) Place in respiratory/secretion precautions

A mother asks about expected motor skills for a 3 year-old child. Which of the following would the nurse emphasize as normal at this age? A) Jumping rope B) Tying shoelaces C) Riding a tricycle D) Playing hopscotch

C) Riding a tricycle

In evaluating the growth of a 12 month-old child, which of these findings would the nurse expect to be present in the infant? A) Increased 10% in height B) 2 deciduous teeth C) Tripled the birth weight D) Head > chest circumference

C) Tripled the birth weight

A client tells the nurse, "I have something very important to tell you if you promise not to tell." The best response by the nurse is A) "I must document and report any information." B) "I can't make such a promise." C) "That depends on what you tell me." D) "I must report everything to the treatment team."

C) Was minimally responsive to voice and touch

The nurse is caring for a client who has chronic obstructive pulmonary disease (COPD) and chest pain related to a recent fall. What nursing intervention requires the greatest caution when caring for a client with COPD? A. Monitoring telemetry and cardiac rhythm B. Assisting client to cough and deep breath C. Administering narcotics for pain relief D. Increasing the client's fluid intake

C. Administering narcotics for pain relief

The nurse is managing 4 clients in the intensive care unit who are mechanically ventilated. After performing a quick visual assessment, the nurse should prioritize care for the client who is exhibiting which finding? A. An audible voice when client is trying to communicate B. High pressure alarm sounds when client is coughing C. Restrained and restless with a low volume alarm sounding D. Diminished breath sounds in the right posterior base

C. Restrained and restless with a low volume alarm sounding

On admission to the Emergency Department, a female client who was diagnosed with bipolar disorder 3 years ago reports that this morning she took a handful of medications and left a suicide note for her family. Which information is most important for the nurse to obtain? A. Which family member has the client's suicide note B. What drugs the client used for the suicide attempt C. When the client last took drugs for bipolar disorder D. Whether the client over attempted suicide in the past

C. When the client last took drugs for bipolar disorder

When assessing a multigravida the first postpartum day, the nurse finds a moderate amount of lochia rubra, with the uterus firm, and three fingerbreadths above the umbilicus. What action should the nurse implement first a. Check for a distended bladder

Check for a distended bladder

Upon assessment, the nurse discovers that a postpartum client has persistent red lochia. Which of the following does this assessment finding suggest to the nurse?

Coagulation disorders

Which assessment finding of a postmenopausal woman necessitates a referral by the nurse to the healthcare provider for evaluation of thyroid functioning? a. Cold sensitivity

Cold sensitivity

A client's subjective data includes dysuria, urgency, and urinary frequency. What action should the nurse implement next? a. Collect a clean-catch specimen

Collect a clean-catch specimen

When evaluating a client's rectal bleeding, which findings should the nurse document? a. Number of blood clots expelled with each stool. b. Unique odor noted with GI bleeding c. Evidence of internal hemorrhoids. d. Color characteristics of each stool.

Color characteristics of each stool Rationale: Color characteristics indicate if blood is coming from high in the GI tract, which would be black and tarry, or from lower area near rectum, which would be bright red blood.

A client exposed to tuberculosis is scheduled to begin prophylactic treatment with isoniazid. Which information is most important for the nurse to note before administering the initial dose? a. Conversion of the client's PPD test from negative to positive. b. Length of time of the exposure to tuberculosis. c. Current diagnosis of hepatitis B. d. History of intravenous drug abuse.

Current diagnosis of hepatitis B Rationale: prophylactic treatment of tuberculosis with isoniazid is contraindicated for persons with liver disease because it may cause liver damage. The nurse should withhold the prescribed dose and contact the healthcare provider. Other options do not provide data indicating the need to question or withhold the prescribed treatment.

Which of these clients who are all in the terminal stage of cancer is least appropriate to suggest the use of patient controlled analgesia (PCA) with a pump? A) A young adult with a history of Down's syndrome B) A teenager who reads at a 4th grade level C) An elderly client with numerous arthritic nodules on the hands D) A preschooler with intermittent episodes of alertness

D) A preschooler with intermittent episodes of alertness

The nurse is teaching the client to select foods rich in potassium to help prevent digitalis toxicity. Which choice indicates the client understands dietary needs? A) Three apricots B) Medium banana C) Naval orange D) Baked potato

D) Baked potato

Which bed position is preferred for use with a client in an extended care facility on falls risk prevention protocol? A) All 4 side rails up, wheels locked, bed closest to door B) Lower side rails up, bed facing doorway C) Knees bent, head slightly elevated, bed in lowest position D) Bed in lowest position, wheels locked, place bed against wall

D) Bed in lowest position, wheels locked, place bed against wall

The nurse is reviewing laboratory results on a client with acute renal failure. Which one of the following should be reported immediately? A) Blood urea nitrogen 50 mg/dl B) Hemoglobin of 10.3 mg/dl C) Venous blood pH 7.30 D) Serum potassium 6 mEq/L

D) Serum potassium 6 mEq/L

After reviewing the Braden Scale findings of residents at a long-term facility, the charge nurse should to tell the unlicensed assistive personnel (UAP) to prioritize skin care for which client? A. A poorly nourished client who requires liquid supplements B. An older adult who is unable to communicate elimination needs C. A woman with osteoporosis who is unable to bear weight D. A older man whose sheets are damp each time he is turned

D. A older man whose sheets are damp each time he is turned

The psychiatric nurse is caring for clients on an adolescent unit. Which client requires the nurse's immediate attention? A. A 14yo client with anorexia nervosa who is refusing to eat the evening snack B. A 16yo client diagnosed with major depression who refuses to participate in group C. A 17yo client diagnosed with bipolar disorder who is pacing around the lobby D. An 18yo client with antisocial behavior who is being yelled at by other clients

D. An 18yo client with antisocial behavior who is being yelled at by other clients

A 3-year-old boy was successfully toilet trained prior to his admission to the hospital for injuries sustained from a fall. His parents are very concerned that the child has regressed in his toileting behaviors. Which information should the nurse provide to the parents? A. A retraining program will need to be initiated when the child returns home. B. Diapering will be provided since hospitalization is stressful to preschoolers C. A potty chair should be brought from home so he can maintain his toileting skills D. Children usually resume their toileting behaviors when they leave the hospital

D. Children usually resume their toileting behaviors when they leave the hospital

A client who recently received a prescription for ramelteon to treat sleep deprivation reports experiencing several side effects since taking the drug. Which side effect should the nurse report to the healthcare provider? A. A change in the sleep-wake cycle B. Mild sedation C. Dizziness reported after initial dose D. Somnambulism

D. Somnambulism

After receiving report on an inpatient acute care unit, which client should the nurse assess first? A. The client with an obstruction of the large intestine who is experiencing abdominal distention B. The client who had surgery yesterday and is experiencing a paralytic ileus with absent bowel sounds C. The client with a small bowel obstruction who has a nasogastric tube that is draining greenish fluid D. The client with a bowel obstruction due to a volvulus who is experiencing abdominal rigidity

D. The client with a bowel obstruction due to a volvulus who is experiencing abdominal rigidity

A client with multiple burn injuries is being treated in the burn trauma unit just hours after the injuries occurred. The healthcare provider instructs the nurse to avoid auto contamination when performing dressing changes. Which intervention is most important for the nurse to implement? A. Dress each wound separately B. Assign equipment to this one client C. Utilize reverse isolation protocol D. Use gown, mask, and gloves with dressing changes

D. Use gown, mask, and gloves with dressing changes

An elderly female client with osteoarthritis reports increasing pain and stiffness in her right knee and asks how to reduce these symptoms. In responding to the client, the nurse recognizes what pathology as the cause of her symptoms? a. Destruction of joint cartilage.

Destruction of joint cartilage.

The nurse is caring for four clients: Client A 94% o2 saturation, Client B, hemoglobin of 8.; Client C, potassium level of 3.8 and Client D appendectomy who has a white blood cell count of 15,000. What should the nurse implement?

Determine if Client B has two units of packed cells available in the blood bank.

A client with persistent low back pain has received a prescription for electronic stimulator (TENS) unit. After the nurse applies the electrodes and turns on the power, the client reports feeling a tingling sensation. How should the nurse respond? a. Determine if the sensation feels uncomfortable. b. Decrease the strength of the electrical signals. c. Remove electrodes and observe for skin redness. d. Check the amount of gel coating on the electrodes.

Determine if the sensation feels uncomfortable. Rational: electronic stimulators, such as a transelectrical nerve stimulator (TENS) unit, have been found to be effective in reducing low back pain by "closing the gate" to pain stimuli. A tingling sensation should be felt when the power is turned on, and the nurse should assess whether the sensation is too strong, causing discomfort or muscle twitching. Decreasing the electrical signal may be indicated if the sensation is too strong. Other options are not necessary because the tingling sensation is expected.

A client arrives on the surgical floor after major abdominal surgery. What intervention should the nurse perform first? a. Administer prescribed pain medication b. Assess surgical site c. Determine the client's vital sign. d. Apply warmed blankets

Determine the client's vital sign. Rationale: The First priority must be to obtain baseline vital signs. A and B should also be accomplished soon, but not until the initial vital signs are determined. C is a nice thing to do.

A mother runs into the emergency department with s toddler in her arms and tells the nurse that her child got into some cleaning products. The child smells of chemicals on hands, face, and on the front of the child's clothes. After ensuring the airway is patent, what action should the nurse implement first? a. Call poison control emergency number. b. Determine type of chemical exposure. c. Obtain equipment for gastric lavage. d. Assess child for altered sensorium.

Determine type of chemical exposure. Rational: once the type of chemical is determined, poison control should be called even if the chemical is unknown. If lavage is recommended by poison control, intubation and nasogastric tube may be needed as directed by poison control. Altered sensorium, such as lethargy, may occur if hydrocarbons are ingested

A female client with acute respiratory distress syndrome (ARDS) is chemically paralyzed and sedated while she is on as assist-control ventilator using 50% FIO2. Which assessment finding warrants immediate intervention by the nurse? a. Premature atrial contractions (PAC) b. Hemoccult-positive nasogastric fluid c. Diminished left lower lobe sounds. d. Increasing endotracheal secretions.

Diminished left lower lobe sounds Rationale: Diminished lobe sounds indicate collapsed alveoli or tension pneumothorax, which required immediate chest tube insertion to re-inflate the lung.

While the nurse is preparing a scheduled intravenous (IV) medication, the client states that the IV site hurts and refuses to allow the nurse to administer a flush to assess the site. Which intervention should the nurse implement? a. Discontinue the painful IV after a new IV is inserted

Discontinue the painful IV after a new IV is inserted

In assessing a client 48 hours following a fracture, the nurse observes ecchymosis at the fracture site, and recognizes that hematoma formation at the bone fragment site has occurred. What action should the nurse implement? a. Document the extent of the bruising in the medical record

Document the extent of the bruising in the medical record

While completing an admission assessment for a client with unstable angina, which closed questions should the nurse ask about the client's pain? a. Does your pain occur when walking short distances?

Does your pain occur when walking short distances?

A client with chronic alcoholism is admitted with a decreased serum magnesium level. Which snack option should the nurse recommend to this client? a. Cheddar cheese and crackers. b. Carrot and celery sticks. c. Beef bologna sausage slices. d. Dry roasted almonds.

Dry roasted almonds. Rational: alcoholism promotes inadequate food intake and gastrointestinal loss of magnesium include green leafy vegetables and nuts and seeds. Other snacks listed provide much lower amounts of magnesium per serving.

A client is being discharged with a prescription for warfarin (Coumadin). What instruction should the nurse provide this client regarding diet? a. Eat approximated the same amount of leafy green vegetables daily so the amount of vitamin K consumed is consistent. b. Avoid eating all foods that contain any vitamin K because it is an antagonist of Coumadin. c. Increase the intake of dark green leafy vegetables while taking Coumadin d. Eat two servings of raw dark green leafy vegetables daily and continue for 30 days after Coumadin therapy is completed.

Eat approximated the same amount of leafy green vegetables daily so the amount of vitamin K consumed is consistent. Rationale: The Coumadin dose is prescribed and adjusted based on the client's normal consumption of foods containing vitamin K (an essential clotting factor that counteracts the effects of Coumadin), so the client should eat a consistent amount of vitamin K food sources (A). (B and C) alter the effectiveness of the already set dosage of Coumadin. (D) provides a consistent amount of vitamin K but does not take into consideration how much is already being consumed by the client.

The nurse note a visible prolapse of the umbilical cord after a client experiences spontaneous rupture of the membranes during labor. What intervention should the nurse implement immediately? a. Administer oxygen by face mask at 6L/mint b. Transport the client for a cesarean delivery c. Elevate the presenting part off the cord. d. Place the client to a knee-chest position.

Elevate the presenting part off the cord Rationale: The nurse should immediately elevate the presenting part off the cord because when the cord prolapses, the presenting part applies pressure to the cord, especially during each contraction, and reduces perfusion to the fetus. A can be delayed until pressure is removed from the cord. B and D are important but do not have priority.

While a child is hospitalized with acute glomerulonephritis, the parents ask why blood pressure readings are taken so often. Which response by the nurse is most accurate? a. Elevated blood pressure must be anticipated and identified quickly

Elevated blood pressure must be anticipated and identified quickly

A male client's laboratory results include a platelet count of 105,000/ mm3 Based on this finding the nurse should include which action in the client's plan of care? a. Cluster care to conserve energy b. Initiate contact isolation c. Encourage him to use an electric razor d. Asses him for adventitious lung sounds

Encourage him to use an electric razor Rationale: This client is at risk for bleeding based on his platelet count (normal 150,000 to 400,000/ mm3). Safe practices, such as using an electric razor for shaving, should be encouraged to reduce the risk of bleeding.

A client is being treated for syndrome of inappropriate antidiuretic hormone (SIADH). On examination, the client has a weight gain of 4.4 lbs (2 kg) in 24 hours and an elev ated blood pressure. Which intervention should the nurse implement first? a. Ensure client takes a diuretic q AM

Ensure client takes a diuretic q AM

In assessing a client twelve hours following transurethral resection of the prostate (TURP), the nurse observes that the urinary drainage tubing contains a large amount of clear pale pink urine and the continuous bladder irrigation is infusing slowly. What action should the nurse implement? a. Increase the rate of the continuous bladder irrigation b. Manually irrigate the catheter with sterile normal saline c. Clamp the catheter above the drainage. d. Ensure that no dependent loops are present in the tubing.

Ensure that no dependent loops are present in the tubing. Rationale: The nurse should ensure that the tubing is not kinked, and adequate flow is maintained to prevent bladder distention. Clear pale pink urine is desirable following TURP and indicates the absence of clots or excessive hemorrhage. A is implemented if the flow is dark red to prevent clot formation, and B if clots is present, to prevent obstruction. C is not a useful action in this situation and causes bladder distention while the bladder irrigation is still infusing

A nurse is planning to teach infant care and preventive measures for sudden infant death syndrome (SIDS) to a group of new parents. What information is most important for the nurse to include? a. Swaddle the infant in a blanket for sleeping b. Place the infant in a prone position whenever possible c. Prop that the infant's crib matter is firm d. Ensure that the infant's crib mattress is firm.

Ensure that the infant's crib mattress is firm Rationale: Sudden infant death syndrome is the unexplained death of infants under the age of one year. Parents should be educated about the methods to reduce the risk of SIDS, which include use of a firm crib mattress, maternal smoking cessation before and after pregnancy, avoidance of pillows in the crib, and placing the infant in the supine position. (Back to Sleep Campaign)

Which type of Leukocyte is involved with allergic responses and the destruction of parasitic worms? a. Neutrophils b. Lymphocytes c. Eosinophils d. Monocytes

Eosinophils Rationale: Eosinophils are involved in allergic responses and destruction of parasitic worms

A client admitted to the psychiatric unit diagnosed with major depression wants to sleep during the day, refuses to take a bath, and refuses to eat. Which nursing intervention should the nurse implement first? a. Assess the client's ability to communicate with the other staff members b. Arrange a meeting with the family to discuss the client's situation c. Administer the client's antidepressant medication as prescribed. d. Establish a structured routine for the client to follow.

Establish a structured routine for the client to follow

The nurse enters a client's room to administer scheduled daily medications and observes the client leaning forward and using pursed lip breathing. Which action is most important for the nurse to implement first? a. Evaluate the oxygen saturation

Evaluate the oxygen saturation

The nurse is assessing a primigravida a 39-weeks gestation during a weekly prenatal visit. Which finding is most important for the nurse to report to the healthcare provider? a. Fetal heart rate of 200 beats/minute

Fetal heart rate of 200 beats/minute

The nurse discovers that an elderly client with no history of cardiac or renal disease has an elevated serum magnesium level. To further investigate the cause of this electrolyte imbalance, what information is most important for the nurse to obtain from the client's medical history? a. Frequency of laxative use for chronic constipation

Frequency of laxative use for chronic constipation

A client's telemetry monitor indicates ventricular fibrillation (VF). After delivering one counter shock, the nurse resumes chest compression, after another minute of compression , the client's rhythm converts to supraventricular tachycardia (SVT) on the monitor, at this point , what is the priority intervention for the nurse? a. Give IV dose of adenosine rapidly over 1-2 seconds.

Give IV dose of adenosine rapidly over 1-2 seconds.

Psych/Alcohol/Tylenol overdose antidote

Give Mucomyst

An elderly male client is admitted to the urology unit with acute renal failure due to a postrenal obstruction. Which questions best assists the nurse in obtaining relevant historical data? a. Have you had any difficulty in starting your urinary stream"

Have you had any difficulty in starting your urinary stream"

A female client reports that her hair is becoming coarse and breaking off, that the outer part of her eyebrows have disappeared, and that her eyes are all puffy. Which follow-up question is best for the nurse to ask? a. "Is there a history of female baldness in your family?" b. "Are you under any unusual stress at home or work?" c. "Do you work with hazardous chemicals?" d. "Have you noticed any changes in your fingernails?"

Have you noticed any changes in your fingernails? Rationale: The pattern of reported manifestations is suggestive of hypothyroidism

A client with arthritis has been receiving treatment with naproxen and now reports ongoing stomach pain, increasing weakness, and fatigue. Which laboratory test should the nurse monitor? a. Sed rate (ESR) b. Hemoglobin c. Calcium d. Osmolality.

Hemoglobin Rational: naproxen can cause gastric bleeding, so the nurse should monitor the client's hemoglobin to assess for possible bleeding. Other options are not likely to be affected by the used of naproxen and are not related to the client's current symptoms.

The nurse is auscultating a client's lung sounds. Which description should the nurse use to document this sound? a. High pitched or fine crackles. b. Rhonchi c. High pitched wheeze d. Stridor

High pitched or fine crackles.

The community health nurse is attempting to address the issue of child abuse in a large metropolitan area. A primary prevention program for child abuse might include which program?

High school child development and parenting classes.

Crohn's Disease - Which has lowest Na+- choose

Horseradish (choices are ketchup, steak sauce, pickle relish, horseradish)

A group of nurse-managers is asked to engage in a needs assessment for a piece of equipment that will be expensed to the organization's budget. Which question is most important to consider when analyzing the cost-benefit for this piece of equipment? a. How many departments can use this equipment? b. Will the equipment require annual repair? c. Is the cost of the equipment reasonable? d. Can the equipment be updated each year?

How many departments can use this equipment?

A client with Addison's disease becomes weak, confused, and dehydrated following the onset of an acute viral infection. The client's laboratory values include; sodium 129 mEq/l (129mmol/l SI), glucose 54 mg/dl (2.97mmol/l SI) and potassium 5.3 mmol/l SI). When reporting the findings to the HCP, the nurse anticipates a prescription for which intravenous medications? a. Hydrocortisone Rationale: Hydrocortisone tablets work as a hormone replacement for a natural hormone called cortisol. You may take hydrocortisone tablets if your body does not make enough cortisol - for example if you have Addison's disease or if you've had your adrenal glands taken out.

Hydrocortisone

An adult client is exhibit the manic stage of bipolar disorder is admitted to the psychiatric unit. The client has lost 10 pounds in the last two weeks and has no bathed in a week "I'm trying to start a new business and "I'm too busy to eat". The client is oriented to time, place, person but not situation. Which nursing problem has the greatest priority? a. Hygiene-self-care deficit b. Imbalance nutrition c. Disturbed sleep pattern d. Self-neglect

Imbalance nutrition Rationale: The client's nutritional status has the highest priority at this time, and finger foods are often provided, so the client who is on the maniac phase of bipolar disease can receive adequate nutrition. Other options are nursing problems that should also be addresses with the client's plan of care, but at this stage in the client's treatment, adequate nutrition is a priority

What action should the school nurse implement to provide secondary prevention to a school-age children? a. Collaborate with a science teacher to prepare a health lesson b. Prepare a presentation on how to prevent the spread of lice c. Initiate a hearing and vision screening program for first-graders d. Observe a person with type 1 diabetes self-administer a dose of insulin

Initiate a hearing and vision screening program for first-grader

A client with pneumonia has arterial blood gases levels at: PH 7.33; PaCO2 49 mm/hg; HCO3 25 mEq/L; PaO2 95. What intervention should the nurse implement based on these results? a. Institute coughing and deep breathing protocols

Institute coughing and deep breathing protocols

The nurse observes an unlicensed assistive personnel (UAP) positioning a newly admitted client who has a seizure disorder. The client is supine and the UAP is placing soft pillows along the side rails. What action should the nurse implement? a. Ensure that the UAP has placed the pillows effectively to protect the client. b. Instruct the UAP to obtain soft blankets to secure to the side rails instead of pillows. c. Assume responsibility for placing the pillows while the UAP completes another task. d. Ask the UAP to use some of the pillows to prop the client in a side lying position.

Instruct the UAP to obtain soft blankets to secure to the side rails instead of pillows Rationale: The nurse should instruct the UAP to pad the side rails with soft blankest because the use of pillows could result in suffocation and would need to be removed at the onset of the seizure. The nurse can delegate paddling the side rails to the UAP

The mother of a 7-month-old brings the infant to the clinic because the skin in the diaper area is excoriated and red, but there are no blisters or bleeding. The mother reports no evidence of watery stools. Which nursing intervention should the nurse implement? a. Instruct the mother to change the child's diaper more often. b. Encourage the mother to apply lotion with each diaper charge c. Tell the mother to cleanse with soap and water at each diaper change d. Ask the mother to decrease the infant's intake of fruits for 24 hours.

Instruct the mother to change the child's diaper more often. Rationale: Changing the diaper more often helps to decrease the amount of time the skin comes in contact with wet soiled diapers and helps heal the irritation.

A client diagnosed with major depression is being allowed a weekend pass from the psychiatric unit. Which instruction should the nurse provide to the client's family?

Involve the client in usual at home activities

The client with which type of wound is most likely to need immediate intervention by the nurse? a. Laceration b. Abrasion c. Contusion d. Ulceration

Laceration Rationale: A laceration is a wound that is produced by the tearing of soft body tissue. This type of wound is often irregular and jagged. A laceration wound is often contaminated with bacteria and debris from whatever object caused the cut

When entering a client's room to administer an 0900 IV antibiotic, the nurse finds that the client is engaged in sexual activity with a visitor. Which actions should the nurse implement? a. Leave the room and close the door quietly

Leave the room and close the door quietly

An adult male client is admitted to the emergency room following an automobile collision in which he sustained a head injury. What assessment data would provide the earliest that the client is experiencing increased intracranial pressure (ICP)? a. Lethargy b. Decorticate posturing c. Fixed dilated pupil d. Clear drainage from the ear.

Lethargy Rationale: Lethargy is the earliest sign of ICP along with slowing of speech and response to verbal commands. The most important indicator of increase ICP is the client's level or responsiveness or consciousness. B and C are very late signs of ICP.

A client is receiving lactulose (Portalac) for signs of hepatic encephalopathy. To evaluate the client's therapeutic response to this medication, which assessment should the nurse obtain? a. Level of consciousness b. Percussion of abdomen c. Serum electrolytes d. Blood glucose.

Level of consciousness Rationale: Colonic bacteria digest lactulose to create a drug-induces acidic and hyperosmotic environment that draws water and blood ammonia into the colon and coverts ammonia to ammonium, which is trapped in the intestines and cannot be reabsorbed into the systemic circulation. This therapeutic action of lactulose is to reduce serum ammonia levels, which improves the client's level of consciousness and mental status.

After placing a stethoscope as seen in the picture, the nurse auscultates S1 and S2 heart sounds. To determine if an S3 heart sound is present, what action should the nurse take first a. Side the stethoscope across the sternum. b. Move the stethoscope to the mitral site c. Listen with the bell at the same location d. Observe the cardiac telemetry monitor

Listen with the bell at the same location Rationale: The nurse uses the bell of the stethoscope to hear low-pitched sounds such as S3 and S4. The nurse listens at the same site using the diaphragm the diaphragm and bell before moving systematically to the next sites.

The nurse is developing a teaching program for the community. What population characteristic is most influential when choosing strategies for implementing a teaching plan? a. Literacy level b. Prevalent learning style c. Median age d. Percent with internet access.

Literacy level Rationale: Reading ability, or literacy level is the most important population characteristic in choosing strategies for implementing teaching plan. If the population cannot read it would be useless to reinforce teaching with written material.

A client is admitted for cellulitis surrounding an insect bite on the lower, right arm and intravenous (IV) antibiotic therapy is prescribed. Which action should the nurse implement before performing venipuncture? a. Lower the left arm below the level of the heart

Lower the left arm below the level of the heart

An adult male with schizophrenia who has been noncompliant in taking oral antipsychotic medications refuses a prescribed IM medication. Which action should the nurse take? a. Notify the healthcare provider of the client's refusal

Notify the healthcare provider of the client's refusal

Four hours after surgery, a client reports nausea and begins to vomit. The nurse notes that the client has a scopolamine transdermal patch applied behind the ear. What action should the nurse take? a. Reposition the transdermal patch to the client's trunk. b. Remove the transdermal patch until the vomiting subsides. c. Notify the healthcare provider of the vomiting. d. Explain that this is a side effect of the medication in the patch.

Notify the healthcare provider of the vomiting. Rational: transdermal scopolamine is used to prevent nausea and vomiting from anesthesia and surgery. The nurse should notify the healthcare provider if the medication is ineffective. The patch should be applied behind the ear and should remain in place to reduce the nausea and vomiting. Nausea and vomiting are no side effects of the medication.

. The nurse is obtaining a medication history for a client with a new prescription for paroxetine (Paxil). The client reports current use of the MAO inhibitor isocarboxazid (Marplan). What intervention is most important for the nurse to implement?

Notify the healthcare provider that the client is currently taking Marplan.

A client with myasthenia Gravis (MG) is receiving immunosuppressive therapy. Review recent laboratory test results show that the client's serum magnesium level has decreased below the normal range. In addition to contacting the healthcare provider, what nursing action is most important? a. Check the visual difficulties b. Note most recent hemoglobin level c. Assessed for he and Hand joint pain d. Observe rhythm on telemetry monitor

Observe rhythm on telemetry monitor Rationale: If not treated a low little Serum magnesium level can affect myocardial depolarization leading to a lethal arrhythmia, and the nurse should assess for dysrhythmias before contacting the healthcare provider. Other choices are common in MG but do not contribute the Safety risk of low magnesium levels.

The nurse applies a blood pressure cuff around a client's left thigh. To measure the client's blood pressure, where should the diaphragm of the stethoscope be placed? (Mark the location on one of the images.) a. On left thigh with arrow pointing to inner thigh"

On left thigh with arrow pointing to inner thigh"

THIS IS WRONG!!! LOL!!! A-12-year old boy has a body mass index (BMI) of 28, a systolic pressure and a glycosylated hemoglobin (HBA1C) of 7.8%. Which selection indicated that his mother understands the management of his diet? a. One whole-wheat bagel with cream cheese, two strips of bacon, six ounces of orange juice.

One whole-wheat bagel with cream cheese, two strips of bacon, six ounces of orange juice.

The charge nurse observes the practical nurse (PN) apply sterile gloves in preparation for performing a sterile dressing change. Which action by the PN requires correction by the charge nurse? a- Opening the package b- Picking up the second glove c- Picking up the first glove d- Positioning of the table

Picking up the second glove

The nurse is demonstrating correct transfer procedures to the unlicensed assisted personnel (UAP) working on a rehabilitation unit. The UAPs ask the nurse how to safely move a physically disabled client from the wheelchair to a bed. What action should the nurse recommended? a. Hold the client at arm's length while transferring to better distribute the body weight. b. Apply the gait belt around the client's waits once standing position has been assumed. c. Place a client's locked wheelchair on the client's strong side next to the bed. d. Pull the client into position by reaching from the opposite side of the bed.

Place a client's locked wheelchair on the client's strong side next to the bed. RATIONALE: Placing the wheelchair on the client's strong side offers the greatest stability for the transfer. Holding the client arm's length or pulling from the opposite site of the bed reflect poor body mechanism. Using a gait belt offers additional safety for the client but should be done after the wheelchair has be put into the proper place and the wheels have been locked and before the client has assumed a standing position.

The nurse is planning to assess a client's oxygen saturation to determine if additional oxygen is needed via nasal cannula. The client needs a bilateral below-the-knee amputation and pedal pulses that are weak and thready. What action should the nurse take? a. Document that an accurate oxygen saturation reading cannot be obtained b. Elevate to client's hands for five minutes prior to obtaining a reading from the finger c. Increase the oxygen based on the clients breathing patterns and lung sounds d. Place the oximeter clip on the ear lobe to obtain the oxygen saturation reading

Place the oximeter clip on the ear lobe to obtain the oxygen saturation reading Rationale: Pulse oximeter clips can be attached to the earlobe to obtain an accurate measurement of oxygen saturation. Other options will not provide the needed assessment.

The nurse is assessing and elderly bedridden client. Which finding indicates that the turning and positioning schedule is effective in protecting the client's skin? a. Reddened skin areas disappear within 15 minutes of being turned and positioned.

Reddened skin areas disappear within 15 minutes of being turned and positioned.

During discharge teaching, the nurse discusses the parameters for weight monitoring with a client who was recently diagnosed with heart failure (HF). Which information is most important for the client to acknowledge? a. Keep a daily weight record b. Obtain weight at the same time every day c. Limit intake of dietary salt. d. Report weight gain of 2 pounds (0.9kg) in 24 hours

Report weight gain of 2 pounds (0.9kg) in 24 hours

The fire alarm goes off while the charge nurse is receiving the shift report. What action should the charge nurse implement first? a. Instruct the client's family member to stay in the visitor waiting area until further notice b. Tell the staff to keep all clients and visitors in the client rooms with the doors closed. c. Direct the nursing staff to evacuate the clients using the stairs in a calm and orderly manner. d. Call the hospital operator to determine if the is indeed a real emergency or a fire drill.

Tell the staff to keep all clients and visitors in the client rooms with the doors closed Rationale: The charge nurse should treat the alarm as an actual fire emergency and instruct all clients and visitors to stay in the clients' room with doors closed until otherwise notified. A should be anxiety producing. Visitors should remain in the rooms with the clients. C is only necessary if the location and severity of the fire make the unit unsafe for inhabitants and would only be implemented after other measures to control de fire had failed. D should not be done until after measures are taken to protect clients and visitors.

For the past 24 hours, an antidiarrheal agent, diphenoxylate, has been administered to a bedridden, older client with infectious gastroenteritis. Which finding requires the nurse to take further action? a. Tented skin turgor

Tented skin turgor

For the past 24 hours, an antidiarrheal agent, diphenoxylate, has been administered to a bedridden, older client with infectious gastroenteritis. Which finding requires the nurse to take further action? a. Loss of appetite b. Serum K 4.0 mEq/or mmol/dl (SI) c. Loose, runny stool d. Tented skin turgor.

Tented skin turgor. Rationale: D indicate dehydration, a serious complication following prolonged diarrhea that requires further interventions by the nurse.

The charge nurse is developing the nursing guidelines for a coronary care unit. Which references is likely to be the most useful in developing these guidelines?

The Scope of Standards of Practice from the American Nurses Association

During the first trimester of pregnancy, a client who was treated for genital herpes with acyclovir (Zovirax) prior to this pregnancy tells the nurse that she is experiencing an episode of genital herpes. Which nursing intervention has the highest priority?

The highest priority in this situation is to determine if the client has taken the drug Zoviarax, because Acyclovir has been associated with birth defects.

A client with hyperthyroidism is admitted to the postoperative after subtotal thyroidectomy. Which of the client's serum laboratory values requires intervention by the nurse? a. Total calcium 5.0 mg/dl

Total calcium 5.0 mg/dl

The nurse is teaching a client about the antiulcer medications ranitidine which was... statement best describes the action of this drug? a- It blocks the effects of histamine, causing decreased secretion of acid b- Ranitidine will neutralize gastric acid and decrease gastric pH c- This drug provides a protective coating over the gastric mucosa d- It effectively blocks 97% of the gastric acid secreted in the stomach

a. It blocks the effects of histamine, causing decreased secretion of acid

A toddler with a history of an acyanotic heart defect is admitted to the pediatric intensive...rate of 60 breaths/ minute, and a heart rate of 150 beats/minute. What action should the nurse take? a. Obtain a pulse oximeter reading b. Assess the child blood pressure c. Perform a neurological assessment d. Initiate peripheral intravenous access.

a. Obtain a pulse oximeter reading

The nurse determines that a client's pupils constricts as they change focus from a far object. What documentation should the nurse enter about this finding? a. Pupils reactive to accommodation b. Nystagmus present with pupillary focus. c. Peripheral vision intact d. Consensual pupillary constriction present

a. Pupils reactive to accommodation

In conducting a health assessment, the nurse determines that both parents of a child with asthma smoke cigarettes. What recommendation is best to the nurse to recommend to the parents? a- avoid smoking in the house b- stop smoking immediately c- decrease the number of cigarettes smoke daily d- obtain nicotine patches to assist in smoking sensation

a. avoid smoking in the house

After removing a left femoral arterial sheath, which assessment findings warrant immediate interventions by the nurse? (Select all that applied.) a- Tenderness over insertion b- Unrelieved back and flank pain. c- Cool and pale left leg and foot. d- Left groin egg-size hematoma. e- Quarter size red drainage at site.

b- Unrelieved back and flank pain. c- Cool and pale left leg and foot. d- Left groin egg-size hematoma.

A male client is admitted with burns to his face and neck. Which position should the nurse place the client to prevent contract? a. Flexed with the chin toward the chest. b. Hyperextended with neck supported by a rolled towel. c. Side-lying with the head on a pillow d. Prone with face supported by an inflated rubber ring.

b. Hyperextended with neck supported by a rolled towel.

The nurse is reviewing a client's electrocardiogram and determines the PR interval (PRI) is prolonged. What does this finding indicate? a- Initiation of the impulses from a location outside the SA node b- Inability of the SA node to initiate an impulse at the normal rate c- Increased conduction time from the SA node to the AV junction d- Interference with the conduction through one or both ventricles

b. Inability of the SA node to initiate an impulse at the normal rate Rationale: A prolonged PRI reflects an increased amount of time for an impulse to travel from the SA node through the AV node and is characteristic of a first-degree heart block.

During the intraoperative phase of care, the circulating nurse observes that the client is not adequately client's privacy. What is the best initial nursing action for the nurse to implement? a- Document the observation in the client's medical record b- Instruct the scrub nurse to re-drape the client c- Ensure that the client in unaware of the surrounding. d- Consult with operating room manager.

b. Instruct the scrub nurse to re-drape the client

A nurse is planning discharge care for a male client with metastatic cancer. The client tells the nurse that he plans to return to work despite pain, fatigue, and impending death. Which goals is most important to include in this client's plan of care? a. Implements decisions about future hospices services within the next 3 months. b. Maintaining pain level below 4 when implementing outpatient pain clinic strategies. c. Request home health care if independence become compromised for 5 days. d. Arranges for short term counseling stressors impact work schedule for 2 weeks.

b. Maintaining pain level below 4 when implementing outpatient pain clinic strategies. Rationale: An outpatient pain clinic provides the interdisciplinary services needed to manage chronic pain. Also, the client has a terminal disease and is being discharge home, hospice and health care are not indicating currently. Short term counseling is not an option.

A client with type 2 diabetes mellitus is admitted for antibiotic treatment for a leg ulcer. To monitor the client for the onset of hyperosmolar hyperglycemic nonketotic syndrome (HHNS), what actions should the nurse take? (Select all that apply) a. Check urine for ketones b. Measure blood glucose c. Monitor vital signs d. Assessed level of consciousness e. Obtain culture of wound

b. Measure blood glucose c. Monitor vital signs d. Assessed level of consciousness Rationale: Blood glucose greater than 600 mg/dl (33.3 mmol/L SI), vital sign changes in mental awareness are indicators of possible HHNS. Urine ketones are monitored in diabetic ketoacidosis. Wound culture is performed prior to treating the wound infection but is not useful in monitoring for HHNS.

A client with superficial burns to the face, neck, and hands resulting from a house fire... which assessment finding indicates to the nurse that the client should be monitored for carbon monoxide...? a. Expiratory stridor and nasal flaring b. Mucous membranes cherry red color c. Carbonaceous particles in sputum d. Pulse oximetry reading of 80 percent

b. Mucous membranes cherry red color

A client who developed syndrome of inappropriate antidiuretic hormone (SIADH) associated with small carcinoma of the lung is preparing for discharge. When teaching the client about self-management with demeclocycline (Declomycin), the nurse should instruct the client to report which condition to the health care provider? a- Insomnia b- Muscle cramping c- Increase appetite d- Anxiety.

b. Muscle cramping Rationale: SIADH causes dilution hyponatremia because of the increased release of ADH, which is treated with water restriction and demeclocycline, a tetracycline derivate that blocks the action of ADH. Signs of hyponatremia (normal 136-145), which indicate the need for increasing the dosage of demeclocycline, should be reported to the healthcare provider. The signs include: plasma sodium level less than 120, anorexia, nausea, weight changes related to fluid disturbance, headache, weakness, fatigue, and muscle cramping. AC& D are not related to hyponatremia.

The nurse is caring for several clients on a telemetry unit. Which client should the nurse assess first? The client who is demonstrating? a. A paced rhythm with 100% capture after pacemaker replacement b. Normal sinus rhythm and complaining of chest pain c. Atrial fibrillation with congestive heart failure and complaining of fatigue d. Sinus tachycardia 3 days after a myocardial infarction

b. Normal sinus rhythm and complaining of chest pain

An older male adult resident of long-term care facility is hospitalized for a cardiac catheterization that occurred yesterday. Since the procedure was conducted, the client has become increasingly disoriented. The night shift nurse reports that he attempted to remove the sandbag from his femoral artery multiple times during the night. What actions should the nurse take? (Select all that apply.) a. Recommend a 24-hour caregiver on discharge to the long-term facility. b. Notify the healthcare provider of the client's change in mental status. c. Include q2 hour's reorientation in the client's plan of care. d. Request immediate evaluation by Rapid Response Team e. Apply soft wrist restraints so that the operative site is protected.

b. Notify the healthcare provider of the client's change in mental status. c. Include q2 hour's reorientation in the client's plan of care. Rationale: The client's condition reflects mental changes that could be related to post procedure stress, sundowner's syndrome, or cerebral complications, the nurses should inform the healthcare provider of the client's change in mental status for the client's safety, q2 hour orientation evaluations and reorientation should be included in the plan of care.

A client is admitted for type 2 diabetes mellitus (DM) and chronic Kidney disease (CKD)... which breakfast selection by the client indicates effective learning? a. Scrambled eggs, bacon, one slice of whole wheat toast with butter and jam. b. Oatmeal with butter, artificial sweetener, and strawberries, and 6 ounces' coffee. c. Banana pancake with maple syrup, sausage links, half grapefruit, and low -fat milk d. Orange juice, yogurt with berries, cold cereal with milk, bran muffin with margarine.

b. Oatmeal with butter, artificial sweetener, and strawberries, and 6 ounces coffee

An older male who is admitted for end stage of chronic obstructive pulmonary disease (COPD) tells the nurse .... The client provides the nurse with a living will and DNR. What action should the nurse implement? a- Inform the family of the client's wishes b- Obtain a prescription for DNR c- Clergy consultation d- Ask the patient why he made this choice

b. Obtain a prescription for DNR

The nurse delegates to an unlicensed assistive personnel (UAP) denture care for a client with...daily leaving. When making this assignment, which instruction is most important for the nurse to do? a- Do not remove the dentures, but instead brush them within the mouth b- Place a washcloth in the sink while cleaning the dentures. c- Use tepid, not hot, water to clean the dentures d- Avoid damaging the dentures using a soft-bristled toothbrush.

b. Place a washcloth in the sink while cleaning the dentures

The nurse is reinforcing home care instructions with a client who is being discharged following...prostate (TURP). Which intervention is most important for the nurse to include in the client... a. Avoid strenuous activity for 6 weeks b. Report fresh blood in the urine. c. Take acetaminophen for fever 101 d. Consume 6 to 8 glasses of water daily.

b. Report fresh blood in the urine

A client who had a gestational trophoblastic disease (GTD) evacuated 2 days ago is being...18 months-old child and lives in a rural area. Her husband takes the family car to work daily...transportation during the day. What intervention is most important for the nurse to implement? a. Teach a client amount the use of a home pregnancy test. b. Schedule a weekly home visit to draw hCG values. c. Make a 5 week follow- up with healthcare provider d. Begin chemotherapy administration during the first home visit

b. Schedule a weekly home visit to draw hCG values Rationale: To monitor for development of choriocarcinoma, a complication TD, level of hCG should be monitor for negative results.

A nurse stops at the site of a motorcycle accident and finds a young adult male lying face down in the road in a puddle of water. It is raining, no one is available to send for help, and the cell phone is in the car about 50 feet away. What action should the nurse take first? a. Examine the victim's body surfaces for arterial bleeding b. Stabilize the victim's neck and roll over to evaluate his status c. Return to the car to call emergency response 911 for help d. Open the airway and initiate resuscitative measures

b. Stabilize the victim's neck and roll over to evaluate his status

The nurse is conducting the initial assessment of an ill client who is from another culture.... What response should the nurse provide? a- Can you read the written instructions is English? b- "What practices do you believe will help you heal?" c- What prescriptions must be strictly followed to get well. d- You must believe that the medications will help you.

b. What practices do you believe will help you heal?"

A toddler presents to the clinic with a barking cough, stridor, refractions with respiration, the child's skin is pink with capillary refill of 2 seconds. Which intervention should the nurse implement? a. Encourage the child to cough b. Obtain a throat specimen for culture c. Administer nebulized epinephrine d. Collect blood for arterial blood gasses

c. Administered Nebulized Epinephrine

When should intimate partner violence (IPV) screening occur? a. As soon as the clinician suspects a problem b. Only when a client presents with an unexplained injury c. As a routine part of each healthcare encounter d. Once the clinician confirms a history of abuse

c. As a routine part of each healthcare encounter

A male client has received a prescription for orlistat for weight and nutrition management. In addition to the medication, the client states he plans to take a multivitamin. What teaching should the nurse provide? a. As a nutritional supplement, orlistat already contains all the recommended daily vitamins and minerals. b. Multivitamins are contraindicated. During treatment with weight-control medications such as orlistat c. Be sure to take the multivitamin and the medication at least two hours apart for best absorption and effectiveness. d. Following a well-balanced diet is a much healthier approach to good nutrition than depending on a multivitamin.

c. Be sure to take the multivitamin and the medication at least two hours apart for best absorption and effectiveness

A 7-year-old boy is brought to the clinic because of facial edema. He reports that he has been voiding small amounts of dark, cloudy, tea-colored urine. The parents state that their son had a sore throat 2 weeks earlier, but it has resolved. After assessing the child's vital signs and weight, what intervention should the nurse implement next? a. Perform an otoscopic examination b. Measure the child's abdominal girth c. Collect a urine specimen for routine urinalysis d. Obtain a blood specimen for serum electrolytes.

c. Collect a urine specimen for routine urinalysis Rationale: Acute glomerulonephritis is an auto-immune reaction to a precursory streptococcus. Manifestation of AGN include oliguria, edema, hypertension.

An adult female client is admitted to the psychiatric unit because of a complex handwashing ritual she performs daily that takes two hours or longer to complete. She worries about staying clean and refuses to sit on any of the chairs in the day area. This client's handwashing is an example of which clinical behavior? a. Addiction b. Phobia c. Compulsion d. Obsession

c. Compulsion

A preeclamptic client who delivered 24h ago remains in the labor and delivery recovery room. She continues to receive magnesium sulfate at 2 grams per hour. Her total input is limited to 125 ml per hour, and her urinary output for the last hour was 850 ml. What intervention should the nurse implement? a. discontinue the magnesium sulfate immediately b. Decrease the client's iv rate to 50 ml per hour c. Continue with the plan of care for this client d. Change the client's to NPO status

c. Continue with the plan of care for this client Rationale: continue with the plan. Diuresis in 24 to 48h after birth is a sign of improvement in the preeclamptic client. As relaxation of arteriolar spasms occurs, kidney perfusion increases. With improvement perfusion, fluid is drawn into the intravascular bed from the interstitial tissue and then cleared by the kidneys

The nurse is caring for a client with an NG tube. Which task can the nurse delegate to the UAP? a- Replace the NG tube as prescribed by the healthcare provider b- Secure the NG tube if it slides out of the client's nasal passage c- Disconnect the NG suction so the client can ambulate in the hallway. d- Reconnect the NG suction when the client returns form ambulating.

c. Disconnect the NG suction so the client can ambulate in the hallway

The nurse administers an oral antiviral to a client with shingles. Which finding is most important for the nurse report to the health care provider? a. Decreased white blood cell count b. Pruritus and muscle aches c. Elevated liver function tests d. Vomiting and diarrhea

c. Elevated liver function tests Rationale: Valacyclovir is an antiviral agent of acyclovir which is used in therapy of herpes simplex and varicella-zoster virus infections (shingles). Valacyclovir has been associated with rare instances mild, clinically apparent liver injury.

Pregnancy Induced Hypertension

high blood pressure, blurred vision, headache, Proteinuria, Abdominal pain

A client has been tentatively diagnosed with Graves' disease (hyperthyroidism). Which of these findings noted on the initial nursing assessment requires quick intervention by the nurse? A) A report of 10 pounds weight loss in the last month B) A comment by the client "I just can't sit still." C) The appearance of eyeballs that appear to "pop" out of the client's eye sockets D) A report of the sudden onset of irritability in the past 2 weeks

is C: The appearance of eyeballs that appear to "pop" out of the client''s eye sockets 7

The nurse needs to add a medication to a liter of 5% Dextrose in Water (D5W) that is already infusing into a client. At what location should the nurse inject the medication? a. medication port

medication port

Electrical burn pt

put him in a cardiac telemetry monitor

UAP using a hand sanitzers that is alcohol for 2 minutes

tell that hand sanitizer use is less > 2min

The nurse is developing the plan of care for a client with pneumonia and includes the nursing diagnosis of "Ineffective airway clearance related to thick pulmonary secretions." Which intervention is most important for the nurse to include in the client's plan of care? a. Increase fluid intake to 3,000 ml/daily b. Administer O2 at 5L/mint per nasal cannula c. Maintain the client in a semi Fowler's position d. Provide frequent rest period.

Increase fluid intake to 3,000 ml/daily Rationale: The plan of care should include an increase in fluid intake (A) to liquefy and thin secretions for easier removal of thick pulmonary secretion which facilitates airway clearance. (B) should be implemented for signs of hypoxia (C) implemented to facilitate lung expansion, and (D) implemented for activity intolerance, but these interventions do not have the priority of (A)

An adult client comes to the clinic and reports his concern over a lump that "just popped up on my neck about a week ago." In performing an examination of the lump, the nurse palpates a large, non-tender, hardened left subclavian lymph node. There is no overlying tissue inflammation. What do these finding suggest? a. Malignancy

Malignancy

The nurse would expect the cystic fibrosis client to receive supplemental pancreatic enzymes along with a diet A) High in carbohydrates and proteins B) Low in carbohydrates and proteins C) High in carbohydrates, low in proteins D) Low in carbohydrates, high in proteins

A) High in carbohydrates and proteins

In order to enhance a client's response to medication for chest pain from acute angina, the nurse should emphasize A) Learning relaxation techniques B) Limiting alcohol use C) Eating smaller meals D) Avoiding passive smoke

A) Learning relaxation techniques

A client is admitted with low T3 and T4 levels and an elevated TSH level. On initial assessment, the nurse would anticipate which of the following assessment findings? A) Lethargy B) Heat intolerance C) Diarrhea D) Skin eruptions

A) Lethargy

A client who gave birth 48 hours ago has decided to bottle feed the infant. During the assessment, the nurse observes that both breasts are swollen, warm, and tender on palpation. Which instruction should the nurse provide? A. Apply ice to the breasts for comfort B. Wear a loose-fitting bra during the day to prevent nipple irritation C. Run warm water over breasts D. Express small amounts of milk from the breasts to relieve pressure

A. Apply ice to the breasts for comfort

A client with a lower respiratory tract infection receives a prescription for ciprofloxacin 500mg PO every 12 hours. When the client requests an afternoon snack, which dietary choice should the nurse provide? A. Cinnamon applesauce B. Vanilla-flavored yogurt C. Calcium-fortified juice D. Low-fat chocolate milk

A. Cinnamon applesauce

A mother brings her 6-year-old child, who has just stepped on a rusty nail, to the pediatrician's office. Upon inspection, the nurse notes that the nail went through the shoe and pierced the bottom of the child's foot. Which action should the nurse implement first? A. Cleanse the foot with soap and water and apply an antibiotic ointment B. Provide teaching about the need for a tetanus booster within the next 72 hours. C. have the mother check the child's temperature q4h for the next 24 hours D. transfer the child to the emergency department to receive a gamma globulin injection

A. Cleanse the foot with soap and water and apply an antibiotic ointment

A male client with cirrhosis has jaundice and pruritus. He tells the nurse that he has been soaking in hot baths at night with no relief of his discomfort. Which action should the nurse take? A. Encourage the client to use cooler water and apply calamine lotion after soaking B. Obtain a PRN prescription for an analgesic that the client can use for symptom relief C. Suggest that the client take brief showers and apply oil-based lotion after showering D. Explain that the symptoms are caused by liver damage and cannot be relieved

A. Encourage the client to use cooler water and apply calamine lotion after soaking

When taking a health history, which information collected by the nurse correlates most directly to a diagnosis of chronic peripheral arterial insufficiency? A. History of intermittent claudication B. A positive Brodie-Trendelenburg test C. Ankle ulceration and edema D. A serum cholesterol level of 250mg/dl (6.47mmol/L)

A. History of intermittent claudication

The nurse is caring for a client with the sexually transmitted infection (STI) chlamydia. The client reports having sex with someone who had many partners. Which response should the nurse provide? A. Inform that follow-up may end after the treatment is finished B. Reassure that complications will not occur if the infection is treated C. Notify that persons with STIs are reported to local health departments D. Explain how the infection is transmitted and the health risks involved

A. Inform that follow-up may end after the treatment is finished.

The nurse is caring for a seated client who is experiencing a tonic-clonic seizure. Which actions should the nurse implement? (Select all that apply) A. Loosen restrictive clothing B. Insert a bite block C. Ease the client to the floor D. Note the duration of the seizure E. Restrain the client

A. Loosen restrictive clothing C. Ease the client to the floor D. Note the duration of the seizure

While assessing a client who is admitted with heart failure and pulmonary edema, the nurse identifies dependent peripheral edema, an irregular heart rate, and a persistent cough that produces pink blood-tinged sputum. After initiating continuous telemetry and positioning the client, which intervention should the nurse implement? A. Obtain sputum sample B. Document degree of edema C. Initiate hourly urine output measurement D. Administer intravenous diuretics

A. Obtain sputum sample

The nurse is caring for client with flail chest secondary to 3 right rib fractures after sustaining a fall from a ladder. The client is anxious, but stable with an oxygen saturation of (SpO2) 93%. Which action should the nurse take? A. Splint affected side B. Insert nasal airway C. Coach through taking deep breaths D. Apply a non-rebreather mask

A. Splint affected side

The nurse is arranging home care for an older client who has a new colostomy following a large bowel resection three days ago. The client plans to live with a family member. Which actions should the nurse implement? (Select all that apply) A. Teach care of ostomy to care provider B. Assess the client for self care ability C. Provide pain medication instructions D. Request a home safety inspection E. Call home care agency to set up oxygen

A. Teach care of ostomy to care provider B. Assess the client for self care ability C. Provide pain medication instructions

The nurse is feeding an older adult who was admitted with aspiration pneumonia. The client is weak and begins coughing while attempting to drink through a straw. Which intervention should the nurse implement? A. Teach coughing and deep breathing exercises B. Assess the client's oral cavity for ulcerations C. Request thick nectar liquids for the client D. Monitor the client when using a straw for liquids

A. Teach coughing and deep breathing exercises

The nurse is caring for a 24-month-old toddler who has sensory sensitivity, difficulty engaging in social interactions, and has not yet spoken two-word phrases. Which assessment should the nurse administer? A. The modified checklist for autism in toddlers (M-CHAT) B. Psychology Systems Questionnaire (PHQ-2) C. Behavioral Style Questionnaire (BSQ) D. The Ages and Stages Questionnaire (ASQ)

A. The Modified Checklist for Autism in Toddlers (M-CHAT)

The mother of a 2 day old infant girl expresses concern about a "flea bite" type rash on her daughter's body. The nurse identifies a pink papular rash with vesicles superimposed over the thorax, back, buttocks, and abdomen. Which explanation should the nurse offer? A. This is a common newborn rash that will resolve after several days B. The rash is due to distended oil glands that will resolve in a few weeks C. The healthcare provider is being notified about the rash D. This rash is characteristic of a medication reaction

A. This is a common newborn rash that will resolve after several days

A combination multi-drug cocktail is being considered for an asymptomatic HIV-infected client with a CD4 cell count of 500. Which nursing assessment of the client is most crucial in determining whether therapy should be initiated? A. Willing to comply with complex drug schedules B. Maintains an adequate social support system C. Qualifies for a prescription assistance program D. States various side effects of retroviral agents

A. Willing to comply with complex drug schedules

A client with end-stage liver failure is declared brain dead. The family wants to discontinue feeding and donate any viable organs. Which action should the nurse take? a- Contact the regional organ procurement agency b- Convene a multidisciplinary care conference c- Explain that client may not be an organ donor candidate d- Discontinue feeding and fluids per the family's request.

a. Contact the regional organ procurement agency

At the end of a preoperative teaching session on pain management techniques, a client starts to cry and states, "I just know I can't handle all the pain." What is the priority nursing diagnosis for this client? a. Knowledge deficit b. Anxiety c. Anticipatory grieving d. Pain (acute)

anxiety Rationale: The client is demonstrating only anxiety. There is no indication that the client is presenting signs of A, C or D

A nurse receive a shift report about a male client with Obsessive compulsive disorder (OCD). The nurse does morning rounds and reaches the client while he is repeatedly washing the top of the same table. What intervention should the nurse implement? a. Encourage the client to be calm and relax for a little while b. Assist the client to identify stimuli that precipitates the activity. c. Allow time for the behavior and then redirect the client to other activities. d. Teach the client thought stopping techniques and ways to refocus.

c. Allow time for the behavior and then redirect the client to other activities

An adult male who was admitted two days ago following a cerebrovascular accident (CVA) is confused and experiencing left-sided weakness. He has tried to get out of bed several times, but is unable to ambulate without assistance. Which intervention is most important for the nurse to implement? a- Ask a family member to sit with the client b- Apply bilateral soft wrist restraints c- Assign staff to check client q15 minutes d- Install a bed exit safety monitoring device

d. Install a bed exit safety monitoring device

The nurse is planning care for a 3 month-old infant immediately postoperative following placement of a ventriculoperitoneal shunt for hydrocephalus. The nurse needs to A) Assess for abdominal distention B) Maintain infant in an upright position C) Begin formula feedings when infant is alert D) Pump the shunt to assess for proper function

is A: Assess for abdominal distention 10

The nurse is caring for a client who was successfully resuscitated from a pulseless dysrhythmia. Which of the following assessments is CRITICAL for the nurse to include in the plan of care? A) Hourly urine output B) White blood count C) Blood glucose every 4 hours D) Temperature every 2 hours

is A: Hourly urine output 4

A client is being treated for paranoid schizophrenia. When the client became loud and boisterous, the nurse immediately placed him in seclusion as a precautionary measure. The client willingly complied. The nurse's action A) May result in charges of unlawful seclusion and restraint B) Leaves the nurse vulnerable for charges of assault and battery C) Was appropriate in view of the client's history of violence D) Was necessary to maintain the therapeutic milieu of the unit

is A: May result in charges of unlawful seclusion and restraint 2

Which nursing intervention will be most effective in helping a withdrawn client to develop relationship skills? A) Offer the client frequent opportunities to interact with 1 person B) Provide the client with frequent opportunities to interact with other clients C) Assist the client to analyze the meaning of the withdrawn behavior D) Discuss with the client the focus that other clients have similar problems

is A: Offer the client frequent opportunities to interact with one person 12

A 2 year-old child is brought to the emergency department at 2:00 in the afternoon. The mother states: "My child has not had a wet diaper all day." The nurse finds the child is pale with a heart rate of 13 What assessment data should the nurse obtain next? A) Status of the eyes and the tongue B) Description of play activity C) History of fluid intake D) Dietary patterns

is A: Status of skin turgor

A client is recovering from a thyroidectomy. While monitoring the client's initial post operative condition, which of the following should the nurse report immediately? A) Tetany and paresthesia B) Mild stridor and hoarseness C) Irritability and insomnia D) Headache and nausea

is A: Tetany and paresthesia 8

A depressed client in an assisted living facility tells the nurse that "life isn't worth living anymore." What is the best response to this statement? A) "Come on, it is not that bad." B) "Have you thought about hurting yourself?" C) "Did you tell that to your family?" D) "Think of the many positive things in life."

is B: "Have you thought about hurting yourself?" 14

A client who is a former actress enters the day room wearing a sheer nightgown, high heels, numerous bracelets, bright red lipstick and heavily rouged cheeks. Which nursing action is the best in response to the client's attire? A) Gently remind her that she is no longer on stage B) Directly assist client to her room for appropriate apparel C) Quietly point out to her the dress of other clients on the unit D) Tactfully explain appropriate clothing for the hospital

is B: Directly assist client to her room for appropriate apparel 14

Upon completing the admission documents, the nurse learns that the 87 year-old client does not have an advance directive. What action should the nurse take? A) Record the information on the chart B) Give information about advance directives C) Assume that this client wishes a full code D) Refer this issue to the unit secretary

is B: Give information about advance directives

The nurse instructs the client taking dexamethasone (Decadron) to take it with food or milk. What is the physiological basis for this instruction? A) Retards pepsin production B) Stimulates hydrochloric acid production C) Slows stomach emptying time D) Decreases production of hydrochloric acid

is B: Stimulates hydrochloric acid production 10

The mother of a 3 month-old infant tells the nurse that she wants to change from formula to whole milk and add cereal and meats to the diet. What should be emphasized as the nurse teaches about infant nutrition? A) Solid foods should be introduced at 3-4 months B) Whole milk is difficult for a young infant to digest C) Fluoridated tap water should be used to dilute milk D) Supplemental apple juice can be used between feedings

is B: Whole milk is difficult for a young infant to digest 8

A 3 year-old child is brought to the clinic by his grandmother to be seen for "scratching his bottom and wetting the bed at night." Based on these complaints, the nurse would initially assess for which problem? A) Allergies B) Scabies C) Regression D) Pinworms

is D: Pinworms 4

A 16 year-old enters the emergency department. The triage nurse identifies that this teenager is legally married and signs the consent form for treatment. What would be the appropriate action by the nurse? A) Ask the teenager to wait until a parent or legal guardian can be contacted B) Withhold treatment until telephone consent can be obtained from the partner C) Refer the teenager to a community pediatric hospital emergency department D) Proceed with the triage process in the same manner as any adult client

is D: Proceed with the triage process in the same manner as any adult client 2

The first paddle has been placed on the chest of a client who needs defibrillation. Where should the nurse place the second paddle? (Mark the location where the second paddle should be placed on the image). right upper chest, left midaxillary

right upper chest, left midaxillary

Which discharge instruction has the greatest priority for the mother of a 6-year-old who was diagnosed with Type 1 diabetes mellitus one week ago?

"Always carry a quick source of sugar

Following a lumbar puncture, a client voices several complaints. What complaint indicated to the nurse that the client is experiencing a complication? a. "I am having pain in my lower back when I move my legs" b. "My throat hurts when I swallow" c. "I feel sick to my stomach and am going to throw up" d. I have a headache that gets worse when I sit up"

"I have a headache that gets worse when I sit up" Rationale: A post-lumbar puncture headache, ranging from mild to severe, may occur as a result of leakage of cerebrospinal fluid at the puncture site. This complication is usually managed by bedrest, analgesic, and hydration.

Ezetimibe (Zetia)

-Class: antihyperlipidemic -Indications/Actions: reduces serum cholesterol, inhibits cholesterol absorption in small intestine -Adverse reactions: may increase liver enzyme, hepatitis, arthralgia, rhabdomyolysis -Nursing implications: don't use with pt with liver disease/damage, get baseline hepatic function labs, monitor cholesterol, report muscle/bone pain

Mesalamine (5-aminosalicylic acid)

-Class: antiinflammatory, salicylic acid derivative -Indications: blocks prostaglandins, diminishes inflammation in colon, used for tx of ulcerative colities, sigmoiditis, and proctitis -Adverse reaction/side effects: abd cramping, flatulence, alopecia (rare) -D/c if rash, fever, abd pain -Nursing implications: encourage fluids, monitor bowel activity, assess for GI disturbances, may turn urine yellow/brown

Reaction formation

-Conscious attitudes and behaviors that are the opposite of what if really felt -Ex: a person who dislikes animals volunteers for Humane Society

Safety concerns with Parkinson's

-Early: falls, decreased mobility, risk for aspiration -Late: wheelchair bound, ulcers, decreased muscle, decreased mobility

Concerning lymph node characteristics

-Enlarged, tender, nonmovable

Repression

-Involuntary exclusion of painful thoughts/memories

Tracheostomy pt teaching

-Maintain airway: suctioning -Communicate with white boards -Cough and suction on the way out -No more than 10 seconds

Treatment for anaphylaxis

-Need meds! -1st: epinepherine for emergent anaphylaxis; can be given IM, SubQ, IV -Benedryl -H2 blocker to stop histamine release

Apgar score for reflex irritability

0 = no response to foot tap 1 = slight response (grimace) 2 = quick foot removal

Why is it important for the nurse to monitor blood pressure in clients receiving antipsychotic drugs? A) Orthostatic hypotension is a common side effect B) Most antipsychotic drugs cause elevated blood pressure C) This provides information on the amount of sodium allowed in the diet D) It will indicate the need to institute anti parkinsonian drugs

A) Orthostatic hypotension is a common side effect

Clients with mitral stenosis would likely manifest findings associated with congestion in the A) Pulmonary circulation B) Descending aorta C) Superior vena cava D) Bundle of His

A) Pulmonary circulation

The nurse is caring for a group of clients with the help of a practical nurse (PN). Which nursing actions should the nurse assign to the PN? (Select all that apply.) A. Administer a dose of insulin per sliding scale for a client with type 2 diabetes mellitus (DM). B. Obtain postoperative vital signs for a client one day following unilateral knee arthroplasty C. Perform daily surgical dressing change for a client who had an abdominal hysterectomy

A. Administer a dose of insulin per sliding scale for a client with type 2 diabetes mellitus (DM). B. Obtain postoperative vital signs for a client one day following unilateral knee arthroplasty C. Perform daily surgical dressing change for a client who had an abdominal hysterectomy

The nurse is developing an educational program for older clients who are being discharged with new antihypertensive medications. The nurse should ensure that the education materials include which characteristics? (Select all that apply) A. Uses common words with few syllables B. Printed using a 12-point type font C. Uses pictures to help illustrate complex ideas D. Contains a list with definitions of unfamiliar terms E. Written at a twelfth-grade reading level

A. Uses common words with few syllables C. Uses pictures to help illustrate complex ideas D. Contains a list with definitions of unfamiliar terms

5.A child is to receive vancomycin (Vancocin) 40 mg/kg IV one hour before a scheduled procedure. The child weighs 44 pounds. How many mg of the medication should the nurse administer

Answer: 800 Rationale: First, convert the child's weight to kg: 44 pounds divided by 2.2pound/kg = 20 kg. Next calculate the mg/kg/dose, 40 mg x 20 kg = 800mg

After administering a proton pump inhibitor (PPI), which action should the nurse take to evaluate the effectiveness of the medication? a. Ask the client about gastrointestinal pain

Ask the client about gastrointestinal pain

. Translator is working with the nurse who is giving discharge instructions to a non-English client. When the translator restates what the nurse is saying it appears that he is saying much more than what the nurse said. What action should the nurse take?

Ask the translator if there is a reason for the lengthiness of the translation

Which problem, noted in the client's history, is important for the nurse to be aware of prior to administration of a newly prescribed selective serotonin reuptake inhibitor (SSRI)? a. Bulimia nervosa b. Obsessive compulsive disorder c. Aural migraine headaches. d. Erectile dysfunction.

Aural migraine headaches

The nurse is caring for residents in a long term care setting for the elderly. Which of the following activities will be most effective in meeting the growth and development needs for persons in this age group? A) Aerobic exercise classes B) Transportation for shopping trips C) Reminiscence groups D) Regularly scheduled social activities

C) Reminiscence groups

The nurse is planning care for a client with a CVA. Which of the following measures planned by the nurse would be most effective in preventing skin breakdown? A) Place client in the wheelchair for four hours each day B) Pad the bony prominence C) Reposition every two hours D) Massage reddened bony prominence

C) Reposition every two hours

While planning care for a preschool aged child, the nurse understands needs. Which of the following would be of the most concern to the nurse? A) Playing imaginatively B) Expressing shame C) Identifying with family D) Exploring the playroom

B) Expressing shame

A client diagnosed with calcium kidney stones has a history of gout. A new prescription for aluminum hydroxide is scheduled to begin at 0730. Which client medication should the nurse bring to the healthcare provider's attention? A. Esinapril B. Allopurinol C. Furosemide D. Aspirin, low dose

B. Allopurinol

A client with bleeding esophageal varices receives vasopressin (Pitressin) IV. What should the nurse monitor for during the IV infusion of this medication? a. Chest pain and dysrhythmia

Chest pain and dysrhythmia

Emphysema classic presentation

Barrel chest

After changing to a new brand of laundry detergent, an adult male reports that he has a fine itchy rash. Which assessment finding warrants immediate intervention by the nurse? Bilateral Wheezing.

Bilateral Wheezing.

A child is diagnosed with acquired aplastic anemia. The nurse knows that this child has the best prognosis with which treatment regimen? a. Bone marrow transplantation

Bone marrow transplantation

The nurse walks into a client's room and finds the client lying still and silent on the floor. The nurse should first A) Assess the client's airway B) Call for help C) Establish that the client is unresponsive D) See if anyone saw the client fall

C) Establish that the client is unresponsive

As the nurse is speaking with a group of teens which of these side effects of chemotherapy for cancer would the nurse expect this group to be more interested in during the discussion? A) Mouth sores B) Fatigue C) Diarrhea D) Hair loss

D) Hair loss

While in the medical records department, the nurse observes several old medical records with names visible in waste container. What action should the nurse implement? a. Place the records in a separate trash bag and tie the bag securely closed b. Point out the record to a worker in the medical records department c. Contact the medical records department supervisor. d. Immediately remove and shred the records.

Contact the medical records department supervisor Rationale: Notify the department supervisor of a Privacy officer alerts the appropriate people to a possible internal procedural problem and provides an opportunity of education a prevention of recurrence.

The nurse is responsible for several elderly clients, including a client on bed rest with a skin tear and hematoma from a fall 2 days ago. What is the best care assignment for this client? A) Assign an RN to provide total care of the client B) Assign a nursing assistant to help the client with self-care activities C) Delegate complete care to an unlicensed assistive personnel D) Supervise a nursing assistant for skin care

D) Supervise a nursing assistant for skin care

Then nurse identifies several nursing problems for client who is immobile and who has been experiencing fecal incontinence and diarrhea for several days. The client's spouse is the primary caregiver. In planning care, which problem has the highest priority? A. Impaired bed mobility B. Caregiver role strain C. Fluid volume deficit D. Bowel incontinence

D. Bowel incontinence

The nurse assesses a client who has bilateral total knee replacements (TKR) four hours ago. The nurse notes that the dressing on the client's right knee is saturated with serosanguineous drainage. What action should the nurse implement? A. Monitor the client's current WBC B. Withhold next scheduled dose of low molecular weight heparin C. Confirm that the continuous passive motion device is intact D. Determine if the wound drainage device is functioning correctly

D. Determine if the wound drainage device is functioning correctly

A client is admitted with a wound on the right hand and associated cellulitis. In assessing the client's hand, which finding required most immediate follow-up by the nurse? a. Cyanotic nailbeds

Cyanotic nailbeds

Know your serum levels?

K- 3.5-5.1, Na - 135-145, Ca 9-11, BUN - 10-20

Incorrect Delegation

LVN/LPN to remove staples from abdominal wound

The nurse is teaching a childbirth education class t prospective parents and describing possible signs of labor. Class participants should be taught that which sign should be reported to the healthcare provider immediately?

Leaking of fluid from the vagina

The nurse assigned unlicensed assistive personnel (UAP) to apply antiembolism stockings to a client. The nurse and UAP enters the room, the nurse observes the stockings that were applying by the UAP. The UAP states that the client requested application of the stockings as seen on the picture, for increased comfort. What action should the nurse take? a. Ask the client if the stocking feel comfortable. b. Supervise the UAP in the removal of the stockings. c. Place a cover over the client's toes to keep them warm. d. Discussed effective use of the stockings with the client and UAP

Discussed effective use of the stockings with the client on UA Rational: antiembolism stockings are designed to fit securely and should be applied so that there are no bands of the fabric constricting venous return. The nurse should discuss the need for correct and effective use of the stockings with both the client and UAP to improve compliance. Other options do not correct the incorrect application of the stockings.

A client in the emergency center demonstrates rapid speech, flight of ideas, and reports sleeping only three hours during the past 48h. Based on these finding, it is most important for the nurse to review the laboratory value for which medication? a. Olanzapine b. Divalproex. c. Lorazepam d. Fluoxetine

Divalproex. Rationale: divalproex is the first line of treatment for bipolar disorder BPD because it has a high therapeutic index, few side effects, and a rapid onset in controlling symptoms and preventing recurrent episodes of mania and depression. The serum value of divalproex should be determined since the client is exhibiting symptoms of mania, which may indicate non-compliance with the medication regimen

Which of the following is a nursing intervention for a 3-year-old client who is going in for sugery in 8 hours that has an order for fluid restriction?

Do not allow the child to drink fluids

An Unna boot is applied to a client with a venous stasis ulcer. One week later, when the Unna boot is removed during a follow-up appointment, the nurse observes that the ulcer site contains bright red tissue. What action should the nurse take in response to this finding? a. Immediately apply a pressure dressing b. Document the ongoing wound healing. c. Irrigate the wound with sterile saline d. Obtain a capillary INR, measurement

Document the ongoing wound healing Rationale: Appearance of granulation tissue is the best indicator of increased venous retuns and ongoing wound healing

Two clients ring their call bells simultaneously requesting pain medication. What action should the nurse implement first? a. Prepared both client's medication and take to them at once b. Determine when each client last received pain medication. c. Evaluate both client's pain using a standardized pain scale d. Provide non-pharmacologic pain management interventions.

Evaluate both client's pain using a standardized pain scale Rationale: Before administering pain medication, each client' s level of pain should be evaluated using a standardizing scale to determine what type and how much pain medication the clients need.

A client who is admitted to the intensive care unit with syndrome of inappropriate antidiuretic hormone (SIADH) has developed osmotic demyelination. Which intervention should the nurse implement first? a. Patch one eye. b. Reorient often. c. Range of motion. d. Evaluate swallow

Evaluate swallow Rational: Osmotic demyelination, also known as central pontine myelinolysis, is nerve damage caused by the destruction of the myelin sheath covering nerve cells in the brainstem. The most common cause is a rapid, drastic change in sodium levels when a client is being treated for hyponatremia, a common occurrence in SIADH. Difficulty swallowing due to brainstem nerve damage should be care but determining the client's risk for aspiration is most important.

A 60-year-old female client with a positive family history of ovarian cancer has developed an abdominal mass and is being evaluated for possible ovarian cancer. Her Papanicolau (Pap) smear results are negative. What information should the nurse include in the client's teaching plan a. Further evaluation involving surgery may be needed b. A pelvic exam is also needed before cancer is ruled out c. Pap smear evaluation should be continued every six month d. One additional negative pap smear in six months is needed.

Further evaluation involving surgery may be needed Rationale: An abdominal mass in a client with a family history for ovarian cancer should be evaluated carefully

In assessing a client at 34-weeks' gestation, the nurse notes that she has a slightly elevated total T4 with a slightly enlarged thyroid, a hematocrit of 28%, a heart rate of 92 beats per minute, and a systolic murmur. Which finding requires follow-up? a. Elevated thyroid hormone level. b. Hematocrit of 28%. c. Heart rate of 92 beats per minute. d. Systolic murmur.

Hematocrit of 28% Rational: although physiologic anemia is expected in pregnancy, a hematocrit of 28% is below pregnant norms and could signify iron-deficiency anemia. Other options are normal finding pregnancy

Heparin

Hematoma assessment

The nurse walks into a client's room and notices bright red blood on the sheets and on the floor by the IV pole. Which action should the nurse take first? a. Clean up the spilled blood to reduce infection transmission. b. Notify the healthcare provider that the client appears to be bleeding. c. Apply direct pressure to the client's IV site. d. Identify the source and amount of bleeding.

Identify the source and amount of bleeding.

A male client with hypertension, who received new antihypertensive prescriptions at his last visit returns to the clinic two weeks later to evaluate his blood pressure (BP). His BP is 158/106 and he admits that he has not been taking the prescribed medication because the drugs make him "feel bad". In explaining the need for hypertension control, the nurse should stress that an elevated BP places the client at risk for which pathophysiological condition? a. Blindness secondary to cataracts b. Acute kidney injury due to glomerular damage c. Stroke secondary to hemorrhage d. Heart block due to myocardial damage

Stroke secondary to hemorrhage Rationale: Stroke related to cerebral hemorrhage is major risk for uncontrolled hypertension.

Which intervention should the nurse include in a long-term plan of care for a client with Chronic Obstructive Pulmonary Disease (COPD)? a. Reduce risks factors for infection b. Administer high flow oxygen during sleep c. Limit fluid intake to reduce secretions d. Use diaphragmatic breathing to achieve better exhalation

Reduce risks factors for infection Rationale: Interventions aimed at reducing the risk factors of infections should be included in the plan of care COPD client are at particular risk for respiratory infection. Prevention and early detection of infections are necessary.

Teenagers Teaching for Diabetic diet

Teenagers Teaching for Diabetic diet - Teach to look out for restaurant that have healthy food.

Priority pt

Ruptured spleen life threatening

A 66-year-old woman is retiring and will no longer have a health insurance through her place of employment. Which agency should the client be referred to by the employee health nurse for health insurance needs? a. Woman, Infant, and Children program b. Medicaid c. Medicare d. Consolidated Omnibus Budget Reconciliation Act provision.

Medicare Rationale: Title XVII of the social security Act of 1965 created Medicare Program to provide medical insurance for person more than 65 years or older, disable or with permeant kidney failure, WIC provides supplemental nutrition to meet the needs of pregnant of breastfeeding woman, infants and children up to age of 6. Medicaid provides financial assistance to pay for medical services for poor older adults, blind, disable and families with dependent children. COBRA(D) health benefit provisions is a limited insurance plan for those who has been laid off or become unemployed.

A client with a history of heart failure presents to the clinic with a nausea, vomiting, yellow vision and palpitations. Which finding is most important for the nurse to assess to the client?

Obtain a list of medications taken for cardiac history

A client who has a suspected brain tumor is scheduled for a computed (CT) scan. When preparing the client for the client for the CT scan, which intervention should the nurse implement? a. Obtain the client's food allergy history

Obtain the client's food allergy history

A client with a general anxiety disorder is pacing the hallway. The client tells the nurse, "My heart is just racing and sometimes it feels like it's fluttering. I'm feeling short of breath and dizzy." What action should the nurse implement first?

Obtain the clients vital signs.

A client with possible acute kidney injury (AKI) is admitted to the hospital and mannitol is prescribed as a fluid challenge. Prior to carrying out this prescription, what intervention should the nurse implement? a. Collect a clean catch urine specimen. b. Instruct the client to empty the bladder. c. Obtain vital signs and breath sounds. d. No specific nursing action is required

Obtain vital signs and breath sounds. Rational: the client's baseline cardiovascular status should be determined before conducting the fluid challenge. If the client manifests changes in the vital signs and breath sounds associated with pulmonary edema, the administration of the fluid challenge should be terminating. Other options would not assure a safe administration of the medication.

The nurse enters a client's room and observe the unlicensed assistive personnel (UAP) making an occupied bed as seen in the picture. What action should the nurse take first? a. Place the side rails in an up position

Place the side rails in an up position

The nurse is collecting sterile sample for culture and sensitivity from a disposable three chamber water-seal drainage system connected to a pleural chest tube. The nurse should obtain the sample from which site on the drainage system? a. Plastic tubing located at the chest insertion site

Plastic tubing located at the chest insertion site

A client with coronary artery disease who is experiencing syncopal episodes is admitted for an electrophysiology study (EPS) and possible cardiac ablation therapy. Which intervention should the nurse delegate to the unlicensed assistive personnel (UAP)? a. Prepare the skin for procedure. b. Identify client's pulse points c. Witness consent for procedure d. Check telemetry monitoring

Prepare the skin for procedure.

The nurse is collecting a sterile urine specimen using a straight catheter tray for culture.... (Arrange from first action to last). Correct Order: (DODU) 1. Drape the client in a recumbent position for privacy 2. Open the urinary catheterization tray 3. Don sterile gloves using aseptic technique 4. Use forceps and swaps to clean the urinary meatus

1. Drape the client in a recumbent position for privacy 2. Open the urinary catheterization tray 3. Don sterile gloves using aseptic technique 4. Use forceps and swaps to clean the urinary meatus

The nurse is preparing a heparin bolus dose of 80 units/kg for a client who weighs 220 pounds. Heparin sodium injection, USP is available in a 3o ml multidose vial with the concentration of 1,000 USP units/ml. how many ml of heparin should the nurse administer? (Enter numeric value only) 8

8 Calculate the client's weigh in kg: 220 pounds divides by 2.2 pounds/kg ꞊100 kg Calculate the client's dose, 80 units x 100 kg ꞊ 8,000 units Use the formula, D / H X Q ꞊ 8,000 units / 1,000 units x 1ml ꞊ 8

A client is receiving a continuous half strength tube feeding at 50ml/hour. To prepare enough of the solution for eight hours, how many ml of full strength feeding will the nurse need? (Enter numeric value only.)

200ml . 25 ml of full strength feeding mixed with 25ml of water provided 50ml of half strength. 25ml × 8 hours= 200ml

The healthcare provider prescribes cephalexin 125mg/5mL oral suspension for a client who weighs 77 pounds. The recommended safe dose 25mg/kg/24 hours in 4 divided doses. Based on the client's weight, how many mL should the nurse administer?

9

A client is receiving an IV of heparin sodium 25000 units in 5% dextrose injection 500 ml at 14 ml/hour...verify that the client is receiving the prescribed amount of heparin. How many units is the client receiving 700

700 Rationale: 25000/500x14=700

The nurse is caring for a 42 year old male client who is excreting less sodium than he is consuming. If this condition continues, what complication can the nurse expect this client to exhibit?

Edema

IV Line

Flush line before and after compatible IV solution

A male client is admitted with a bowel obstruction and intractable vomiting for the last several hours despite the use of antiemetics. Which intervention should the nurse implement first? pH 7.50; PaCo2 42; HCO3 33; pO2 92 a. Infuse 0.9 % sodium chloride 500 ml bolus

Infuse 0.9 % sodium chloride 500 ml bolus

A client with multiple sclerosis (MS) has decreased motor function after taking a hot bath (Uhthoff's sign). Which pathophysiological mechanism supports this response a. Arterial Constriction b. Temporary vasodilation c. Poor temperature control d. Severe dehydration.

Temporary vasodilation Rationale: Uhthoff's sign results from temporary vasodilation

When assessing a mildly obese 35-year-old female client, the nurse is unable to locate the gallbladder when palpating below the liver margin at the lateral border of the rectus abdominal muscle. What is the most likely explanation for failure to locate the gallbladder by palpation? a. The client is too obese b. Palpating in the wrong abdominal quadrant c. The gallbladder is normal d. Deeper palpation technique is needed

The gallbladder is normal Rationale: a normal healthy gallbladder is not palpable

Dialysis

educate client of the reality of condition

Apgar score for color

0 = dusky/cyanotic 1 = acrocyanotic 2 = pink

Korotkoff sounds

-Heard over brachial artery

Normal lymph node characteristics

-Pea sized, nontender, movable

Which client should the nurse assist first?

An inguinal hernia who has developed abdominal distention and fever in the last 8 hours.

A client who is admitted with complications related to hypopituitarism is diaphoretic and hypotensive. Which assessment finding warrants immediate intervention by the nurse?

Lethargy

Spiritual/Documentation

Native American - Allow pt's family to stay in the room

Diagnosis of breast cancer

Needle aspiration or biopsy.

A child with heart failure (HF) is taking digitalis. Which signs indicates to the nurse that the child may be experiencing digitalis toxicity? a. Vomiting

Vomiting

Pyelonephritis symptoms

elevates temperature

Ankle brachial index

ensure that you provide meticulous foot care to client

While assessing a client with wrist restraints the nurse first slides two fingers under the restraints and then notes that the ties are secured to the side rail using a quick-release tie. What action should the nurse implement?

Reposition the restraints ties, securing them to the bed frame

A client is admitted to isolation with the diagnosis of active tuberculosis (TB). Which infection control measures should the nurse implement? a. Negative pressure environment b. Contact precautions c. Droplet precautions d. Protective environment

a. Negative pressure environment

Prolonged exposure to high concentrations of supplemental oxygen over several days can cause which pathophysiological effect? A. Disrupted surfactant production B. Metabolic acidosis C. Aphasia and memory loss D. Deep sleep or coma

A. Disrupted surfactant production

Triage question with a train wreck The psychiatric nurse is called to a train derailment that was likely caused by a terrorist bomb. In triaging those in need of immediate care, what is the priority ranking for these cases? (arrange these cases in order or priority with the top item requiring the most immediate care and the bottom item requiring the least priority care)

A mother and father have just arrived on the . A crying child being held by another passenger. A woman sitting on the ground with a blanket. A middle-aged man who is wandering around

The nurse-manager of a perinatal units is notified that one client from the medical-surgical unit needs to be transferred to make room for new admissions. Which client should the nurse recommend for transfer to the antepartal unit?

A 35-year-old with lupus erythematosus

Which location should the nurse choose as the best for beginning a screening program for hypothyroidism? a. A business and professional women's group. b. An African-American senior citizens center c. A daycare center in a Hispanic neighborhood d. An after-school center for Native-American teens

A business and professional women's group Rationale: The population at highest risk is A so this is the group that would benefit the most for a screening program of hypothyroidism and occurs between 35 and 60 years of age and is most common in females.

Which of the actions suggested to the RN by the PN during a planning conference for a 10 month-old infant admitted 2 hours ago with bacterial meningitis would be acceptable to add to the plan of care? A) Measure head circumference B) Place in airborne isolation C) Provide passive range of motion D) Provide an over-the-crib protective top

A) Measure head circumference

While caring for a toddler with croup, which initial sign of croup requires the nurse's immediate attention? A) Respiratory rate of 42 B) Lethargy for the past hour C) Apical pulse of 54 D) Coughing up copious secretions

A) Respiratory rate of 42

While caring for a client who was admitted with myocardial infarction (MI) 2 days ago, the nurse notes today's temperature is 101.1 degrees Fahrenheit (38.5 degreesCelsius). The appropriate nursing intervention is to A) Call the health care provider immediately B) Administer acetaminophen as ordered as this is normal at this time C) Send blood, urine and sputum for culture D) Increase the client's fluid intake

B) Administer acetaminophen as ordered as this is normal at this time

A nurse has just received a medication order which is not legible. Which statement best reflects assertive communication? A) "I cannot give this medication as it is written. I have no idea of what you mean." B) "Would you please clarify what you have written so I am sure I am reading it correctly?" C) "I am having difficulty reading your handwriting. It would save me time if you would be more careful." D) "Please print in the future so I do not have to spend extra time attempting to read your writing."

B) "Would you please clarify what you have written so I am sure I am reading it correctly?"

A client with type 2 diabetes mellitus arrives to the clinic reporting episodes of weakness and palpitations. Which finding should the nurse identify may indicate an emerging situation? A. Potassium 3.5 mEq/L B. Fingertips feel numb C. Sodium 135 mEq/L D. Cervical spine stiffness

B. Fingertips feel numb

After working with a very demanding client, an unlicensed assistive personnel(UAP) tells the nurse, "I have had it with that client. I just can't do anything that pleases him. I'm not going in there again." The nurse should respond by saying A) "He has a lot of problems. You need to have patience with him. "B) "I will talk with him and try to figure out what to do." C) "He is scared and taking it out on you. Let's talk to figure out what to do." D) "Ignore him and get the rest of your work done. Someone else can take care of him forthe rest of the day."

C) "He is scared and taking it out on you. Let's talk to figure out what to do."

A client is receiving enoxaparin 30mg subcutaneously twice a day. In assessing for adverse effects of the medication, which serum laboratory value is most important for the nurse to monitor? A. Glucose B. Calcium C. Platelet count D. White blood cell count

C. Platelet count

The nurse is teaching a client with dysrhythmia about the electrical pathway of an impulse as it travels through the heart. Which of these demonstrates the normal pathway? A) AV node, SA node, Bundle of His, Purkinje fibers B) Purkinje fibers, SA node, AV node, Bundle of His C) Bundle of His, Purkinje fibers, SA node , AV node D) SA node, AV node, Bundle of His, Purkinje fibers

D) SA node, AV node, Bundle of His, Purkinje fibers

A client with a diagnosis of bipolar disorder has been referred to a local boarding home for consideration for placement. The social worker telephoned the hospital unit for information about the client's mental status and adjustment. The appropriate response of the nurse should be which of these statements? A) I am sorry. Referral information can only be provided by the client's health care providers. B) "I can never give any information out by telephone. How do I know who you are?" C) Since this is a referral, I can give you this information. D) I need to get the client's written consent before I release any information to you.

D) I need to get the client's written consent before I release any information to you.

A home health nurse is caring for a client with a pressure sore that is red, with serous drainage, is 2 inches in diameter with loss of subcutaneous tissue. The appropriate dressing for this wound is A) A transparent film dressing B) Wet dressing with debridement granules C) Wet to dry with hydrogen peroxide D) Moist saline dressing

D) Moist saline dressing

Which nursing action is a priority as the plan of care is developed for a 7 year-old child hospitalized for acute glomerulonephritis? A) Assess for generalized edema B) Monitor for increased urinary output C) Encourage rest during hyperactive periods D) Note patterns of increased blood pressure

D) Note patterns of increased blood pressure

Which nursing intervention has the highest priority for a multigravida who delivered? a. Maintain cold packs to the perineum for 24 hrs. b. Assess the client pain level frequently c. Observe for appropriate interaction with the infants. d. Assess fundal tone and lochia flow

D. Assess fundal tone and lochia flow Rationale D is the priority intervention because is a multigravida and this pregnancy predisposes the client to uterine atony which could result in hemorrhage.

A client is admitted with a diagnosis of urolithiasis. Which finding is most important for the nurse to report to the healthcare provider? A. Volume of each voiding is more than 300mL B. Serum potassium that is elevated C. Relief of flank pain that radiated into the groin D. Hematuria that is beginning to turn pink

D. Hematuria that is beginning to turn pink

A client at 12 weeks gestation is admitted to the antepartum unit with a diagnosis of hyperemesis gravidarum. Which action is most important for the nurse to implement? A. Obtain the client's 24-hour dietary recall B. Document mucosal membrane status C. Schedule a consult with a nutritionist D. Initiate prescribed intravenous fluids

D. Initiate prescribed intravenous fluids

3 Middle ear bones

Incus, Maleus (hammer), Stapes

A mother brings her 4-month-old son to the clinic with a quarter taped over his umbilicus, and tells the nurse the quarter is supposed to fix her child's hernia. Which explanations should the nurse provide? a. This hernia is a normal variation that resolves without treatment. b. Restrictive clothing will be adequate to help the hernia go away. c. An abdominal binder can be worn daily to reduce the protrusion. d. The quarter should be secured with an elastic bandage wrap.

This hernia is a normal variation that resolves without treatment. Rational: an umbilical hernia is a normal variation in infants that occurs due to an incomplete fusion of the abdominal musculature through the umbilical ring that usually resolves spontaneously as the child learns to walk. Other choices are ineffective and unnecessary.

The nurse is developing a plan of care for a middle-aged woman who is diagnosed with type 2 diabetes mellitus (DM). To lower her blood glucose and increase her serum high-density lipoprotein (HDL) levels, which instruction is most important for the nurse to provide? a- Exercise at least three times weekly b- Monitor blood glucose levels daily c- Limit intake of foods high in saturated fat d- Learn to read all food product labels

a. Exercise at least three times weekly

A male client recently released from a correctional facility arrives at the clinic with a cough, fever, and chills. His history reveals active tuberculosis (TB) 10 years ago. What action should the nurse implement? (Select all that apply) a- Administer a PPD test b- Schedule the client for the chest radiograph c- Obtain sputum for acid fast bacillus (AFB) testing d- Place a mask on the client until he is moved to isolation. e- Send the client home with instructions for a prescribe antibiotic.

b- Schedule the client for the chest radiograph c- Obtain sputum for acid fast bacillus (AFB) testing d- Place a mask on the client until he is moved to isolation. Rationale: Client with history of TB a chest x-ray and sputum are indicated. The client sign and symptoms indicate the pt should wear mask to protect others.

The nurse is managing the care of a client with Cushing's syndrome. Which interventions should the nurse delegate to the unlicensed assistive personnel (UAP)? (Select all that apply) a. Evaluate the client for sleep disturbances b. Weigh the client and report any weight gain. c. Report any client complaint of pain or discomfort. d. Assess the client for weakness and fatigue e. Note and report the client's food and liquid intake during meals and snacks.

b. Weigh the client and report any weight gain. c. Report any client complaint of pain or discomfort. e. Note and report the client's food and liquid intake Rationale: B, C and E are functions within the scope of practice for the UAO include reporting client complaints of pain.

An older woman who has difficulty hearing is being discharged from day surgery following a cataract extraction & lens implantation. Which intervention is most important for the nurse to implement to ensure the client's compliance with self-care? a- Speak clearly and face the clients for lip reading b- Provide written instructions for eyes drop administration c- Ensure that someone will stay with the client for 24 hours. d- Have the client vocalize the instructions provided.

d. Have the client vocalize the instructions provided. Rationale: A client with both hearing and visual sensory deficit should be repeat the instruction provided so the nurse needs to be sure the clients understand the self-care instructions.

Degenerative Joint Disease

-Assess kidney function before giving pain meds -Assess pain -Pain meds: NSAIDS (motrin, aspirin, naproxin sodium) are better for inflammation -Nonpharmacological measures: ice, rest, warm compresses

Polycystic Kidney Disease treatments

-Need pain relievers -NO NSAIDS (advil, naproxin sodium, motrin) -Can have tylenol

Trapeze bar and considerations

-Need upper body strength/upper extremities -Get history from pt: make sure they don't have any problems with dislocated shoulder -Allows more independence and participation with repositioning

Mormon Diet

-No alcohol -No caffeine -Give milk, juice, or water -If diabetic, no juice -Church = latter day saints

When washing soiled hands, the nurse first wets the hands and applies soap. The nurse should complete additional actions in which sequence? (Arrange from first action on top last action on bottom.) 1. Rub hands palm to palm. 2. Interlace the fingers, 3. Dry hands with paper towel. 4. Turn off the water faucet.

1. Rub hands palm to palm. 2. Interlace the fingers, 3. Dry hands with paper towel. 4. Turn off the water faucet.

After a client has an enteral feeding tube inserted, the most accurate method for verification of placement is A) Abdominal x-ray B) Auscultation C) Flushing tube with saline D) Aspiration for gastric contents

A) Abdominal x-ray

The nurse who works in labor and delivery is reassigned to the cardiac care unit for the day because of a low census in labor and delivery. Which assignment is best for the charge nurse to give this nurse? A. Assist cardiac nurses with their assignments B. Monitor the central telemetry C. Perform the admission of a new client D. Transfer a client to another unit

A. Assist cardiac nurses with their assignments

The nurse identifies an electrolyte imbalance, a weight gain of 4.4lbs (2kg) in 24 hours and an elevated central venous pressure for a client with full thickness burns. Which intervention should the nurse implement? A. Auscultate for irregular heart rate B. Review arterial blood gases results C. Measure ankle circumference D. Document abdominal girth

A. Auscultate for irregular heart rate

A female client presents in the emergency department and tells the nurse that she was raped last night. Which question is most important for the nurse to ask? A. Has she taken a bath since the rape occurred? B. Is the place where she lives a safe place? C. Does she know the person who raped her? D. Did she report the rape to the police department?

A. Has she taken a bath since the rape occurred?

During an admission assessment, a client reports currently using heroin. Which information is most important for the nurse to consider in the plan of care? A. History of suicide attempts B. Feelings of disorientation C. Undiagnosed social anxiety symptoms (SAD) D. Family history of schizophrenia

A. History of suicide attempts

A client with rheumatoid arthritis (RA) starts a new prescription for etanercept subcutaneously once weekly. The nurse should emphasize the importance of reporting which problem to the healthcare provider? A. Joint stiffness B. Persistent fever C. Headache D. Increased hunger and thirst

A. Joint stiffness

A client with Type 1 diabetes mellitus and a large draining ulcer of the right foot is admitted with a suspected Staphylococcus aureus infection. Which interventions should the nurse implement? (Select all that apply) A. Monitor the client's white blood cell count B. Explain the purpose of a low bacteria diet C. Send wound drainage for culture and sensitivity D. Institute contact precautions for staff and visitors E. Use standard precautions and wear a mask

A. Monitor the client's white blood cell count C. Send wound drainage for culture and sensitivity D. Institute contact precautions for staff and visitors

During discharge teaching, an overweight client heart failure (HF) is asked to make a grocery list for the nurse to review. Which food choices included on the client's list should the nurse encourage A. natural whole almonds B. lightly salted potatoe chips C. Cheddar cheese cubes D. Canned fruits in heavy syrup E. plain, air-popped popcorn

A. Natural whole almonds E. Plain, air-popped popcorn

A client with multiple sclerosis is receiving beta-1b interferon every other day. To assess for possible bone marrow suppression caused by the medication, which serum laboratory test findings should the nurse monitor? (Select all that apply) A. Platelet count B. Red blood cell count (RBC) C. White blood cell count (WBC).

A. Platelet count B. Red blood cell count (RBC) C. White blood cell count (WBC).

Two days prior to discharge from the rehabilitation facility, the nurse is teaching a client who is recovering from Guillain-Barre syndrome about home care. Which actions should the nurse include when providing discharge teaching to the client and spouse? (Select all that apply) A. Review safe transfer strategies B. Develop a nutritional plan C. Help identify community support D. Initiate a rigorous exercise routine E. Provide cooking instructions

A. Review safe transfer strategies B. Develop a nutritional plan C. Help identify community support

A toddler presenting with a history of intermittent skin rashes, hives, abdominal pain, and vomiting that occurs after ingesting of milk products arrives to the clinic accompanied by the parents. Which type of testing should the nurse provide education to the toddler's family about? A. Serum immunoglobulin E (IgE) B. Intradermal test C. Atopy patch test D. Placebo-controlled food challenge

A. Serum immunoglobulin E (IgE)

The nurse is triaging several children as they present to the emergency room after an accident. Which child requires the most immediate intervention by the nurse? a. An 11-year-old with a headache, nausea, and projectile vomiting

An 11-year-old with a headache, nausea, and projectile vomiting

The nurse is preparing a client who had a below-the-knee (BKA) amputation for discharge to home. Which recommendations should the nurse provide this client? (Select all that apply) A. Avoid range of motion exercises B. Use a residual limb shrinker C. Apply alcohol to the stump after bathing D. Inspect skin for redness E. Wash the stump with soap and water

B. Use a residual limb shrinker D. Inspect skin for redness E. Wash the stump with soap and water

A female client who is admitted to the mental health unit for opiate dependency is receiving clonidine 0.1 mg PO for withdrawal symptoms. The client begins to complain of feeling nervous and tells the nurse that her bones are itching. Which finding should the nurse identify as a contraindication for administering the medication? a. Blood pressure 90/76 mm Hg

Blood pressure 90/76 mm Hg

The primary nursing diagnosis for a client with congestive heart failure with pulmonary edema is A) Pain B) Impaired gas exchange C) Cardiac output altered: decreased D) Fluid volume excess

C) Cardiac output altered: decreased

The nurse is auscultating a client's lung sounds. Which description should the nurse use to document this sound? A. Stridor B. Low pitched or coarse crackles C. High pitched or fine crackles D. High pitched wheeze

C. High pitched or fine crackles

When checking a third grader's height and weight, the school nurse notes that these measurements have not changed in the last year. The child is currently taking daily vitamins, albuterol, and methylphenidate for attention deficit hyperactivity disorder (ADHD). Which intervention should the nurse implement? A. Report findings to the parents. B. Document findings in the child's school file. C. Refer child to the family healthcare provider. D. Encourage child to get more sleep.

C. Refer child to the family healthcare provider. Rationale: ADHD is most commonly managed with methylphenidate, which causes insomnia due to CNS stimulation and growth suppression secondary to appetite suppression. The child should be referred to the healthcare provider (C) because a change in the administration schedule of methylphenidate or discontinuing the drug is indicated until the child's growth increases. (A and B) may not ensure referral for a valuation of the medication's impact on the child's growth pattern. (D) is ineffective.

Following a motor vehicle collision (MCV), a male adult in severe pain is brought to the emergency department via ambulance. His injured left leg is edematous, ecchymotic around the impact of injury on the thigh, and shorter than his right leg. Based on these findings, the client is at greatest risk for which complication? a. Arterial ischemia b. Tissue necrosis c. Fat embolism d. Nerve damage

Fat embolism

A client who had a small bowel resection acquired methicillin resistant staphylococcus aureus (MRSA) while hospitalized. He treated and released, but is readmitted today because of diarrhea and dehydration. It is most important for the nurse to implement which intervention. a. Maintain contact transmission precaution b. Review white blood cell (WBC) count daily c. Instruct visitors to gown and wash hands d. Collect serial stool specimens for culture

Maintain contact transmission precaution Rationale: The client may have residual postoperative MRSA infection, a resistant and highly contagious healthcare-associated infection (HAI), that requires strict contact precautions (A), as recommend by the Center for Disease Control (CDC).

A 59-year-old male client comes to the clinic and reports his concern over a lump that, "just popped up on my neck about a week ago." In performing an examination of the lump, the nurse palpates a large, nontender, hardened left subclavian lymph node. There is not overlying tissue inflammation. What do these findings suggest? a. Malignancy

Malignancy

Following an open reduction of the tibia, the nurse notes bleeding on the client's cast. Which action should the nurse implement? a. Outline the area with ink and check it every 15 minutes to see if the area has increased

Outline the area with ink and check it every 15 minutes to see if the area has increased

An infant is receiving gavage feedings via nasogastric tube. At the beginning of the feeding, the infant's heart rate drops to 80 beats/minute. What action should the nurse take? a. Slow the feeding and monitor the infant's response.

Slow the feeding and monitor the infant's response.

Azithromycin is prescribed for an adolescent female who has lower lobe pneumonia and recurrent chlamydia. What information is most important for the nurse to provide to this client a. Have partner screened for human immunodeficiency virus b. Report a sudden onset arthralgia to the healthcare provider c. Decrease intake of high-fat-foods, caffeine, and alcohol d. Use two forms of contraception while taking this drug.

Use two forms of contraception while taking this drug. Rationale: Antibiotic, especially broad-spectrum drugs, like azithromycin, decrease the effectiveness of oral contraceptives and some spermicides, so the adolescent should be encouraging to use at least two forms of contraception to prevent pregnancy

A nurse is preparing to feed a 2-month-old male infant with heart failure who was born with congenital heart defect. Which intervention should the nurse implement? a. Feed the infant when he cries b. Allow the infant to rest before feeding c. Weigh before and after feeding. d. Insert a nasogastric feeding tube.

b. Allow the infant to rest before feeding

A client in her first trimester of pregnancy complains of nausea. Which complementary therapy should the nurse recommend? a. Eat food high in garlic with the evening meal b. Drink chamomile tea at breakfast and in the evening. c. Increase cocoa in the diet and drink before bedtime d. Join a yoga class that meets at least weekly

b. Drink chamomile tea at breakfast and in the evening. Rationale: Chamomile tea is used to aid with digestion and is in fact sometimes used for indigestion. C should not be used by breastfeeding woman or at night when trying to go to sleep. D is for improve circulation, stimulate the internal organs, stretch the body, restore....

The mother of a one-month-old boy born at home brings the infant to his first well...was born two weeks after his due date, and that he is a "good, quiet baby" who almost... hypothyroidism, what question is most important for the nurse to ask the mother? a. Has your son had any immunizations yet? b. Is your son sleepy and difficult to feed? c. Are you breastfeeding or bottle feeding your son? d. Were any relatives born with birth defects?

b. Is your son sleepy and difficult to feed? Rationale: Like adults with hypothyroidism, excess fatigue is common and a "good" baby is of.... occurs with hypothyroidism and can result in poor sucking.

A client with hypertension receives a prescription for enalapril, an angiotensin... instruction should the nurse include in the medication teaching plan? a. Increase intake of potassium-rich foods b. Report increased bruising of bleeding c. Stop medication if a cough develops d. Limit intake of leafy green vegetables

b. Report increased bruising of bleeding Rationale: ACEIs can cause thrombocytopenia and increased risk for bruising and bleeding. A is not necessary because is a potassium-sparing

Which serum blood findings with diabetic ketoacidosis alerts the nurse that immediate action is required? A) pH below3 B) Potassium of 5.0 C) HCT of 60 D) Pa O2 of 79

is C: HCT of 60 7

The healthcare provider prescribes potassium chloride 25 mEq in 500 ml D_5W to infuse over 6 hours. The available 20 ml vial of potassium chloride is labeled, "10 mEq/5ml." how many ml of potassium chloride should the nurse add the IV fluid? (Enter numeric value only. If is rounding is required, round to the nearest tenth.) 12.5

12.5 Rationale: Using the formula D / H X Q: 25 mEq / 10 mEq x 5ml ꞊12.5ml

A client currently receiving an infusion labeled Heparin Sodium 25,000 Units in 5% Dextrose Injection 500 mL at 14 mL/hour. A prescription is received to change the rate of the infusion to 900 units of Heparin per hour. The nurse should set the infusion pump to deliver how many mL/hour? (Enter numeric value only) 18

18 Rationale: 500 mL x 900 units = 450,000/25000 = 18 mL/hour

The nurse has complete the diet teaching of a client who is being discharged following treatment of a leg wound. A high protein diet is encouraged to promote wound healing. Which lunch choice by the client indicates that the teaching was effective? A. A tuna fish sandwich with chips and ice cream B. A salad with three kinds of lettuce and fruit C. A peanut butter sandwich with soda and cookies D. Vegetable soup, crackers, and milk

A. A tuna fish sandwich with chips and ice cream

A client diagnosed with calcium kidney stones has a history of gout. A new prescription for aluminum hydroxide (Amphogel) is scheduled to begin at 0730. Which client medication should the nurse bring to the healthcare provider's attention? a. Allopurinol (Zyloprim) b. Aspirin, low dose c. Furosemide (lasix) d. Enalapril (vasote)

Allopurinol (Zyloprim) Rationale: The effectiveness of allopurinol is diminished when aluminum hydroxide is used leading to an increased chance for gout flare ups. The healthcare provider should be alerted about the allopurinol interaction so any changes in medication regimen may be considered.

In preparing assignments for the shift, which client is best for the charge nurse to assign to a practical nurse?

An elderly client with Alzheimer's disease complicated by dysphagia.

The nurse reviews the laboratory findings of a client with an open fracture of the tibia. The white blood cell (WBC) count and erythrocyte sedimentation rate (ESR) are elevated. Before reporting this information to the healthcare provider, what assessment should the nurse obtain? a. Appearance of wound

Appearance of wound

Several months after a foot injury, and adult woman is diagnosed with neuropathic pain. The client describes the pain as severe and burning and is unable to put weight on her foot. She asks the nurse when the pain will "finally go away." How should the nurse respond? a. Explain the healing from injury can take many months b. Assist the client in developing a goal of managing the pain. c. Encourage the client to verbalize her fears about the pain d. Complete an assessment of the client's functional ability.

Assist the client in developing a goal of managing the pain Rationale: Neuropathic pain is chronic pain and the nurse should first help the client understand the need to learn to manage the pain.

A client in the intensive care unit is being mechanically ventilated, has an indwelling urinary catheter in place, an exhibiting signs of restlessness. Which action should the nurse take fist? a. Auscultate bilateral breath sounds

Auscultate bilateral breath sounds

A client with a chronic health problem has difficulty ambulating short distance due to generalized weakness but can bear weight on both legs. To assist with ambulation and provide the greatest stability, what assistive device is best for this client? a. A quad cane b. Crutches with 2-point gait. c. Crutches with 3-point gait. d. Crutches with 4-point gait.

Crutches with 4-point gait. Rationale: Crutches using a 4-point gait provide stability and require weight bearing on both legs, which this client should be able to provide. A is used when is partial or complete leg paralysis or some hemiplegia. B requires at least partial weight bearing on each foot but does not provide the stability of D. C is useful when the client must bear all the weight on one foot and this is not the problem experienced by this client.

During the change-of-shift report the assigned nurse notes a Catholic client is scheduled to be admitted for the delivery of a ninth child. Which comment stated angrily to a colleague by this nurse indicates an attitude of prejudice? A) "I wonder who is paying for this trip to the hospital? "B) "I think she needs to go to the city hospital. "C) "All those people indulge in large families! "D) "Doesn't she know there's such a thing as birth control?"

D) "Doesn't she know there's such a thing as birth control?"

The nurse is caring for a client in hypertensive crisis in an intensive care unit. The priority assessment in the first hour of care is A) Heart rate B) Pedal pulses C) Lung sounds D) Pupil responses

D) Pupil responses

During a visit to the planned parenthood clinic, a young woman tells the nurse that she is going to discontinue taking the oral contraceptives she has taken for three years because she wants to get pregnant. History indicates that her grandfather has adult onset diabetes and that she was treated for chlamydia six months ago, which factor in this client's history poses the greatest risk for this woman's pregnancy? A. Family history of adult onset diabetes. B. Treatment for chlamydia in the past year C. Client's age and previous sexual behavior D. Three year history of taking oral contraceptives

D. Three year history of taking oral contraceptives

The nurse is preparing to administer an oral antibiotic to a client with unilateral weakness, ptosis, mouth drooping and, aspiration pneumonia. What is the priority nursing assessment that should be done before administering this medication a. Ask the client about soft foods preferences b. Auscultate the client's breath sounds c. Obtain and record the client's vital signs d. Determine which side of the body is weak.

Determine which side of the body is weak.

During a clinic visit, a client with a kidney transplant ask, "What will happen if chronic rejection develops?" which response is best for the nurse to provide? a. Dialysis would need to be resumed if chronic rejection becomes a reality

Dialysis would need to be resumed if chronic rejection becomes a reality

A client is admitted with a diagnosis of acute renal failure. The nurse should monitor closely for what?

Drug toxicity

One day following a total knee replacement, a male client tells the nurse that he is unable to transfer because it is too painful. What action should the nurse implement? a- Encourage use of analgesics before position change b- Assess anxiety about transferring to commode chair c- Assist client during transfer on the first two days d- Review use of assistive devices for weight bearing.

Encourage use of analgesics before position change

When development a teaching plan for a client newly diagnosed type 1 diabetes, the nurse should explain that an increase thirst is an early sing of diabetes ketoacidosis (DKA), which action should the nurse instruct the client to implement if this sign of DKA occur? a. Resume normal physical activity b. Drink electrolyte fluid replacement c. Give a dose of regular insulin per sliding scale d. Measure urinary output over 24 hours.

Give a dose of regular insulin per sliding scale Rationale: As hyperglycemia persist, ketone body become a fuel source, and the client manifest early signs of DKA that include excessive thirst, frequent urination, headache, nausea and vomiting. Which result in dehydration and loss of electrolyte. The client should determine fingerstick glucose level and self-administer a dose of regular insulin per sliding scale.

After six days on a mechanical ventilator, a male client is extubated and place on 40% oxygen via face mask. He is awake and cooperative, but complaining of a severe sore throat. While sipping water to swallow a medication, the client begins coughing, as if strangled. What intervention is most important for the nurse to implement? a. Administer PRN medication b. Titrate the oxygen to keep saturation above 92% c. Hold oral intake until swallow evaluation is done. d. Elevate the head of his bed at least 45 degrees.

Hold oral intake until swallow evaluation is done. Rationale: After oral intubation, the client is at high risk for swallowing difficulties. A swallowing evaluation should be done to determine what consistency of liquids the client can tolerate without aspirating. A, B and D helps but does not have the priority.

The nurse should teach the parents of a 6 year-old recently diagnosed with asthma that the symptom of acute episode of asthma are due to which physiological response? Inflammation of the mucous membrane & bronchospasm

Inflammation of the mucous membrane & bronchospasm

During a left femoral artery aortogram, the healthcare provider inserts an arterial sheath and initiate...through the sheath to dissolve an occluded artery. Which interventions should the nurse implement? (SATA) Correct Answers: a. Instruct the client to keep the left leg straight b. Observe the insertion site for a hematoma c. Circle first noted drainage on the dressing

Instruct the client to keep the left leg straight Observe the insertion site for a hematoma Circle first noted drainage on the dressing

An elderly client with limited mobility reports frequent episodes of nocturia. To reduce the risk for urinary incontinence, what action should the nurse implement?

Keep the call bell button within the client's reach.

The nurse inserts an indwelling urinary catheter as seen in the video what action should the nurse take next? a. Remove the catheter and insert into urethral opening b. Observe for urine flow and then inflate the balloon. c. Insert the catheter further and observe for discomfort. d. Leave the catheter in place and obtain a sterile catheter.

Leave the catheter in place and obtain a sterile catheter.

A client with Alzheimer's disease falls in the bathroom. The nurse notifies the charge nurse and completes a fall follow-up assessment. What assessment finding warrants immediate intervention by the nurse? a. Urinary incontinence b. Left forearm hematoma c. Disorientation to surroundings d. Dislodge intravenous site

Left forearm hematoma Rationale: The left forearm hematoma may be indicative an injury, such as broken bone, that requires immediate intervention. A may be likely be due to the inability to use the toilet due to the fall. Disorientation is a common symptom of Alzheimer's disease. IV Dislodged is not an urgent concern.

A client with a history of cirrhosis and alcoholism is admitted with severe dyspnea and ascites. Which assessment finding warrants immediate intervention by the nurse? a. Jaundice skin tone b. Muffled heart sounds c. Pitting peripheral edema d. Bilateral scleral edema

Muffled heart sounds Rationale: Muffled heart sounds may indicative fluid build-up in the pericardium and is life- threatening. The other one is sign of end stage liver disease related to alcoholism but are not immediately life- threatening.

Which assessment finding indicates a client's readiness to leave the nursing unit for a Bronchoscopy?

On-call sedation administered.

A client on a long-term mental health unit repeatedly takes own pulse regardless of the circumstance. What action should the nurse implement? a. Overlook the client's behavior. b. Distract client to interfere with the ritual. c. Ask why the client checks the pulse. d. Hold client's hand to stop the behavior.

Overlook the client's behavior.

A client has an intravenous fluid infusing in the right forearm. To determine the client's distal pulse rate most accurately, which action should the nurse implement? a. Elevate the client's upper extremity before counting the pulse rate b. Auscultate directly below the IV site with a Doppler stethoscope c. Turn off the intravenous fluids that are infusing while counting the pulse. d. Palpate at the radial pulse site with the pads of two or three fingers.

Palpate at the radial pulse site with the pads of two or three fingers Rationale: The radial pulse is easily accessible and palpable unless an IV is placed at the client wrist. A may make the pulse more difficult to palpate B places the stethoscope over a vein rather than an artery and is unlikely to provide an accurate pulse rate. The pulse rate can be accurately counted without implementing.

At 1615, prior to ambulating a postoperative client for the first time, the nurse reviews the client's medical record. Based on date contained in the record, what action should the nurse take before assisting the client with ambulation: a. Remove sequential compression devices. b. Apply PRN oxygen per nasal cannula. c. Administer a PRN dose of an antipyretic. d. Reinforce the surgical wound dressing.

Remove sequential compression devices. Rationale: Sequential compression devices should be removed prior to ambulation and there is no indication that this action is contraindicated. The client's oxygen saturation levels have been within normal limits for the previous four hours, so supplemental oxygen is not warranted.

An older adult male is admitted with complications related to chronic obstructive pulmonary disease (COPD). He reports progressive dyspnea that worsens on exertion and his weakness has increased over the past month. The nurse notes that he has dependent edema in both lower legs. Based on these assessment findings, which dietary instruction should the nurse provide? a. Limit the intake of high calorie foods. b. Eat meals at the same time daily. c. Maintain a low protein diet. d. Restrict daily fluid intake.

Restrict daily fluid intake. Rationale: the client is exhibiting signs of Cor pulmonale, a complication of COPD that causes the right side of the heart to fail. Restricting fluid intake to 1000 to 2000 ml/day, eating a high-calorie diet at small frequent meals with foods that are high in protein and low in sodium can help relieve the edema and decrease workload on the right-side of the heart.

What is the most important symptom the nurse should monitor the client for while assisting with the insertion of a subclavian central venous catheter?

Shortness of breath.

When assessing a 6-month old infant, the nurse determines that the anterior fontanel is bulging. In which situation would this finding be most significant? a. Crying b. Straining on stool c. Vomiting d. Sitting upright.

Sitting upright. Rationale: The anterior fontanel closes at 9 months of age and may bulge when venous return is reduced from the head, but a bulging anterior fontanel is most significant if the infant is sitting up and may indicated an increase in cerebrospinal fluid. Activities that reduce venous return from the head, such as crying, a Valsalva maneuver, vomiting or a dependent position of the head, cause a normal transient increase in intracranial pressure.

A client who has localized eczematous eruptions on both hands is diagnosed as having contact dermatitis. What instructions should the nurse include on the clients discharge teaching plan?

Take prescribed antihistamine near bedtime.

While the nurse is inserting a nasogastric tube, the client becomes cyanotic. What intervention should the nurse implement?

Withdraw the nasogastric tube

An adult man reports that he recently experienced an episode of chest pressure and breathlessness when he was jogging in the neighborhood. He expresses concern because both of his deceased parents had heart disease and his father was a diabetic. He lives with his male partner, is a vegetarian, and takes atenolol which maintain his blood pressure at 138/74. Which risk factors should the nurse explore further with the client? Select all that apply a- History of hypertension. b- Homosexual lifestyle c- Vegetarian diet d- Excessive aerobic exercise e- Family heath history.

a- History of hypertension. e- Family heath history. Rationale: Based on the client's family history and medication for management of hypertension, the nurse should further explore these risk for ischemic heart disease.

A client with acute pancreatitis is complaining of pain and nausea. Which interventions should the nurse implement (Select all that apply) a- Monitor heart, lung, and kidney function. b- Notify healthcare provider of serum amylase and lipase levels. c- Position client on abdomen to provide organ stability d- Encourage an increased intake of clear oral fluids e- Review client's abdominal ultrasound findings.

a- Monitor heart, lung, and kidney function. b- Notify healthcare provider of serum amylase and lipase levels. e- Review client's abdominal ultrasound findings.

A primigravida client is 36 weeks gestation is admitted to labor and delivery unit because her membranes ruptured 30minutes ago. Initial assessment indicates 2cm dilation, 50% effaced, -2 station, vertex presentation greenish colored amniotic fluid, and contractions occurring 3-5 minutes with a low FHR after the last contraction peaks: a. Administer Oxygen via face mask b. Apply an internal fetal heart monitor c. Notify the healthcare provider d. Use a vibroacoustic stimulator

a. Administer Oxygen via face mask Rationale: The nurse should administer oxygen to increase the amount of oxygen available for the fetus, because is presenting characteristics of late decelerations, caused by uteroplacental insufficiency.

A client who had an open cholecystectomy two weeks ago comes to the emergency department with complaints of nausea, abdominal distention, and pain. Which assessment should the nurse implement? a. Auscultate all quadrants of the abdomen. b. Perform a digital rectal exam c. Palpate the liver and spleen d. Obtain a hemoccult of the client's stool

a. Auscultate all quadrants of the abdomen.

Following insertion of a LeVeen shunt in a client with cirrhosis of the liver, which assessment finding indicates to the nurse that the shunt is effective? a- Decreased abdominal girth b- Increased blood pressure c- Clear breath sounds d- Decrease serum albumin.

a. Decreased abdominal girth

The charge nurse of the Intensive Care Unit is making assignments for the permanent staff and one RN who was floated from a medical unit. The client with which condition is the best to assign to the float nurse? a. Diabetic ketoacidosis and titrated IV insulin infusion b. Emphysema extubated 3 hours ago receiving heated mist c. Subdural hematoma with an intracranial monitoring device d. Acute coronary syndrome treated with vasopressors

a. Diabetic ketoacidosis and titrated IV insulin infusion

The nurse is caring for a client who is experiencing a tonic-clonic seizure. Which actions should the nurse implement? (Select all that apply) a. Ease the client to the floor b. Loosen restrictive clothing c. Note the duration of the seizure

a. Ease the client to the floor b. Loosen restrictive clothing c. Note the duration of the seizure

Which assessment finding indicates to the nurse a client's readiness for pulmonary function tests? a- Expresses an understanding of the procedure. b- NPO for 6 hrs. c- No known drug allergies d- Intravenous access intact.

a. Expresses an understanding of the procedure

A client with type 2 diabetes mellitus is admitted for frequent hyperglycemic episodes and a glycosylated hemoglobin (HbA1c) of 10%. Insulin glargine 10 units subcutaneously once a day at bedtime and a sliding scale with insulin aspart q6h are prescribed. What action should the nurse include in this client's plan of care? a. Fingerstick glucose assessment q6h with meals b. Mix bedtime dose of insulin glargine with insulin aspart sliding scale dose c. Review with the client proper foot care and prevention of injury d. Do not contaminate the insulin aspart so that it is available for iv use e. Coordinate carbohydrate controlled meals at consistent times and intervals f. Teach subcutaneous injection technique, site rotation and insulin management

a. Fingerstick glucose assessment q6h with meals c. Review with the client proper foot care and prevention of injury e. Coordinate carbohydrate controlled meals at consistent times and intervals f. Teach subcutaneous injection technique, site rotation and insulin management

When obtaining a rectal temperature with an electronic thermometer, which action is most important for the nurse to perform? a. Hold the thermometer in place. b. Place the disposable pad under buttocks c. Instruct the client to breathe deeply d. Return the probe to the charger.

a. Hold the thermometer in place

A nurse is caring for a client with Diabetes Insipidus. Which assessment finding warrants immediate intervention by the nurse? a- Hypernatremia b- Excessive thirst c- Elevated heart rate d- Poor skin turgor

a. Hypernatremia Rationale: Hypernatremia can lead to neurological symptoms, such as overactivity in the brain and nerve muscles, confusion, seizures, or even coma. Without treatment, central diabetes inspidus can lead to permanent kidney damage.

The health care provider prescribes atenolol 50 mg daily for a client with angina pectoris...to the health care provider before administering this medication? a. Irregular pulse b. Tachycardia c. Chest pain d. Urinary frequency

a. Irregular pulse

An adult female client with chronic kidney disease (CKD) asks the nurse if she can continue...Medications. Which medication provides the greatest threat to this client? a. Magnesium hydroxide (Maalox). b. Birth control pills c. Cough syrup containing codeine d. Cold medication containing alcohol

a. Magnesium hydroxide (Maalox) Rationale: Some aluminum containing antacids (Maalox, for example) may cause Acute Kidney Failure if used over long periods of time. Antacids can also disrupt the electrolyte balance of people with CKD.

The nurse notes an increase in serosanguinous drainage from the abdominal surgical wound from an obese client. What action should the nurse implement? a. Observe the wound for dehiscence b. Teach the client to splint the incision while coughing c. Assess the skin surrounding the wound for maceration d. Obtain a culture of the wound drainage.

a. Observe the wound for dehiscence

The nurse observes an adolescent client prepare to administer a prescribed corticosteroid medication using a metered dose inhaler as seen in the picture. What action should the nurse take? a. Remind the client to hold his breath after inhaling the medication b. Confirm that the client has correctly shaken the inhaler c. Affirm that the client has correctly positioned the inhaler d. Ask the client if he has a spacer to use for this medication

a. Remind the client to hold his breath after inhaling the medication

A client with HIV and pulmonary coccidioidomycosis is receiving amphotericin B. which assessment finding should the nurse report to the healthcare provider? a- Urinary output of 25mL per hour b- Hemoglobin level of 10 g/dL or 100 g/L (S1) c- Hyperactive bowel sounds d- Oral temperature of 100.4 F (38 C)

a. Urinary output of 25mL per hour

When conducting diet teaching for a client who was diagnosed with hypoparathyroidism, which foods should the nurse encourage the client to eat? a- Nuts b- Yogurt. c- Fresh turkey d- Fresh chicken e- Processed cheese.

b- Yogurt. e- Processed cheese. Rationale: In hypoparathyroidism, the client's diet should be supplemented with calcium rich foods which include dairy products.

The nurse assesses a female client with obstructive sleep apnea syndrome (OSAS) who is 5 feet tall (152 cm) and weighs 155 pounds (70 kg), the client's 24 hour diet history includes: no breakfast, cheeseburger and fries for lunch; lasagna, chocolate ice cream and a cola drink for dinner, and 2 glasses of wine in the evening before going to bed for a total caloric intake of 3500 calories. What instructions should the nurse provide? (Select all that apply) a. Maintain current caloric intake b. Avoid use of alcohol as a sleep aide at bedtime c. Reduce intake of dairy products d. Start a weight loss program e. Set a goal of increasing BMI (Body Mass Index)

b. Avoid use of alcohol as a sleep aide at bedtime d. Start a weight loss program

The nurse is developing an educational program for older clients who are being discharged with new antihypertensive medications. The nurse should ensure that the educational materials include which characteristics? Select all that apply a. Written at a twelfth-grade reading level b. Contains a list with definitions of unfamiliar terms c. Uses common words with few Syllables d. Printed using a 12-point type font e. Uses pictures to help illustrate complex ideas

b. Contains a list with definitions of unfamiliar terms c. Uses common words with few Syllables e. Uses pictures to help illustrate complex ideas Rationale: During the aging process older clients often experience sensory or cognitive changes, such as decreased visual or hearing acuity, slower thought or reasoning processes, and shorter attention span. Materials for this age group should include at least of terms, such as a medical terminology that incline may not know and use common words that expresses information clearly and simply. Simple, attractive pictures help hold the learner's attention. The reading level of material should be at the 4th to 5th grade level. Materials should be printed using large font (18-point or higher), not the standard 12-point font.

The nurse plans to use an electronic digital scale to weight a client who is able to stand. Which intervention should the nurse implement to ensure that measurement of the client's weight is accurate? a. Ask the client to remove shoes before stepping on the scale b. Ensure that the scale is calibrated before a weight is obtained. c. Slide the balancing weights until the scale is at zero. d. Compare client's weight at various time of the day.

b. Ensure that the scale is calibrated before a weight is obtained

A male client with impaired renal function who takes ibuprofen daily for chronic arthritis...gastrointestinal (GI) bleeding. After administering IV fluids and a blood transfusion, his blood pressure is 100/70, and his renal output is 20 ml / hour. Which intervention should the nurse include in hours? a. Maintain the client NPO during the diuresis phase b. Evaluate daily serial renal laboratory studies for progressive elevations. c. Observe the urine character for sedimentation and cloudy appearance. d. Monitor for onset of polyuria greater than 150ml/hr.

b. Evaluate daily serial renal laboratory studies for progressive elevations.

The nurse observes an unlicensed assistive personnel (UAP) using an alcohol-based gel hand cleaner before performing catheter care ...tray to the room. The UAP rub both hands thoroughly for 2 minutes while standing at the...should the nurse take? a. Encourage the UAP to remain in the client's room, until completed b. Explain that the hand rub can be completed in less than 2 minutes. c. Inform the UAP that handwashing helps to promote better asepsis. d. Determine why the UAP was not wearing gloves in the client's room

b. Explain that the hand rub can be completed in less than 2 minutes

The nurse is interacting with a female client who is diagnosed with postpartum depression. Which finding should the nurse document as an objective signs of depression? (Select all that apply) a. Report feeling sad b. Interacts with a flat affect. c. Avoids eye contact. d. Has a disheveled appearance. e. Express suicidal thoughts.

b. Interacts with a flat affect. c. Avoids eye contact. d. Has a disheveled appearance. Rationale: Observed finding are objective and include the client's appearance, such as flat affect, lack of eyes contact, and disheveled appearance. A and E are subject only the client can express verbally.

During the transfer of a client who had major abdominal surgery this morning, the post anesthesia care unit (PACU) nurse reports that the client, who is awake and responsive continues to report pain and nausea after receiving morphine 2 mg IV and ondansetron 4 mg IV 45 mints ago. Which elements of SBAR communication are missing from the report given by the PACU nurse? (Select all that apply) a- Situation b- Background c- Assessment d- Recommendation e- Rationales.

c- Assessment d- Recommendation e- Rationales.

The nurse is assessing a first day postpartum client. Which finding is most indicative of a postpartum infection? a. White blood count of 19,000 mm3 b. Oral temperature of 100.2 F (37.9 C) c. Moderate amount of foul-smelling lochia. d. Blood pressure 122/74 mm Hg

c. Moderate amount of foul-smelling lochia.

The nurse makes a supervisory home visit to observe an unlicensed assistive personnel (UAP) who is providing personal care for a client with Alzheimer's disease. The nurse observes that whenever the client gets upset, the UAP changes the subject. What action should the nurse take in response to this observation? a. Tell the UAP to offer more choices during the personal care to prevent anxiety b. Meet with the UAP later to role model more assertive communication techniques c. Assume care of the client to ensure that effective communication is maintained. d. Affirm that the UAP is using an effective strategy to reduce the client's anxiety.

d. Affirm that the UAP is using an effective strategy to reduce the client's anxiety. Rationale: Reduction is an effective technique is managing the anxiety of client with Alzheimer's disease, so the nurse should affirm the UAP is using an effective strategy (A). Nurse assertive communication and offering more choices (B) may increase... an agitation (C) is not indicated since the UAP is using redirection, an effective strategy.

Oral antibiotics are prescribed for an 18-month-old toddler with severe otitis media. An antipyrine and benzocaine-otic also prescribed for pain and inflammation. What instruction should the nurse emphasize concerning the installation of the antipyrine/ benzocaine otic solution? a. Place the dropper on the upper outer ear canal and instill the medication slowly. b. Warm the medication in the microwave for 10 seconds before instilling. c. Keep the medication refrigerated between administrations. d. Have the child lie with the ear up for one to two minutes after installation.

d. Have the child lie with the ear up for one to two minutes after installation.

A male infant born at 28-weeks gestation at an outlying hospital is being prepared for transport to a respiration are 92 breaths/minute and his heart rate is 156 beats/minute. Which drug is the transport administration to this infant? a- Give ampicillin 25 mg/kg slow IV push b- Deliver 1:10,000 epinephrine 0.1 ml/kg per endotracheal tube c- Administer digoxin 20 mcg/kg IV d- Instill beractant 100 mg/kg in endotracheal tube.

d. Instill beractant 100 mg/kg in endotracheal tube

A client with anorexia is hospitalized on a medical unit due to electrolyte imbalance and cardiac dysrhythmias. Additional assessment findings that the nurse would expect to observe are A) Brittle hair, lanugo, amenorrhea B) Diarrhea, nausea, vomiting, dental erosion C) Hyperthermia, tachycardia, increased metabolic rate D) Excessive anxiety about symptoms

is A: Brittle hair, lanugo, amenorrhea 14

The mother of a 2 month-old baby calls the nurse 2 days after the first DTaP, IPV, Hepatitis B and HIB immunizations. She reports that the baby feels very warm, cries inconsolably for as long as 3 hours, and has had several shaking spells. In addition to referring her to the emergency room, the nurse should document the reaction on the baby's record and expect which immunization to be most associated to the findings in the infant? A) DTaP B) Hepatitis B C) Polio D) H. Influenza

is A: DTaP 5

The nurse is assigned to care for a client who had a myocardial infarction (MI) 2 days ago. The client has many questions about this condition. What area is a priority for the nurse to discuss at this time? A) Daily needs and concerns B) The overview cardiac rehabilitation C) Medication and diet guideline D) Activity and rest guidelines

is A: Daily needs and concerns 13

A client is admitted to the emergency room following an acute asthma attack. Which of the following assessments would be expected by the nurse? A) Diffuse expiratory wheezing B) Loose, productive cough C) No relief from inhalant D) Fever and chills

is A: Diffuse expiratory wheezing 3

A nurse assigned to a manipulative client for 5 days becomes aware of feelings for a reluctance to interact with the client. The next action by the nurse should be to A) Discuss the feeling of reluctance with an objective peer or supervisor B) Limit contacts with the client to avoid reinforcement of the manipulative behavior C) Confront the client about the negative effects of behaviors on other clients and staff D) Develop a behavior modification plan that will promote more functional behavior

is A: Discuss the feeling of reluctance with an objective peer or supervisor 2

Which statements by the client would indicate to the nurse an understanding of the issues with end stage renal disease? A) I have to go at intervals for epoetin (Procrit) injections at the health department. B) I know I have a high risk of clot formation since my blood is thick from too many red cells. C) I expect to have periods of little water with voiding and then sometimes to have a lot of water. D) My bones will be stronger with this disease since I will have higher calcium than normal.

is A: I have to go at intervals for epoetin (Procrit) injections at the health department. 15

The nurse is observing a client with an obsessive-compulsive disorder in an inpatient setting. Which behavior is consistent with this diagnosis? A) Repeatedly checking that the door is locked B) Verbalized suspicions about thefts C) Preference for consistent care givers D) Repetitive, involuntary movements

is A: Repeatedly checking that the door is locked 13

An 80 year-old client admitted with a diagnosis of possible cerebral vascular accident has had a blood pressure from 180/110 to 160/100 over the past 2 hours. The nurse has also noted increased lethargy. Which assessment finding should the nurse report immediately to the health care provider? A) Slurred speech B) Incontinence C) Muscle weakness D) Rapid pulse

is A: Slurred speech 4

The nurse admits a 7 year-old to the emergency room after a leg injury. The x-rays show a femur fracture near the epiphysis. The parents ask what will be the outcome of this injury. The appropriate response by the nurse should be which of these statements? A) "The injury is expected to heal quickly because of thin periosteum." B) "In some instances the result is a retarded bone growth." C) "Bone growth is stimulated in the affected leg." D) "This type of injury shows more rapid union than that of younger children."

is B: "In some instances the result is a retarded bone growth." 10

Which of these children at the site of a disaster at a child day care center would the triage nurse put in the "treat last" category? A) An infant with intermittent bulging anterior fontanel between crying episodes B) A toddler with severe deep abrasions over 98% of the body C) A preschooler with 1 lower leg fracture and the other leg with an upper leg fracture D) A school-age child with singed eyebrows and hair on the arms

is B: A toddler with severe deep abrasions over 98% of the body .

The nurse is teaching an elderly client how to use MDI's (multi-dose inhalers). The nurse is concerned that the client is unable to coordinate the release of the medication with the inhalation phase. What is the nurse's best recommendation to improve delivery of the medication? A) Nebulized treatments for home care B) Adding a spacer device to the MDI canister C) Asking a family member to assist the client with the MDI D) Request a visiting nurse to follow the client at home

is B: Adding a spacer device to the MDI canister 7

A nurse administers the influenza vaccine to a client in a clinic. Within 15 minutes after the immunization was given, the client complains of itchy and watery eyes, increased anxiety, and difficulty breathing. The nurse expects that the first action in the sequence of care for this client will be to A) Maintain the airway B) Administer epinephrine 1:1000 as ordered C) Monitor for hypotension with shock D) Administer diphenhydramine as ordered

is B: Administer epinephrine 1:1000 as ordered .

A 2 year-old child is brought to the health care provider's office with a chief complaint of mild diarrhea for 2 days. Nutritional counseling by the nurse should include which statement? A) Place the child on clear liquids and gelatin for 24 hours B) Continue with the regular diet and include oral rehydration fluids C) Give bananas, apples, rice and toast as tolerated D) Place NPO for 24 hours, then rehydrate with milk and water

is B: Continue with the regular diet and include oral rehydration fluids 7

The nurse is planning care for a 14 year-old client returning from scoliosis corrective surgery. Which of the following actions should receive priority in the plan? A) Antibiotic therapy for 10 days B) Teach client isometric exercises for legs C) Assess movement and sensation of extremities D) Assist to stand up at bedside within the first 24 hours

is C: Assess movement and sensation of extremities 9

A child is injured on the school playground and appears to have a fractured leg. The first action the school nurse should take is A) Call for emergency transport to the hospital B) Immobilize the limb and joints above and below the injury C) Assess the child and the extent of the injury D) Apply cold compresses to the injured area

is C: Assess the child and the extent of the injury 4

A 72 year-old client with osteomyelitis requires a 6 week course of intravenous antibiotics. In planning for home care, what is the most important action by the nurse? A) Investigating the client's insurance coverage for home IV antibiotic therapy B) Determining if there are adequate hand washing facilities in the home C) Assessing the client's ability to participate in self care and/or the reliability of a caregiver D) Selecting the appropriate venous access device

is C: Assessing the client''s ability to participate in self care and/or the reliability of a caregiver 4

Which activity can the RN ask an unlicensed assistive personnel (UAP) to perform? A) Take a history on a newly admitted client B) Adjust the rate of a gastric tube feeding C) Check the blood pressure of a 2 hours post operative client D) Check on a client receiving chemotherapy

is C: Check the blood pressure of a 2 hours post operative client 4

An 8 year-old child is hospitalized during the edema phase of minimal change nephrotic syndrome. The nurse is assisting in choosing the lunch menu. Which menu is the best choice? A) Bologna sandwich, pudding, milk B) Frankfurter, baked potato, milk C) Chicken strips, corn on the cob, milk D) Grilled cheese sandwich, apple, milk

is C: Chicken strips, corn on the cob, milk 9

The nurse is assessing a 17 year-old female client with bulimia. Which of the following laboratory reports would the nurse anticipate? A) Increased serum glucose B) Decreased albumin C) Decreased potassium D) Increased sodium retention

is C: Decreased potassium 11

Which of the following findings contraindicate the use of haloperidol (Haldol) and warrant withholding the dose? A) Drowsiness, lethargy, and inactivity B) Dry mouth, nasal congestion, and blurred vision C) Rash, blood dyscrasias, severe depression D) Hyperglycemia, weight gain, and edema

is C: Rash, blood dyscrasias, severe depression 9

A client diagnosed with hepatitis C discusses his health history with the admitting nurse. The nurse should recognize which statement by the client as the most important? A) I got back from Central America a few weeks ago. B) I had the best raw oysters last week. C) I have many different sex partners. D) I had a blood transfusion 15 years ago.

is D: I had a blood transfusion 8

During the initial home visit a nurse is discussing the care of a newly diagnosed client with Alzheimer's disease with family members. Which of these interventions would be most helpful at this time? A) Leave a book about relaxation techniques B) Write out a daily exercise routine for them to assist the client to do C) List actions to improve the client's daily nutritional intake D) Suggest communication strategies

is D: Suggest communication strategies 5

Education for splenomegaly

-Avoid heavy lifting/straining -Avoid contact sports -Cant go right back to the norm, unless they just sit at a desk all day -Wear a seatbelt in the car

Palliative care patient care

-Goal: improve quality of life, caring not curing -Still treat symptoms like pain/whatever impacts QOL -Still take meds for managing problems

Care and assessment for adrenalectomy

-Pt at risk for electrolyte abnormalities -Put on telemetry -Daily serum electrolytes -Monitor VS -Watch for dysrhythmias

The nurse is caring for a 5 year-old child with Reye's syndrome. What goal has the highest priority in caring for this child?

Reduce cerebral edema and lower intracranial pressure.

Grave's disease test

-Radioactive Iodine Uptake to detect how the thyroid is working -Ultrasound -Bloodwork -Imaging tests

Muslim Diet

-Ramadan: fast for one month each year from dawn till after dark -Respect this -No alcohol

Isolation

-Separation -Ex: ER nurse is able to care for pts by isolating feelings and emotions r/t pts injuries

Trouble shooting for pulse ox

-Warm up the extremity -Change site: nose, toes, earlobes

Aseptic handwashing

-Wash clean to dirty -Point arms down so dirty water runs from clean to dirty

Considerations when choosing an IV site

-Avoid restricted extremities (fistula, breast CA (mastectomy/lymphedema), cellulitis) -If no upper extremities available, can use legs -Infiltration is a problem, dont reinstill IV access; keep IV in but start a new line

Manual bladder irrigation and CBI

-CBI: 3 way catheter -Check for kinks if not flowing properly -Manually irrigate CBI if there's a clot -If a clot is blocking, the pressure in the bladder could cause leakage around the catheter site

Seizure in the community interventions

-Call for help -Turn on side -Move any objects surrounding them -Don't put anything in the mouth -Keep the patient safe

Seizure in the hospital interventions

-Call for help -Turn on side -O2 -Don't put anything in the mouth -Never restrain -Ativan IV to break the seizure -Pad siderails -Document time and length of seizure

Identification

-Attempt to be like someone else -Ex: teenager dresses like favorite singer

While caring for a client's postoperative dressing, the nurse observes purulent drainage at the wound. Before reporting this finding to the healthcare provider, the nurse should review which of the client's laboratory values? A. Serum albumin B. Culture for sensitive organisms C. Serum blood glucose level D. Creatinine level

B. Culture for sensitive organisms

A male client with cancer is admitted to the oncology unit and tells the nurse that he is in the hospital for palliative care measures. The nurse notes that the client's admission prescription include radiation therapy. What action should the nurse implement?

Consult with the client about his expected goals for his hospitalization and current treatment plan

The nurse is teaching a smoking cessation class and notices there are 2 pregnant women in the group. Which information is a priority for these women? A) Low tar cigarettes are less harmful during pregnancy B) There is a relationship between smoking and low birth weight C) The placenta serves as a barrier to nicotine D) Moderate smoking is effective in weight control

B) There is a relationship between smoking and low birth weight

One reason that domestic violence remains extensively undetected is A) Few battered victims seek medical care B) There is typically a series of minor, vague complaints C) Expenses due to police and court costs are prohibitive D) Very little knowledge is currently known about batterers and battering relationships

B) There is typically a series of minor, vague complaints

The nurse is evaluating a client's symptoms, and formulates the nursing diagnosis, "high risk for injury due to possible urinary tract infection." Which symptoms indicate the need for this diagnosis? a. Hematuria and proteinuria b. Azotemia and anorexia c. Fever and dysuria. d. Straining on urination and nocturia

Fever and dysuria.

The nurse receives a newborn within the first minutes after a vaginal delivery and intervenes to establish adequate respirations. What priority issue should the nurse address to ensure the newborn's survival? a. Hypoglycemia b. Fluid balance c. Heat loss d. Bleeding tendencies

Heat loss Rationale: Adequate thermoregulation is the nurse next priority. The newborn is at risk for significant heat loss due to a large surface area exposed to the environment, a thin layer of subcutaneous fat, and distribution of brow fat. Heat loss increases the neonate's metabolic pathway's utilization of oxygen and glucose.

A client with bipolar disorder began taking valproic acid (Depakote) 250 mg PO three times daily two months ago. Which finding provides the best indication that the medication regimen is effective? a. The nurse note that no pills remain in the prescription bottle. b. The client serum Depakote level is 125 mcg/ml c. The family reports a great reduction in client's maniac behavior d. The client denies any occurrence of suicidal ideation.

The family reports a great reduction in client's maniac behavior

Right sided heart failure symptoms

-Peripheral edema (legs and hands common) -Weight gain -JVD

During change of shift, the nurse reports that a male client who had abdominal surgery yesterday increasingly confused and disoriented during the night. He wandered into other clients rooms, saying that there are men in his room trying to hurt him. Because of continuing disorientation and the client's multiple attempts to get of bed, soft restrains were applied at 0400. In what order should the nurse who is receiving report implement these interventions? (Arrange from first action on top to last on the bottom) 1. Assess the client's skin and circulation for impairment related to the restrains 2. Evaluate the client's mentation to determine need to continue the restrains 3. Assign unlicensed assistive personnel to remove restrains and remain with client 4. Contact the client's surgeon and primary healthcare provider

1. Assess the client's skin and circulation for impairment related to the restrains 2. Evaluate the client's mentation to determine need to continue the restrains 3. Assign unlicensed assistive personnel to remove restrains and remain with client 4. Contact the client's surgeon and primary healthcare provide

Nurses working on a surgical unit are concerned about the physicians treatment of clients during invasive procedures, such as dressing changes and insertion of IV lines. Clients are often crying during the procedures, and the physician is usually unconcerned or annoyed by the client's response. To resolve this problem, what actions should the nurses take? (Arrange from the first action on the top of the list on the bottom) 1. Talk to the physician as a group in a non-confrontational manner. 2. Document concerns and report them to the charge nurse. 3. Submit a written report to the director of nursing. 4. Contact the hospital's chief of medical services. 5. File a formal complaint with the state medical board.

1. Talk to the physician as a group in a non-confrontational manner. 2. Document concerns and report them to the charge nurse. 3. Submit a written report to the director of nursing. 4. Contact the hospital's chief of medical services. 5. File a formal complaint with the state medical board. Rational: nurses have both an ethical and legal responsibility to advocate for clients' physical and emotional safety. Talking with the physician in a non-confrontational manner is the first step in conflict resolution. If this is not effective, the organizational chain of ineffective, a formal complaint with the state medical board should be implemented.

The healthcare provider prescribes penicillin G benzathine 2,400,000 units intramuscularly for a client who has a postoperative wound infection. The prefilled syringe is labeled, penicillin G benzathine 1,200,000 units/2mL. How many mL should the nurse administer to this client?

4mL

A client receives a new prescription for simvastatin (Zocor) 5 mg PO daily at bedtime. What action should the nurse take? a. Provide a bedtime snack to be eaten before taking the medication. b. Administer the medication as prescribed with a glass of water c. Contact the prescriber about changing the time of administration. d. Check the client's blood pressure prior to administering the med.

Administer the medication as prescribed with a glass of water Rationale: Simvastatin (Zocor), a HMG co-enzyme A reductase inhibitor, interferes with cholesterol synthesis pathway. Zocor can be taken at any time.

A client with a traumatic brain injury becomes progressively less responsive to stimuli. The client has a "Do Not Resuscitate" prescription, and the nurse observes that the unlicensed assistive personnel (UAP) has stopped turning the client from side to side as previously schedules. What action should the nurse take? a. Advise the UAP to resume positioning the client on schedule

Advise the UAP to resume positioning the client on schedule

A client at 38 weeks gestation is in active labor, and a vaginal birth after Cesarean section (VBAC) is planned. Vaginal exam indicates that the client is 6cm dilated, 90% effaced, and the station 0 with intact membranes. As the client's contraction become stronger, the fetal heart rate decelerates during the contraction but returns to baseline. What action should be taken?

Continue to monitor the client's labor progress.

After a sudden loss of consciousness, a female client is taken to the ED and initial assessment indicate that her blood glucose level is critically low. Once her glucose level is stabilized, the client reports that was recently diagnosed with anorexia nervosa and is being treated at an outpatient clinic. Which intervention is more important to include in this client's discharge plan? a. Describe the signs and symptoms of hypoglycemia. b. Encourage a low-carbohydrate and high-protein diet c. Reinforce the need to continue outpatient treatment d. Suggest wearing a medical alert bracelet at all time.

Encourage a low-carbohydrate and high-protein diet Rationale: A client with anorexia nervosa with long term starvation or who self-restrict intake can sign.... Reserves. Providing the client with dietary selections such as low-carbohydrate, high protein.... Hypoglycemic episodes, which can become life-threating.

The nurse ends the assessment of a client by performing a mental status exam. Which statement correctly describes the purpose of the mental status exam? a. Determine the client's level of emotional functioning' b. Assess functional ability of the primary support system. c. Evaluate the client's mood, cognition and orientation. d. Review the client's pattern of adaptive coping skill

Evaluate the client's mood, cognition and orientation. Rational: the mental status exam assesses the client for abnormalities in cognitive functioning; potential thought processes, mood and reasoning, the other options listed are all components of the client's psychosocial assessment.

The healthcare provider prescribes acarbose (Precose), an alpha-glucosidase inhibitor, for a client with Type 2 diabetes mellitus. Which information provides the best indicator of the drug's effectiveness? a. Body max index (BMI) between 20 and 24 b. Blood pressure reading less than 120/80 mm Hg c. Hemoglobin A1C (HbA1C) reading less than 7% d. Self-reported glucose levels of 120-150 mg/dl.

Hemoglobin A1C (HbA1C) reading less than 7% Rationale: Acarbose (Precose) delays carbohydrate absorption in the GI tract and causes the blood glucose to rise slowly after a meal. The best indicator of acarbose effectiveness is a serum hemoglobin A1 no greater than 7%, an indication of glucose level over time. Acarbose has no effect on pain or blood pressure. Self-reported glucose levels of 120-150 reflect the blood sugar at the time taken and are not the best indicator of drug effectiveness.

After administering the first dose of newly prescribed medications to four clients within a thirty minute time frame, the nurse evaluates each client for therapeutic responses or any adverse reactions. Which medication should the nurse evaluate first?

Hydromorphone (Dilaudid)

An older adult female admitted to the intensive care unit (ICU) with a possible stroke is intubated with ventilator setting of tidal volume 600, PlO2 40%, and respiratory rate of 12 breaths/minute. The arterial blood gas (ABG) results after intubation are PH 7.31. PaCO2 60, PaO2 104, SPO2 98%, HCO3 23. To normalize the client's ABG finding, which action is required? a. Report the results to the healthcare provider. b. Increase ventilator rate. c. Administer a dose of sodium carbonate. d. Decrease the flow rate of oxygen.

Increase ventilator rate. Rationale: This client is experience respiratory acidosis. Increasing the ventilator rate depletes CO2 a, which returns the PH toward normal. Report findings is important but only after increasing ventilator rate

An adult is admitted to the emergency department following ingestion of a bottle of antidepressants secondary to chronic paint. A nasogastric tube and a left subclavian venous catheter are placed. The nurse auscultates audible breath sounds on the right side, faint sounds procedure should the nurse prepare for first? a. Insertion of a left- sided chest tube.

Insertion of a left- sided chest tube.

A male client who had a small bowel resection acquired methicillin- resistant Staphylococcus aureus (MRSA) while hospitalized. He was treated and released, but is readmitted today because of diarrhea and dehydration. It is most important for the nurse to implement which intervention? a. Maintain contact transmission precautions

Maintain contact transmission precautions

A client with acute renal failure (ARF) is admitted for uncontrolled type 1 diabetes Mellitus and hyperkalemia. The nurse administers an IV dose of regular insulin per sliding scale. Which intervention is the most important for the nurse to include in this client's plan of care? a. Monitor the client's cardiac activity via telemetry. b. Maintain venous access with an infusion of normal saline. c. Assess glucose via fingerstick q4 to 6 hours. d. Evaluate hourly urine output for return of normal renal function

Monitor the client's cardiac activity via telemetry. Rational: as insulin lowers the blood glucose of a client with diabetic ketoacidosis (DKA), potassium returns to the cell but may not impact hyperkalemia related to acute renal failure. The priority is to monitor the client for cardiac dysrhythmias related to abnormal serum potassium levels. IV access, assessment of glucose level, and monitoring urine output are important interventions, but do not have the priority of monitoring cardiac function.

A male client with an antisocial personality disorder is admitted to an in-patient mental health unit for multiple substance dependency. When providing a history, the client justifies to the nurse his use of illicit drugs. Based on this pattern of behavior this client's history is most likely to include which finding? a. Multiple convictions for misdemeanors and class B felonies.

Multiple convictions for misdemeanors and class B felonies.

In early septic shock states, what is the primary cause of hypotension? a. Peripheral vasoconstriction b. Peripheral vasodilation c. Cardiac failure d. A vagal response

Peripheral vasodilation Rationale: Toxins released by bacteria in septic shock create massive peripheral vasodilation and increase microvascular permeability at the site of the bacterial invasion.

The nurse is preparing to gavage feed a premature infant through an orogastric tube. During insertion of the tube, the infant's heart rate drops to 60 beats/minute. Which action should the nurse take? a. Postpone the feeding until the infant's vital signs and stable

Postpone the feeding until the infant's vital signs and stable

Which intervention is most important for the nurse to include in the plan of care for an older woman with osteoporosis? a. Evaluate the client's orientation to time and place b. Place the client on fall precautions c. Encourage the client to drink milk with meals d. Assess the client's breath sounds daily.

Place the client on fall precautions Rationale: Osteoporosis causes bone to become brittle, fragile and less dense with age, which increases an older client's risk for falls and fractures which increases their risk for another pathology.

Within four weeks of childbirth, a client is admitted to the hospital for disorganized speech bizarre behavior, and strange thoughts about her infant being possessed by demons. The nurse identifies a nursing diagnosis of "Altered thought processes, secondary to" what condition?

Postpartum psychosis

When preparing a client for discharge from the hospital following a cystectomy and a urinary diversion to treat bladder cancer, which instruction is most important for the nurse to include in the client's discharge teaching plan? a. Report any signs of cloudy urine output. b. Seek counseling for body image concerns c. Follow instruction for self-care toileting d. Frequently empty bladder to avoid distension.

Report any signs of cloudy urine output. Rationale Infection can be life-threatening and cloudy urine output is a sign of urinary tract infection, which should be reported immediately.

A client is admitted to a mental health unit after attempting suicide by taking a handful of medications. In developing a plan of care for this client, which goal has the highest priority? a. Signs a no-self-harm contract.

Signs a no-self-harm contract.

A school nurse is called to the soccer field because a child has a nose bleed (epistaxis). In what position should the nurse place the child Sitting up and leaning forward

Sitting up and leaning forward

The nursing staff on a medical unit includes a registered nurse (RN), practical nurse (LPN), and unlicensed assistant personnel (UAP). Which task should the charge nurse assign to the RN? a. Supervised a newly hired graduate nurse during an admission assessment

Supervise a newly hired graduate nurse during an admission assessment

One year after being discharged from the burn trauma unit, a client with a history of 40% full-thickness burns is admitted with bone pain and muscle weakness. Which intervention should the nurse include in the clients plan of care a. Encourage Progressive active range of motion b. Teach need for dietary and supplementary vitamin D3 c. Explain the need for skin exposure to sunlight without sunscreen d. Instruct the client to use of muscle strengthening exercises

Teach need for dietary and supplementary vitamin D3 Rationale: Burn injury results in the acute loss of bone as well as the development of progressive vitamin D deficiency because burn scar tissue and adjacent normal-appearing skin cannot convert normal quantities of the precursors for vitamin D3 that is synthesized from ultraviolet sun rays which is needed for strong bones. Clients with a history of full thickness burns should increase their dietary resources of vitamin D and supplemental D3 (B). range of motion (A) and muscle strengthening exercises (D) do not treat he is underlying causes of the bone pain and weakness unprotected sunlight (C) should be avoided.

The nurse plans to collect a 24- hour urine specimen for a creatinine clearance test. Which instruction should the nurse provide to the adult male client? a. Clearance around the meatus, discard first portion of voiding, and collect the rest in a sterile bottle b. Urinate at specific time, discard the urine, and collect all subsequent urine during the next 24 hours. c. For the next 24 hours, notify the nurse when the bladder is full, and the nurse will collect catheterized specimens. d. Urinate immediately into a urinal, and the lab will collect specimen every 6 hours, for the next 24 hours.

Urinate at specific time, discard the urine, and collect all subsequent urine during the next 24 hours. Rationale: Urinate at specific time, discard the urine, and collect all subsequent urine during the next 24 hours is the correct procedure for collecting 24-hour urine specimen. Discarding even one voided specimen invalidate the test.

In caring for a client receiving the aminoglycoside antibiotic gentamicin, it is most important for the nurse to monitor which diagnostic test? a- Urinalysis b- Serum creatinine c- Serum osmolarity d- Liver enzymes.

b- Serum creatinine Rationale: Aminoglycosides can cause nephrotoxicity, so it is important for the nurse to monitor the serum creatinine level which can monitor the renal function.

Child w/ hemophila

have parent pad the corners of the furniture due to bleeding.

A 65-year-old Hispanic-Latino client with prostate cancer rates his pain as a 6 on a 0- to-10 scale. The client refuses all pain medication other than Motrin, which does not relieve his pain. The next action for the nurse to take is to A) Ask the client about the refusal of certain pain medications B) Talk with the client's family about the situation C) Report the situation to the health care provider D) Document the situation in the notes

A) Ask the client about the refusal of certain pain medications

A young adult visits the client reporting symptoms associated with gastritis. Which information in the client's history is most important for the nurse to address in the teaching plan? A. Consumes 10 or more drinks of alcohol every weekend B. Snacks on foods with very high salt content on a daily basis C. Exercises vigorously every evening right before going to bed D. Recently became a vegetarian and eats a lot of high fiber foods

A. Consumes 10 or more drinks of alcohol every weekend

A nurse-manager is preparing an annual budget for the unit. Nursing salaries should be included in which component of the budget?

Operating budget

A neonate born 12 hours ago to a methadone maintained woman is exhibiting a hyperactive MORO reflex and slight tremors. The newborn passes loose, watery stool. Which of these is a nursing priority? A) Hold the infant at frequent intervals. B) Assess for neonatal withdrawal syndrome C) Offer fluids to prevent dehydration D) Administer paregoric to stop diarrhea

B) Assess for neonatal withdrawal syndrome

After administering a proton pump inhibitor (PPI), which action should the nurse take to evaluate the effectiveness of the medication? A. Auscultate for bowel sounds in all quadrants B. Ask the client about gastrointestinal pain C. Monitor the client's serum electrolyte levels D. Measure the client's fluid intake and output

B. Ask the client about gastrointestinal pain

A client with multiple sclerosis (MS) is admitted to the medical unit. The client reports...which action should the nurse implement to reduce the client's risk for falls? Select all that apply. (SATA) A.)Assign the client a wheel chair B.)The utilization of crutches C.)Schedule frequent rest periods D.)Provide assistance to bedside commode E.)Teach to patch one eye when ambulating

C.) Schedule frequent rest periods D.) Provide assistance to bedside commode E.) Teach to patch one eye when ambulating

6. While assisting a postpartum client with perineal care, the nurse notes that her vaginal bleeding spurts rather than trickles from the vagina. The uterine fundus is firm and the client's vital signs are pulse 88 beats/minute; respiratory rate, 21 breaths/minute; and blood pressure, 104/68 mmHg. What action should the nurse take next?

Compare current vital signs with previous vital signs

The nurse is caring for a client following a myelogram. Which assessment finding should the nurse report to the healthcare provider immediately? a. Complaint of headaches and stiff neck

Complain of headaches and stiff neck

Suicide precautions are initiated for a child admitted to the mental health unit following an intentional narcotic overdose. After a visitor leaves, the nurse finds a package of cigarettes in the client's room. Which intervention is most important for the nurse to implement? a. Assign a sitter for constant observation b. Screen future visitors for contraband c. Document suicide monitoring frequently d. Remove cigarettes for the client's room.

Remove cigarettes for the client's room Rationale: Safety is the priority, and any items that might cause self-harm, such as cigarettes should be removed immediately to create a safe environment.

During shift report, the central electrocardiogram (EKG) monitoring system alarms. Which client alarm should the nurse investigate first? a. Respiratory apnea of 30 seconds b. Oxygen saturation rate of 88% c. Eight premature ventricular beats every minute d. Disconnected monitor signal for the last 6 minutes.

Respiratory apnea of 30 seconds Rationale: The priority is the client whose alarm indicating respiratory apnea that should be assessed first.

The nurse is preparing to administer an IV dose of ciprofloxacin to a client with urinary tract infection. Which client data requires the most immediate intervention by the nurse? a. Serum creatinine of 4.5 mg/dl (398 mcmol/L SI) Rationale: The typical range for serum creatinine is: For adult men, 0.74 to 1.35 mg/dL. For adult women, 0.59 to 1.04 mg/dL.

Serum creatinine of 4.5 mg/dl (398 mcmol/L SI)

The RN is assigned to care for four surgical clients. After receiving report, which client should the nurse see first? The client who is: a. Two days postoperative bladder surgery with continuous bladder irrigation infusing. b. One day postoperative laparoscopic cholecystectomy requesting pain medication. c. Three days postoperative colon resection receiving transfusion of packed RBCs. d. Preoperative, in buck's traction, and scheduled for hip arthroplasty within the next 12 hours.

Three days postoperative colon resection receiving transfusion of packed RBCs.

To obtain an estimate of a client's systolic B/P. What action should the nurse take first? a. Palpate the client's brachial pulse b. Pump up the blood pressure cuff c. Position the stethoscope diaphragm d. Release the blood pressure cuff valve

a. Palpate the client's brachial pulse

Which interventions should the nurse include in a long-term plan of care for a client with COPD? a- Reduce risk factors for infection b- Administer high flow oxygen during sleep c- Limit fluid intake to reduce secretions d- Use diaphragmatic breathing to achieve better exhalation

a. Reduce risk factors for infection Rationale: Interventions aimed at reducing the risk factors of infections should be included in the plan of care COPD client are at particular risk for respiratory infection. Prevention and early detection of infections are necessary

A client, recovering from alcoholism, asks the nurse, "What can I do when I start recognizing relapse triggers within myself?" How might the nurse best respond? A) "When you have the impulse to stop in a bar, contact a sober friend and talk with him." B) "Go to an AA meeting when you feel the urge to drink." C) "It is important to exercise daily and get involved in activities that will cause you not to think about drug use." D) "Identify your relapse triggers as part of getting better."

is D: "Identify your relapse triggers as part of getting better." 14

A client with paranoid delusions stares at the nurse over a period of several days. The client suddenly walks up to the nurse and shouts "You think you're so perfect and pure and good." An appropriate response for the nurse is A) "Is that why you've been starring at me?" B) "You seem to be in a really bad mood." C) "Perfect? I don't quite understand." D) "You are angry right now."

is D: "You are angry right now." 14

A client has an order for 1000 ml of D5W over an 8 hour period. The nurse discovers that 800 ml has been infused after 4 hours. What is the priority nursing action? A) Ask the client if there are any breathing problems B) Have the client void as much as possible C) Check the vital signs D) Auscultate the lungs

is D: Auscultate the lungs

MRSA Precautions

-Precautions if they've already had MRSA -Contact isolation -Single rm or cohorted

A 10 year old who has terminal brain cancer asks the nurse, "What will happen to my body when I die?" How should the nurse respond? a. "Your mother & father will be here soon. Talk to them about that." b. "Why do you want to know about what will happen to your body when you die?" c. "The heart will stop beating & you will stop breathing." d. "Are you concerned about where your spirit will go?"

"The heart will stop beating & you will stop breathing."

UTI

-Pt doesn't always report sx! -Urine sample is foul smelling and cloudy -Send sample to lab for urinalysis -Order a secondary test (culture and sensitivity) to choose the correct tx

A female nurse who took drugs from the unit for personal use was temporarily released from duty. After completion of mandatory counseling, the nurse has asked administration to allow her to return to work. When the nurse administrator approaches the charge nurse with the impaired nurse request, which action is best for the charge nurse to take? a. Since treatment is completed, assign the nurse to the routine RN responsibilities b. Ask to meet with impaired nurse's therapist before allowing her back on the unit. c. Allow the impaired nurse to return to work and monitor medication administration d. Meet with staff to assess their feelings about the impaired nurse's return to the unit.

Allow the impaired nurse to return to work and monitor medication administration

Splint

Chest compression or chest splint after surgery - they used it for comfort

After administering a medication through a nasogastric tube connected to suction, what action should the nurse take first?

Clamp the tube

The nurse is changing a client's IV tubing and closes the roller clamp on the new tubing setup when the bag of solution is _____. Which action should the nurse take to ensure adequate filling of the drip chamber? a. Lower the IV bag to a flat surface b. Compress the drip chamber c. Open the roller clamp d. Squeeze the bag of IV solution

Compress the drip chamber

A nurse who is new to the pediatric unit is positioning a 6 month-old for an injection of penicillin V (Pen V) in the dorsogluteal muscle. Which action should the nurse-manager who is supervising this nurse take first?

Instruct the nurse to select another injection site.

A male client with cancer who has lost 10 pounds during the last months tells the nurse that beef, chicken, and eggs, which used to be his favorite foods, now they taste "bitter". He complains that he simply has no appetite. What action should the nurse implement? a. Suggest the use of alternative sources of protein such as dairy products and nuts

Suggest the use of alternative sources of protein such as dairy products and nuts

The healthcare provider prescribes the antibiotic Cefdinir (cephalosporin) 300mg PO every 12 h for a client with postoperative wound infections. Which foods should the nurse encourage this client to eat? a. Yogurt and/or buttermilk. b. Avocados and cheese c. Green leafy vegetables d. Fresh fruits

Yogurt and/or buttermilk. Rationale: A should be encouraging to help maintain intestinal flora and decrease diarrhea, which is a common side effect of antibiotic therapy, particularly cephradine. B and C are contraindicated because they can increase bowel elimination, thereby exacerbating diarrhea as a side effect.

The nurse requests a meals tray for a client follows Mormon beliefs and who is on clear liquid diet following abdominal surgery. Which meal item should the nurse request for this client? (Select all that apply) a- Apple juice b- Chicken broth. c- Hot chocolate d- Orange juice e- Black coffee

a- Apple juice b- Chicken broth.

During a left femoral artery aortogram, the healthcare provider inserts an arterial sheath and initiate. Through the sheath to dissolve an occluded artery. Which interventions should the nurse implement? a- Instruct the client to keep the left leg straight b- Keep the head of bed at 60-degree angle. c- Observe the insertion site for a hematoma d- Manually flush the arterial sheath hourly e- Circle first noted drainage on the dressing

a- Instruct the client to keep the left leg straight c- Observe the insertion site for a hematoma e- Circle first noted drainage on the dressing

What is the primary goal when planning nursing care for a client with degenerative joint disease (DJD)? a. Obtain adequate rest and sleep b. Achieve satisfactory pain control. c. Improve stress management skills d. Reduce risk for infection.

b. Achieve satisfactory pain control.

The nurse is assessing an older adult with type 2 diabetes mellitus. Which assessment finding indicates that the client understands long- term control of diabetes? a. The fasting blood sugar was 120 mg/dl this morning. b. Urine ketones have been negative for the past 6 months c. The hemoglobin A1C was 6.5g/100 ml last week d. No diabetic ketoacidosis has occurred in 6 months.

c. The hemoglobin A1C was 6.5g/100 ml last week Rationale: A hemoglobin A1C level reflects he average blood sugar the client had over the previous 2 to 3 month, and level of 6.5 g/100 ml suggest that the client understand long-term diabetes control. Normal value in a diabetic patient is up to 6.5 g/100 ml.

Chron's disease

decrease intestinal activity

When providing diet teaching for a client with cholecystitis, which types of food choices the nurse recommend to the client? a. High protein b. Low fat c. Low sodium d. High carbohydrate.

low fat Rationale: A client with cholecystitis is at risk of gall stones that can be move into the biliary tract and cause pain or obstruction. Reducing dietary fat decrease stimulation of the gall bladder, so bile can be expelled, along with possible stones, into the biliary tract and small intestine.

An Elderly man is having trouble urinating and unsteady he uses a wheelchair to walk to the bathroom what should the nurse implement?

provide a raised toilet seat

A nurse working on a mental health unit where a diagnosis of bulimia nervosa is being admitted. Which of the following symptoms would the nurse most likely NOT expect to find with this client?

severe over-or under-weight

Know how the chest tube vac machine works - there is a picture of a chest tube vac machine.

this q is very easy just mark the spot on the tube for the amount of 125 ml of drainage

Influences on blood sugar levels

-Illness/infection: check glucose more frequently -Check sugar levels before/after meals and at night -Exercise can decrease sugar: monitor before

Stapedectomy pt teaching

-Improvement of hearing isn't immediate, can initially be worse -Hearing improves in 3 weeks, maximum hearing takes 6 months -Dressing on the ear

Morbidity

-Incidence of a disease across a population during a single year

Introjection

-Incorporation of values or qualities of an admired person/group into one's own ego -Ex: young man deal with business in same fashion father did

Cushing's triad

-Increase in systolic BP -Increase in pulse pressure (systolic and diastolic numbers get further apart) -Decrease in HR -Late sign -Seen with increased ICP -Adverse effect of prednisone

Passive-aggression

-Indirectly expressing aggression toward others -Ex: employee arrives late and disrupts others after being reminded of the meeting earlier that day and promising to be on time

Ecoli investigation

-Infected from eating/drinking something contaminated with animal/human fecal matter -Look at method of storage -Temperature of storage

How RN would handle a safety concern reported by a team member

-Intervene if there's a concern for immediate danger (ex : doing CPR wrong) -Nurse should assess/address any equipment not working (ex: hoyer lift wont work, go look at it before calling maintenance)

Left sided heart failure symptoms

-Left = lungs (and heart) -Crackles, increased HR, SOB, palpitations, dizzy, lightheaded, confused, restless, cough, dyspnea

Assessment of pt on a ventilator

-Listen to lungs -Possible reasons for alarms: increased/decreased pressure in lungs, secretions, kinked tubing, pain/anxiety bc sedation is wearing off, biting on tube (sign of resistance/sedating wearing), increased HR

Assessment of postpartum pt

-Massage the fundus -Soft or boggy = concern, massage -If fundus is off to the side, empty bladder -Fundus should be midline -Assess blood: saturating a pad in an hour is too much

Considerations for diabetic mother and baby

-Monitor glucose regularly -Before birth, monitor mom more -After birth, monitor baby more -Baby has higher risk for hypoglycemia; check regularly bc sx are hard to recognize

Enema administration and nursing considerations

-Monitor pt for cramping and abd pain -Coach and support pt through this pain -Need to get procedure done, take break, deep breaths, then continue

Teaching points for multiple sclerosis

-Need more rest periods -They will have periods of exacerbation -Be aware of triggers: infection, trauma, immunization, childbirth, stress, change in climate

Heart Failure Diet

-No canned/processed food -No cheese or salami -Limit sodium (fluid retention) -Vegetables and fruits

Vaginal laceration care plan considerations

-No straining, stool softeners -Ice packs to minimize edema -Sitz baths -Stitches

Neuropathic pain

-Numbing, burning, shooting, stabbing, sharp pain -Caused by trauma, inflammation, infection (herpes zoster, HIV), diabetes -Get a good pain assessment

Pt receiving blood

-Nurse takes vitals for first 15-30 min, then can delegate to UAP -Nurse documents final vitals -VS: watch for significant changes -No specific diet

Diagnostics helpful in mental health assessment

-O2 levels -Glucose -Blood test -Xray, CT scan -Ammonia levels (increased causes confusion) -Find out if this is a mental health problem or from an outside problem -Assess current medications (urine test, blood test) -Assess use of alcohol (breathalyzer)

Myxedema Coma

-Occurs with long standing hypothyroidism -Need to give IV thyroid hormone (levothyroxine)

Rationalization

-Offering an acceptable, logical explanation for unacceptable feelings/behavior -Ex: Student did bad on test and says the course was poorly taught

Stapedectomy

-Performed for otosclerosis -Incision behind inner ear, stapes removed and implant inserted -Allows sound and vibrations to pass from eardrum to inner ear fluids

Magnesium Sulfate

-Pharmacological treatment for preterm labor -Therapeutic level is 4-7.5 -Watch for adverse effects: loss of DTR, resp rate <12, UO <30 ml/hr -Anecdote = calcium gluconate

Education late in pregnancy

-Prepare for childbirth -Low heeled shoes, no heavy lifting -Tour L&D area -Parent orientation, take classes for delivery

Dealing with oppositional behavior in teens/adolescents

-Present a cause/effect of how something will impact their life -Discuss specific behaviors -Ex: if you do this then you'll get in trouble and wont be able to play sports

What to do if there's a fire

-RACE: rescue, alarm, confine, evacuate -Close the pt doors (containment)

Herpes Zoster

-Rash along the nerves of the side of the body (usually one side) -Very painful

Mortality

-Rate of death in a population

Secondary interventions

-Reduce a problem that already exists -Screenings

Fat embolism symptoms

-SOB -Confusion -Anxious/restless

Emphysema treatment

-Sit pt up in bed -Pursed lip breathing doesn't make the pt work less -Need to rest...might need bipap

Nursing interventions for a pt with COPD

-Sit upright (positioning) -Pursed lip breathing -Diaphragmatic breathing -HOB up -Not too much O2: decreases stimulus to breathe -O2 is not first choice!

How cultural characteristics can affect mental heath assessment

-Some cultures act differently and this is their norm -Interpretations may not be accurate -Gender comes into play (female being assessed by male nurse)

Sublimation

-Substitution of an acceptable feeling by a more socially acceptable one -Ex: a student who feels to small to play football becomes a marathon swimmer; pt takes unacceptable feelings and takes them out during contact sports

Viral meningitis

-Supportive care, rest, fluids -Sx: fever, lethargy -Biggest risk factor: being exposed to other viruses/having other infection -Vaccinations decrease the risk of getting this

General teaching points for pt taking meds

-Take the full dose of antibiotics (can build resistance) -Pay attention to timing and whether you take with food or on an empty stomach

Job responsibilities only LPN can do

-Take the more stable patients -Insert NG tubes, foleys, IVs -Wound care

Actions of a supervisor when an employer has poor behavior

-Talk to employer 1:1 -Document the incident (time, day, exactly what happened)

What to do if there's a power outage

-Tell pts to stay in their rooms and that you'll be around to check up on them

Teaching considerations for adolescent pt

-They're concerned about being with friends -Can teach 2 pts same age at the same time -Involve friends in treatment/education

Herniated disc

-Thorough pain assessment -Assess quality of pain -Radiating pain and numbness bc the bulge is pressing on nerves

Dealing with oppositional behavior in young children

-Time out and rewards -Reinforcement of rules -Behavioral modification

Displacement

-Transference of feelings to another group/object -Ex: after being yelled at by supervisor, person comes home and kicks dog for barking

Von Willebrand's disease treatment

-Treatment: stopping/preventing bleeding with meds/apply pressure to bleeding -Educate about preventing injury

Herpes Zoster treatment

-Try to stop the replication of the virus; can still have symptoms -Pain meds throughout: narcotics

Pheochromocytoma

-Tumor on the adrenals that causes release of catecholamines (epi and norepi) -Can cause hypertensive crisis (above 240) -Monitor BP frequently -Mosly cardio sx: increased HR and BP

Intellectualization

-Using reason to avoid emotional conflicts -Ex: wife of substance abuser describes the dynamics of enabling behavior yet continues to call her husband's work and report he was missing bc of illness

Job responsibilities only UAP can do

-Vitals -Can do home visits to help a pt shower -No dressing changes

Risk for adrenal crisis

-When pt with congenital adrenal hyperplasia becomes symptomatic -Stopping steroid treatment abruptly

After teaching a male client with chronic kidney disease (CKD) about therapeutic diet... which menu of foods indicates that the teaching was effective? Select all that apply a- A slice of whole grain toast b- Half cup of black beans c- A ham and cheese sandwich d- A bowl of cream of wheat e- Two bananas.

a- A slice of whole grain toast d- A bowl of cream of wheat Rationale: Patient with CKD have elevated serum potassium, sodium and protein levels. A and D are low in potassium, sodium and protein, Beans are rich in proteins. C are high in sodium and potassium and E are rich in potassium.

The nurse is preparing to discharge an older adult female client who is at risk for hyPOCALCEMIA nurse include with this client's discharge teaching? (SATA) a- Report any muscle twitching or seizures b- Take vitamin D with calcium daily c- Avoid seafood, particularly shellfish d- Low fat yogurt is a good source of calcium e- Keep a diet record to monitor calcium intake

a- Report any muscle twitching or seizures b- Take vitamin D with calcium daily d- Low fat yogurt is a good source of calcium e- Keep a diet record to monitor calcium intake Rationale: Twitching and seizure are signs of low calcium. (A) Vit D supplement with calcium to enhance calcium absorption, especially in older adults. Dairy product should be included in the diet. Keeping a food record is a good healthcare practice. Foods high in calcium are recommended to maintain normal calcium level and it is important to verify if the client has allergy to shellfish.

A school-aged child was recently diagnosed with celiac disease. Which instruction should the nurse give the classroom teacher? a- The child should avoid eating homemade cookies and cupcakes during parties. b- No products containing any form of peanuts should be allowed in the classroom c- Report a runny nose or head cold to the nurse immediately for further revaluation. d- Avoiding direct contact sports and games will reduce the child's risk of bruising.

a- The child should avoid eating homemade cookies and cupcakes during parties. Rationale: Celiac disease is an intolerance to products containing gluten. Wheat flour,

The husband of a client with advanced ovarian cancer wants his wife to have every treatment available. When the husband leaves, the client tells the nurse that she has had enough chemotherapy and wants to stop all treatments but knows her husband will sign the consent form for more treatment. The nurse's response should include which information? a- The husband cannot sign the consent for the client, her signature is required b- The client's specific wishes should be discussed with her healthcare provider c- Counseling should be sought to resolve the husband's desire to control his wife d- The healthcare team will formulate a plan of care to keep the client comfortable e- The client should seek a second medical opinion before deciding to stop treatment.

a- The husband cannot sign the consent for the client, her signature is required b- The client's specific wishes should be discussed with her healthcare provider d- The healthcare team will formulate a plan of care to keep the client comfortable Rationale: An adult client who is mentally competent has the autonomy and the client's right to make her own decision regarding her treatment.

Client skin integrity for client with chronic venous insufficiency

around the ankle with little pain.

An adult female client is admitted to the psychiatric unit with a diagnosis of major depressive...medication therapy, the nurse notices the client has more energy, is giving her belongings...mood. Which intervention is best for the nurse to implement? a- Support the client by telling her what wonderful progress she is making. b- Ask the client if she has had any recent thoughts of harming herself. c- Reassure the client that the antidepressant drugs are apparently effective d- Tell the client to keep her belongings because she will need hem at discharge.

b- Ask the client if she has had any recent thoughts of harming herself.

The nurse is teaching a client with atrial fibrillation about a newly prescribed medication, dronedarone. Which information should the nurse include in client interactions? (Select all that apply) a- Discontinue medication when palpitation subside. b- Avoid eating grapefruit or drinking grapefruit juice. c- Report changes in the use of daily supplements d- Notify your health care provider if your skin looks yellow e- If a dose is missed, the next dose should be double.

b- Avoid eating grapefruit or drinking grapefruit juice. c- Report changes in the use of daily supplements d- Notify your health care provider if your skin looks yellow Rationale: Side effects can increase if the client consumes grapefruit. OTC medications or herbals should be reported for possible drug interactions. Hepatic injury can occur, and the client should report sign of jaundice or itching, or right upper quadrant pain.

The nurse weighs a 6-month-old infant during a well-baby check-up and determines that the baby's weight has tripled compared to the birth weight of 7 pounds 8 ounces. The mother asks if the baby is gaining enough weight. What response should the nurse offer? a- Your baby is gaining weight right on schedule b- What food does your baby usually eat in a normal day? c- The baby is below the normal percentile for weight gain d. What was the baby's weight at the last well-baby clinic visit

b- What food does your baby usually eat in a normal day? Rationale: The normal weight gain in the first year of life is approx. twice the birth weight

A 3-year-old boy with a congenital heart defect is brought to the clinic by his mother... During the assessment, the mother asks the nurse why her child is at the 5th percent...response is best for the nurse to provide? a. Does your child seem mentally slower than his peers also? b. "His smaller size is probably due to the heart disease" c. Haven't you been feeding him according to recommended daily allowances for children? d. You should not worry about the growth tables. They are only averages for children

b. "His smaller size is probably due to the heart disease" Rationale: Poor growth patterns are associated with heart disease.

A client is admitted to a voluntary hospital mental health unit due to suicidal ideation. The client has been on the unit for 2 days and now states "I demand to be released now " The appropriate action is for the nurse to A) You cannot be released because you are still suicidal. B) You can be released only if you sign a no suicide contract. C) Let's discuss your decision to leave and then we can prepare you for discharge. D) You have a right to sign out as soon as we get an order from the health care provider's discharge order.

! is C: Let's discuss your decision to leave and then we can prepare you for discharge. 3

A nurse entering the room of a postpartum mother observes the baby lying at the edge of the bed while the woman sits in a chair. The mother states," This is not my baby, and I do not want it." The nurse's best response is A) "This is a common occurrence after birth, but you will come to accept the baby." B) "Many women have postpartum blues and need some time to love the baby." C) "What a beautiful baby Her eyes are just like yours." D) "You seem upset; tell me what the pregnancy and birth were like for you."

! is D: "You seem upset; tell me what the pregnancy and birth were like for you." 15

The nurse is preparing a client who will undergo a myelogram. Which of the following statements by the client indicates a contraindication for this test? A) "I can't lie in 1 position for more than thirty minutes. "B) "I am allergic to shrimp." C) "I suffer from claustrophobia. "D) "I developed a severe headache after a spinal tap."

"B) "I am allergic to shrimp."

A client with pneumococcal pneumonia had been started on antibiotics 16 hours ago.During the nurse's initial evening rounds the nurse notices a foul smell in the room. The client makes all of these statements during their conversation. Which statement would alert the nurse to a complication? A) "I have a sharp pain in my chest when I take a breath. "B) "I have been coughing up foul-tasting, brown, thick sputum. " C) "I have been sweating all day. "D) "I feel hot off and on."

"B) "I have been coughing up foul-tasting, brown, thick sputum.

A client is diagnosed with a spontaneous pneumothorax necessitating the insertion of a chest tube. What is the best explanation for the nurse to provide this client? A) "The tube will drain fluid from your chest. "B) "The tube will remove excess air from your chest." C) "The tube controls the amount of air that enters your chest. " D) "The tube will seal the hole in your lung."

"B) "The tube will remove excess air from your chest."

A nurse is doing pre conceptual counseling with a woman who is planning a pregnancy. Which of the following statements suggests that the client understands the connection between alcohol consumption and fetal alcohol syndrome? A) "I understand that a glass of wine with dinner is healthy. "B) "Beer is not really hard alcohol, so I guess I can drink some. "C) "If I drink, my baby may be harmed before I know I am pregnant. " D) "Drinking with meals reduces the effects of alcohol."

"C) "If I drink, my baby may be harmed before I know I am pregnant.

After diagnosis and initial treatment of a 3-year-old child with cystic fibrosis, the nurse provides home care instructions to the mother. Which statement by the child's mother indicates that she understands home care treatment to promote pulmonary function?

"Cough suppressants can be used four times a day."

A client diagnosed with acute epididymitis secondary to a sexually transmitted disease receives a prescription for ceftriaxone (Rocephin). Prior to administering the prescription, which question should the nurse ask the client?

"Have you ever had an allergic reaction to any other antibiotics?"

The nurse is teaching a client with COPD about health promotioin activities. What is the most important advice the nurse should give this client?

"If you have not already done so, quit smoking"

A client is receiving a continuous infusion of normal saline at 125ml per hour. The nurse prepares to change the primary IV tubing and hang a new bag of normal saline to maintain the prescription. In which sequence should the nurse implement the procedure?

(Arrange with first on top and last on bottom.) Spike a new bag- open the clamp-close the clamp-place the pumpattach the distal end-open all tubing clamps

The nurse completes the Leopold maneuvers for a primipara who is admitted in active labor and determines that the fetus is in the right sacral anterior (RSA) position. On which quadrant should the nurse place the external fetal heart transducer?

(Click the chosen location. To change, click on the new location.)

Nurse given a change in drug orders via phone from physician for a client headed to radiology, place in order what steps the nurse should take first to last?

(Place in order) 1- read order back 2-enter verbal prescription 3- administer prn meds 4- prepare client for x-ray procedure

Assessing radial and pedal pulses

-If you can't find a pulse, use doppler ultrasound; keep looking! -Don't take pulse at same time as BP -Compare pulses bilaterally

Stages of CKD

-1,2,and 3: pt still functioning well, living faily normal life, may never progress or advance -Stage 5: dialysis 3x/week -Can't go on a week long vacation without dialysis; will end up back in the hospital -Fistula = restricted extremity (no BP or IV)

Interventions for low biophysical profile score

-<2: labor induction -4: labor induction if >32 weeks; repeat the test the same day if <32 weeks then induce labor if <6 -6: Labor induction if >36 weeks, repeat testing in 24 hrs if <36 weeks -8: labor induction if presence of oligohydramnios (lack of amniotic fluid surrounding fetus)

Assessment for hypocalcemia

-<8.5 -Chvostek's sign: stroke face and face twitches -Trousseau's sign: pump up BP cuff, hand/wrist flexes -Possibly from thyroid surgery

Congenital adrenal hyperplasia

-A healthy baby starts to develop problems: start telemetry -Severe form symptoms: abnormal heart rhythm, change in electrolytes, poor feeding, vomiting, dehydration -Exams: check electrolytes -Treatment: goal is to return hormones to normal/near normal, cortisol, steroids

BMI assessment

-Age/sex specific -Assess pts nutrition -Do they have money for food? -Assess activity level -Assess who is providing the pt with food and assess their knowledge of food/educate that person on healthy choices

Primary interventions

-Aimed to prevent -Immunizations, education (ex: posters around the hospital)

TB Precautions

-Airborne isolation -N-95 masks -Negative pressure, single room

Catheter Insertion

-Always ask for allergies: could be allergic to the Iodine/Bedadine used to sterilize -Document time, size of foley, fluid in the balloon -Landmark = meatus inbetween clitoris and vaginal opening

Restraints

-Assess circulation every hour -ROM exercises -Monitor safety and necessity

Assessment points for pt with constipation

-Assess current medications -Check measures that they're currently taking (daily enemas, laxatives, etc) -Assessment: no bowel sounds, abd distention -Make a doctors appointment -Assess current nutrition, diet, and exercise -Drink more water

Assessment and nursing interventions for hip fx

-Assess for pain -Hip fracture sometimes shortens the leg/rotates it -Treat like its a hip fracture even if no testing done yet -Assess circulation (pulse, cap refill, feeling, movement, wiggle toes, color) -Don't elevate the leg -No extra motion until hip is stabilized -Assess skin and circulation if in traction

Education early in pregnancy

-Assess their support system -Are they homeless? This impacts care -Assess environment, living situation, age, cultural/religious expectations -Are they happy about the pregnancy?

Biophysical profile

-Assessment of the fetus -Failure of stress test indicates you need the biophysical profile -Scores 0-10; 2 points for normal, 0 for abnormal -Low score = increased rate of infant mortality; possible emergent c-section -High score = continued monitoring

Job responsibilities only RN can do

-Assessments -Fall risk assessment -Assessing knowledge of other team members -Always assigned the more high risk patients -New admissions -Suicidal pts -1st day post op pts -Critical pts -Acute problem pts -New splits

Projection

-Attributing one's own thoughts or impulses to another person -Ex: sexual feelings toward her teacher tells her friends the teacher is coming onto her

Osteomyelitis

-Bone infection -Treat with antibiotics -Poor healing, injury entered the bone

Steps of the chain of infection

-Causative agent -Reservoir -Portal of exit: ex: wound, dressing change -Mode of transmission: nurse could be mode if they wear a gown in the hallway -Portal of entry -Susceptible host: decreased immune system, also have wounds (entry site) *Ways to block transmission: precautions, wash hands

Terbutaline

-Causes uterine muscle relaxation -Helps delay birth for hours/days -Increases mom/fetal HR -d/c if HR mom HR is >130 -Notify HCP

Hospital emergency response plan in case of an external disaster

-Central control center manages care to decrease chaos

Sx of shock

-Change in LOC -Increased HR, decreased BP -Take manual BP and make sure its not below 90 -Manage with aggressive fluids

Penicillin G Potassium

-Class: antibiotic -Indications/action: treatment of infections, including sepsis, meningitis, endocarditis, pneumonia -Adverse reactions/side effects: electrolyte imbalances, may increase liver enzymes, risk for seizures (rare) -Nursing implications: monitor I&O, use caution in renal/hepatic impairment, hypersensitivity to cephalosporins, seizure disorder

Linezolid (Zyvox)

-Class: antibiotic -Indications/actions: treatment of nosocomial infections, VRI -Adverse reactions: thrombocytopenia, myelodepression -Other superinfections: watch for severe watery diarrhea (may result from altered bacterial balance in gut) -Nursing implications: monitor bowel consistency, GI effects, monitor CBC, platelets, hgb, electrolytes, monitor for superinfections, avoid tyramine containing foods/alcohol

IM Injections

-If you stick yourself, fill out appropriate paper work and get pts blood sample -If you think you're in the wrong space, get a larger needle -If you aspirate and get blood return, take out and throw everything away

Phenytoin (dilantin)

-Class: anticonvulsant, antiarrhythmic -Indications/action: limits spread of seizure activity, stabilizes threshold against hyperexcitability -Side effects: drowsiness, lethargy, slurred speech -*Abrupt withdrawal may induce seizure -Nursing implications: monitor for therapeutic serum level (10-20), multiple food & drug contraindicatons

Diphenhydramine (Benedryl)

-Class: antihistamine -Indications/Action: decreases inflammatory reaction associated with allergy, decreases urticaria -Adverse reactions/side effects: dry mouth, may have paradoxical effect in children (euphoria, agitation), may cause drowsiness, confusion, dizziness, watch for CNS depression with overdose -Nursing implications: avoid alcohol, safety considerations associated with drowsiness, dizziness

Prasugrel (Effient)

-Class: antiplatelet agent, inhibits platelet aggregation -Indications: reduction of CV events (MI/CVA, thrombosis) for pts with ACS undergoing PCI (percutaneous coronary intervention) -Adverse reactions/side effects: hemorrhage can occur, contraindicated with active bleeding, may decrease hgb/hct, wbc, platelets -Nursing implications: monitor for bleeding, in stool/urine, bruising, SOB, petichial rash, obtain baseline labs, EKG, vitals, watch lfts, report fever, weakness, and yellowing skin/eyes

Sulcrafate (Carafate)

-Class: antiulcer -Indications/action: protective agent in stomach, forms adhesive barrio on mucosa in stomach/duodenum, treatment of ulcer, ulcer prevention -Adverse reactions: constipation, can make bezoars -Nursing implications: take 1 hr before meals and before bedtime

Ramelteon (Rozeram)

-Class: hypnotic, melatonin receptor agonist -Indications/action: prevents insomnia, characterized with difficulty falling asleep -Adverse reactions: may cause sleepwalking, sleepcooking, sleepeating (somnambulism), may decrease testosterone, increase prolactin -Nursing implications: don't use with alcohol, take 30 min before bed, provide restful and quiet environment

Orlistat (Xenical, Alli)

-Class: obesity management agent, gastric/pancreatic lipase inhibitor) -Indications/Action: inhibits absorption of dietary fats, may improve weight loss -Adverse reactions: headache, flatulence, abd discomfort, fatty stool, fecal urgency -Nursing implications: for use with reduced calorie diet, fat soluble vitamins should be taken at least two hours apart from med to allow for absorption, administer during or up to 1 hr after meal containing fat

Mannitol (osmitrol)

-Class: osmotic diuretic -Indication/actions: produces diuresis, oliguria prevention tx (in AKI), reduces intraocular pressure (IOP), reduces ICP and cerebral edema -Adverse reactions: can cause f/e imbalances, pulmonary edema -Side effect: thirst, dry mouth -Nursing implications: obtain baseline wt/chemistries, monitor serum and urine osmolarity, monitor VS and electrolytes, monitor I&Os, monitor for arrhythmias

Undoing

-Communication or behavior done to negate a previously unacceptable act -Ex: young man who used to hunt animals is now on committee for protecting them; mom yells at child then offers to get icecream

AKI assessment

-Decreased kidney function because of secondary condition and output decreases -Oliguria: less than 400 cc in 24 hrs -Kidneys aren't producing urine -Possibly can't produce urine sample

Pneumothorax and tension pneumothorax

-Difference in LS from one lung to the other -SOB, asymmetrical chest expansion, unequal chest rise and fall -Trachial deviation = tension pneumo, squishing the heart -Medical emergency!

Pertussis (whooping cough) Precautions

-Droplet isolation and contact precautions -Facemask

Grave's disease signs and symptoms

-Enlarged thyroid -Acceleration of body process: weight loss, increased appetite, diarrhea, heat intolerance, tachycardia, palpitations, increased BP, diaphoresis, nervousness, insomnia -Exopthalamos -Increased T3 and T4 levels -Low TSH levels

Distracting Injury

-Ex: traumatic amputation, so focused on treating that you don't realize the pt isn't breathing -ABCs -C: know the cause and how to control it

Decerebrate posturing

-Extension of extremities -2 on GCS (worse than decorticate) -Indicates brain injury

Decorticate posturing

-Flexing towards core -3 on GCS -Indicates brain injury

Caring for the burn patient

-Fluids in the first 24-48 hours -Pain management -Airway is most important: possible intubation -Check vitals: HR, BP -Increase or decrease of HR/BP could mean too much fluid. Assess lungs for fluid in lungs

Osteomalacia and implications for pt teaching

-Focus on nutritional support -Sunlight is a good source of vitamin D -Vitamin D is important to absorb calcium and phosphorous -Check vitamin D levels

AKI Interventions

-Follow up and look at what drugs they're on -Monitor electrolytes -Monitor weight and edema -Monitor VS: HTN/HypoTN, increased HR -Aggressive fluids: watch for fluid overload, check lung sounds and vitals -Lasix or Mannitol

Duchenne muscular dystrophy

-Genetic, inherited -Progressive muscular degneration -Slowly loses ability to walk, most kids in wheelchair by 10 -At end, have problems with talking/breathing -Support groups, teach parents how to recognize when child is getting worse -Provide resources -Continued education

Trouble shooting for BP machine

-Get a different machine!

What to do if there's a flood

-Get the pt out of the area

Explain safe insulin administration practice and how to teach another nurse

-Give in SubQ fat, rotate spots -Sliding scale dose is based on most recent glucose level -Check orders in the computer if forget dose/med -Clean area with alcohol -Make sure you have the right pt

Grave's disease treatment

-Goal: inhibit production of thyroid hormones and block effect on the body -Radioactive iodine therapy: destroys overactive thyroid cells over time -Propylthiouracil, methimazole -Betablockers -Subtotal or full thyroidectomy (risk of damaging vocal cords and parathyroid glands)

The nurse administers an antibiotic to a client with respiratory tract infection. To evaluate the medication's effectiveness, which laboratory values should the nurse monitor? Select all that apply -White blood cell (WBC) count -Sputum culture and sensitivity

-White blood cell (WBC) count -Sputum culture and sensitivity

Apgar score for Heart Rate

0 = absent 1 = <100 2 = >100

An adolescent receives a prescription for an injection of s-matriptan succinate 4 mg subcutaneously for a migraine headache. Using a vial labeled, 6 mg/ 0.5 ml, how many ml should the nurse administer? (Enter the numerical value only. If rounding is required, round to the nearest hundredth. 0.33

0.33 mL Rationale: 4mg x 0.5 ml=2/6=0.33 ml

An infant is receiving penicillin G procaine 220,000 units IM. The drug is supplied as 600,000 units/ml. How many ml should the nurse administer? (Enter numeric value only. If rounding is required, round to the nearest tenth) 0.4

0.4 Rationale: Calculate using the formula, desired dose (220,000 units) over dose on hand (600,000 units) x the volume of the available dose (1 ml). 220,000 / 600,000 x 1 ml = 0.36 = 0.4 ml

The healthcare provider prescribes celtazidime (Fortax) 35 mg every 8 hours IM for an infant. The 500 mg vial is labeled with the instruction to add 5.3 ml diluent to provide a concentration of 100 mg/ml. How many ml should the nurse administered for each dose? (Enter numeric value only. If rounding is required, round to the nearest tenth. 0.4

0.4 rationale: 35mg/100mg x 1 = 0.35 = 0.4 ml

A client with a serum sodium level of 125 meq/mL should benefit most from the administration of which intravenous solution? a. 0.9% sodium chloride solution (normal saline) b. 0.45% sodium chloride solution (half normal saline) c. 10% Dextrose in 0.45% sodium chloride d. 5% dextrose in 0.2% sodium chloride

0.9% sodium chloride solution (normal saline) Rationale: Normal range = 135-145

An older adult male reporting abdominal pain is admitted to the hospital from a long-term care facility. It has been 7 days since his last bowel movement, his abdomen is distended, and he just vomited 150mL of dark brown emesis. In what order should the nurse implement these interventions? (Highest to lowest priority)

1. Send emesis sample to the lab 2. Elevate the head of the bed 3. Complete focused assessment 4. Offer PRN pain medication

The development of atherosclerosis is a process of sequential events. Arrange the pathophysiological events in orders of occurrence. (Place the first event on top and the last on the bottom) 1. Arterial endothelium injury causes inflammation 2. Macrophages consume low density lipoprotein (LDL), creating foam cells 3. Foam cells release growth factors for smooth muscle cells 4. Smooth muscle grows over fatty streaks creating fibrous plaques 5. Vessel narrowing results in ischemia.

1. Arterial endothelium injury causes inflammation 2. Macrophages consume low density lipoprotein (LDL), creating foam cells 3. Foam cells release growth factors for smooth muscle cells 4. Smooth muscle grows over fatty streaks creating fibrous plaques 5. Vessel narrowing results in ischemia Rationale: Prolonged low blood pressure leads to renal ischemia, which is the common etiology of acute tubular necrosis(ATN) Decreasing urine output is an early indicator of ATN.

A client admitted to the telemetry unit is having unrelieved chest pain after receiving 3 sublingual nitroglycerin tablets and morphine 8 mg IV. The electrocardiogram reveals sinus bradycardia with ST elevation. In what order should the nurse implement the nursing actions? (Arrange first to last) Correct Order: 1. Call the rapid response team to assist 2. Move the crash cart to the client room 3. Notify the client's healthcare provider 4. Inform the family of the critical situation

1. Call the rapid response team to assist 2. Move the crash cart to the client room 3. Notify the client's healthcare provider 4. Inform the family of the critical situation

A client diagnosed with bipolar disorder is going home on a week-end pass. Which suggestions should give the client's family to help them prepare for the visit? 1. Discuss the importance of continuing the usual at-home activities 2. Encourage the family to plan daily activities to keep the client busy 3. Have friends and family visit the client at a welcome party. 4. Instruct family to monitor the client's choice of television programs.

1. Discuss the importance of continuing the usual at-home activities Rationale: Week-end pass are schedules to help the client ease back into the family's routine, so the client can back to normal activities.

5 variables of biophysical profile

1. Fetal breathing movements 2. Gross body movements 3. Fetal tone 4. Reactive FHR 5. Qualitative amniotic fluid volume

An unlicensed assistive personnel UAP leaves the unit without notifying the staff. In what order should the unit manager implement this intervention to address the UAPs behavior? (Place the action in order from first on top to last on bottom.) 1. Note date and time of the behavior. 2. Discuss the issue privately with the UAP. 3. Plan for scheduled break times. 4. Evaluate the UAP for signs of improvement.

1. Note date and time of the behavior. 2. Discuss the issue privately with the UAP. 3. Plan for scheduled break times. 4. Evaluate the UAP for signs of improvement. Rationale: Noting the date and time of the behavior is the first action that is important in providing factual information. The unit manager should discuss the behavior with the UAP and describe the problems the behaviors causes for the staff, when a problem is identified, it is important to plan and implement solutions, such as scheduled break times during the shift. These interventions should be evaluated based on the UAP's signs of improvements.

A client who is admitted for primary hypothyroidism has early signs of myxedema coma. In assessing the client, in which sequence should the nurse complete these actions? (descending order)

1. Observe breathing patterns 2. Assess blood pressure 3. Measure body temperature 4. Palpate for pedal edema

Which actions should the nurse implement with auscultating anterior breath sounds? (Place the first action on top and last action on the bottom.) Correct order: 1. Place stethoscope in suprasternal area to auscultate from bronchial sounds 2. Auscultate bronchovesicular sounds from side to side of the first and second intercostal spaces 3. Displace female breast tissue and apply stethoscope directly on chest wall to hear vesicular sounds 4. Document normal breath sounds and location of adventitious breath sounds

1. Place stethoscope in suprasternal area to auscultate from bronchial sounds 2. Auscultate bronchovesicular sounds from side to side of the first and second intercostal spaces 3. Displace female breast tissue and apply stethoscope directly on chest wall to hear vesicular sounds 4. Document normal breath sounds and location of adventitious breath sounds

The nurse is caring for a 4-year-old male child who becomes unresponsive as his heart rate decreases to 40 beats/minute. His blood pressure is 88/70 mmHg, and his oxygen saturation is 70% while receiving 100% oxygen by non-rebreather face mask. In what sequence, from first to last, should the nurse implement these actions? (Place the first action on top and last action on the bottom.) 1. Start chest compressions with assisted manual ventilations 2. Administer epinephrine 0.01 mg/kg intraosseous (IO) 3. Apply pads and prepare for transthoracic pacing 4. Review the possible underlying causes for bradycardia.

1. Start chest compressions with assisted manual ventilations 2. Administer epinephrine 0.01 mg/kg intraosseous (IO) 3. Apply pads and prepare for transthoracic pacing 4. Review the possible underlying causes for bradycardia Rationale: The American Heart Association guidelines recommend that the basic life support (BLS) algorithm should be initiated immediately in pediatric clients who are unresponsive or have a heart rate below 60 beats/minutes*** and exhibit signs of poor perfusion. This child is manifesting poor perfusion as evidenced by a low blood pressure and poor oxygenation, so chest compression and assisted manual ventilation should be provided first, followed by administration of drug therapy for persistent bradycardia. Preparation with pad placement for transthoracic pacing should be implemented next, followed by treatment indicated for the underlying cause of the child's bradycardia.

The HCP prescribes methotrexate 7.5 mg PO weekly, in 3 divides doses for a child with rheumatoid arthritis whose body surface area (BSA) is 0.6 m2. The therapeutic dosage of methotrexate PO is 5 to 15 mg/m2/week. How many mg should the nurse administer in each of the three doses given weekly? (Enter the numeric value only. If round is required, round to the nearest tenth.) 2.5

1.5

The healthcare provider prescribes Morphine Sulfate Oral Solution 38 mg PO q4 hours for a client who is opioid-tolerant. The available 30 mL bottle is labeled, 100 mg/5 mL (20mg/mL), and is packaged with a calibrated oral syringe to provide to provide accurate dose measurements. How many mL should the nurse administer? (Enter the numerical value only. If rounding is required, round to the nearest tenth.) 1.9

1.9 Rationale: 38/20x1=1.9 m

An IV antibiotic is prescribed for a client with a postoperative infection. The medication is to be administered in 4 divided doses. What schedule is best for administering this prescription? a. 1000, 1600, 2200, 0400

1000, 1600, 2200, 0400

A heparin infusion is prescribed for a client who weighs 220 pounds. After administering a bolus dose of 80 units/kg, the nurse calculates the infusion rate for the heparin solution at 18 units/kg/hr. The available solution is Heparin Sodium 25,000 units in 5% Dextrose injection 250mL. The nurse should program the infusion pump to deliver how many mL/hour?

18

Dopamine protocol is prescribed for a male client who weigh 198 pounds to maintain the mean arterial pressure (MAP) greater than 65 mmHg. His current MAP is 50 mmHg, so the nurse increases the infusion to 7 mcg/kg/minute. The infusion is labeled dextrose 5% in water (D5W) 500 ml with dopamine 400 mg. The nurse should program the infusion pump to deliver how many ml/hour? 47

47

After administering a 12 ounce can of nutritional supplement, 3 teaspoons of medication, and 120 mL of water, the nurse should document the client's fluid intake as how many mL?

495

A 154 pound client with diabetic ketoacidosis is receiving an IV of normal saline 100 ML with regular insulin 100 units. The healthcare provider prescribes a rate of 0.1 units/kg/hour. To deliver the correct dosage, the nurse should set the infusion pump to Infuse how many ml/hour? enter numeric value only 7

7 Rationale: Convert the client's weight to kg, 2.2 pound: 1 kg:: 154 pounds: x kg = 154/2.2 = 70kg. Calculate the client infusion rate, 0.1 x 70 kg = 7 units/hour. Using the formula, D/H x Q = 7 units/hour / 100 units x 100 ml = 7ml / hour

The healthcare provider prescribes an IV solution of isoproterenol (Isuprel) 1 mg in 250 ml of D5W at 300 mcg/hour. The nurse should program the infusion pump to deliver how many ml/hour? (Enter numeric value only.) 75

75 Rationale: Convert mg to mcg and use the formula D/H x Q. 300 mcg/hour / 1,000 mcg x 250 ml = 3/1 x 25 = 75 ml/hour

A female client reports that she drank a liter of a solution to cleanse her intestines... immediately. How many ml of fluid intake should the nurse document? Whole number 760

760 Rationale: 1L=1000ml Subtract the emesis, 1 cup (8 oz)=240ml 1000-240=760 ml

During the admission assessment, the nurse auscultates heart sounds for a client with no history of cardiovascular disease. Where should the nurse listen when assessing the client's point of maximal impulse (PMI) (Click the chosen location. To change, click on a new location)

4-5th intercostal space midclavicular

The charge nurse is planning for the shift and has a registered nurse (RN) and a practical nurse (PN) on the team. Which client should the charge nurse assign to the RN? a. A 64-year-old client who had a total hip replacement the previous day. b. A 75-year-old client with renal calculi who requires urine straining. c. An adolescent with multiple contusions due to a fall that occurred 2 days ago. d. A 30-year-old depressed client who admits to suicide ideation.

A 30 year old depressed client who admits to suicide ideation RATIONALE: A client who is suicidal requires psychological assessment, therapeutic communication and knowledge beyond the educational level of a practical nurse (RN). Other clients could be cared for by the PN or the UAP, with supervision by the registered nurse.

The nurse-manager of a perinatal units is notified that one client from the medical-surgical unit needs to be transferred to make room for new admissions. Which client should the nurse recommend for transfer to the antepartal unit?

A 35-year-old with lupus erythematosus 121. Who is at risk for rib fracture - Elderly pt.

A nurse working on an endocrine unit should see which client first? a. An adolescent male with diabetes who is arguing about his insulin dose. b. An older client with Addison's disease whose current blood sugar level is 62mg/dl (3.44 mmol/l). c. An adult with a blood sugar of 384mg/dl (21.31mmol/l) and urine output of 350 ml in the last hour. d. A client taking corticosteroids who has become disoriented in the last two hours.

A client taking corticosteroids who has become disoriented in the last two hours. Rational: meeting the client's need for safety is a priority intervention. Mania and psychosis can occur during corticosteroids therapy, places the client at risk for injury, so the patient taking corticosteroids should be seen first.

The home health nurse is preparing to make daily visits to a group of clients. Which client should the nurse visit first? a. A client with congestive heart failure who reports a 3 pound weight gain in the last two days

A client with congestive heart failure who reports a 3 pound weight gain in the last two days

The nurse is instructing a 65 year-old female client diagnosed with osteoporosis. The most important instruction regarding exercise would be to A) Exercise doing weight bearing activities B) Exercise to reduce weight C) Avoid exercise activities that increase the risk of fracture D) Exercise to strengthen muscles and thereby protect bones

A) Exercise doing weight bearing activities

The nurse working in the psychiatric clinic has phone messages from several clients. Which call should the nurse return first? a. A young man with schizophrenia who wants to stop taking his medication b. The mother of a child who was involved in a physical fight at school today. c. A client diagnosed with depression who is experiencing sexual dysfunction. d. A family member of a client with dementia who has been missing for five hours

A family member of a client with dementia who has been missing for five hours Rationale: safety is always the priority concern and the family member of the missing client with dementia needs assistance with contacting authorities as well as psychological support during this time.

A client is admitted with an exacerbation of heart failure secondary to COPD. Which observations by the nurse require immediate intervention to reduce the likelihood of harm to this client? (Select all that apply). A full pitcher of water is on the bedside table The client is lying in a supine position in bed

A full pitcher of water is on the bedside table The client is lying in a supine position in bed

A nurse with 10 years experience working in the emergency room is reassigned to the perinatal unit to work an 8 hour shift. Which client is best to assign to this nurse? a. A client who is leaking clear fluid b. A mother who just delivered a 9 pounds boy c. A mother with an infected episiotomy. d. A client at 28- weeks' gestation in pre-term labor.

A mother with an infected episiotomy Rationale: An infected episiotomy is essentially an infected surgical incision, and an experienced emergency room nurse is likely be able to care for such a client. A, B and D required specialized maternity nursing care.

The charge nurse is making assignment on a psychiatric unit for a practical nurse (PN) and newly license register nurse (RN). Which client should be assigned to the RN? a. An adult female who has been depressed for the past several months and denies suicidal ideations. b. A middle-age male who is in a depressive phase of bipolar disease and is receiving Lithium. c. A young male with schizophrenia who said voices are telling him to kill his psychiatrist. d. An elderly male who tells the staff and other clients that he is superman and he can fly.

A young male with schizophrenia who said voices is telling him to kill his psychiatric. Rationale: The RN should deal with the client with command hallucinations and these can be very dangerous if the client's acts on the commands, especially if the command is a homicidal in nature. Other client present low safety risk

The nurse is making a home visit to a client with chronic obstructive pulmonary disease (COPD). The client tells the nurse that he used to be able to walk from the house to the mailbox without difficulty. Now, he has to pause to catch his breath halfway through the trip. Which diagnosis would be most appropriate for this client based on this assessment? A) Activity intolerance caused by fatigue related to chronic tissue hypoxia B) Impaired mobility related to chronic obstructive pulmonary disease C) Self-care deficit caused by fatigue related to dyspnea D) Ineffective airway clearance related to increased bronchial secretions

A) Activity intolerance caused by fatigue related to chronic tissue hypoxia

An antibiotic IM injection for a 2 year-old child is ordered. The total volume of the injection equals 2.0 ml The correct action is to A) Administer the medication in 2 separate injections B) Give the medication in the dorsal gluteal site C) Call to get a smaller volume ordered D) Check with pharmacy for a liquid form of the medication skip

A) Administer the medication in 2 separate injections

The nurse who working in the ED is obtaining evidence for a rape kit from a woman who reports that she was raped. Which intervention is most important for the nurse to implement? A) Do not allow client to shower until all evidence is obtained. B) Report incident to the university's security department. C) Listen attentively to the client's description of the event. D) Determine the client's personal reaction to the reported rape.

A) Do not allow client to shower until all evidence is obtained. Rationale: It is most important to gather evidence and a shower distorts such evidence. The client should not be allowed to shower until all the evidence is collected.

A client has received her first dose of fluphenazine (Prolixin) 2 hours ago. She suddenly experiences torticollis and involuntary spastic muscle movement. In addition to administering the ordered anticholinergic drug, what other measure should the nurse implement? A) Have respiratory support equipment available B) Immediately place her in the seclusion room C) Assess the client for anxiety and agitation D) Administer PRN dose of IM antipsychotic medication

A) Have respiratory support equipment available

The health care provider order reads "aspirate nasogastric feeding (NG) tuber every 4 hours and check pH of aspirate." The pH of the aspirate is 10. Which action should the nurse take? A) Hold the tube feeding and notify the provider B) Administer the tube feeding as scheduled C) Irrigate the tube with diet cola soda D) Apply intermittent suction to the feeding tube

A) Hold the tube feeding and notify the provider

The nurse enters a 2 year-old child's hospital room in order to administer an oral medication. When the child is asked if he is ready to take his medicine, he immediately says, "No!" What would be the most appropriate next action? A) Leave the room and return five minutes later and give the medicine B) Explain to the child that the medicine must be taken now C) Give the medication to the father and ask him to give it D) Mix the medication with ice cream or applesauce

A) Leave the room and return five minutes later and give the medicine

When assessing a client, it is important for the nurse to be informed about cultural issues related to the client's background because A) Normal patterns of behavior may be labeled as deviant, immoral, or insane B) The meaning of the client's behavior can be derived from conventional wisdom C) Personal values will guide the interaction between persons from 2 cultures D) The nurse should rely on her knowledge of different developmental mental stages

A) Normal patterns of behavior may be labeled as deviant, immoral, or insane

Which type of accidental poisoning would the nurse expect to occur in children under age 6? A) Oral ingestion B) Topical contact C) Inhalation D) Eye splashes

A) Oral ingestion

Alcohol and drug abuse impairs judgment and increases risk taking behavior. What nursing diagnosis best applies? A) Risk for injury B) Risk for knowledge deficit C) Altered thought process D) Disturbance in self-esteem

A) Risk for injury

The nurse assesses delayed gross motor development in a 3 year-old child. The inability of the child to do which action confirms this finding? A) Stand on 1 foot B) Catch a ball C) Skip on alternate feet D) Ride a bicycle

A) Stand on 1 foot

During the care of a client with a salmonella infection, the primary nursing intervention to limit transmission is which of these approaches? A) Wash hands thoroughly before and after client contact B) Wear gloves when in contact with body secretions C) Double glove when in contact with feces or vomitus D) Wear gloves when disposing of contaminated linens

A) Wash hands thoroughly before and after client contact

A client with acute pancreatitis is admitted with severe, piercing abdominal pain and an elevated serum amylase. Which additional information is the client most likely to report to the nurse? A. Abdominal pain decreases when lying supine B. Pain lasts an hour and leaves the abdomen tender C. Right upper quadrant pain refers to right scapula D. Drinks alcohol until intoxicated at least twice weekly.

A. Abdominal pain decreases when lying supine

The nurse is caring for a group of clients with the help of a practical nurse (PN). Which nursing actions should the nurse assign to the PN? (Select all that apply) A. Administer a dose of insulin per sliding scale for a client with Type 2 DM B. Start the second blood transfusion for a client 12 hours following a BKA C. Initiate patient controlled analgesia (PCA) pumps for two clients immediately postoperatively D. Perform daily surgical dressing change for a client who had an abdominal hysterectomy E. Obtain postoperative vital signs for a client one day following unilateral knee arthroplasty

A. Administer a dose of insulin per sliding scale for a client with Type 2 DM D. Perform daily surgical dressing change for a client who had an abdominal hysterectomy E. Obtain postoperative vital signs for a client one day following unilateral knee arthroplasty

A female client with a history of heart failure (HF) arrives at the clinic after what she describes as a very long trip. Following the initial physical assessment and chart review, which priority action should the nurse implement? A. Administer the prescribed diuretic B. Give a potassium supplement C. Reteach medication regimen D. Auscultate lung and heart sounds

A. Administer the prescribed diuretic

The nurse is caring for a preterm newborn with nasal flaring, grunting, and sternal retractions. After administering surfactant, which assessment is most important for the nurse to monitor? A. Arterial blood gasses B. Breath sounds C. Oxygen saturation D. Respiratory rate

A. Arterial blood gasses

A client presses the call bell and requests pain medication for a severe headache. To assess the quality of the client's pain, which approach should the nurse use? A. Ask the client to describe the pain B. Observe body language and movement C. Identify effective pain relief measures D. Provide a numeric pain scale

A. Ask the client to describe the pain

A pediatric client is taking the beta-adrenergic blocking agent propranolol. In developing a teaching plan, the nurse should teach the parents to report which sign of overdose? A. Bradycardia B. Tachypnea C. Hypertension D. Coughing

A. Bradycardia

The nurse assessing a client who reports falling 2 days ago and has a history of gouty arthritis that is controlled with allopurinol. The client states the left knee is swollen and extremely pain to touch. Which instruction should the nurse include in the discharge teaching? A. Decrease consumption of red meat and most seafood B. Substitute natural fruit juices for carbonated drinks C. Limit use of mobility equipment to avoid muscle atrophy D. Use electric heating pad when pain is at its worse

A. Decrease consumption of red meat and most seafood

The nurse is caring for four clients...postoperative hemoglobin of 8.7 mg/dl; client C, newly admitted with potassium...an appendectomy who has a white blood cell count of 15,000mm3. What intervention... A. Determine the availability of two units of packed cells in the blood bank for client B B. Increase the oxygen flow rate to 4 liters/minute per face mask for client A C. Remove any foods, such as banana or orange juice, for the breakfast tray for client C D. Inform client D that surgery is likely to be delayed until the infection responds to antibiotics

A. Determine the availability of two units of packed cells in the blood bank for client B

A young male client is admitted to rehabilitation following a right above-knee amputation (AKA) for a severe traumatic injury. He is in the commons room and anxiously calls out to the nurse, stating that his "right foot is aching". The nurse offers reassurance and support. Which additional intervention is most important for the nurse to implement? A. Encourage discussion of feelings about the loss of his limb B. Administer a prescription for gabapentin, a neuroleptic agent C. Tech the client how to wrap the stump with an elastic bandage D. Offer to assist the client to a quieter location so he can relax

A. Encourage discussion of feelings about the loss of his limb

The home care nurse visits a client who has cancer. The client reports having a good appetite but experiencing nausea when smelling food cooking. Which action should the nurse implement? A. Encourage family members to cook meals outdoors and bring the cooked food inside B. Assess the client's mucous membranes and report the findings to the healthcare provider C. Advise the client to replace cooked foods with a variety of different nutritional supplements D. Instruct the client to take an antiemetic before every meal to prevent excessive vomiting

A. Encourage family members to cook meals outdoors and bring the cooked food inside

An older client's daughter calls the home health nurse and reports that her mother has become forgetful and is very confused at night. The daughter stated that her mother's behavior changed suddenly a few days ago and is now getting worse. Which actions should the nurse take? A. Encourage increased intake of high protein foods B. Instruct the daughter to check her mother's temperature C. Review the client's current food and medication allergies D. Ask if the mother is experiencing any pain with urination E. Determine if the mother has recently experienced a fall.

A. Encourage increased intake of high protein foods B. Instruct the daughter to check her mother's temperature D. Ask if the mother is experiencing any pain with urination

A middle-aged client, admitted to a critical care unit several weeks ago because of serious injuries sustained in a motor vehicle accident, is currently in stable condition. Based on the client's age and recent life-threatening crisis, which intervention should the nurse implement? A. Encourage the client to reflect on personal goals and priorities B. Allow long periods of uninterrupted rest in order to reduce fatigue C. Discuss the cause of the accident with the client and his family D. Provide a routine schedule of activities to facilitate trust

A. Encourage the client to reflect on personal goals and priorities

An adult female client tells the nurse that though she is afraid her abusive boyfriend might one day kill her, she keeps hoping that he will change. Which action should the nurse take first? A. Explore client's readiness to discuss the situation B. Determine the frequency and type of client's abuse C. Report the finding to the police department D. Discuss treatment options for abusive partners

A. Explore client's readiness to discuss the situation

The pathophysiological mechanism are responsible for ascites related to liver failure? (Select all that apply)

A. Fluid shifts from intravascular to interstitial area due to decreased serum protein B. Increased hydrostatic pressure in portal circulation increases fluid shifts into abdomen C. Increased circulating aldosterone levels that increase sodium and water retention

The nurse is taking the blood pressure measurement of a client with Parkinson's disease. Which information in the client's admission assessment is relevant to the nurse's plan for taking the blood pressure reading? (Select all the apply) A. Frequent syncope B. Occasional nocturia C. Flat affect D. Blurred vision E. Frequent drooling

A. Frequent syncope C. Flat affect D. Blurred vision

When conducting diet teaching for a client who was diagnosed with a myocardial infarction, which snack foods should the nurse encourage the client to eat? (Select all that apply). A. Fresh turkey slices and berries B. Fresh vegetables with mayonnaise dip C. Soda crackers and peanut butter D. Chicken bouillon soup and toast E. raw unsalted almonds and apples

A. Fresh turkey slices and berries E. raw unsalted almonds and apples

The nurse is wearing personal protective equipment (PPE) while caring for a client. When exiting the room, which PPE should be removed first? A. Gloves B. Mask C. Eyewear D. Gown

A. Gloves

A client with Addison's crisis is admitted for treatment with adrenal cortical supplementation. Based on the client's admitting diagnosis, which findings require immediate action by the nurse? (Select all that apply.) A. Headache and tremors. B. Postural hypotension. C. Pallor and diaphoresis. D. Skin hyperpigmentation. E. Irregular heart beat.

A. Headache and tremors. B. Postural hypotension. C. Pallor and diaphoresis. E. Irregular heart beat. Rationale: (A, B, C, and E) are correct. Addison's crisis results from an acute lack of adrenal cortical hormones. Headache and tremors (A), as well as pallor and diaphoresis (C) may indicate significant hypoglycemia. Hypotension (B) with pale, diaphoretic skin (C) are indicative of progressing signs of life-threatening shock that also require immediate nursing intervention. An irregular heart rate (E) may be the result of hyperkalemia and also requires immediate intervention. (D) is consistent with chronic secondary adrenal insufficiency and does not require immediate nursing intervention.

A child newly diagnosed with sickle cell anemia (SCA) is being discharged from the hospital. Which information is most important for the nurse to provide the parents prior to discharge? A. Instructions about how much fluid the child should drink daily. B. Signs of addiction to opioid pain medications C. Information about non-pharmaceutical pain relief measures D. Referral for social services for the child and family

A. Instructions about how much fluid the child should drink daily

Following a cardiac catheterization and placement of a stent in the right coronary artery, the nurse administers prasugrel, a platelet inhibitor, to the client. To monitor for adverse effects from the medication, which assessment is most important for the nurse to include in this client's plan of care? A. observe color of urine B. Measure body temperature C. Assess skin turgor D. Check for pedal edema

A. Observe color of urine

A client taking clopidogrel reports the onset of diarrhea. Which nursing action should the nurse implement first? A. Observe the appearance of the stool B. Assess the elasticity of the client's skin C. Review the client's laboratory values D. Auscultate the client's bowel sounds

A. Observe the appearance of the stool

A client presents to the emergency department with muscle aches, headache, fever, and describes a recent loss of taste and smell. The nurse obtains a nasal swab for COVID-19 testing. Which action is most important for the nurse to take? A. Place the nasal swab specimen for COVID-19 directly into a biohazard bag B. Move the client to a private room, keep the door closed, and initiate droplet precautions. C. Teach the client to wear a mask, hand wash, and social distance to prevent spreading the virus D. Explain to the client to inform others that they may have been potentially exposed in the last 14 days.

A. Place the nasal swab specimen for COVID-19 directly into a biohazard bag

A middle-aged woman, diagnosed with Graves' disease, asks the nurse about this condition. Which etiological pathology should the nurse include in the teaching plan about hyperthyroidism? (Select all that apply.) A.) Graves' disease, an autoimmune condition, affects thyroid stimulating hormone receptors. B.) T3 and T4 hormone levels are increased C.) Large protruding eyeballs are a sign of hyperthyroid function D.) Weight gain is a common complaint in hyperthyroidism E.) Early treatment includes levothyroxine (Synthroid).

A.) Graves' disease, an autoimmune condition, affects thyroid stimulating hormone receptors. B.) T3 and T4 hormone levels are increased C.) Large protruding eyeballs are a sign of hyperthyroid function

The nurse who is working on a surgical unit receives change of shift report on a group of clients for the upcoming shift. A client with which condition requires the most immediate attention by the nurse? a. Gunshot wound three hours ago with dark drainage of 2 cm noted on the dressing. b. Mastectomy 2 days ago with 50 ml bloody drainage noted in the Jackson-pratt drain. c. Collapsed lung after a fall 8h ago with 100 ml blood in the chest tube collection container d. Abdominal-perineal resection 2 days ago with no drainage on dressing who has fever and chills.

Abdominal-perineal resection 2 days ago with no drainage on dressing who has fever and chills. Rationale: the client with an abdominal- perineal resection is at risk for peritonitis and needs to be immediately assessed for other signs and symptoms for sepsis.

A 14-year-old male client arrives at the emergency room in status epilepticus. He was diagnosed with a seizure disorder in childhood. What is the most likely cause of his present condition?

Acute withdrawal from anticonvulsant medication.

A client is receiving a full strength continuous enteral tube feeding at 50 ml/hour and has developed diarrhea. The client has a new prescription to change the feeding to half strength. What intervention should the nurse implement? a. Add equal amounts of water and feeding to a feeding bag and infuse at 50ml/hour b. Continue the full strength feeding after decreasing the rate of infusion to 25 ml/hr. c. Maintain the present feeding until diarrhea subsides and the begin the next new prescription. d. Withhold any further feeding until clarifying the prescription with healthcare provides.

Add equal amounts of water and feeding to a feeding bag and infuse at 50ml/hour Rationale: Diluting the formula can help alleviate the diarrhea. Diarrhea can occur as a complication of enteral tube feeding and can be due to a variety of causes including hyperosmolar formula.

To reduce the risk of being named in malpractice lawsuit, which action is most important for the nurse to take? a. Adhere consistently to standards of care.

Adhere consistently to standards of care.

A postoperative female client has a prescription for morphine sulfate 10 mg IV q3 hours for pain. One dose of morphine was administered when the client was admitted to the post anesthesia care unit (PACU) and 3 hours later, the client is again complaining of pain. Her current respiratory rate is 8 breaths/minute. What action should the nurse take? a. Provide oxygen 100% via facemask b. Check peripheral tendon reflexes c. Give another IV dose of morphine d. Administer Naloxone IV

Administer Naxolone IV Rationale: naloxone, the antidote for morphine, is indicated for respiratory depression below 10 breath/minute A is indicative for oxygen saturation is low. B is not indicated at this time. Another dose of morphine within the prescribed time interval © can cause a further decline in the respiratory rate, which could be life-threatening.

A male client is admitted with a severe asthma attack. For the last 3 hours he has experienced increased shortness of breath. His arterial blood gas results are: pH 7.22 PaCO2 55 mmHg; HCO3 25 mEq/L or mmol/L (SI). Which intervention should the nurse implement? a. Administer PRN dose of albuterol

Administer PRN dose of albuterol

While making rounds, the charge nurse notices that a young adult client with asthma who was admitted yesterday is sitting on the side of the bed and leaning over the bed-side-table. The client is currently receiving at 2 liters/minute via nasal cannula. The client is wheezing and is using pursed-lip breathing. Which intervention should the nurse implement? a. Assist the client to lie back in bed b. Call for an Ambu resuscitating bag c. Increase oxygen to 6 liters/minute d. Administer a nebulizer Treatment

Administer a nebulizer Treatment Rationale: The client needs an immediate medicated nebulizer treatment. Sitting in an upright position with head and arms resting on the over-bed table is an ideal position to promote breathing because it promotes lung expansion. Other actions me be accurate but not yet indicated.

The nurse caring for a 3-month-old boy one day after a pylorostomy notices that the infant is restless, is exhibiting facial grimaces, and is drawing his knees to his chest. What action should the nurse take? a. Administer a prescribed analgesia for pain. b. Increase IV infusion rate for rehydration c. Provide additional blankets to increase body temperature d. Feed one ounce of formula to correct hypoglycemia.

Administer a prescribed analgesia for pain Rationale: Since this child is exhibiting signs of pain, the prescribed analgesic should be administered. The behavioral signs of pain in an infant are facial grimaces, restlessness or agitation, and guarding the area of pain, in this case by pulling the knees to the chest

A 6 -year-old who has asthma is demonstrating a prolonged expiratory phase and wheezing, and has 35% personal best peak expiratory flow rate (PEFR). Based on these finding, which action should the nurse implement first? a. Administer a prescribed bronchodilator. b. Report finding to the healthcare provider. c. Encourage the child to cough and deep breath d. Determine what trigger precipitated this attack.

Administer a prescribed bronchodilator. Rationale: If the PEFR is below 50% in as asthmatic child, there is severe narrowing of the airway, and a bronchodilator should be administered immediately. B should be implemented after A. C will not alleviate the symptoms and D is not a priority.

A client was admitted to the cardiac observation unit 2 hours ago complaining of chest pain. On admission, the client's EKG showed bradycardia, ST depression, but no ventricular ectopy. The client suddenly reports a sharp increase in pain, telling the nurse, "I feel like an elephant just stepped on my chest" The EKG now shows Q waves and ST segment elevations in the anterior leads. What intervention should the nurse perform? a. Increase the peripheral IV flow rate to 175 ml/hr to prevent hypotension and shock b. Administer prescribed morphine sulfate IV and provide oxygen at 2 L/min per nasal cannula. c. Obtain a stat 12 lead EKG and perform a venipuncture to check cardiac enzymes levels. d. Notify the healthcare provider of the client's increase chest pain a call for the defibrillator crash cart.

Administer prescribed morphine sulfate IV and provide oxygen at 2 L/min per nasal cannula. Rationale: Administer morphine sulfate can increase oxygen supply are the priority intervention for symptoms of acute MI and should be supplemented with nitroglycerin and aspirin administration. A may result in overload that the impaired myocardium cannot handdle effectively. C and D are helpful but after. B

A client is scheduled to receive an IV dose of ondansetron (Zofran) eight hours after receiving chemotherapy. The client has saline lock and is sleeping quietly without any restlessness. The nurse caring for the client is not certified in chemotherapy administration. What action should the nurse take? a. Ask a chemotherapy-certified nurse to administer the Zofran b. Administer the Zofran after flushing the saline lock with saline c. Hold the scheduled dose of Zofran until the client awakens d. Awaken the client to assess the need for administration of the Zofran.

Administer the Zofran after flushing the saline lock with saline Rationale: Zofran is an antiemetic administered before and after chemotherapy to prevent vomiting. The nurse should administer the antiemetic using the accepter technique for IV administration via saline lock. Zofran is not a chemotherapy drug and does not need to be administered by a chemotherapy- certified nurse.

The healthcare provider changes a client's medication prescription from IV to PO administration and double the dose. The nurse notes in the drug guide that the prescribed medication, when given orally, has a high first-pass effect and reduce bioavailability. What action should the nurse implement? a. Continue to administer the medication via the IV route b. Give half the prescribed oral dose until the provider is consulted. c. Administer the medication via the oral route as prescribed. d. Consult with the pharmacist regarding the error in prescription.

Administer the medication via the oral route as prescribe Rationale: Bioavailability refers to the percentage of drug available in the systemic circulation. An increase in dosage is necessary to provide a therapeutic effect for oral medications with significantly reduce bioavailability.

Following surgery, a male client with antisocial personality disorder frequently requests that a specific nurse be assigned to his care and is belligerent when another nurse is assigned. What action should the charge nurse implement? a. Ask the client to explain why he constantly request the nurse b. Encourage the client to verbalize his feelings about the nurse c. Reassure the client that his request will be met whenever possible. d. Advise the client that assignments are not based on client requests

Advise the client that assignments are not based on clients requests Rationale: Those with antisocial personality disorders are manipulative in order to meet their own needs. The charge nurse must set limits on this behavior. The client's superficial charm and emotional maturity prevent effective therapeutic communication and (A and B) will be used to the client's advantage. C encourage further manipulative behavior.

Magnesium

Alcohol pt (Magnesium 1.5 to 2.5)

A young adult who is hit with a baseball bat on the temporal area of the left skull is conscious when admitted to the ED and is transferred to the Neurological Unit to be monitored for signs of closed head injury. Which assessment finding is indicative of a developing epidural hematoma? a. Altered consciousness within the first 24 hours after injury. b. Cushing reflex and cerebral edema after 24 hours c. Fever, nuchal rigidity and opisthotonos within hours d. Headache and pupillary changes 48 hours after a head injury

Altered consciousness within the first 24 hours after injury.

Family members of a client who is in hospice care discuss with the nurse their fears that their loved one's death will be painful. Which intervention should the nurse implement?

Although the client may or may not experience pain as death approaches, the nurse should provide teaching that pain control methods will be used, and are helpful in managing pain that may occur.

The charge nurse of critical care unit informed at beginning of shift that less than optimal number registered nurses be working that shift. In planning assignments, which client should receive most care hours by a registered nurse a. A 34 yo admitted today after emergency appendendectomy who has peripheral intravenous catheter, Foley catheter. b. A 48 yo marathon runner w/a central venous catheter experiencing nausea, vomiting due to electrolyte disturbance following a race. c. A 63 yo chain smoker w/ chronic bronchitis receiving O2 nasal cannula and a saline-locked peripheral intravenous catheter. d. An 82 yo client with Alzheimer's disease newly-fractures femur w/Foley catheter and soft wrist restrains applied

An 82-year-old client with Alzheimer's disease newly-fractures femur who has a Foley catheter and soft wrist restrains applied Rationale: (D) describe the client at the most risk for injury and complications because of the factor listed. (A) has complete the recovery period form anesthesia but requires critical care because of the invasive lines and new abdominal incision. (B) is likely to be in excellent physical condition and has one invasive line needed for rehydration. (C) is essentially stable, despite having a chronic condition.

Which client is at the greatest risk for developing delirium? a. An adult client who cannot sleep due to constant pain. b. an older client who attempted 1 month ago c. a young adult who takes antipsychotic medications twice a day d. a middle-aged woman who uses a tank for supplemental oxygen

An adult client who cannot sleep due to constant pain. Rationale: Client who are in constant pain ad have difficulty sleeping or resting are at high risk for delirium. Supplemental oxygen may cause confusion. B is taking medication so is not at high risk of delirium.

In making client care assignment, which client is best to assign to the practical nurse (PN) working on the unit with the nurse? a. An immobile client receiving low molecular weight heparin q12 h. b. A client who is receiving a continuous infusion of heparin and gets out of bed BID c. A client who is being titrated off heparin infusion and started on PO warfarin (Coumadin) d. An ambulatory client receiving warfarin (Coumadin) with INR of 5 second.

An immobile client receiving low molecular weight heparin q12 h. Rationale: A describe the most stable client. The other ones are at high risk for bleeding problems and require the assessment skills.

A 40-year-old client with type 1 diabetes developed chronic kidney (CKD) disease 6 months ago. Because of secondary complications often associated with this illness the nurse plans to carefully assess for signs of what condition?

Anemia

A 16-year-old adolescent with meningococcal meningitis (83) is receiving a continuous IV infusion of penicillin G, which is prescribed as 20 million units in a total volume of 2 liters of normal saline every 24 hr. The pharmacy delivers 10 million units/ liters of normal saline. How many ml/hr should the nurse program the infusion pump? (Enter numeric value only. If rounding is required, round to the nearest whole number.) 83

Answer 83 Rationale: 1000 ml-----12hr. 1000/12 = 83.33

The nurse is preparing an older client for discharge following cataract extraction. Which instruction should be include in the discharge teaching? a. Do not read without direct lighting for 6 weeks. b. Avoid straining at stool, bending, or lifting heavy objects. c. Irrigate conjunctiva with ophthalmic saline prior to installing antibiotic ointment. d. Limit exposure to sunlight during the first 2 weeks when the cornea is healing.

Avoid straining at stool, bending, or lifting heavy objects Rationale: after cataract surgery, the client should avoid activities which increase pressure and place strain on the suture line.

A client is receiving and oral antibiotic suspension labeled 250 mg/2ml. The healthcare provider prescribes 200mg every 6 hours. How many ml should the nurse administer at each dose? (Enter numerical value only. If rounding is required, round to the nearest tenth) 1.6

Answer: 1.6 Rational: using the formula D/H x Q 200mg/250 mg x 2ml = 200/250 = 1.6 ml

The nurse uses the parkland formula (4ml x kg x total body surface area = 24 hours fluid replacement) to calculate the 24-hours IV fluid replacement for a client with 40% burns who weighs 76kg. How many ml should the client receive? (Enter numeric value only.) 12,160

Answer: 12,160 Rationale: 4ml x 76kg x 40 (bsa) =12,160 ml

Which intervention should the nurse implement during the administration of vesicant chemotherapeutic agent via an IV site in the client's arm? a. Explain the temporary burning of the IV site may occur. b. Assess IV site frequently for signs of extravasation c. Apply a topical anesthetic of the infusion site for burning d. Monitor capillary refill distal to the infusion site.

Assess IV site frequently for signs of extravasation Rationale: Infiltration of a vesicant can cause severe tissue damage and necrosis, so the IV site should be assessed regularly for extravasation (B) of the chemotherapeutic agent. The client should be instructed to report any discomfort at the site (A). If pain and burning occur, the IV should be stopped and C is not indicated. Peripheral pulses, not D, provide the best assessment of perfusion distal to the infusion should the drug extravasate or infiltrate.

An antacid is prescribed for a client with gastroesophageal (GERD). The client asks the nurse, "How does this help my GERD?" What is the best response by the nurse? a. Antacids will neutralize the acid in your stomach

Antacids will neutralize the acid in your stomach

The nurse is caring for a client with acute kidney injury (AKI) secondary to gentamicin therapy the client's serum blood potassium is elevated, which finding requires immediate action by the nurse? a. Anuria for the last 12 hours. Rationale: Anuria is nonpassage of urine, in practice is defined as passage of less than 100 milliliters of urine in a day. Anuria is often caused by failure in the function of kidneys. It may also occur because of some severe obstruction like kidney stones or tumours. It may occur with end stage kidney disease.

Anuria for the last 12 hours.

A male client with ulcerative colitis received a prescription for a corticosteroid last month, but because of the side effect he stopped taking the medication 6 days ago. Which finding warrants immediate intervention by the nurse? a. Anxiety and restlessness.

Anxiety and restlessness.

In assessing an older female client with complication associated with chronic obstructive pulmonary disease (COPD), the nurse notices a change in the client's appearance. Her face appears tense and she begs the nurse not to leave her alone. Her pulse rate is 100, and respirations are 26 per min. What is the primary nursing diagnosis? a. Impaired gas exchange related to narrowing of small airways b. Death anxiety related to concern about prognosis c. Anxiety related to fear of suffocation. d. Ineffective coping related to knowledge deficit about COPD

Anxiety related to fear of suffocation. Rationale: A common problem with clients who have COPD is anxiety. These clients cannot aerate their bodies, so they feel a perpetual state of suffocation which is worse during exacerbation of their COPD. A classic descriptor of COPD id impaired gas exchange (A). Because the client has typically adapted to impaired gas exchange over a long period of time, and the nurse has assessed a change in her appearance (A) is not the primary diagnosis at this time. Based on the data presented (B and D) are not the best diagnoses in this situation.

After repositioning an immobile client, the nurse observes an area of hyperemia. To assess for blanching, what action should the nurse take? a. Note the skin color around the area b. Measure the degree of... c. Apply light pressure over the area. d. Palpate the temperature of the area.

Apply light pressure over the area. Rationale: To assess for blanching the nurse should apply pressure to the area of hyperemia with one finger and when the finger is removed evaluate for return of erythema (blanching hyperemia)

The healthcare provider explains through an interpreter the risks and benefits of a scheduled surgical procedure to a non-English speaking female client. The client gives verbal consent and the healthcare provider leaves, instructing the nurse to witness the signature on the consent form. The client and the interpreter then speak together in the foreign language for an additional 2 minutes until the interpreter concludes, "She says it is OK." What action should the nurse take next? a. Ask for a full explanation from the interpreter of the witnessed discussion

Ask for a full explanation from the interpreter of the witnessed discussion

A new member joins the nursing team spreads books on the table, puts items on two chairs, and sits on a third chair. The members of the group are forced to move closer and remove their possessions from the table what action should the nurse leader take? a. Move to welcome and accommodate a new person b. Ask the new person to move belonging to accommodate others c. Tell the new person to move belongings because of limited space d. Bring in additional chairs so that all staff members can be seated

Ask the new person to move belonging to accommodate others

A client who had a percutaneous transluminal coronary angioplasty (PTCA) two weeks ago returns to the clinic for a follow up visit. The client has a postoperative ejection fraction ejection fraction of 30%. Today the client has lungs which are clear, +1 pedal edema, and a 5pound weight gain. Which intervention the nurse implement? a. Arrange transport for admission to the hospital. b. Insert saline lock for IV diuretic therapy. c. Assess compliance with routine prescriptions. d. Instruct the client to monitor daily caloric intake.

Assess compliance with routine prescriptions. Rationale: Fluid retention may be a sign that the client is not taking the medication as prescribed or that the prescriptions may need adjustment to manage cardiac function post-PTCA (normal ejection fraction range is 50 to 75%)

An older male comes to the clinic with a family member. When the nurse attempts to take the client's health history, he does not respond to questions in a clear manner. What action should the nurse implement first a. Ask the family member to answer the questions. b. Provide a printed health care assessment form c. Assess the surroundings for noise and distractions. d. Defer the health history until the client is less anxious.

Assess the surroundings for noise and distractions.

A client with gestational diabetes at 39-weeks gestation is in the second stage of labor. After delivery of the fetal head, the nurse recognizes that shoulder dystocia is occurring. What intervention should the nurse implement first?

Assist the client to sharply flex her thighs up against the abdomen

While removing staples from a male client's postoperative wound site, the nurse observes that the client's eyes are closed and his face and hands are clenched. The client states, "I just hate having staples removed." After acknowledging the client's anxiety, what action should the nurse implement? a. Attempt to distract the client with general conversation

Attempt to distract the client with general conversation

While receiving a male postoperative client's staples the nurse observes that the client's eyes are closed and his face and hands are clenched. The client states, "I just hate having staples removed". After acknowledgement the client's anxiety, what action should the nurse implement? a. Encourage the client to continue verbalize his anxiety b. Attempt to distract the client with general conversation c. Explain the procedure in detail while removing the staples d. Reassure the client that this is a simple nursing procedure.

Attempt to distract the client with general conversation Rational: Distract is an effective strategy hen a client experience anxiety during an uncomfortable procedure. (A & D) increase the client's anxiety.

A male client, who is 24 hours postoperative for an exploratory laparotomy, complains that he is "starving" because he has had no "real food" since before the surgery. Prior to advancing his diet, which intervention should the nurse implement? a. Discontinue intravenous therapy b. Obtain a prescription for a diet change c. Assess for abdominal distention and tenderness. d. Auscultate bowel sounds in all four quadrants

Auscultate bowel sounds in all four quadrants

The nurse identifies an electrolyte imbalance, an elevated pulse rate, and elevated BP for a client with chronic kidney disease. Which is the most important action for the nurse to take? a. Monitor daily sodium intake. b. Record usual eating patterns. c. Measure ankle circumference. d. Auscultate for irregular heart rate.

Auscultate for irregular heart rate. Rational: Chronic kidney failure (CKF) is a progressive, irreversible loss of kidney functions, decreasing glomerular filtration rate (GFR), and the kidney's inability to excrete metabolic waste products and water, resulting in fluid overload, elevated pulse, elevated BP and electrolytes imbalances. The most important action for the nurse to implement is to auscultate for irregular heart rate (D) due to the decreased excretion of potassium by the kidneys. (A, B, and C) are not as important as monitoring for fatal cardiac dysrhythmias related to hyperkalemia.

The nurse notes that a client has been receiving hydromorphone (Dilaudid) every six hours for four days. What assessment is most important for the nurse to complete? a. Auscultate the client's bowel sounds b. Observe for edema around the ankles c. Measure the client's capillary glucose level d. Count the apical and radial pulses simultaneously

Auscultate the client's bowel sounds Rationale: hydromorphone is a potent opioid analgesic that slows peristalsis and frequently causes constipation, so it is most important to Auscultate the client's bowel sounds

Oxygen at 5l/min per nasal cannula is being administered to a 10 year old child with pneumonia. When planning care for this child, what principle of oxygen administration should the nurse consider? a. Avoid administration of oxygen at high levels for extended periods.

Avoid administration of oxygen at high levels for extended periods.

A client with angina pectoris is being discharge from the hospital. What instruction should the nurse plan to include in this discharge teaching? a. Engage in physical exercise immediately after eating to help decrease cholesterol levels. b. Walk briskly in cold weather to increase cardiac output c. Keep nitroglycerin in a light-colored plastic bottle and readily available. d. Avoid all isometric exercises but walk regularly.

Avoid all isometric exercises, but walk regularly Rationale: Isometric exercise can raise blood pressure for the duration of the exercise, which may be dangerous for a client with cardiovascular disease, while walking provides aerobic conditioning that improves ling, blood vessel, and muscle function. Client with angina should refrain from physical exercise for 2 hours after meals, but exercising does not decrease cholesterol levels. Cold water cause vasoconstriction that may cause chest pain. Nitroglycerin should be readily available and stored in a dark-colored glass bottle not C, to ensure freshness of the medication.

The nurse is preparing a discharge teaching plan for a client who had a liver transplant. Which instruction is most important to include in this plan? a. Limit intake fatty foods for one month after surgery. b. Notify the healthcare provider if edema occurs. c. Increase activity and exercise gradually, as tolerated. d. Avoid crowds for first two months after surgery.

Avoid crowds for first two months after surgery.

A male adult is admitted because of an acetaminophen overdose. After transfer to the mental health unit, the client is told he has liver damage. Which information is most important for the nurse to include in the client's discharge plan? a. Avoid exposure to large crowds

Avoid exposure to large crowds

A 30 month-old child is admitted to the hospital unit. Which of the following toys would be appropriate for the nurse to select from the toy room for this child? A) Cartoon stickers B) Large wooden puzzle C) Blunt scissors and paper D) Beach ball

B) Large wooden puzzle

A nurse is stuck in the hand by an exposed needle. What immediate action should the nurse take? A) Look up the policy on needle sticks B) Contact employee health services C) Immediately wash the hands with vigor D) Notify the supervisor and risk management

is C: Immediately wash the hands with vigor 1

The nurse is teaching a class on HIV prevention. Which of the following should be emphasized as increasing risk? A) Donating blood B) Using public bathrooms C) Unprotected sex D) Touching a person with AIDS

is C: Unprotected sex 6

The emergency room nurse admits a child who experienced a seizure at school. The father comments that this is the first occurrence, and denies any family history of epilepsy. What is the best response by the nurse? A) "Do not worry. Epilepsy can be treated with medications." B) "The seizure may or may not mean your child has epilepsy." C) "Since this was the first convulsion, it may not happen again." D) "Long term treatment will prevent future seizures."

B) "The seizure may or may not mean your child has epilepsy."

The mother of a 15 month-old child asks the nurse to explain her child's lab results and how they show her child has iron deficiency anemia. The nurse's best response is A) "Although the results are here, your doctor will explain them later. "B) "Your child has less red blood cells that carry oxygen. "C) "The blood cells that carry nutrients to the cells are too large." D) "There are not enough blood cells in your child's circulation."

B) "Your child has less red blood cells that carry oxygen."

A nurse from the maternity unit is floated to the critical care unit because of staff shortage on the evening shift. Which client would be appropriate to assign to this nurse? A client with A) A Dopamine drip IV with vital signs monitored every 5 minutes B) A myocardial infarction that is free from pain and dysrhythmias C) A tracheotomy of 24 hours in some respiratory distress D) A pacemaker inserted this morning with intermittent capture

B) A myocardial infarction that is free from pain and dysrhythmias

Which of these clients with associated lab reports is a priority for the nurse to report to the public health department within the next 24 hours? A) An infant with a positive culture of stool for Shigella B) An elderly factory worker with a lab report that is positive for acid-fast bacillus smear C) A young adult commercial pilot with a positive histopathological examination from an induced sputum for Pneumocystis carinii D) A middle-aged nurse with a history of varicella-zoster virus and with crops of vesicles on an erythematous base that appear on the skin

B) An elderly factory worker with a lab report that is positive for acid-fast bacillus smear

A Hispanic client in the postpartum period refuses the hospital food because it is "cold." The best initial action by the nurse is to A) 1Have the unlicensed assistive personnel (UAP) reheat the food if the client wishes B) Ask the client what foods are acceptable or bad C) Encourage her to eat for healing and strength D) Schedule the dietitian to meet with the client as soon as possible

B) Ask the client what foods are acceptable or bad

An explosion has occurred at a high school for children with special needs and severe developmental delays. One of the students accompanied with a parent is seen at a community health center a day later. After the initial assessment the nurse concludes that the student appears to be in a crisis state. Which of these interventions based on crisis intervention principles is appropriate to do next? A) Help the student to identify a specific problem B) Ask the parent to identify the major problem C) Ask the student to think of different alternatives D) Examine with the parent a variety of options

B) Ask the parent to identify the major problem

A client is admitted for first and second degree burns on the face, neck, anterior chest and hands. The nurse's priority should be A) Cover the areas with dry sterile dressings B) Assess for dyspnea or stridor C) Initiate intravenous therapy D) Administer pain medication

B) Assess for dyspnea or stridor

An 86 year-old nursing home resident who has decreased mental status is hospitalized with pneumonic infiltrates in the right lower lobe. When the nurse assists the client with a clear liquid diet, the client begins to cough. What should the nurse do next? A) Add a thickening agent to the fluids B) Check the client's gag reflex C) Feed the client only solid foods D) Increase the rate of intravenous fluids

B) Check the client's gag reflex

Following a cocaine high, the user commonly experiences an extremely unpleasant feeling called A) Craving B) Crashing C) Outward bound D) Nodding out

B) Crashing

The nurse admits a client newly diagnosed with hypertension. What is the best method for assessing the blood pressure? A) Standing and sitting B) In both arms C) After exercising D) Supine position

B) In both arms

A nurse in the emergency department suspects domestic violence as the cause of a client's injuries. What action should the nurse take first? A) Ask client if there are any old injuries also present B) Interview the client without the persons who came with the client C) Gain client's trust by not being hurried during the intake process D) Photograph the specific injuries in question

B) Interview the client without the persons who came with the client

Immediately following an acute battering incident in a violent relationship, the batterer may respond to the partner's injuries by A) Seeking medical help for the victim's injuries B) Minimizing the episode and underestimating the victim's injuries C) Contacting a close friend and asking for help D) Being very remorseful and assisting the victim with medical care

B) Minimizing the episode and underestimating the victim's injuries

Constipation is one of the most frequent complaints of elders. When assessing this problem, which action should be the nurse's priority? A) Obtain a complete blood count B) Obtain a health and dietary history C) Refer to a provider for a physical examination D) Measure height and weight

B) Obtain a health and dietary history

What nursing assessment of a paralyzed client would indicate the probable presence of a fecal impaction? A) Presence of blood in stools B) Oozing liquid stool C) Continuous rumbling flatulence D) Absence of bowel movements

B) Oozing liquid stool

Which these findings would the nurse more closely associate with anemia in a 10 month-old infant? A) Hemoglobin level of 12 g/dI B) Pale mucosa of the eyelids and lips C) Hypoactivity D) A heart rate between 140 to 160

B) Pale mucosa of the eyelids and lips

The nurse is caring for a client who requires a mechanical ventilator for breathing.The high pressure alarm goes off on the ventilator. What is the first action the nurse should perform? A) Disconnect the client from the ventilator and use a manual resuscitation bag B) Perform a quick assessment of the client's condition C) Call the respiratory therapist for help D) Press the alarm re-set button on the ventilator

B) Perform a quick assessment of the client's condition

A nurse is reinforcing teaching with a client about compromised host precautions. The client is receiving filgrastim (Neupogen) for neutropenia. The selection of which lunch suggests the client has learned about necessary dietary changes? A) Grilled chicken sandwich and skim milk B) Roast beef, mashed potatoes, and green beans C) Peanut butter sandwich, banana, and iced tea D) Barbecue beef, baked beans, and cole slaw

B) Roast beef, mashed potatoes, and green beans

The nurse has been teaching a client with congestive heart failure about proper nutrition. The selection of which lunch indicates the client has learned about sodium restriction? A) Cheese sandwich with a glass of 2% milk B) Sliced turkey sandwich and canned pineapple C) Cheeseburger and baked potato D) Mushroom pizza and ice cream

B) Sliced turkey sandwich and canned pineapple

At the day treatment center a client diagnosed with Schizophrenia - Paranoid Type sits alone alertly watching the activities of clients and staff. The client is hostile when approached and asserts that the doctor gives her medication to control her mind. The client's behavior most likely indicates A) Feelings of increasing anxiety related to paranoia B) Social isolation related to altered thought processes C) Sensory perceptual alteration related to withdrawal from environment D) Impaired verbal communication related to impaired judgment

B) Social isolation related to altered thought processes

A client is being discharged with a prescription for chlorpromazine (Thorazine).Before leaving for home, which of these findings should the nurse teach the client to report? A) Change in libido, breast enlargement B) Sore throat, fever C) Abdominal pain, nausea, diarrhea D) Dsypnea, nasal congestion

B) Sore throat, fever

A client is admitted with a diagnosis of schizophrenia. The client refuses to take medication and states "I don't think I need those medications. They make me too sleepy and drowsy. I insist that you explain their use and side effects." The nurse should understand that A) A referral is needed to the psychiatrist who is to provide the client with answers B) The client has a right to know about the prescribed medications C) Such education is an independent decision of the individual nurse whether or not to teach clients about their medications D) Clients with schizophrenia are at a higher risk of psychosocial complications when they know about their medication side effects

B) The client has a right to know about the prescribed medications

Which of these observations made by the nurse during an excretory urogram indicate a complicaton? A) The client complains of a salty taste in the mouth when the dye is injected B) The client's entire body turns a bright red color C) The client states "I have a feeling of getting warm." D) The client gags and complains " I am getting sick."

B) The client's entire body turns a bright red color

During the evaluation phase for a client, the nurse should focus on A) All finding of physical and psychosocial stressors of the client and in the family B) The client's status, progress toward goal achievement, and ongoing re-evaluation C) Setting short and long-term goals to insure continuity of care from hospital to home D) Select interventions that are measurable and achievable within selected timeframes

B) The client's status, progress toward goal achievement, and ongoing re-evaluation

What principle of HIV disease should the nurse keep in mind when planning care for a newborn who was infected in utero? A) The disease will incubate longer and progress more slowly in this infant B) The infant is very susceptible to infections C) Growth and development patterns will proceed at a normal rate D) Careful monitoring of renal function is indicated

B) The infant is very susceptible to infections

The nurse is assessing a child for clinical manifestations of iron deficiency anemia. Which factor would the nurse recognize as cause for the findings? A) Decreased cardiac output B) Tissue hypoxia C) Cerebral edema D) Reduced oxygen saturation

B) Tissue hypoxia

Privacy and confidentiality of all client information is legally protected. In which of these situations would the nurse make an exception to this practice? A) When a family member offers information about their loved one B) When the client threatens self-harm and harm to others C) When the health care provider decides the family has a right to know the client's diagnosis D) When a visitor insists that the visitor has been given permission by the client

B) When the client threatens self-harm and harm to others

The nurse should recognize that physical dependence is accompanied by what findings when alcohol consumption is first reduced or ended? A) Seizures B) Withdrawal C) Craving D) Marked tolerance

B) Withdrawal

A nurse working on an Endocrine Unit should see which client first? A. An older client with Addison's disease whose current blood sugar level is 62 mg/dL B. A client taking corticosteroids who has become disoriented in the last two hours C. An adolescent male with type 1 diabetes who is arguing about his insulin dose D. An adult with a blood sugar of 384 mg/dL and a urine output of 350mL in the last hour

B. A client taking corticosteroids who has become disoriented in the last two hours

A client's morning assessment includes bounding peripheral pulses, weight gain of 2 pounds (0.91 kg), pitting ankle edema, and moist crackles bilaterally. Which intervention is most important for the nurse to include in this client's plan of care? A. Maintain accurate intake and output B. Administer prescribed diuretic C. Weigh client every morning D. Restrict daily fluid intake to 1500mL

B. Administer prescribed diuretic

Which class of drugs is the only source of a cure for septic shock? A. Antihypertensives. B. Antiinfectives. C. Antihistamines. D. Anticholestermics.

B. Antiinfectives. Rationale: Anti-infective agents (B), such as antibiotics, are the only drugs that eliminate bacteria. The only way to halt the destruction to organ systems in septic shock is to eliminate the production of endotoxins by bacterial invaders. (A) is contraindicated due to the low cardiac output which results in low blood pressure and occurs in late septic shock. While (C) may reduce some of the destructive effects of massed cell release occurring with the inflammatory response that may occur, endotoxin release would not be stopped. (D) has no therapeutic effect relevant to septic shock.

A preschool-aged boy is admitted to the pediatric unit following successful resuscitation from a near-drowning incident. While providing care to the child, the nurse begins talking with his preadolescent brother who rescued the child from the swimming pool and initiated resuscitation. The nurse notices the older boy becomes withdrawn when asked about what happened. Which action should the nurse take? A. Develop a water safety teaching plan for the family B. Ask the older brother how he felt during the incident C. Tell the older brother that he seems depressed D. Commend the older brother for his heroic actions

B. Ask the older brother how he felt during the incident

A nurse receives report on a client who is four hours post-total abdominal hysterectomy. The previous nurse reports that it was necessary to change the client's perineal pad hourly and that it is again saturated. The previous nurse also reports that the client's urinary output has decreased. Which action should the nurse implement first? A. Evaluate the skin turgor B. Assess for weakness or dizziness C. Change the perineal pad D. Measure the urinary output

B. Assess for weakness or dizziness

The nurse is caring for a client who is still experiencing light sedation after undergoing an emergency colectomy for bowel obstruction. Which postoperative pain intervention should the nurse implement first? A. Review medical records to obtain pain tolerance expectations B. Attempt to obtain a self-report of pain level from the client C. Provide the first medication prescribed for pain management D. Wait until the client is awake before providing pain management

B. Attempt to obtain a self-report of pain level from the client

A client with chronic kidney disease has an arteriovenous fistula in the left forearm. Which observation by the nurse indicates that the fistula is patent? A. Assessment of a bruit on the left forearm B. Auscultation of a thrill on the left forearm C. The left radial pulse is 2+ bounding.

B. Auscultation of a thrill on the left forearm

The nurse is evaluating the diet teaching of a client with hypertension. What dinner selection indicates that the client understands the dietary recommendations for hypertension? A. Grilled steak, baked potato with sour cream, green beans, coffee, and raisin cream pie B. Baked pork chops, applesauce, corn on the cob, 1% milk, and key-lime pie C. Tomato soup, grilled cheese sandwich, pickles, skim milk, and lemon meringue pie D. Beef stir fry, fried rice, egg drop soup, diet soda, and pumpkin pie

B. Baked pork chops, applesauce, corn on the cob, 1% milk, and key-lime pie

An unconscious client is admitted to the intensive care unit and is placed on a ventilator. The ventilator alarms continuously and the client's oxygen saturation level is 62%. What action should the nurse take first? A. Call respiratory therapy. B. Begin manual ventilation immediately. C. Monitor oxygen saturation levels q5 minutes. D. Silence the alarm and call the technician.

B. Begin manual ventilation immediately. Rationale: The first action that must be taken is to begin manual ventilation. Remember the ABC's — airway, breathing and circulation! The nurse's highest priority is to ensure that the client is receiving oxygen. Also, remember Maslow — safety is a primary human need and breathing is fundamental to safety. (A, C, and D) do not have the priority of initiating manual ventilation.

Which self care measure is most important for the nurse to include in the plan of care of a client recently diagnosed with type 2 diabetes mellitus? A. Self-injection techniques B. Blood glucose monitoring C. Diabetic diet meal planning D. A realistic exercise plan

B. Blood glucose monitoring

The nurse is providing discharge teaching to the parents of a 13 month old child who underwent repair for an atrial septal defect. The healthcare provider prescribes aspirin and an antibiotic for the first 6 months postoperatively to prevent infective endocarditis (IE). What information is most important for the nurse discuss with the parents about the child's recovery and prevention of IE? A. Refer the mother to the healthcare provider to discuss infective endocarditis B. Brush the child's teeth every day and ensure the child receives regular dental followup C. Give the child acetaminophen for pain or fever and visit the surgeon for follow-up D. Monitor the child for regular bowel movements and urine output that exceeds intake

B. Brush the child's teeth every day and ensure the child receives regular dental followup

The nurse is caring for a client with heart failure. Which method is used in computing the cardiac index to measure how the client's heart is functioning? A. Mean arterial pressure minus right atrial pressure B. Cardiac output divided by body surface area C. Stroke volume divided by end diastolic volume D. Stroke volume multiplied by heart rate

B. Cardiac output divided by body surface area

The nurse assesses a client one hour after starting a transfusion of packed red blood cells and determines that there are no indications of a transfusion reaction. What instruction should the nurse provide the unlicensed assistive personnel (UAP) who is working with the nurse? A. Notify the nurse when the transfusion has finished, so further client assessment can be done B. Continue to measure the client's vital signs every thirty minutes until the transfusion is complete C. Monitor the client carefully for the next three hours and report the onset of a reaction immediately D. Since a reaction did not occur, the priority is to maintain client comfort during the transfusion

B. Continue to measure the client's vital signs every thirty minutes until the transfusion is complete

The nurse is evaluating the chest drainage system of a client with a chest tube inserted to treat a left hemothorax. Which finding requires intervention by the nurse? A. Rise and fall of water level with respiration B. Continuous bubbling in the water-seal chamber C. Total fluid level in water-seal chamber unchanged D. An average collection of 50 mL/hr drainage

B. Continuous bubbling in the water-seal chamber

An S3 heart sound is auscultated in a client in her third trimester of pregnancy. What intervention should the nurse take? A. Prepare the client for an echocardiogram B. Document in the client's record C. Notify the healthcare provider D. Limit the client's fluids

B. Document in the client's record

An adult client is admitted to the emergency department after falling from the ladder. While waiting to have a computed tomography (CT) scan, the client requests something for a severe headache. When the nurse offers a prescribed dose of acetaminophen, the client asks for something stronger. Which intervention should the nurse implement? A. Review client's history for use of illicit drugs B. Explain the reason for using only non-narcotics C. Assess client's pupils for their reaction to light D. Request that the CT scan be done immediately

B. Explain the reason for using only non-narcotics

When conducting diet teaching for a client who was diagnosed with a myocardial infarction, which snack foods should the nurse encourage the client to eat? (Select all that apply) A. Fresh vegetables with mayonnaise dip B. Fresh turkey slices and berries C. Chicken bouillon soup and toast D. Soda crackers and peanut butter E. Raw unsalted almonds and apples

B. Fresh turkey slices and berries C. Chicken bouillon soup and toast E. Raw unsalted almonds and apples

A client asks the nurse for information about how to reduce risk factors for benign prostatic hyperplasia (BPH). Which information should the nurse provide? A. Consume a high protein diet B. Increase physical activity C. Take vitamin supplements D. Obtain a prostate-specific antigen blood level test

B. Increase physical activity

Sublingual nitroglycerin is administered to a male client with unstable angina who complains of crushing chest pain. Five minutes later the client becomes nauseated and his blood pressure drops to 60/40. Which intervention should the nurse implement? A. Administer second dose of nitroglycerin. B. Infuse a rapid IV normal saline bolus. C. Begin external chest compressions. D. Give a PRN antiemetic medication.

B. Infuse a rapid IV normal saline bolus. Rationale: When chest pain is treated with a vasodilator, such as nitroglycerin, and the blood pressure falls to a critical level, a right ventricular infarction may have occurred which requires immediate infusion of IV fluid (B). (A and D) may worsen the condition if implemented prior to rapid infusion of fluids. Chest compressions (C) are not indicated when the client has a pulse.

An unlicensed assistive personnel (UAP) is assigned to ambulate a client with influenza who has droplet precautions implemented. The UAP requests a change in assignment, stating the reason of having not been fitted yet for a N95 respirator mask. Which action should the nurse take? A. send the UAP to be fitted for a particulate filter mask immediately so she can provide care to this client. B. Instruct the UAP that a standard face mask is sufficient for the provision of care for the assigned client C. Before changing assignments, determine which staff members have fitted particulate filter masks D. Advise the UAP to wear a standard face mask to take vital signs, and then get fitted for a filter mask before providing personal care

B. Instruct the UAP that a standard face mask is sufficient for the provision of care for the assigned client

Which intervention should the nurse include in the plan of care for a child with tetanus? A. Encourage coughing and deep breathing B. Minimize the amount of stimuli in the room C. Reposition from side to side every hour D. Open window shades to provide natural light

B. Minimize the amount of stimuli in the room

The nurse is reviewing the diagnostic tests prescribed for a client with a positive skin test. Which subjective findings reported by the client supports the diagnosis of tuberculosis? A. Barking cough and vomiting B. Mucopurulent cough and night sweats C. Dry cough and chest tightness D. Chronic cough and fatty stools

B. Mucopurulent cough and night sweats

A client with bacterial meningitis is receiving phenytoin. Which assessment finding indicates to the nurse that the client is experiencing a therapeutic response?

B. Normal electroencephalogram after drug administration

In evaluating the effectiveness of a postoperative client's intermittent pneumatic compression devices, which assessment is most important for the nurse to complete? A. Monitor the amount of drainage from the client's incision B. Observe both lower extremities for redness and swelling C. Evaluate the client's ability to use an incentive spirometer D. Palpate all peripheral pulse points for volume and strength

B. Observe both lower extremities for redness and swelling

The nurse is demonstrating correct transfer procedures to the unlicensed assistive personnel (UAP) working on a rehabilitation unit. The UAP asks the nurse how to safely move a physically disabled client from the wheelchair to a bed. Which action should the nurse recommend? A. apply a gait belt around the client's waist once a standing position has been assumed B. Place the client's locked wheelchair on the client's strong side next to the bed C. Pull the client into position by reaching from the opposite side of the bed D. Hold the client at arm's length while transferring to better distribute the body weight

B. Place the client's locked wheelchair on the client's strong side next to the bed

The nurse is providing education to a client who experiences recurrent levels of moderate anxiety to situations and perceived stress. In addition to information about prescribed medication and administration, which instruction should the nurse include in the teaching? A. Find outlets for more social interaction B. Practice using muscle relaxation techniques C. Center attention on positive upbeat music D. Think about reasons the episodes occur

B. Practice using muscle relaxation techniques

A male client approaches the nurse with an angry expression on his face and raises his voice, saying "My roommate is the most selfish, self-centered, angry person I have ever met and if he loses his temper one more time with me, I am going to punch him out!" The nurse recognizes that the client is using which defense mechanism? A. Splitting B. Projection C. Rationalization D. Denial

B. Projection

An older client with a long history of coronary artery disease (CAD), hypertension (HTN), and heart failure (HF) arrives in the Emergency Department (ED) in respiratory distress. The healthcare provider prescribes furosemide IV. Which therapeutic response to furosemide should the nurse expected in the client with acute HF? A. Increased cardiac contractility B. Reduced preload C. Relaxed vascular tone D. Decreased afterload

B. Reduced preload

When providing client care the nurse identifies a problem and develops a related clinical question. Next, the nurse intends to gather evidence so that the decision-making process in response to the problem and clinical question is evidence-based. When gathering evidence, which consideration is most important? A. Past experience with similar problems B. Relevance to the situation C. Related personal values D. Frequency that the problem occurs

B. Relevance to the situation

Three days after initiating parenteral fluids for a newborn with a ventricular septal defect (VSD), the nurse assesses an increase in heart rate and blood pressure. Which intervention is most important for the nurse to implement? A. View the graph of daily weights B. Restrict intake of oral fluids C. Assess bilateral lung sounds D. Decrease IV flow rate

B. Restrict intake of oral fluids

A client with type 2 diabetes mellitus is admitted for frequent hyperglycemic episodes and a glycosylated hemoglobin (A1C) of 10%. Insulin glargine 10 units subcutaneously once a day at bedtime and a sliding scale of insulin aspart every 6h are prescribed. What actions should the nurse include in this client's plan of care? (Select all that apply) A. Do not contaminate the insulin aspart so that it is available for IV use B. Review with the client proper foot care and prevention of injury C. Teach subcutaneous injection technique, site rotation, and insulin management D. Coordinate carbohydrate controlled meals at consistent times and intervals. E. Mix bedtime dose of insulin glargine with insulin aspart sliding scale dose F. Fingerstick glucose assessments every 6h with meals

B. Review with client proper foot care and prevention of injury C. Teach subcutaneous injection technique, site rotation, and insulin management D. Coordinate carbohydrate controlled meals at consistent times and intervals F. Fingerstick glucose assessments every 6h with meals

An older adult client with systemic inflammatory response syndrome (SIRS) has a temperature of 101.8F, heart rate of 110 beats/minute, and respiratory rate of 24 breaths/minute. Which additional finding is most important to report to the healthcare provider? A. Capillary glucose reading of 110 mg/dL B. Serum creatinine of 2.0 mg/dL C. Blood pressure of 130/88 mmHg D. Hemoglobin of 12 g/dL

B. Serum creatinine of 2.0 mg/dL

The nurse is completing the admission assessment of a 3-year old who is admitted with bacterial meningitis and hydrocephalus. Which assessment finding is evidence that the child is experiencing increased intracranial pressure (ICP)? A. Tachycardia and tachypnea B. Sluggish and unequal pupillary responses C. Increased head circumference and bulging fontanels D. Blood pressure fluctuations and syncope

B. Sluggish and unequal pupillary responses

A client is recovering in the critical care unit following a cardiac catheterization. IV nitroglycerin and heparin are infusing. The client is sedated but responds to verbal instructions. After changing positions, the client complains of pain at the right groin insertion site. What action should the nurse implement? A. Check femoral site for hematoma formation B. Stimulate the client to take deep breaths C. Evaluate the integrity of the IV insertion site D. Assess distal lower extremity capillary refill

B. Stimulate the client to take deep breaths

When conducting diet teaching for a client who is on a postoperative full liquid diet, which foods should the nurse encourage the client to eat? (Select all that apply) A. Cheese B. Tea C. Lentils D. Whole grain breads E. Potato soup

B. Tea C. Lentils E. Potato soup

The nurse identifies the presence of clear fluid on the surgical dressing of a client who just returned to the unit following lumbar spinal surgery. Which action should the nurse implement immediately? A. Change the dressing using a compression bandage B. Test the fluid on the dressing for glucose C. Document the findings in the electronic medical record D. Mark the drainage area with a pen and continue to monitor

B. Test the fluid on the dressing for glucose

The nurse is planning care for a client who admits having suicidal thoughts. Which client behavior indicates the highest risk for the client acting on these suicidal thoughts? a. Express feelings of sadness and loneliness b. Neglects personal hygiene and has no appetite c. Lacks interest in the activity of the family and friends d. Begin to show signs of improvement in affect

Begin to show signs of improvement in affect

The nurse observes that a postoperative client with a continuous bladder irrigation has a large blood clot in the urinary drainage tubing. What action should the nurse perform first? A. determine the client's blood pressure and apical pulse B. observe the amount of urine in the clients urinary drainage bag C. obtain a pulse oximeter to assess the client's oxygen saturation D. review the medication record for recently administered medications

B. observe the amount of urine in the clients urinary drainage bag. Rationale: If blood clots are present, the nurse should first determine if urinary output has become obstructed by observing the amount of urine in the urinary drainage bag (B) Continuous bladder irrigation is performed to prevent blood clots that may form and obstruct the outflow of urine

A client receiving chemotherapy has severe neutropenia. Which snack is best for the nurse to recommend to the client? a. Plain yogurt with sweetened with raw honey b. Peanuts in the shell, roasted or un-roasted. c. Aged farmer's cheese with celery sticks d. Baked apples topped with dried raisins

Baked apples topped with dried raisins Rationale: A patient with chemotherapy-induced severe neutropenia is at high risk for infection. A low bacteria diet is required D is a healthy snack for a client receiving chemotherapy. A, B and C have a high bacterial count and should be avoided.

The nurse is evaluating the diet teaching of a client with hypertension. What dinner selection indicates that the client understands the dietary recommendation for hypertension? a. Tomato soup, grilled cheese sandwich, pickles, skim milk, and lemon meringue pie. b. Baked pork chop, applesauce, corn on the cob, 2% milk, and key-lime pie. c. Grilled steak, baked potato with sour cream, green beans, coffee and raisin cream pie. d. Beed stir fry, fried rice, egg drop soup, diet coke and pumpkin pie.

Baked pork chop, applesauce, corn on the cob, 2% milk, and key-lime pie Rationale: B is limited in sodium, is high in fiber, and no additional fat is added through cooking, so it is the best choice for an antihypertensive meal. A high in sodium and cholesterol, which should be avoid. C is high in fat and caffeine which can elevate the BP D is high in sodium and cholesterol and includes caffeine.

A male infant born at 30-weeks gestation at an outlying hospital is being prepared for transport to a level IV neonatal facility. His respirations are 90/min, and his heart rate is 150 beats per min. Which drug is the transport team most likely to administer to the infant?

Beractant (SURVANTA) 100mg/kg per endotracheal tube.

A client is receiving mesalamine 800 mg PO TID. Which assessment is most important for the nurse to perform to assess the effectiveness of the medication? a. Pupillary response b. Oxygen saturation c. Peripheral pulses d. Bowel patterns

Bowel patterns Rationale: the client should be assessed for a change in bowel patterns to evaluate the effectiveness of this medication because Mesalamine is used to treat ulcerative colitis (a condition which causes swelling and sores in the lining of the colon [large intestine] and rectum) and also to maintain improvement of ulcerative colitis symptoms. Mesalamine is in a class of medications called anti-inflammatory agents. It works by stopping the body from producing a certain substance that may cause inflammation.

The mother of a child with cerebral palsy (CP) ask the nurse if her child's impaired movements will worsen as the child grows. Which response provides the best explanation? a. Brain damage with CP is not progressive but does have a variable course b. CP is one of the most common permanent physical disability in children c. Severe motor dysfunction determines the extent of successful habilitation d. Continued development of the brain lesion determines the child's outcome.

Brain damage with CP is not progressive but does have a variable course Rationale: CP is nonprogressive cerebral insult due to asphyxia, brain malformation, or toxicity, such as kernicterus. It is characterized by impair movement, posturing and may include perceptual, expressive and intellectual deficits, but the motor disabilities can vary as the child grows (A) and as interventions are implemented to prevent disuse complications.

The nurse is performing a pre-kindergarten physical on a 5 year old. The last series of vaccines will be administered. What is the preferred site for injection by the nurse? A) Vastus intermedius B) Gluteus rainlinus C) Vastus lateralis D) DorsogluteaI

is C: Vastus lateralis 12

The school nurse is teaching the faculty the most effective methods to prevent the spread of lice in the school. The information that would be most important to include would be which of these statements? A) "The treatment requires reapplication in 8 to 10 days." B) "Bedding and clothing can be boiled or steamed." C) Children are not to share hats, scarves and combs. D) Nit combs are necessary to comb out nits.

C) Children are not to share hats, scarves and combs.

Which statement made by a client to the admitting nurse suggests that the client is experiencing a manic episode? A) "I think all children should have their heads shaved. "B) "I have been restricted in thought and harmed." C) "I have powers to get you whatever you wish, no matter the cost. " D) "I think all of my contacts last week have attempted to poison me."

C) "I have powers to get you whatever you wish, no matter the cost."

While providing home care to a client with congestive heart failure, the nurse is asked how long diuretics must be taken. What is the nurse's best response? A) "As you urinate more, you will need less medication to control fluid. "B) "You will have to take this medication for about a year." C) "The medication must be continued so the fluid problem is controlled. "D) "Please talk to your health care provider about medications and treatments."

C) "The medication must be continued so the fluid problem is controlled.

A 2 year-old child has just been diagnosed with cystic fibrosis. The child's father asks the nurse "What is our major concern now, and what will we have to deal with in the future?" Which of the following is the best response? A) "There is a probability of life-long complications." B) "Cystic fibrosis results in nutritional concerns that can be dealt with." C) "Thin, tenacious secretions from the lungs are a constant struggle in cystic fibrosis." D) "You will work with a team of experts and also have access to a support group that the family can attend."

C) "Thin, tenacious secretions from the lungs are a constant struggle in cystic fibrosis."

A nurse is assessing several clients in a long term health care facility. Which client is at highest risk for development of decubitus ulcers? A) A 79 year-old malnourished client on bed rest B) An obese client who uses a wheelchair C) A client who had 3 incontinent diarrhea stools D) An 80 year-old ambulatory diabetic client

C) A client who had 3 incontinent diarrhea stools

A client in a long term care facility complains of pain. The nurse collects data about the client's pain. The first step in pain assessment is for the nurse to A) Have the client identify coping methods B) Get the description of the location and intensity of the pain C) Accept the client's report of pain D) Determine the client's status of pain

C) Accept the client's report of pain

A 35 year-old client with sickle cell crisis is talking on the telephone but stops as the nurse enters the room to request something for pain. The nurse should A) Administer a placebo B) Encourage increased fluid intake C) Administer the prescribed analgesia D) Recommend relaxation exercises for pain control

C) Administer the prescribed analgesia

A 60 year-old male client had a hernia repair in an outpatient surgery clinic. He is awake and alert, but has not been able to void since he returned from surgery 6 hours ago.He received 1000 mL of IV fluid. Which action would be most likely to help him void? A) Have him drink several glasses of water B) Crede' the bladder from the bottom to the top C) Assist him to stand by the side of the bed to void D) Wait 2 hours and have him try to void again

C) Assist him to stand by the side of the bed to void

A client is admitted with a diagnosis of hepatitis B. In reviewing the initial laboratory results, the nurse would expect to find elevation in which of the following values? A) Blood urea nitrogen B) Acid phosphatase C) Bilirubin D) Sedimentation Rate

C) Bilirubin

The nursing student is discussing with a preceptor the delegation of tasks to an unlicensed assistive personnel (UAP). Which tasks, delegated to a UAP, indicates the student needs further teaching about the delegation process? A) Assist a client post cerebral vascular accident to ambulate B) Feed a 2 year-old in balanced skeletal traction C) Care for a client with discharge orders D) Collect a sputum specimen for acid fast bacillus

C) Care for a client with discharge orders

The school nurse suspects that a third grade child might have Attention Deficit Hyperactivity Disorder. Prior to referring the child for further evaluation, the nurse should A) Observe the child's behavior on at least 2 occasions B) Consult with the teacher about how to control impulsivity C) Compile a history of behavior patterns and developmental accomplishments D) Compare the child's behavior with classic signs and symptoms

C) Compile a history of behavior patterns and developmental accomplishments

The nurse is caring for a child receiving chest physiotherapy (CPT). Which of the following actions by the nurse would be appropriate? A) Schedule the therapy thirty minutes after meals B) Teach the child not to cough during the treatment C) Confine the percussion to the rib cage area D) Place the child in a prone position for the therapy

C) Confine the percussion to the rib cage area

A mother wants to switch her 9 month-old infant from an iron fortified formula to whole milk because of the expense. Upon further assessment, the nurse finds that the baby eats table foods well, but drinks less milk than before. What is the best advice by the nurse? A) Change the baby to whole milk B) Add chocolate syrup to the bottle C) Continue with the present formula D) Offer fruit juice frequently

C) Continue with the present formula

The nurse is caring for a post myocardial infarction client in an intensive care unit. It is noted that urinary output has dropped from 60 -70 ml per hour to 30 ml per hour. This change is most likely due to A) Dehydration B) Diminished blood volume C) Decreased cardiac output D) Renal failure

C) Decreased cardiac output

The nurse is talking with a client. The client abruptly says to the nurse, "The moon is full. Astronauts walk on the moon. Walking is a good health habit." The client's behavior most likely indicates A) Neologisms B) Dissociation C) Flight of ideas D) Word salad

C) Flight of ideas

A client with heart failure has Lanoxin (digoxin) ordered. What would the nurse expect to find when evaluating for the therapeutic effectiveness of this drug? A) Diaphoresis with decreased urinary output B) Increased heart rate with increase respirations C) Improved respiratory status and increased urinary output D) Decreased chest pain and decreased blood pressure

C) Improved respiratory status and increased urinary output

A 38 year-old female client is admitted to the hospital with an acute exacerbation of asthma. This is her third admission for asthma in 7 months. She describes how she doesn't really like having to use her medications all the time. Which explanation by the nurse best describes the long-term consequence of uncontrolled airway inflammation? A) Degeneration of the alveoli B) Chronic broncho constriction of the large airways C) Lung remodeling and permanent changes in lung function D) Frequent pneumonia

C) Lung remodeling and permanent changes in lung function

Which of these parents' comment for a newborn would most likely reveal an initial finding of a suspected pyloric stenosis? A) I noticed a little lump a little above the belly button. B) The baby seems hungry all the time. C) Mild vomiting that progressed to vomiting shooting across the room. D) Irritation and spitting up immediately after feedings.

C) Mild vomiting that progressed to vomiting shooting across the room.

The nurse is talking to parents about nutrition in school aged children. Which of the following is the most common nutritional disorder in this age group? A) Bulimia B) Anorexia C) Obesity D) Malnutrition

C) Obesity

The nurse is caring for a client who is in the late stage of multiple myeloma. Which of the following should be included in the plan of care? A) Monitor for hyperkalemia B) Place in protective isolation C) Precautions with position changes D) Administer diuretics as ordered

C) Precautions with position changes

The nurse is performing a physical assessment on a client who just had an endotracheal tube inserted. Which finding would call for immediate action by the nurse? A) Breath sounds can be heard bilaterally B) Mist is visible in the T-Piece C) Pulse oximetry of 88 D) Client is unable to speak

C) Pulse oximetry of 88

Post-procedure nursing interventions for electroconvulsive therapy include A) Applying hard restraints if seizure occurs B) Expecting client to sleep for 4 to 6 hours C) Remaining with client until oriented D) Expecting long-term memory loss

C) Remaining with client until oriented

A 20 year-old client has an infected leg wound from a motorcycle accident, and the client has returned home from the hospital. The client is to keep the affected leg elevated and is on contact precautions. The client wants to know if visitors can come. The appropriate response from the home health nurse is that: A) Visitors must wear a mask and a gown B) There are no special requirements for visitors of clients on contact precautions C) Visitors should wash their hands before and after touching the client D) Visitors

C) Visitors should wash their hands before and after touching the client

The nurse receives a report on an older adult client with middle stage dementia.What information suggests the nurse should do immediate follow up rather than delegate care to the nursing assistant? The client A) Has had a change in respiratory rate by an increase of 2 breaths B) Has had a change in heart rate by an increase of 10 beats C) Was minimally responsive to voice and touch D) Has had a blood pressure change by a drop in 8 mmHg systolic

C) Was minimally responsive to voice and touch

The nurse is giving instructions to the parents of a child with cystic fibrosis. The nurse would emphasize that pancreatic enzymes should be taken A) Once each day B) 3 times daily after meals C) With each meal or snack D) Each time carbohydrates are eaten

is C: With each meal or snack 6

An adolescent, whose mother recently died, comes to the school nurse complaining of a headache. Which statement made by the students should warrant further explanation by the nurse? A. "I've had dreams about Mon since she died." B. "I've been very sad and cry a lot at night." C. "I miss Mom and would like to go see her'". D. " it's hard to concentrate on my homework"

C. "I miss Mom and would like to go see her'".

A client tells the nurse about working out with a personal trainer and swimming three times a week in an effort to lose weight and sleep better. The client states that it still is taking hours to fall asleep at night. Which action should the nurse implement? A. Advise the client that lifestyle changes often take several weeks to be effective B. Encourage the client to exercise everyday to eliminate bedtime wakefulness C. Ask the client for a description of the exercise schedule that is being followed D. Determine the amount of weight the client has lost since increasing activity

C. Ask the client for a description of the exercise schedule that is being followed

A preoperative client states he is not allergic to any medications. What is the most important nursing action for the nurse to implement next? A. Record "no known drug allergies" on preoperative checklist B. Assess client's allergies to non-drug substances C. Assess client's knowledge of an allergy response D. Flag "no known drug allergies" on the front of the chart

C. Assess client's knowledge of an allergy response

The nurse is preparing a discharge teaching plan for a client who had a liver transplant. Which instruction is most important to include in this plan? A. Increase activity and exercise gradually, as tolerated B. Limit intake of fatty foods for one month after surgery C. Avoid crowds for first two months after surgery D. Notify the healthcare provider if edema occurs

C. Avoid crowds for first two months after surgery

In assessing a client with type 1 diabetes mellitus, the nurse notes that the client's respirations have changed from 16 breaths/min with a normal depth to 32 breaths/min and deep, and the client become lethargic. Which assessment data should the nurse obtain next? A. Temperature B. Breath sounds C. Blood glucose D. White blood cell count

C. Blood glucose

A nurse who is working in the emergency department triage area is presented with four clients at the same time. The client presenting with which symptoms requires the most immediate intervention by the nurse? A. One inch bleeding laceration on the chin of crying 5 year old B. Low grade fever, headache and malaise for the past 72 hours C. Chest discomfort one hour after consuming a large, spicy meal D. Unable to bear weight on the left food, with swelling and bruising

C. Chest discomfort one hour after consuming a large, spicy meal

The nurse implements a tertiary prevention program for type 2 diabetes in a rural health clinic. Which outcome indicates that the program was effective? A. Only 30% of clients did not attend self-management education sessions. B. More than 50% of at-risk clients were diagnosed early in their disease process C. Clients who developed disease complications promptly received rehabilitation D. Average client scores improved on specific risk factor knowledge tests

C. Clients who developed disease complications promptly received rehabilitation

While the nurse is assessing an older client's fall risk, the client reports living at home alone and never falling. Which action should the nurse take? A. Inform the client that falls occur more often in the hospital than at home B. Record a minimal risk for falls, documenting the client's statement C. Continue to obtain client data needed to complete the fall risk survey D. Place the client on a high fall risk protocol because of advanced age

C. Continue to obtain client data needed to complete the fall risk survey

Prior to surgery, written consent must be obtained. Which is the nurse's legal responsibility with regard to obtaining written consent? A. Explain the surgical procedure to the client and ask the client to sign the consent form B. Ask the client or a family member to sign the surgical consent form C. Determine that the surgical consent form has been signed and is included in the client's record. D. Validate the client's understanding of the surgical procedure to be conducted

C. Determine that the surgical consent form has been signed and is included in the client's record

A client in the emergency center demonstrates rapid speech, flight of ideas, and reports sleeping only three hours during the past 48 hours. Based on these findings, it is most important for the nurse to review the laboratory value for which medication? A. Lorazepam B. Fluoxetine C. Divalproex D. Olanzapine

C. Divalproex

A male client tells the nurse that he is concerned that he may have a stomach ulcer, because he is experiencing heartburn and a dull gnawing pain that is relieved when he eats. Which is the best response by the nurse? A. Instruct the client that these mild symptoms can generally be controlled with changes in his diet B. Advise the client that he needs to seek immediate medical evaluation and treatment of these symptoms C. Encourage the client to obtain a complete physical exam, since these symptoms are consistent with an ulcer D. Assure the client that his symptoms may only reflect reflux, since ulcer pain is not relieved with food

C. Encourage the client to obtain a complete physical exam, since these symptoms are consistent with an ulcer

A client in the third trimester of pregnancy reports that she fells some "lumpy places" in her breasts and that her nipples sometimes leak a yellowish fluid. She has an appointment with her healthcare provider in two weeks. What action should the nurse take? A. Tell the client to begin nipple stimulation to prepare for breast feeding. B. Reschedule the client's prenatal appointment for the following day C. Explain that this normal secretion can be assessed at the next visit D. Recommend that the client start wearing a supportive brassiere

C. Explain that this normal secretion can be assessed at the next visit

The nurse is ready to insert an indwelling urinary catheter as seen in the picture. At this point in the procedure, what actions should the nurse take before inserting the catheter? (Select all that apply) C. Gently palpate the client's bladder for distention D. Hold the catheter 3 - 4 inches (7.5 - 10 cm) from its tip E. Secure the urinary drainage bag to the bed frame

C. Gently palpate the client's bladder for distention D. Hold the catheter 3 - 4 inches (7.5 - 10 cm) from its tip E. Secure the urinary drainage bag to the bed frame

An adolescent who was diagnosed with diabetes mellitus Type 1 at the age of 9, is admitted to the hospital in diabetic ketoacidosis. Which occurrence is the most likely cause of the ketoacidosis? A. Ate an extra peanut butter sandwich before gym class B. incorrectly administered too much insulin C. Had a cold and ear infection for the past two days D. Skipped eating lunch

C. Had a cold and ear infection for the past two days

A young woman with multiple sclerosis just received several immunizations in preparation for moving into a college dormitory. Two days later, she reports to the nurse that she is experiencing increasing fatigue and visual problems. What teaching should the nurse provide? A. Plans to move into the dormitory need to be postponed for at least a semester B. These are common side effects of the vaccines and will resolve in a few days C. Immunizations can trigger a relapse of the disease, so get plenty of extra rest D. these early signs of an infection may require medical treatment with antibiotics

C. Immunizations can trigger a relapse of the disease, so get plenty of extra rest

An older client is brought to the ED with a sudden onset of confusion that occurred after experiencing a fall at home. The client's daughter, who has power of attorney, has brought the client's prescriptions. Which information should the nurse provide first when reporting to the healthcare provider using SBAR communication? A. currently prescribed medications B. Client's healthcare power of attorney C. Increasing confusion of the client D. Fall at home as reason for admission

C. Increasing confusion of the client

A mother brings her 3-week-old son to the clinic because he is vomiting "all the time." In performing a physical assessment, the nurse notes that the infant has poor skin turgor, has lost 20% of his birth weight, and has a small palpable oval-shaped mass in his abdomen. What intervention should the nurse implement first? A. Give the infant 5% dextrose in water orally B. Insert a nasogastric tube for feeding C. Initiate a prescribed IV for parental fluid D. Feed the infant 3 ounces of Isomil

C. Initiate a prescribed IV for parental fluid

When admitting a client with a diagnosis of transient ischemic attack (TIA), which intervention is most important for the nurse to include in this client's plan of care? A. Assess bilateral breath sounds B. Review client's daily medications C. Initiate neurological monitoring every 2 hours D. Palpate suprapubic region for urinary retention

C. Initiate neurological monitoring every 2 hours

A client who is scheduled for a bronchoscopy in the morning is anxious and asking the nurse numerous questions about the procedure. In preparing the client for the procedure, which intervention has the highest priority? A. Allow client to gargle with warm salt water B. Administer a sedative to alleviate anxiety C. Instruct client to write down the questions D. Deny client's request for a midnight snack

C. Instruct client to write down the questions

A client with a new diagnosis of Raynaud's disease lives alone. Which instruction should the nurse include in the client's discharged teaching plan? A. Hire a caregiver for eight hours daily. B. Develop a walking exercise routine. C. Keep room temperature 80. D. Wear TED stockings at night.

C. Keep room temperature 80 Rationale: Keeping the environment warm (C) may minimize vasoconstriction which decreases blood flow and causes the pain associated with Raynaud's disease. The client is not helpless and does not require a caregiver (A). Exercise (B) may increase pain. TED stockings (D) have no therapeutic value for those with Raynaud's disease.

The nurse observes a client prepare a meal in the kitchen of a rehabilitation facility prior to discharge. Which behaviors indicates the client understands how to maintain balance safely? (Select all that apply) A. Brings a heavy can close to body before lifting B. Leans forward to pull on a pan from a high shelf C. Locks knees while preparing food on the counter D. Bends from the waist to pick trash off the floor E. Widens stance while working near the sink

C. Locks knees while preparing food on the counter D. Bends from the waist to pick trash off the floor

After an older client receives treatment for drug toxicity, the healthcare provider prescribes a 24-hour creatinine clearance test. Prior to starting the urine collection, the nurse notes that the client's serum creatinine is 0.3 mg/dL. Which action should the nurse implement? A. Evaluate the client's serum BUN level B. Initiate the urine collection as prescribed C. Notify the healthcare provider of the results D. Assess the client for signs of hypokalemia

C. Notify the healthcare provider of the results

The father of a 4-year-old has been battling metastatic lung cancer for the past 2 years. After discussing the remaining options with his healthcare provider, the client requests that all treatment stop and that no heroic measures be taken to save his life. When the client is transferred to the palliative care unit, which action is most important for the nurse working on the palliative care unit to take in facilitating continuity of care? A. Reassure the client that his child will be allowed to visit B. Provide the client written information about end-of-life care C. Obtain a detailed report from the nurse transferring the client D. Mark the chart with client's request for no heroic measures

C. Obtain a detailed report from the nurse transferring the client

The nurse is providing care for a client with severe peripheral arterial disease (PAD). The client reports a history of rest ischemia, with leg pain that occurs during the night. Which action should the nurse take in response to this finding? A. Elevate the legs to assess for color changes B. Provide a heating pad for PRN use C. Offer cold packs when the pain occurs D. Suggest dangling the legs when pain occurs

C. Offer cold packs when the pain occurs

The nurse has received funding to design a health promotion project for African-American women who are at risk for developing breast cancer. Which resource is most important in designing this program? A. A listing of African-American women who live in the community B. Morbidity data for breast cancer in women of all races C. Participation of community leaders in planning the program D. Technical assistance to produce a video on breast self-examination

C. Participation of community leaders in planning the program

A client is diagnosed with Meniere's disease. Which problem should the nurse identify as most important in the plan of care? A. Risk for ineffective self-health management related to deficient knowledge B. Ineffective coping related to personal vulnerability C. Risk for injury related to vertigo D. Anxiety related to disruption of lifestyle

C. Risk for injury related to vertigo.

Which laboratory finding for an adult client is most critical for the nurse to report to the healthcare provider? A. Serum sodium 142mEq/L B. Serum potassium 3.9mEq/L C. Serum glucose 62 mg/dL D. Blood urea nitrogen 18 mg/dL

C. Serum glucose 62 mg/dL

The nurse is developing a plan of care for a client who reports tingling of the feet and who is newly diagnosed with peripheral vascular disease. Which outcome should the nurse include in the plan of care for this client? A. The client's skin on the lower legs will be intact at the next clinic visit B. The client will express acceptance of their newly diagnosed health status C. The client's blood pressure readings will be less than 160/90 mmHg D. The nurse will encourage the client to walk thirty minutes every day

C. The client's blood pressure readings will be less than 160/90 mmHg

After a third hospitalization 6 months ago, a client is admitted to the hospital with ascites and malnutrition. The client is drowsy but responding to verbal stimuli and reports recently spitting up blood. What assessment finding warrants immediate intervention by the nurse? a. Bruises on arms and legs b. Round and tight abdomen c. Pitting edema in lower legs d. Capillary refill of 8 seconds

Capillary refill of 8 seconds Rationale: The client is bleeding and hypovolemia is likely. Capillary refill is greater than 3 to 5 seconds indicates poor perfusion and requires immediate attention

The mother of the 12- month-old with cystic fibrosis reports that her child is experiencing increasing congestion despite the use of chest physical therapy (CPT) twice a day, and has also experiences a loss of appetite. What instruction should the nurse provide? a. Perform CPT after meals to increase appetite and improve food intake. b. CPT should be performed more frequently, but at least an hour before meals. c. Stop using CPT during the daytime until the child has regained an appetite. d. Perform CPT only in the morning, but increase frequency when appetite improves.

CPT should be performed more frequently, but at least an hour before meals. Rationale: CPT with inhalation therapy should be performed several times a day to loosen the secretions and move them from the peripheral airway into the central airways where they can be expectorated. CPT should be done at least one hour before meals or two hours after meals.

The unit clerk reports to the charge nurse that a healthcare provider has written several prescriptions that are illegible and it appears the healthcare provider used several unapproved abbreviations in the prescriptions. What actions should the charge nurse take? a. Call the healthcare provider who wrote the prescription

Call the healthcare provider who wrote the prescription

A child with heart failure is receiving the diuretic furosemide (Lasix) and has serum potassium level 3.0 mEq/L. Which assessment is most important for the nurse to obtain? a. Cardiac rhythm and heart rate. b. Daily intake of foods rich in potassium. c. Hourly urinary output d. Thirst ad skin turgor.

Cardiac rhythm and heart rate Rationale: Hypokalemia is a side effect of potassium-wasting diuretics, such as Lasix, and manifest as muscle weakness, hypotension, tachycardia, and cardiac dysrhythmias, so changes in the child's heart rate and cardiac rhythm should be reported to the healthcare provider. Although BCD can affect the serum potassium level, the most important finding is the effect of hypokalemia on the child's cardiac rate and rhythm.

During a home visit, the nurse observed an elderly client with diabetes slip and fall. What action should the nurse take first? a. Give the client 4 ounces of orange juice b. Call 911 to summon emergency assistance c. Check the client for lacerations or fractures d. Asses clients blood sugar level

Check the client for lacerations or fractures Rationale: After the client falls, the nurse should immediately assess for the possibility of injuries and provide first aid as needed

A nurse who is working in the emergency department triage area is presented with four clients at the same time. The client presented with which symptoms requires the most immediate intervention by the nurse? a. Low-grade fever, headache, and malaise for the past 72 hours b. Unable to bear weight on the left foot, with the swelling and bruising c. Chest discomfort one hour after consuming a large, spicy meal d. One-inch bleeding laceration on the chain of the crying five-year-old

Chest discomfort one hour after consuming a large, spicy meal Rationale: Emergency triage involves quick assessment to prioritize the need for further evaluation and care. Those with trauma, chest pain, respiratory distress, or acute neurological changes are priority. In this example, while clients with other conditions require attention, the client with chest discomfort is at greatest risk and is a priority.

After diagnosis and initial treatment of a 3 year old with Cystic fibrosis, the nurse provides home care instructions to the mother, which statement by the child's mother indicates that she understands home care treatment to promote pulmonary functions? a. Chest physiotherapy should be performed twice a day before a meal.

Chest physiotherapy should be performed twice a day before a meal.

An older woman who was recently diagnosed with end stage metastatic breast cancer is admitted because she is experiencing shortness of breath and confusion. The client refuses to eat and continuously asks to go home. Arterial blood gases indicate hypoxia. Which intervention is most important for the nurse to implement? a. Clarify end of life desires

Clarify end of life desires

A mother brings her 6-year-old child, who has just stepped on a rusty nail, to the pediatrician's office. Upon inspection, the nurse notes that the nail went through the shoe and pierced the bottom of the child's foot. Which action should the nurse implement first? a. Cleanse the foot with soap and water and apply an antibiotic ointment b. Provide teaching about the need for a tetanus booster within the next 72 hours. c. have the mother check the child's temperature q4h for the next 24 hours d. transfer the child to the emergency department to receive a gamma globulin injection

Cleanse the foot with soap and water and apply an antibiotic ointment Rationale: The nurse should cleanse the wound first and implement B next.

The nurse is completing a head to be assessment for a client admitted for observation after falling out of a tree. Which finding warrants immediate intervention by the nurse? a. Sluggish pupillary response to light b. Clear fluid leaking from the nose. c. Complaint of severe headache d. Periorbital ecchymosis of right eye.

Clear fluid leaking from the nose Rationale: Clear fluid from nose or ear may be cerebrospinal fluid related to a basilar skull fracture and require immediate intervention.

The mother of a child recently diagnosed with asthma asks the nurse how to help protect her child from having asthmatic attacks. To avoid triggers for asthmatic attacks, which instructions should the nurse provide the mother? (Select all that apply) Close car windows and use air conditioner Avoid sudden changes in temperature Keep away from pets with long hair Stay indoors when grass is being cut

Close car windows and use air conditioner Avoid sudden changes in temperature Keep away from pets with long hair Stay indoors when grass is being cut

A client who is at 10-weeks gestation calls the clinic because she has been vomiting for the past 24 hours. The nurse determines that the client has no fever. Which instructions should the nurse give to this client? a. Come to the clinic to be seen by a healthcare provider

Come to the clinic to be seen by a healthcare provider

An older male client with type 2 diabetes mellitus reports that has experiences legs pain when walking short distances, and that the pain is relieved by rest. Which client behavior indicates an understanding of healthcare teaching to promote more effective arterial circulation? a. Consistently applies TED hose before getting dressed in the morning. b. Frequently elevated legs thorough the day. c. Inspect the leg frequently for any irritation or skin breakdown d. Completely stop cigarette/ cigar smoking.

Completely stop cigarette/ cigar smoking Rationale: Stopping cigarette smoking helps to decrease vasoconstriction and improve arterial circulation to the extremity.

A client with Alzheimer's disease (AD) is receiving trazodone (Desyrel), a recently prescribed atypical antidepressant. The caregiver tells the home health nurse that the client's mood and sleep patterns are improved, but there is no change in cognitive ability. How should the nurse respond to this information? a. Explain that it may take several weeks for the medication to be effective b. Confirm the desired effect of the medication has been achieved. c. Notify the health care provider than a change may be needed. d. Evaluate when and how the medication is being administered to the client.

Confirm the desired effect of the medication has been achieved. Rationale: Trazodone oR Desyrel, an atypical antidepressant, is prescribed for client with AD to improve mood and sleep.

The nurse is caring for a client receiving continuous IV fluids through a single lumen central venous catheter (CVC). Based on the CVC care bundle, which action should be completed daily to reduce the risk for infection? a. Remind staff to follow protective environment precautions b. Gently flush the catheter lumen with sterile saline solution c. Cleanse the site and change the transparent dressing. d. Confirm the necessity for continued use of the CVC.

Confirm the necessity for continued use of the CVC Rationale: Increase the length of use increase the risk for infection. The CVC care bundle includes the review of the need for continued use of the CVC. Effective hand hygiene and standard precautions should be maintained but protective environment precautions are not needed. B is not needed if continuous IV fluid are infused, ad may introduce contaminants. Use of a transparent dressing allows the site to be visualized for any signs of infection but changing the dressing daily increases the risk for infection.

A client who sustained a head injury following an automobile collision is admitted to the hospital. The nurse include the client's risk for developing increased intracranial pressure (ICP) in the plan of care. Which signs indicate to the nurse that ICP has increased? a. Increased Glasgow coma scale score. b. Nuchal rigidity and papilledema. c. Confusion and papilledema d. Periorbital ecchymosis.

Confusion and papilledema Rationale: papilledema is always an indicator of increased ICP, and confusion is usually the first sign of increased ICP. Other options do not necessarily reflect increased ICP.

A nurse plans to call the healthcare provider to report an 0600 serum potassium level of 2 mEq/L or mmol/L (SI), but the charge nurse tells the nurse that the healthcare provider does not like to receive early morning calls and will make rounds later in the morning. What action should the nurse make? a. Contact the healthcare provider immediately to report the laboratory value regardless of the advice

Contact the healthcare provider immediately to report the laboratory value regardless of the advice

A client with osteoporosis related to long-term corticosteroid therapy receives a prescription for calcium carbonate. Which client's serum laboratory values requires intervention by the nurse? a. Total calcium 9 mg/dl (2.25 mmol/L SI) b. Creatinine 4 mg/dl (354 micromol/L SI) c. Phosphate 4 mg/dl (1.293 mmol/L SI) d. Fasting glucose 95 mg/dl (5.3 mmol/L SI)

Creatinine 4 mg/dl (354 micromol/L SI

A primigravida a 40-weeks gestation with preeclampsia is admitted after having a seizure in the hot tub at a midwife's birthing center. Based on documentation in the medical record, which action should the nurse implement? (Click on each chart tab for additional information. Please be sure to scroll to the bottom right corner of each tab to view all information contained in the client's medical record.) a. Continue to monitor the client's blood pressure hourly. b. Inform the healthcare provider of CBC results c. Update the nursery staff on the client's status d. Give a dose of calcium gluconate per preeclampsia protocol.

Continue to monitor the client's blood pressure hourly Rationale: The laboratory results, urinary output, FHR, and vital signs are within expected ranges for a client who is receiving magnesium sulfate for preeclampsia. The client remains hypertensive, son continued hourly monitoring A is indicated Client magnesium center therapeutic range (5 to 7 mEq/L)

Based on the information provided in this client's medical record during labor, which should the nurse implement? (Click on each chart tab for additional information. Please be sure to scroll to the bottom right corner of each tab to view all information contained in the client's medical record.) a. Apply oxygen 10 l/mask b. Stop the oxytocin infusion c. Turn the client to the right lateral position. d. Continue to monitor the progress of labor.

Continue to monitor the progress of labor Rationale: Early deceleration are indicative of head compression as the fetus descends in the birth canal, which is a normal patter during active labor, so labor progression should continue to be monitored

The nurse is caring for a client who is entering the second stage of labor. Which action should the nurse implement first? a. Convey to the client that birth is imminent. b. Prepare the client for spinal anesthesia c. Empty the client's bladder using a straight catheter d. Prepare the coach to accompany the client to delivery

Convey to the client that birth is imminent Rationale: The second stage of labor occurs when the client is fully dilated, and the fetus is crowning, so completing preparations and informing the client that birth is imminent, so A is the first action. B is usually administered immediately prior to delivery. C is usually performed prior of after delivery D is not the priority action at this time.

The nurse is using a straight urinary catheter kit to collect a sterile urine specimen from a female client. After positioning am prepping this client, rank the actions in the sequence they should be implemented. (Place to first action on the top on the last action on the bottom.) ODCP 1. Open the sterile catheter kit close to the client's perineum. 2. Don sterile gloves and prepare to sterile field 3. Cleanse the urinary meatus using the solution, swabs, and forceps provided 4. Place distal end of the catheter in sterile specimen cup and insert catheter into meatus

Correct : ODCP 1. Open the sterile catheter kit close to the client's perineum. 2. Don sterile gloves and prepare to sterile field 3. Cleanse the urinary meatus using the solution, swabs, and forceps provided 4. Place distal end of the catheter in sterile specimen cup and insert catheter into meatus Rationale: First the kit should be open near the clients to minimize the risk of contamination during the collection of the sterile specimen. Once the kit is opened, sterile gloves should be donned to prepare the sterile field. Then the clients' meatus should be cleansed, and the catheter inserted while to distal end of the catheter drains urine into the sterile specimen cup or receptacle.

A client is complaining of intermittent, left, lower abdominal pain that began two days ago...in what order would the nurse implement the following interventions? Correct order: (DPIA) 1. Determine when the client had last bowel movement 2. Position client supine with knees bent 3. Inspect abdominal contour 4. Auscultate all four abdominal quadrants

Correct order: (DPIA) 1. Determine when the client had last bowel movement 2. Position client supine with knees bent 3. Inspect abdominal contour 4. Auscultate all four abdominal quadrants

Which action should the nurse implement with auscultating anterior breath sounds? (Place the first action on top and last action on the bottom) Correct order: PADD 1. Place stethoscope in suprasternal area to auscultate for bronchial sounds 2. Auscultate bronchovesicular sounds from side to side the first and second intercostal spaces 3. Displace female breast tissue and apply stethoscope directly on chest wall to hear vesicular sounds 4. Document normal breath sounds and location of adventitious breath sounds

Correct order: (PADD)

It is most important for the nurse to use an IV pump and/or Buretrol, an in-line volumecontrol device, when initiating IV therapy for a client following which surgical procedure?.

Craniotomy

While caring for a client's postoperative dressing, the nurse observes purulent drainage at the wound. Before reporting this finding to the healthcare provider, the nurse should review which of the client's laboratory values? a. Serum albumin b. Creatinine level c. Culture for sensitive organisms. d. Serum blood glucose (BG) level

Culture for sensitive organisms. RATIONALE: A client who has a postoperative dressing with purulent drainage from the wound is experiencing an infection. The nurse should review the client's laboratory culture for sensitive organisms (C) before reporting to the healthcare provider. (A, B and D) are not indicated at this time.

Which of these nursing diagnoses of 4 elderly clients would place 1 client at the greatest risk for falls? A) Sensory perceptual alterations related to decreased vision B) Alteration in mobility related to fatigue C) Impaired gas exchange related to retained secretions D) Altered patterns of urinary elimination related to nocturia

D) Altered patterns of urinary elimination related to nocturia

After talking with her partner, a client voluntarily admitted herself to the substance abuse unit. After the second day on the unit the client states to the nurse, "My husband told me to get treatment or he would divorce me. I don't believe I really need treatment but I don't want my husband to leave me." Which response by the nurse would assist the client? A) "In early recovery, it's quite common to have mixed feelings, but unmotivated people can't get well." B) "In early recovery, it's quite common to have mixed feelings, but I didn't know you had been pressured to come." C) "In early recovery it's quite common to have mixed feelings, perhaps it would be best to seek treatment on an out client bases. " D) " In early recovery, it's quite common to have mixed feelings. Let's discuss the benefits of sobriety for you."

D) " In early recovery, it's quite common to have mixed feelings. Let's discuss the benefits of sobriety for you."

A victim of domestic violence states to the nurse, "If only I could change and be how my companion wants me to be, I know things would be different." Which would be the best response by the nurse? A) "The violence is temporarily caused by unusual circumstances, don't stop hoping for a change. "B) "Perhaps, if you understood the need to abuse, you could stop the violence. "C) "No one deserves to be beaten. Are you doing anything to provoke your spouse into beating you?" D) "Batterers lose self-control because of their own internal reasons, not because of what their partner did or did not do."

D) "Batterers lose self-control because of their own internal reasons, not because of what their partner did or did not do."

An unlicensed assistive personnel (UAP), who usually works on a surgical unit is assigned to float to a pediatric unit. Which question by the charge nurse would be most appropriate when making delegation decisions? A) "How long have you been a UAP and what units you have worked on?" B) "What type of care do you give on the surgical unit and what ages of clients?" C) "What is your comfort level in caring for children and at what ages?" D) "Have you reviewed the list of expected skills you might need on this unit?"

D) "Have you reviewed the list of expected skills you might need on this unit?"

Which task could be safely delegated by the nurse to an unlicensed assistive personnel (UAP)? A) Be with a client who self-administers insulin B) Cleanse and dress a small decubitus ulcer C) Monitor a client's response to passive range of motion exercises D) Apply and care for a client's rectal pouch

D) Apply and care for a client's rectal pouch

The nursing intervention that best describes treatment to deal with the behaviors of clients with personality disorders include A) Pointing out inconsistencies in speech patterns to correct thought disorders B) Accepting client and the client's behavior unconditionally C) Encouraging dependency in order to develop ego controls D) Consistent limit-setting enforced 24 hours per day

D) Consistent limit-setting enforced 24 hours per day

A client frequently admitted to the locked psychiatric unit repeatedly compliments and invites one of the nurses to go out on a date. The nurse's response should be to A) Ask to not be assigned to this client or to work on another unit B) Tell the client that such behavior is inappropriate C) Inform the client that hospital policy prohibits staff to date clients D) Discuss the boundaries of the therapeutic relationship with the client

D) Discuss the boundaries of the therapeutic relationship with the client

The nurse is performing a developmental assessment on an 8 month-old. Which finding should be reported to the health care provider? A) Lifts head from the prone position B) Rolls from abdomen to back C) Responds to parents' voices D) Falls forward when sitting

D) Falls forward when sitting

A client says, "It's raining outside and it's raining in my heart. Did you know that St. Patrick drove the snakes out of Ireland? I've never been to Ireland." The nurse would document this behavior as A) Perseveration B) Circumstantiality C) Neologisms D) Flight of ideas

D) Flight of ideas

A client has been receiving lithium (Lithane) for the past two weeks for the treatment of bipolar illness. When planning client teaching, what is most important to emphasize to the client? A) Maintain a low sodium diet B) Take a diuretic with lithium C) Come in for evaluation of serum lithium levels every 1-3 months D) Have blood lithium levels drawn during the summer months

D) Have blood lithium levels drawn during the summer months

A newly admitted adult client has a diagnosis of hepatitis A. The charge nurse should reinforce to the staff members that the most significant routine infection control strategy, in addition to hand washing, to be implemented is which of these? A) Apply appropriate signs outside and inside the room B) Apply a mask with a shield if there is a risk of fluid splash C) Wear a gown to change soiled linens from incontinence D) Have gloves on while handling bedpans with feces

D) Have gloves on while handling bedpans with feces

Which of these clients who call the community health clinic would the nurse ask to come in that day to be seen by the health care provider? A) I started my period and now my urine has turned bright red. B) I am an diabetic and today I have been going to the bathroom every hour. C) I was started on medicine yesterday for a urine infection. Now my lower belly hurts when I go to the bathroom. D) I went to the bathroom and my urine looked very red and it didn't hurt when I went.

D) I went to the bathroom and my urine looked very red and it didn't hurt when I went.

A client is admitted with a pressure ulcer in the sacral area. The partial thickness wound is 4cm by 7cm, the wound base is red and moist with no exudate and the surrounding skin is intact. Which of the following coverings is most appropriate for this wound? A) Transparent dressing B) Dry sterile dressing with antibiotic ointment C) Wet to dry dressing D) Occlusive moist dressing

D) Occlusive moist dressing

The nurse is about to assess a 6 month-old child with nonorganic failure-to thrive (NOFTT). Upon entering the room, the nurse would expect the baby to be A) Irritable and "colicky" with no attempts to pull to standing B) Alert, laughing and playing with a rattle, sitting with support C) Skin color dusky with poor skin turgor over abdomen D) Pale, thin arms and legs, uninterested in surroundings

D) Pale, thin arms and legs, uninterested in surroundings

A client was admitted to the psychiatric unit with a diagnosis of bipolar disorder. He constantly bothers other clients, tries to help the housekeeping staff, demonstrates pressured speech and demands constant attention from the staff. Which activity would be best for the client? A) Reading B) Checkers C) Cards D) Ping-pong

D) Ping-pong

The nurse receives an order to give a client iron by deep injection. The nurse know that the reason for this route is to A) Enhance absorption of the medication B) Ensure that the entire dose of medication is given C) Provide more even distribution of the drug D) Prevent the drug from tissue irritation Skip

D) Prevent the drug from tissue irritation Skip

The nurse asks a client with a history of alcoholism about the client's drinking behavior. The client states "I didn't hurt anyone. I just like to have a good time, and drinking helps me to relax." The client is using which defense mechanism? A) Denial B) Projection C) Intellectualization D) Rationalization

D) Rationalization

A nurse checks a client who is on a volume-cycled ventilator. Which finding indicates that the client may need suctioning?A) Drowsiness B) Complaint of nausea C) Pulse rate of 92 D) Restlessness

D) Restlessness

The nurse is discussing nutritional requirements with the parents of an 18 month-old child. Which of these statements about milk consumption is correct? A) May drink as much milk as desired B) Can have milk mixed with other foods C) Will benefit from fat-free cow's milk D) Should be limited to 3-4 cups of milk daily

D) Should be limited to 3-4 cups of milk daily

While the school nurse is teaching a group of 14-year-olds, one of the participants remarks, "You are too young to be our teacher! You're not much older than we are!" How should the nurse respond? A. "I think I am qualified to teach this group." B. "How old do you think I am?" C. "Do you think you can teach it any better?" D. "We need to stay focused on the topic."

D. "We need to stay focused on the topic." Rationale: (D) is the best response since the nurse should keep the students focused and avoid entering into an argument with them. (A) is defensive; there is no need for the nurse to defend her/his position. (B) is irrelevant; it does not matter how old the student thinks the nurse is. (C) is sarcastic; the nurse should avoid this kind of exchange and remain professional.

A young adult male who is being seen at the employee health care clinic for an annual assessment tells the nurse that his mother was diagnosed with schizophrenia when she was his age and that life with a schizophrenic mother was difficult indeed. Which response is best for the nurse to provide? A. Encourage the client to seek genetic counseling to determine his risk for mental illness B. Inform the client that his mother's schizophrenia has affected his psychological development C. Tell the client that mental illness has a familial predisposition so he should see a psychiatrist D. Ask the client if he is worried about becoming schizophrenic at the age his mother was diagnosed

D. Ask the client if he is worried about becoming schizophrenic at the age his mother was diagnosed

A client with postpartum depression, who is admitted to the behavioral health unit, refuses to leave her room or eat meals. In addition to maintaining physical safety, which short-term goal should the nurse include in the plan of care? A. Sleeps at least 6 hours per night B. Consumes 3 meals and 1500 mL of fluid per day C. Engages in one client to client interaction daily D. Attends one group activity per day

D. Attends one group activity per day

The nurse is preparing a client for discharge who was hospitalized with an acute flare of systemic lupus erythematosus (SLE) symptoms. Which instruction is most important for the nurse to include? A. Use a walker when weakness occurs B. Take prescribed cortisone accurately C. Decrease daily intake of sodium in diet D. Avoid extreme environmental temperatures

D. Avoid extreme environmental temperatures

When the nurse enters the room of a male client who was admitted for a fractured femur, his cardiac monitor displays a normal sinus rhythm, but he has no spontaneous respirations and his carotid pulse is not palpable. Which intervention should the nurse implement? A. Analyze the cardiac rhythm in another lead B. Obtain a 12-lead electrocardiogram C. Observe for swelling at the fracture site D. Begin chest compressions at 100/minute

D. Begin chest compressions at 100/minute

A client presents to the labor and delivery unit with a report of leaking fluid that is greenish-brown vaginal discharge. Which action should the nurse take first? A. Start an intravenous infusion B. Administer oxygen via facemask C. Perform a vaginal exam D. Begin continuous fetal monitoring

D. Begin continuous fetal monitoring

An older male client, who is a retired chef, is hospitalized with a diabetic ulcer on his foot. His daughter tells the nurse that her father has become increasingly obsessed with the way his food is prepared in the hospital. The nurse's response should be based on what information? A. The client probably has an organic brain disease and will likely have Alzheimer's disease within a few years B. The family needs a social worker to talk to them about how to handle their father when he becomes annoying C. The daughter is under stress and should be encouraged to think about happier times D. If the client was compulsive about food when he was younger, the aging process can magnify this

D. If the client was compulsive about food when he was younger, the aging process can magnify this

The nurse instructs a client in use of a incentive spirometer. The client performs a return demonstration as seen in the video. Which action should the nurse take in response to the return demonstration? A. Auscultate the client's lungs for adventitious sounds B. Encourage the client to practice until successful C. Emphasize the need to inhale slowly into the spirometer D. Remind the client to cough after using the spirometer

D. Remind the client to cough after using the spirometer

The healthcare provider prescribes a sepsis protocol for a client with multi-organ failure caused by a ruptured appendix. Which intervention is most important for the nurse to include in the plan of care? A. Assess warmth of extremities B. Keep head of bed raised 45 degrees C. Monitor blood glucose level D. Maintain strict intake and output

D. Maintain strict intake and output

A 26-year-old female client is admitted to the hospital for treatment of a simple goiter, and levothyroxine sodium (Synthroid) is prescribed. Which symptoms indicate to the nurse that the prescribed dosage is too high for this client? The client experiences: A. Bradycardia and constipation B. Lethargy and lack of appetite C. Muscle cramping and dry, flushed skin D. Palpitations and shortness of breath

D. Palpitations and shortness of breath

A client with urge incontinence was treated with onabotuilinumtoxinA injections and is now experiencing urinary retention. Which action should the nurse include in the client's plan of care? A. Provide a bedside commode for immediate use in the client's room B. Teach the client techniques for performing intermittent catheterization C. Explain the need to limit intake of oral fluids to reduce client discomfort D. Remind the client to practice pelvic floor (Kegel) exercises regularly

D. Remind the client to practice pelvic floor (Kegel) exercises regularly

A teenager presents to the emergency department with palpitations after vaping at a party. The client is anxious, fearful, and hyperventilating. The nurse anticipates the client developing which acid base imbalance? A. Respiratory acidosis B. Metabolic alkalosis C. Metabolic acidosis D. Respiratory alkalosis

D. Respiratory alkalosis

A 7-year old is admitted to the hospital with persistent vomiting, and a nasogastric tube attached to low intermittent suction is applied. Which finding is most important for the nurse to report to the healthcare provider? A. Shift intake of 640mL IV fluids plus 30mL PO ice chips B. Serum pH of 7.45 C. Gastric output of 100 mL in the last 8 hours D. Serum potassium of 3.0 mg/dL

D. Serum potassium of 3.0 mg/dL

A client is scheduled for a spiral computed tomography (CT) scan with contrast to evaluate for pulmonary embolism. Which information in the client's history requires follow-up by the nurse? A. CT scan that was performed 6 months earlier B. Metal hip prosthesis was placed 20 years ago C. Report of client's sobriety for the last 5 years D. Takes metformin for type 2 diabetes mellitus

D. Takes metformin for type 2 diabetes mellitus

An unlicensed assistive personnel (UAP) informs the nurse who is giving medications that a female client is crying. The client was just informed that she has a malignant tumor. What action should the nurse implement first? A. Provide the client with a PRN antianxiety medication and allow privacy for her to grieve. B. Instruct the UAP to notify the client's spiritual advisor of her need for counseling. C. Ask another nurse to finish giving medications and attend to the client immediately. D. Tell the client that the nurse will be back to talk to her after medications are given.

D. Tell the client that the nurse will be back to talk to her after medications are given. Rationale: The nurse should first acknowledge the client's grief and arrange with the client a time to provide support, then complete the administration of medications (D). During that time, the nurse should assess the client to determine what intervention is best to offer the grieving client (A). The nurse, not the UAP (B), should talk with the client to determine if the client needs her spiritual advisor's counsel and support. The client's grief is not an emergency situation, so after acknowledging it and informing her that the nurse will return to talk with her, the nurse should finish administering the medications (C) and then spend time supporting the grieving client.

While caring for a toddler receiving oxygen (02) via face mask, the nurse observes that the child's lips and nares are dry and cracked. Which intervention should the nurse implement? A.) Ask the mother what she usually uses on the child's lips and nose B.) Apply a petroleum jelly (Vaseline) to the child's nose and lips C.) Use a topical lidocaine (Zylocaine viscous) analgesic for cracked lips D.) Use a water soluble lubricant on affected oral and nasal mucosa

D.) Use a water soluble lubricant on affected oral and nasal mucosa

A client with hyperthyroidism is receiving propranolol (Inderal). Which finding indicates that the medication is having the desired effect a. Decrease in serum T4 levels b. Increase in blood pressure c. Decrease in pulse rate d. Goiter no longer palpable.

Decrease in pulse rate Rationale: Beta blockers such as propranolol help control the symptoms of hyperthyroidism, such as palpitations or tachycardia, but do not alter thyroid hormone levels, B is not a desired effect in hyperthyroidism. Beta blocker do not impact the presence of a goiter.

An increased number of elderly persons are electing to undergo a new surgical procedure which cures glaucoma. What effect is the nurse likely to note as a result of this increases in glaucoma surgeries? a. Decrease morbidity in the elderly population b. Decrease prevalence of glaucoma in the population. c. Increase mortality in the elderly population d. Increased incidence of glaucoma in the population.

Decrease prevalence of glaucoma in the population.

The nurse is preparing to administer a histamine 2-receptor antagonist to a client with peptic ulcer disease. What is the primary purpose of this drug classification? a. Neutralize hydrochloric (HCI) acid in the stomach b. Decreases the amount of HCL secretion by the parietal cells in the stomach c. Inhibit action of acetylcholine by blocking parasympathetic nerve endings. d. Destroys microorganisms causing stomach inflammation.

Decreases the amount of HCL secretion by the parietal cells in the stomach Rationale: B correctly describe the action of histamine 2 receptor antagonist in helping to prevent peptic ulcer disease.

An elderly male client is admitted to the mental health unit with a sudden onset of global disorientation and is continuously conversing with his mother, who died 50 years ago. The nurse reviews the multiple prescriptions he is currently taking and assesses his urine specimen, which is cloudy, dark yellow, and has foul odor. These findings suggest that his client is experiencing which condition? a. Delirium b. Depression c. Dementia d. Psychotic episode

Delirium Rationale: The client's clinical findings-polypharmia, urinary tract infection, and possible fluid imbalance are the most common causes of cognition and memory impairment, which is characteristic of delirium.

A female client is admitted for diabetic crisis resulting from inadequate dietary practices. After stabilization, the nurse talks to the client about her prescribed diet. What client characteristic is most import for successful adherence to the diabetic diet? a. Demonstrates willingness to adhere to the diet consistently

Demonstrates willingness to adhere to the diet consistently

A client with hyperthyroidism is being treated with radioactive iodine (I-131). Which explanation should be included in preparing this client for this treatment? a. Explain the need for using lead shields for 2 to 3 weeks after the treatment b. Describe the signs of goiter because this is a common side effect of radioactive iodine c. Explain that relief of the signs/symptoms of hyperthyroidism will occur immediately d. Describe radioactive iodine as a tasteless, colorless medication administered by the healthcare provider

Describe radioactive iodine as a tasteless, colorless medication administered by the healthcare provider Rationale: A single dose of tasteless, colorless radioactive iodine is administered by mouth and the client is observed for signs of thyroid storm. 85% of clients are cure by one dose. The dose of radioactivity is not enough to warrant (A). B is indicated for a client receiving iodine or iodine compound medications in the treatment of hyperthyroidism. It takes 3 to 4 weeks for sings of hyperthyroidism to subside.

An adolescent with major depressive disorder has been taking duloxetine (Cymbalta) for the past 12 days. Which assessment finding requires immediate follow-up a. Describes life without purpose b. Complains of nausea and loss of appetite c. States is often fatigued and drowsy d. Exhibits an increase in sweating.

Describes life without purpose Rationale: Cymbalta is a selective serotonin and norepinephrine reuptake inhibitor that is known to increase the risk of suicidal thinking in adolescents and young adults with major depressive disorder. B, C and D are side effects

After a colon resection for colon cancer, a male client is moaning while being transferred to the Postanesthesia Care Unit (PACU). Which intervention should the nurse implement first? a. Assess the client's dressing for bleeding b. Determine client's pulse, blood pressure, and respirations c. Administer a PRN dose of IV Morphine d. Check the client's orientation to time and place.

Determine client's pulse, blood pressure, and respirations Rationale: Colon resection, a major abdominal surgical procedure, causes severe pain in the immediate postoperative period and requires administration of IV morphine regularly to maintain analgesic serum level. Before administering a central nervous system depressing analgesia, the client's vital signs should be assessed to determine the client's current level of CNS depression. In the immediate postoperative period, during administration to PACU (A, C and D) should be evaluated.

A new mother tells the nurse that she is unsure if she will be able to transition into parenthood. What action should the nurse take? a. Provide reassurance to the client that these feeling are normal after delivery b. Discuss delaying the client's discharge from the hospital for another 24 hrs. c. Determine if she can ask for support from family, friend, or the baby's father. d. Explain the differences between postpartum blues and postpartum depression.

Determine if she can ask for support from family, friend, or the baby's father Rationale: Emotional support of significant family and friends can help a new mother cope with anxiety about transitioning to parenthood. The nurse should ask the client who is available to support her.

The nurse is preparing an intravenous (IV) fluid infusion using an IV pump. Within 30 seconds of turning on the machine, the pump's alarm beeps "occlusion". What action should the nurse implement first? a. Flush the vein with 3 ml of sterile normal saline. b. Assess the IV catheter insertion site for infiltration. c. Verify the threading of the tubing through the IV pump. d. Determine if the clamp on the IV tubing is released

Determine if the clamp on the IV tubing is released Rational: When the pump immediately beeps, it is often because the IV tubing clamp is occluding the flow, so the clamp should be checked first to ensure that it is open. If the alarm is not eliminated after the tubing clamp is released, flushing the IV site with saline is a common practice to clean the needle or to identify resistance due to another source. Local signs of infiltration may indicate the need to select another vein, but the pump's beeping-this early in the procedure is likely due to a mechanical problem. If beeping continues after verifying that the clamp is released the placement or threading of the tubing through the pump should be verified.

A young boy who is in a chronic vegetative state and living at home is readmitted to the hospital with pneumonia and pressure ulcers. The mother insists that she is capable of caring for her son and which action should the nurse implement next? a. Report the incident to the local child protective services. b. Find a home health agency that specializes in brain injuries. c. Determine the mother's basic skill level in providing care. d. Consult the ethics committee to determine how to proceed.

Determine the mother's basic skill level in providing care Rational: Although the mother states she is a capable caregiver, the client is manifesting disuse syndrome complications, and the mother's skill in providing basic care should be determined. Further assessment is needed before implementing other nursing actions.

The nurse tests a client's visual acuity and determines that the uncorrected vision is 20/100 in the right eye and 20/80 in the left eye. What does this finding indicate?

Difficulty visualizing objects at a distance also known as myopia.

A female client reports she has not had a bowel movement for 3 days, but now is defecating frequent small amount of liquid stool. Which action should the nurse implement? Digitally check the client for a fecal impaction

Digitally check the client for a fecal impaction

An infant who is admitted for surgical repair of a ventricular septal defect (VSD) is irritable and diaphoretic with jugular vein distention. Which prescription should the nurse administer first? a. Spironolactone b. Potassium c. Ampicillin sodium parental d. Digoxin.

Digoxin. Rationale: This infant is demonstrating early signs of heart failure due to an increase right ventricular workload caused by a left to right shunt through the VSD, son an inotropic, such as digoxin should be administered first to improve the efficiency of myocardial contractility. Next a high ceiling diuretic to reduce fluid volume and workload of the heart. If hypokalemia occurs as result of potassium-wasting diuretic, should be given to reduce the risk of digoxin toxicity.

The nurse and an unlicensed assistive personnel (UAP) are providing care for a client with a nasogastric tube (NGT) when the client begins to vomit. How should the nurse manage this situation? a. Direct the UAP to measure the emesis while the nurse irrigates the NGT

Direct the UAP to measure the emesis while the nurse irrigates the NGT

Before preparing a client for the first surgical case of the day, a part-time scrub nurse asks the circulating nurse if a 3 minute surgical hand scrub is adequate preparation for this client. Which response should the circulating nurse provide? a. Ask a more experience nurse to perform that scrub since it is the first time of the day b. Validate the nurse is implementing the OR policy for surgical hand scrub c. Inform the nurse that hand scrubs should be 3 minutes between cases. d. Direct the nurse to continue the surgical hand scrub for a 5-minute duration.

Direct the nurse to continue the surgical hand scrub for a 5 minute duration Rationale: The surgical hand scrub should last for 5 to 10 mints, so the nurse should be directed to continue the vigorous scrub using a reliable agent for the total duration of 5 mints. It is not necessary to reassign staff (A). The length of the hand scrub and subsequent scrubs during the day require the same process for the same amount of time, (B and C)

During a routine clinic visit, an older female adult tells the nurse that she is concerned that the flu season is coming soon, but is reluctant to obtain the vaccination. What action should the nurse take first? a. Determine when the client last had an influenza vaccination. b. Discuss the concerns expressed by the client about the vaccination. c. Ask about any recent exposure to persons with the flu or other viruses. d. Review the informed consent form for the vaccination with the client.

Discuss the concerns expressed by the client about the vaccination Rationale: the nurse should first address the concerns identified by the client, before taking other actions, such as obtaining information about past vaccinations, exposure to the flu, or reviewing the informed consent form.

Which assessment is more important for the nurse to include in the daily plan of care for a client with a burned extremity? a. Distal pulse intensity

Distal pulse intensity

Which assessment is more important for the nurse to include in the daily plan of care for a client with a burned extremity a. Range of Motion b. Distal pulse intensity c. Extremity sensation d. Presence of exudate

Distal pulse intensity Rationale: Distal pulse intensity assesses the blood flow through the extremity and is the most important assessment because it provides information about adequate circulation to the extremity. Range of motions evaluates the possibility of long term contractures sensation. C evaluates neurological involvement, and exudate. D provides information about wound infection, but this assessment does not have the priority of determining perfusion to the extremity.

The nurse is preparing to administer 1.6 ml of medication IM to a 4 month old infant. Which action should the nurse include? a. Select a 22 gauge 1 ½ inch (3.8 cm) needle for the intramuscular injection b. Administer into the deltoid muscle while the parent holds the infant securely c. Divide the medication into two injections with volumes under 1ml d. Use a quick dart-like motion to inject into the dorsogluteal site.

Divide the medication into two injection with volumes under 1ml Rationale: IM injection for children under 3 of age should not exceed 1ml. divide the dose into smaller volumes for injection in two different sites.

A woman just received the Rubella vaccine after a delivery of a normal new born, has two children at home, ages 13 months and 3 years. Which instruction is most important to provide to the client? a- Refrain from eating foods containing eggs for 24 hors b- Breast feeding is recommended to prevent ovulation. c- Do not get pregnant for at least 3 months. d- Avoid exposure to the sunlight for 36 hours.

Do not get pregnant for at least 3 months Rationale: The rubella vaccine can be harmful to an unborn child who is conceived within 3 month of the vaccination.

In assessing an adult client with a partial rebreather mask, the nurse notes that the oxygen reservoir bag does not deflate completely during inspiration and the client's respiratory rate is 14 breaths / minute. What action should the nurse implement a. Encourage the client to take deep breaths b. Remove the mask to deflate the bag c. Increase the liter flow of oxygen d. Document the assessment data

Document the assessment data Rational: reservoir bag should not deflate completely during inspiration and the client's respiratory rate is within normal limits.

To prevent infection by auto contamination during the acute phase of recovery from multiple burns, which intervention is most important for the nurse to implement? a. Dress each wound separately. b. Avoid sharing equipment between multiple clients. c. Use gown, mask and gloves with dressing change. d. Implement protective isolation.

Dress each wound separately. Rational: each wound should be dressed separately using a new pair of sterile glove to avoid auto contamination (the transfer of microorganisms' form one infected wound to a non-infected wound). The other choices do not prevent auto contamination.

The nurse is planning a class for a group of clients with diabetes mellitus about blood glucose monitoring. In teaching the class as a whole, the nurse should emphasize the need to check glucose levels in which situation? a. Prior to exercising b. Immediately after meals c. Before going to bed d. During acute illness.

During acute illness Rationale: Client should be instructed to always check their blood glucose whenever they feel sick or nauseated. There is great variability in recommendations for frequency of blood glucose testing. When first diagnosed, clients are often advised to test before and after meals and at bedtime, then after meals and at bedtime for a short period. Once they are stable, clients may be advised to test as often as four times a day or as little as once each week, depending on the consistency of their diet and exercise.

Which instruction should the nurse provide a pregnant client who is complaining of heartburn? a. Limit fluids between meals to avoid over distension of the stomach b. Take an antacid at bedtime and whenever symptoms worsen c. Maintain a sitting position for two hours after eating. d. Eat small meal throughout the day to avoid a full stomach.

Eat small meal throughout the day to avoid a full stomach. Rationale: Eating small frequent meals throughout the day decreases stomach fullness and helps decrease heartburn. Fluids should not be consumed with foods because they further distend the stomach, but fluids not be limited between meals (A) because this puts the client at risk for dehydration. (B) is not recommended during pregnancy unless prescribed by the health provider because they place the client at risk for electrolyte imbalance (sodium), constipation (aluminum, or diarrhea (magnesium) (C) is less effective than (D) preventing heartburn.

Who is at risk for rib fracture

Elderly pt.

A client is admitted to the surgical unit with symptoms of a possible intestinal obstruction. When preparing to insert a nasogastric (NG) tube, which intervention should the nurse implement? a. Elevate the head of the bed 60 to 90 degrees

Elevate the head of the bed 60 to 90 degrees

A female client is extremely anxious after being informed that her mammogram was abnormal and needs to be repeated. Client is tearful and tells the nurse her mother died of breast cancer. What action should the nurse take? a. Provide the client with information about treatment options for breast cancer. b. Reassure the client that the final diagnosis has not been made. c. Encourage the client to continue expressing her fears and concerns. d. Suggest to the client that she seek a second opinion.

Encourage the client to continue expressing her fears and concerns. Rational: the nurse should show support for the client by encouraging her to continue expressing her concerns. A diagnosis has not yet been made, so it is too early to discuss treatment options. Other options dismiss the client's feelings or are premature given that the diagnosis is not yet made.

A resident of a long-term care facility, who has moderate dementia, is having difficulty eating in the dining room. The client becomes frustrated when dropping utensils on the floor and then refuses to eat. What action should the nurse implement? a. Allow client to choose foods from a menu b. Assign a staff member to feed the client c. Have meals brought to the client's room d. Encourage the client to eat finger foods.

Encourage the client to eat finger foods Rationale: Eyes-hand coordination is often affected with dementia. Providing a way to eat without using utensils is likely to help the client maintain independence while obtaining adequate nutrition. A: increase frustration.

A male client tells the nurse that he is concerned that he may have a stomach ulcer, because he is experiencing heartburn and a dull growing pain that is relieved when he eats. What is the best response by the nurse? a. Encourage the client to obtain a complete physical exam since these symptoms are consistent with an ulcer

Encourage the client to obtain a complete physical exam since these symptoms are consistent with an ulcer

An elderly female is admitted because of a change in her level of sensorium. During the evening shift, the client attempts to get out bed and falls, breaking her left hip. Buck's skin traction is applied to the left leg while waiting for surgery. Which intervention is most important for the nurse to include in this client's plan care? a. Evaluate her response to narcotic analgesia b. Asses the skin under the traction moleskin c. Place a pillow under the involved lower left leg d. Ensure proper alignment of the leg in traction.

Ensure proper alignment of the leg in traction. Rationale: A fractured hip results in external rotation and shortening of the affected extremity. With the application of Buck's skin traction proper alignment ensures the transaction S pull is exerted to align the fracture hip with the distal leg, immobilize the fractured bone, and minimize muscle spasms and surrounding tissue injury related to the fracture. A should be implement but improper pull of traction can increase pain and soft tissue damage. B and C should be implemented but the greatest risk is improper alignment of the traction.

Before leaving the room of a confused client, the nurse notes that a half bow knot was used to attach the client's wrist restraints to the movable portion of the client's bed frame. What action should the nurse take before leaving the room? a. Ensure that the knot can be quickly released. b. Tie the knot with a double turn or square knot. c. Move the ties so the restraints are secured to the side rails. d. Ensure that the restraints are snug against the client's wrist.

Ensure that the knot can be quickly released.

A vacuum-assistive closure (VAC) device is being use to provide wound care for a client who has stage III pressure ulcer on a below-the- knee (BKA) residual limb. Which intervention should the nurse implement to ensure maximum effectiveness of the device? a. Empty the device every 8 hours and change the dressing daily ensure sterility b. Extended the transparent film dressing only to edge of wound to prevent tension. c. Ensure the transparent dressing has no tears that might create vacuum leaks d. Use an adhesive remover when changing the dressing to promote comfort.

Ensure the transparent dressing has no tears that might create vacuum leak Rationale: The nurse should ensure that the VAC transparent film is intact, without tears or loose edges C) because a break in the seal resulting in drying the wound and decreasing the vacuum. The vacuum-assisted closure (VAC) device uses an open sponge in the wound bed, sealed with a transparent film dressing and tube extrudes to a suction device that exert negative pressure to remove excess wound fluid, reduce the bacterial count and stimulate granulation. The VAC is changed every other day or third day, not (A) depending on the stage of wound healing and emptied when full or weekly. The transparent wound dressing should extend 3 to 5 cm beyond the wound edges, not (B) to ensure and airtight seal. Adhesive removers leave a reduce that binder transparent film adherence (D)

After receiving the first dose of penicillin, the client begins wheezing and has trouble breathing. The nurse notifies the healthcare provider immediately and received several prescriptions. Which medication prescription should the nurse administer first? a. Epinephrine Injection, USP IV b. Diphenhydramine IV c. Albuterol (Ventolin) inhaler d. Methylprednisolone IV

Epinephrine Injection, USP IV Rationale: Epinephrine should be administered immediately to open the airway and raise the blood pressure by vasoconstricting the blood vessels. All other medications should be administered after the epinephrine is given.

Denial

Ex: woman with cancer says her tests are negative

The nurse ask the parent to stay during the examination of a male toddler's genital area. Which intervention should the nurse implement? a. Examine the genitalia as the last part of the total exam. b. Use soothing statements to facilitate cooperation c. Allow the child to keep underpants on to examine genitalia d. Work slowly and methodically so not to stress the child

Examine the genitalia as the last part of the total exam. Rationale: Examination of a child's genitalia is particularly stressful to toddles, so this assessment is best left as the last part of the examination. B are best done by a parent, not the nurse. The genitals must be completely visualized and sometimes palpates underwear for a brief period of.

A young adult woman visits the clinic and learns that she is positive for BRCA1 gene mutation and asks the nurse what to expect next. How should the nurse respond? a. Explain that counseling will be provided to give her information about her cancer risk. b. Gather additional information about the client's family history for all types of cancer. c. Offer assurance that there are a variety of effective treatments for breast cancer. d. Provide information about survival rates for women who have this genetic mutation.

Explain that counseling will be provided to give her information about her cancer risk Rational: BRACA1or BRACA2 genetic mutation indicates an increased risk for developing breast or ovarian cancer and genetic counseling should be provided to explain the increased risk (A)to the client along with options for increased screening or preventative measures. (B) Is completed by the genetic counselor before the client undergoes genetic testing. a positive BRACA1test is not an indicator of the presence of cancer and (C and D) are not appropriate responses prior to genetic counseling.

A young adult female with chronic kidney disease (CKD) due to recurring pyelonephritis is hospitalized with basilar crackles and peripheral edema. She is complaining of severe nausea and the cardiac monitor indicates sinus tachycardia with frequent premature ventricular contraction. Her blood pressure is 200 /110 mm Hg, and her temperature is 101 F which PRN medication should the nurse administers first? a. Furosemide Rationale: Furosemide is given to help treat fluid retention (edema) and swelling that is caused by congestive heart failure, liver disease, kidney disease, or other medical conditions. It works by acting on the kidneys to increase the flow of urine.

Furosemide

The husband of an older woman, diagnosed with pernicious anemia, calls the clinic to report that his wife still has memory loss and some confusion since she received the first dose of nasal cyanocobalamin two days ago. He tells the nurse that he is worried that she might be getting Alzheimer's disease. What action should the nurse take? a- Explain that memory loss and confusion are common with vitamin B12 deficiency. b- Ask if the client is experiencing any changes in bowel habits c- Determine if the client is taking iron and folic acid supplements d- Encourage the husband to bring the client to the clinic for a complete blood count.

Explain that memory loss and confusion are common with vitamin B12 deficiency Rationale: Pernicious anemia is related to the absence of intricic factor in gastric secretions, leading to malabsorption of vit B12, and commonly causes memory loss, confusion and cognitive problems, and GI manifestations. The nurse should reassure the husband that the client's signs are consistent with the primary disease. Although B, C and D provide additional information about the client's compliance and response to therapy, a quick and dramatic response can occur after 72 hrs. of B12 injections.

When performing postural drainage on a client with Chronic Obstructive Pulmonary Disease (COPD), which approach should the nurse use? a. Perform the drainage immediately after meals b. Instruct the client to breath shallow and fast c. Obtain arterial blood gases (ABG's) prior to procedure d. Explain that the client may be placed in five positions

Explain that the client may be placed in five positions Rationale: Frequently, the client is placed in five positions (head down, prone, right and left lateral, and sitting upright) to aid in drainage of each of the five lobes of the lungs (D). Postural drainage should be performed before meals to prevent nausea, vomiting and aspiration(A). The client should breath slow and exhale through pursed lips to help keep airway open so that secretions can be drained while assuming the various positions. C is not required

An adult male is admitted to the emergency department after falling from a ladder. While waiting to have a computed tomography (CT) scan, he requests something for a severe headache. When the nurse offers him a prescribed does of acetaminophen, he asks for something stronger. Which intervention should the nurse implement? a. Request that the CT scan be done immediately b. Review the client's history for use of illicit drugs c. Assess client's pupils for their reaction to light. d. Explain the reason for using only non-narcotics.

Explain the reason for using only non-narcotics. Rationale: The client needs to understand that any pain medication that can mask declining neurological symptoms, such as narcotics should be avoid. There is no indication that the CT scan needs to be done immediately. In the absent of additional information B is presumptive. Regular neurological assessment is necessary, but they do not address the client's pain.

An adult female client tells the nurse that though she is afraid her abusive boyfriend might one day kill her, she keeps hoping that he will change. What action should the nurse take first? a. Report the finding to the police department b. Discuss treatment options for abusive partners c. Determine the frequency and type of client's abuse d. Explore client's readiness to discuss the situation.

Explore client's readiness to discuss the situation Rationale: By assessing the client's level of readiness to discuss her situation the nurse can begin to stablish trust so that further action can be taken to protect her. The nurse needs the client's permission to report the abuse to the police department, which may be obtained after trust is established. B might be an option during the discussion it is most important that the client has a safe refuge even if the abusive partner does not commit seeking help.

A female client on the mental health unit frequently asks the nurse when she can be discharged. Then, becoming more anxious, she begins to pace the hallway. What intervention should the nurse implement first? a. Explore the client's reasons for wanting to be discharged.

Explore the client's reasons for wanting to be discharged.

Diagnostic studies indicate that the elderly client has decreased bone density. In providing client teaching, which area of instruction is most important for the nurse to include? a. Application of joint splints b. Effective body mechanisms c. Fall prevention measures. d. Low fat, high protein diet

Fall prevention measures Rationale: Client instruction should include measures to prevent falls, because elderly clients with decrease bone density are at high risk for bone fractures and impaired bone healing in fracture should occur.

. Which strategy is most important for the nurse to use when assisting a client with myasthenia gravis to devise a daily routine?

Female with a head injury whose pupils are fixed and dilated.

The nurse is assisting the mother of a child with phenylketonuria (PKU) to select foods that are in keeping with the child's dietary restrictions. Which foods are contraindicated for this child? a. Wheat products b. Foods sweetened with aspartame. c. High fat foods d. High calories foods.

Foods sweetened with aspartame Rationale: Aspartame should not be consumed by a child with PKU because ut is converted to phenylalanine in the body. Additionally, milk and milk products are contraindicated for children with PKU.

During a staff meeting, a nurse verbally attacks the nurse manager conducting the meeting, stating, "you always let your favorites have holidays off give then easier assignments. You are unfair and prejudiced" how should the nurse-manager respond? a. Give me specific examples to support your statements.

Give me specific examples to support your statements.

In caring for a client with Cushing syndrome, which serum laboratory value is most important for the nurse to monitor? a. Lactate b. Glucose c. Hemoglobin d. Creatinine

Glucose Rationale: Cushing syndrome, caused by excess corticosteroids causes hyperglycemia and the client's serum glucose level should be monitor for this side effect.

A 6-year old boy was hit with a bat while playing at school. He has a splinter of wood imbedded in his eye. Which action should the school nurse take?

Have the parent take the child for emergency help

A 60-year-old female client asks the nurse about hormones replacement therapy (HRT) as a means preventing osteoporosis. Which factor in the client's history is a possible contraindication for the use of HRT? a. Her mother and sister have a history of breast cancer

Her mother and sister have a history of breast cancer

A 75-year-old female client is admitted to the orthopedic unit following an open reduction and internal fixation of a hip fracture. On the second postoperative day, the client becomes confused and repeatedly asks the nurse she is. What information for the nurse to obtain? a. History of alcohol use

History of alcohol use,

The nurse is caring for a client who was admitted two hours ago with confusion, Kussmaul respirations, and warm, flushed skin. The healthcare provider determines the client is in acute renal failure (ARF). Which intervention is most important for the nurse to include in this clients plan of care?

Hourly neurological assessments.

A client is admitted to the intensive care unit with diabetes insipidus due to a pituitary gland tumor. Which potential complication should the nurse monitor closely? a. Hypokalemia b. Ketonuria. c. Peripheral edema d. Elevated blood pressure

Hypokalemia Rational: pituitary tumors that suppress antidiuretic hormone (ADH) result in diabetes insipidus, which causes massive polyuria and serum electrolyte imbalances, including hypokalemia, which can lead to lethal arrhythmias

A male client arrives at the clinic with a severe sunburn and explains that he did not use sun screen because it was an overcast day. Large blisters are noted over his back and chest and his shirt is soaked with serosanguinous fluid. Which assessment finding warrants immediate intervention by the nurse? a. Hypotension. b. Fever and chills c. Dizziness d. Headache

Hypotension

The nurse is assessing a client with hypothyroidism and knows that these clients are at risk for myxedema coma. What symptoms indicate that the client is developing this condition?

Hypothermia, decreased cardiac output, and decreased respiratory functioning

To auscultate for a carotid bruit, the nurse places the stethoscope at what location. (Select the location on the image with a red dot).

I placed the red dot on the base of the neck on the right side

When implementing a disaster intervention plan, which intervention should the nurse implement first? a. Initiate the discharge of stable clients from hospital units b. Identify a command center where activities are coordinated c. Assess community safety needs impacted by the disaster d. Instruct all essential off-duty personnel to report to the facility.

Identify a command center where activities are coordinated

An older male client with a history of diabetes mellitus, chronic gout, and osteoarthritis comes to the clinic with a bag of medication bottles. Which intervention should the nurse implement first? a. Identify pills in the bag

Identify pills in the bag

An older male client with history of diabetes mellitus, chronic gout, and osteoarthritis comes to the clinic with a bag of medication bottles. Which intervention should the nurse implement first? a. Obtain a medical history b. Record pain evaluation c. Assess blood glucose d. Identify pills in the bag.

Identify pills in the bag Rationale: Comorbidity places the client at risk for multiple drug interaction and side effects, and the client's gout therapy may need to be modified. A review of the medication in the bag (D) is the most important way to analyze the client's polypharmacy. And therapeutic response for comorbidities. Obtain a medical history (A), pain evaluation (B), and assessing blood glucose level (C) should be done in a timely manner.

A client presents in the emergency room with right-sided facial asymmetry. The nurse asks the client to perform a series of movements that require use of the facial muscles. What symptoms suggest that the client has most likely experience a Bell's palsy rather than a stroke? a. Slow onset of facial drooping associated with headache b. Inability to close the affected eye, raise brow, or smile c. A flat nasolabial fold on the right resulting in facial asymmetry. d. Drooling is present on right side of the mouth, but not on the left.

Inability to close the affected eye, raise brow, or smile Rationale: Because the motor function controlling eye closure, brow movement and smiling are all carried on the 7th cranial (facial) nerve, the combination of symptoms directly relating to an impairment of all branches of the facial nerve indicate that Bell's palsy has occurred.

A 46-year-old male client who had a myocardial infarction 24-hours ago comes to the nurse's station fully dressed and wanting to go home. He tells the nurse that he is feeling much better at this time. Based on this behavior, which nursing problem should the nurse formulate? a. Ineffective coping related to denial

Ineffective coping related to denial

A woman with an anxiety disorder calls her obstetrician's office and tells the nurse of increased anxiety since the normal vaginal delivery of her son three weeks ago. Since she is breastfeeding, she stopped taking her antianxiety medications, but thinks she may need to start taking them again because of her increased anxiety. What response is best for the nurse to provide this woman? a. Describe the transmission of drugs to the infant through breast milk b. Encourage her to use stress relieving alternatives, such as deep breathing exercises c. Inform her that some antianxiety medications are safe to take while breastfeeding d. Explain that anxiety is a normal response for the mother of a 3-week-old.

Inform her that some antianxiety medications are safe to take while breastfeeding Rationale: There are several antianxiety medications that are not contraindicated for breastfeeding mothers. The woman is apparently aware that drugs can be transmitted through breast milk, so A is not helpful. C might be helpful, but the client's history suggest that nonpharmacological methods of anxiety management do not produce the best outcome. (D) the mother's history places her at risk for severe anxiety.

The psychiatric nurse is talking to a newly admitted client when a male client diagnosed with antisocial behavior intrudes on the conversation and tells the nurse, "I have to talk to you right now! It is very important!" how should the nurse respond to this client? a. Put his behavior on extinction and continue talking with the newly admitted. b. Inform him that the nurse is busy admitting a new client and will talk to him later. c. Encourage him to go to the nurse's station and talk with another nurse. d. Introduce him to the newly admitted client and ask him to him to join in the conversation.

Inform him that the nurse is busy admitting a new client and will talk to him later. Rational: the psychiatric nurse must set limits with antisocial behavior so that appropriate behavior is demonstrated. Interrupting a conversation is rude and inappropriate, so telling the client that they can talk later is the best course of action. Other options may cause the client to become angry and they do not address the client's behavior. The nurse should not involve this client with newly admitted client's admission procedure.

At 0600 while admitting a woman for a schedule repeat cesarean section (C-Section), the client tells the nurse that she drank a cup a coffee at 0400 because she wanted to avoid getting a headache. Which action should the nurse take first? a. Ensure preoperative lab results are available b. Start prescribed IV with lactated Ringer's c. Inform the anesthesia care provider d. Contact the client's obstetrician.

Inform the anesthesia care provider Rationale: Surgical preoperative instruction includes NPO after midnight the day of surgery to decrease the risk of aspiration should vomiting occur during anesthesia. While it is possible the C-section will be done on schedule or rescheduled for later in the day, the anesthesia provider should be notified first.

After placement of a left subclavian central venous catheter (CVC), the nurse receives report of the x-ray findings that indicate the CVC tip is in the client's superior vena cava. Which action should the nurse implement? a. Initiate intravenous fluid as prescribed b. Notify the HCP of the need to reposition the catheter c. Remove the catheter and apply direct pressure for 5 minutes. d. Secure the catheter using aseptic technique

Initiate intravenous fluid as prescribed Rationale: Venous blood return to the heart and drains from the subclavian vein into the superior vena cava. The X-ray findings indicate proper placement of the CVC, so prescribed intravenous fluid can be started. A and B are not indicated at this time. The catheter should be secure immediate following insertion (C)

A client experiencing withdrawal from the benzodiazepines alprazolam (Xanax) is demonstrating severe agitation and tremors. What is the best initial nursing action? a. Administer naloxone (Narcan) per PNR protocol b. Initiate seizure precautions c. Obtain a serum drug screen d. Instruct the family about withdrawal symptoms.

Initiate seizure precaution Rationale: Withdrawal of CNS depressants, such as Xanax, results in rebound over-excitation of the CNS. Since the client exhibiting tremors, the nurse should anticipate seizure activity and protect the client

What is the priority nursing action when initiating morphine therapy via an intravenous patient-controlled analgesia (PCA) pump? a. Assess the client's ability to use a numeric pain scale b. Initiate the dosage lockout mechanism on the PCA pump c. Instruct the client to use the medication before the pain become severe d. Assess the abdomen for bowel sounds

Initiate the dosage lockout mechanism on the PCA pump Rationale: Morphine depress respiration, so ensuring that the client cannot overdose on the medications

If the nurse is initiating IV fluid replacement for a child who has dry, sticky mucous membranes, flushed skin, and fever of 103.6 F. Laboratory finding indicate that the child has a sodium concentration of 156 mEq/L. What physiologic mechanism contributes to this finding? a. The intravenous fluid replacement contains a hypertonic solution of sodium chloride b. Urinary and Gastrointestinal fluid loss reduce blood viscosity and stimulate thirst c. Insensible loss of body fluids contributes to the hemoconcentration of serum solutes d. Hypothalamic resetting of core body temperature causes vasodilation to reduce body heat

Insensible loss of body fluids contributes to the hemoconcentration of serum solutes Rationale: Fever causes insensible fluid loss, which contribute to fluid volume and results in hemoconcentration of sodium (serum sodium greater than 150 mEq/L). Dehydration, which is manifested by dry, sticky mucous membranes, and flushed skin, is often managed by replacing lost fluids and electrolytes with IV fluids that contain varying concentration of sodium chloride. Although other options are consistent with fluid volume deficit, the physiologic response of hypernatremia is explained by hem concentration.

Know how to prevent SIDS death

Instruct parents to have a firm mattress and avoid laying infant in prone position.

The charge nurse observes a new nurse preparing to insert an intravenous (IV) catheter. The new nurse has gathered supplies, including intravenous catheters, an intravenous insertion kit, and a 4x4 sterile gauze dressing to cover and secure the insertion site. What action should the charge nurse take? a. Instruct the nurse to use a transparent dressing over the site

Instruct the nurse to use a transparent dressing over the site

A child newly diagnosed with sickle cell anemia (SCA) is being discharged from the hospital. Which information is most important for the nurse to provide the parents prior to discharge? a. Instructions about how much fluid the child should drink daily b. information about non-pharmaceutical pain reliever measures c. Referral for social services for the child and family d. Signs of addiction to opioid and medications

Instructions about how much fluid the child should drink daily Rationale: It is essential that the child and family understands the importance of adequate hydration in preventing the stasis-thrombosis-ischemia cycle of a crisis that has a specific plan for hydration is developed so that a crisis can be delayed. Other choices listed are not the most important topics to include in the discharge teaching.

The nurse is caring for a client who is taking a macrolide to treat a bacterial infection. Which finding should the nurse report to the healthcare provider before administering the next dose? a. Jaundice b. Nausea c. Fever d. Fatigue

Jaundice Rationale: Macrolides can cause hepatotoxicity, which is manifested by jaundice and should be reported to the healthcare provider before further doses of the medication are administered, B is a common side effect of macrolides. Fever and Fatigue are expected finding when a client has an infection.

The nurse is teaching a client how to perform colostomy irrigations. When observing the client's return demonstration, which action indicated that the client understood the teaching? a. Turns to left the side to instill the irrigating solution into the stoma b. Keeps the irrigating container less than 18 inches above the stoma c. Instills 1,200 ml of irrigating solution to stimulate bowel evacuation d. Inserts irrigating catheter deeper into stoma when cramping occurs

Keeps the irrigating container less than 18 inches above the stoma Rationale: Keeping the irrigating container less than 18 inches above the stoma permits the solution to flow slowly with little excessive peristalsis does not cause immediate release of stool.

The nurse caring for a client with acute renal fluid (ARF) has noted that the client has voided 800 ml of urine in 4 hours. Based on this assessment, what should the nurse anticipate that client will need? a. Treatment for acute uremic symptoms within 24 hours b. Change to a regular diet c. Large amounts of fluid and electrolyte replacement. d. Unrestricted sodium intake

Large amounts of fluid and electrolyte replacement. Rationale: This client, whose output is significantly high will need fluids and electrolyte replacement. The diuretic stage of ARF begins when the client has greater than 500 ml of urine in 24 hrs. A is associated with the oliguric and anuric stage of ARF. B and D should not occur until the client's BUN and electrolytes indicate a significant improvement that will allow for such changes.

A young adult female presents at the emergency center with acute lower abdominal pain. Which assessment finding is most important for the nurse to report to the healthcare provider? a. Pain scale rating at 9 on a 0-10 scale b. Last menstrual period was 7 weeks ago c. Reports white curdy vaginal discharge d. History of irritable bowel syndrome IBS

Last menstrual period was 7 weeks ago Rationale: Acute lower abdominal pain in A young adult female can be indicative of an ectopic pregnancy, which can be life threatening. Since the clients last menstrual period was seven weeks ago a pregnancy test to be obtained to ruled out ectopic pregnancy, which can result in intra-abdominal hemorrhage caused by a ruptured Fallopian tube. Although the severity of pain requires treatment, the most significant finding is the clients last menstrual period. Other options are not the most important concerns.

A client admitted to the hospital is suspected of having meningitis. The nurse should plan to prepare the client for which diagnostic test?

Lumbar puncture

A client is admitted with metastatic carcinoma of the liver, ascites, and bilateral 4+ pitting edema of both lower extremities. When the client complains that the antiembolic stocking are too constricting, which intervention should the nurse implement? a. Maintain both lower extremities elevated on pillows. b. Remove the contracting antiembolic stocking c. Administer diuretics in the morning hours d. Restrict PO fluid intake to 500 ml per shift

Maintain both lower extremities elevated on pillows Rationale: Hepatocellular failure and hypertension contribute to third spacing of fluids. The clients complain best addresses by maintaining both extremities in an elevated position on pillows, which uses gravity to facilitate venous return and decrease peripheral edema. Stockings should be reapplied evenly to relieve constriction, but no removed.

A client at 30 week gestation is admitted due to preterm labor. A prescription of terbutaline sulfate 8.35 mg is given subcutaneously. Based on which finding should the nurse withhold the next dose of this drug? a. Maternal blood pressure of 90/60 b. Fetal heart rate of 170 beats per minute for 15 mints c. Maternal pulse rate of 162 beats per min d. Serum potassium of 2.3 mg/dl

Maternal pulse rate of 162 beats per min Rationale: The nurse checks the maternal pulse prior to administering the beta sympathomimetic drug terbutaline and notify the healthcare provider before administration of the drug if the pulse is over 140 beats in within normal limits because peripheral vasodilation accompanies pregnancy and causes the BP decrease.

The nurse is caring for a client who had gastric bypass surgery yesterday. Which intervention is most important for the nurse to implement during the first 24 postoperative hours? a. Insert an indwelling urinary catheter b. Monitor for the appearance of an incisional hernia c. Instruct the client to eat small frequent meals d. Measure hourly urinary output.

Measure hourly urinary output. Rationale: a serious early complications of gastric bypass surgery is an anastomoses leak, often resulting in death.

While changing a client's chest tube dressing, the nurse notes a crackling sensation when gentle pressure is applied to the skin at the insertion site. What is the best action for the nurse to take? a. Apply a pressure dressing around the chest tube insertion site. b. Assess the client for allergies to topical cleaning agents. c. Measure the area of swelling and crackling. d. Administer an oral antihistamine per PRN protocol.

Measure the area of swelling and crackling. Rational: a crackling sensation, or crepitus, indicates subcutaneous emphysema, or air leaking into the skin. This area should be measured, and the finding documented. Other options are not indicated for crepitus.

While assisting a client who recently had a hip replacement into a bed pan, the nurse notices that there is a small amount of bloody drainage on the surgical dressing, the client's skin is warm to the touch, and there is a strong odor from the urine. Which action should the nurse take? a. Obtain a urine sample from the bed pan b. Remove dressing and assess surgical site c. Insert an indwelling urinary catheter d. Measure the client's oral temperature

Measure the client's oral temperature Rationale: The strong odor from the urine and skin that is warm to the touch may indicate that the client has a urinary tract infection. Assessing the client's temperature provides objective information regarding infection that can be reported to the healthcare provider. Urine should be obtained via a clean catch, not the bed pan where it has been contaminated. The drainage on the dressing is normal and does not require direct conservation currently. An indwelling catheter should be avoided if possible because it increases the risk of infection.

A female client has been taking a high dose of prednisone, a corticosteroid, for several months. After stopping the medication abruptly, the client reports feeling "very tired". Which nursing intervention is most important for the nurse to implement? a. Measure vital signs b. Auscultate breath sounds c. Palpate the abdomen d. Observe the skin for bruising

Measure vital signs Rationale: Abrupt withdrawal of an exogenous corticosteroids can precipitate adrenal insufficiency and hypoglycemia, hypokalemia, and circulatory collapse can occur. Is most important for the nurse to assess vital sign to impending shock.

Ten years after a female client was diagnosed with multiple sclerosis (MS), she is admitted to a community palliative care unit. Which intervention is most important for the nurse to include in the client's plan of care? a. Allow the family to visit whenever they wish b. Medicate as needed for pain and anxiety. c. Allow client to participate in care provided d. Maintain quiet, low lighting environment

Medicate as needed for pain and anxiety. Rationale: Neuropathic pain in MS is related to damage to peripheral nerves or structures in the CNS can be sudden, intense or lingering, and shooting, electric shock-like sensations that results for paroxysmal firing of injured nerves. Once the client enters palliative care, the primary goal is comfort.

Which intervention should the nurse include in the plan of care for a child with tetanus? a. Open window shades to provide natural light b. Reposition side to side every hour. c. Minimize the number of stimuli in the room. d. Encourage coughing and deep breathing

Minimize the amount of stimuli in the room Rationale: Tetanus is an acute, preventable, and often fatal disease caused by an exotoxin produces by the anaerobic spore forming gram positive bacillus clostridium tetani, which affect neuromuscular junction and causes painful muscular rigidity. In planning caring for a child with tetanus, any environmental stimulation should be minimized.

An older client is admitted to the intensive care unit with severe abdominal pain, abdominal distention, and absent bowel sound. The client has a history of smoking 2 packs of cigarettes daily for 50 years and is currently restless and confused. Vital signs are: temperature 96`F, heart rate 122 beats/minute, respiratory rate 36 breaths/minute, mean arterial pressure(MAP) 64 mmHg and central venous pressure (CVP) 7 mmHg. Serum laboratory findings include: hemoglobin 6.5 grams/dl, platelets 6o, 000, and white blood cell count (WBC) 3,000/mm3. Based on these findings this client is at greatest risk for which pathophysiological condition? a. Multiple organ dysfunction syndrome (MODS) b. Disseminated intravascular coagulation (DIC) c. Chronic obstructive disease. d. Acquired immunodeficiency syndrome (AIDS)

Multiple organ dysfunction syndrome (MODS) Rational: MODS are a progressive dysfunction of two or more major organs that requires medical intervention to maintain homeostasis. This client has evidence of several organ systems that require intervention, such as blood pressure, hemoglobin, WBC, and respiratory rate. DIC may develop as a result of MODS. The other options are not correct.

The nurse is auscultating a client's heart sounds. Which description should the nurse use to document this sound? (Please listen to the audio first to select the option that applies) a. S1 S2 b. S1 S2 S3 c. Murmur d. Pericardial friction rub.

Murmur Rationale: A murmur is auscultated as a swishing sound that is associated with the blood turbulence created by the heart or valvular defect. B is associate with Heart Failure.

Which problem reported by a client taking lovastatin requires the most immediate follow up by the nurse? a. Diarrhea and flatulence b. Abdominal cramps c. Muscle pain d. Altered taste

Muscle pain Rationale: statins can cause rhabdomyolysis, a potentially fatal disease of skeletal muscle characterized by myoglobinuria and manifested with muscle pain, so this symptom should immediately be reported to the HCP.

The nurse is preparing a teaching plan for an older female client diagnosed with osteoporosis. What expected outcome has the highest priority for this client? a. Identifies 2 treatments for constipation due to immobility. b. Names 3 home safety hazards to be resolve immediately. c. State 4 risk factors for the development of osteoporosis. d. Lists 5 calcium-rich foods to be added to her daily diet.

Names 3 home safety hazards to be resolve immediately Rational: a major teaching goal for an elderly client with osteoporosis is maintenance of safety to prevent falls. Injury due to a fall, usually resulting in a hip fracture, can result in reduced mobility and associated complications. Other goals are also important when teaching clients who have osteoporosis, but they do not have the priority of preventing falls, which relates to safety.

When caring for a client with traumatic brain injury (TBI) who had a craniotomy for increased intracranial pressure (ICP), the nurse assesses the client using the Glasgow coma scale (GCS) every two hours. For the past 8 hours the client's GCS score has been 14. What does this GCS finding indicate about the client? a. Neurologically stable without indications of an increased IC

Neurologically stable without indications of an increased IC

One day following an open reduction and internal fixation of a compound fracture of the leg, a male client complains of "a tingly sensation" in his left foot. The nurse determines the client's left pedal pulses are diminished. Based on these finding, what is the client's greatest risk? a. Reduce pulmonary ventilation and oxygenation related to fat embolism. b. Neurovascular and circulation compromise related to compartment syndrome. c. Wound infection and delayed healing to fractured bone protrusion. d. Venous stasis and thrombophlebitis related to postoperative immobility.

Neurovascular and circulation compromise related to compartment syndrome Rationale: Inflammation from the traumatic injury produces swelling and edema inside the closed space under the skin that produces pressure, which decreases blood flow to capillaries and nerves, causing altered perfusion related to compartment syndrome

The nurse is preparing a community education program on osteoporosis. Which instruction is helpful in preventing bone loss and promoting bone formation? a. Recommend weigh bearing physical activity b. Reduce intake of foods high in vitamin D c. Decrease intake of foods high in fat d. Minimize heavy lifting and bending.

Recommend weigh bearing physical activity Rationale: Active weight-bearing exercise is a primary preventive measure for osteoporosis. C is indicated for client with cardiac and liver diseases. D may decrease injuries but is not directed toward slowing bone loss and promoting bone formation.

A male client who was diagnosed with viral hepatitis A 4 weeks ago returns to the clinic complaining of weakness and fatigue. Which finding is most important for the nurse to report to the healthcare provider? a. Dark yellow-brown colored urine b. Nonspecific muscle and joint pain c. New onset of purple skin lesions. d. Weakness when getting up to walk.

New onset of purple skin lesions. Rationale: During the convalescence period of hepatitis A, the client major complain is malaise and fatigability. Purple skin lesions may be indicative of the liver's impaired ability to produce clothing elements and should be reported to the healthcare provider (C) for further analysis. Urine may become dark when excess bilirubin is excreted by the kidney, which is expected even when the client is not jaundice during the acute phase hepatitis (A). Myalgia and arthralgia (B) are intermittent complains with ongoing malaise, fatigue and weakness (D) during convalescence of hepatitis A.

A client is admitted to the emergency department with a respiratory rate of 34 breaths per minute and high pitched wheezing on inspiration and expiration, the medical diagnosis is severe exacerbation of asthma. Which assessment finding, obtained 10 min after the admission assessment, should the nurse report immediately to the emergency department healthcare provider? a. An apical pulse of 120 beats per minute b. Extreme agitation with staff and family c. Client report being anxious d. No wheezing upon auscultation of the chest.

No wheezing upon auscultation of the chest. Rationale: No wheezing an auscultation indicates that the client is not exchanging air and is highly compromised immediate action is indicated A, B, and C are sign of hypoxia but no as critical as D

An older adult client with heart failure (HF) develops cardiac tamponade. The client has muffled, distant, heart sounds, and is anxious and restless. After initiating oxygen therapy and IV hydration, which intervention is most important for the nurse to implement? a. Observe neck for jugular vein distention b. Notify healthcare provider to prepare for pericardiocentesis c. Asses for paradoxical blood pressure d. Monitor oxygen saturation (Sp02) via continuous pulse oximetry

Notify healthcare provider to prepare for pericardiocentesis Rationale: Cardiac tamponade is pressure on the heart that occurs when blood or fluid builds up in the space between the heart muscle (myocardium) and the outer covering sac of the heart (pericardium). In this condition, blood or fluid collects in the pericardium, the sac surrounding the heart. This prevents the heart ventricles from expanding fully. The excess pressure from the fluid prevents the heart from working properly. As a result, the body does not get enough blood.

A client with hyperthyroidism who has not been responsive to medications is admitted for evaluation. What action should the nurse implement? (Click on each chart tab for additional information. Please scroll to the bottom right corner of each tab to view all information contained in the client's medical record.) a. Notify the healthcare provider

Notify the healthcare provider

A mother brings her 3-year-old son to the emergency room and tells the nurse the he has had an upper respiratory infection for the past two days. Assessment of the child reveals a rectal temperature of 102 F. he is drooling and becoming increasingly more restless. What action should the nurse take first? a. Put a cold cloth on his head and administer acetaminophen. b. Listen to lung sounds and place him in a mist tent. c. Notify the healthcare provider and obtain a tracheostomy tray d. Assist the child to lie down and examine his throat.

Notify the healthcare provider and obtain a tracheostomy tray Rationale: This child exhibiting signs and symptoms of epiglottitis, a bacterial infection causing acute airway obstruction, so is the immediate action to take.

A client with leukemia undergoes a bone marrow biopsy. The client's laboratory values indicate the client has thrombocytopenia. Based on this data, which nursing assessment is most important following the procedure? a. Assess body temperature b. Monitor skin elasticity c. Observe aspiration site. d. Measure urinary output

Observe aspiration site.

One day after abdominal surgery, an obese client complains of pain and heaviness in the right calf. What action should the nurse implement? a. Observe for unilateral swelling

Observe for unilateral swelling

A client with a peripherally inserted central catheter (PICC) line has a fever. What client assessment is most important for the nurse to perform? Observe the antecubital fossa for inflammation.

Observe the antecubital fossa for inflammation.

A client presents to the labor and delivery unit, screaming "THE BABY IS COMING" which action should the nurse implement first. a. Observe the perineum

Observe the perineum

An elderly client seems confused and reports the onset of nausea, dysuria, and urgency with incontinence. Which action should the nurse implement a. Auscultate for renal bruits b. Obtain a clean catch mid-stream specimen c. Use a dipstick to measure for urinary ketone d. Begin to strain the client's urine.

Obtain a clean catch mid-stream specimen Rationale: This elderly is experiencing symptoms of urinary tract infection. The nurse should obtain a clean catch mid-stream specimen to determine the causative agent so an anti-infective agent can be prescribed.

A client with a history of heart failure presents to the clinic with a nausea, vomiting, yellow vision and palpitations. Which finding is most important for the nurse to assess to the client? a. Determine the client's level of orientation and cognition b. Assess distal pulses and signs of peripheral edema c. Obtain a list of medications taken for cardiac history. d. Ask the client about exposure to environmental heat.

Obtain a list of medications taken for cardiac history Rationale: The client is presenting with signs of digitalis toxicity. A list of medication, which is likely to include digoxin (Lanoxin) for heart failure, can direct further assessment in validating digitalis toxicity with serum labels greater than 2 mg/ml that is contributing to client's presenting clinical picture.

The nurse enters a client's room and observes the client's wrist restraint secured as seen in the picture. What action should the nurse take? a. Use a full knot to secure the restrain tie. b. Reposition the restraint tie onto the bedframe. c. Raise the button side rail of the client's bed d. Document that the restrain is secured.

Reposition the restraint tie onto the bedframe. Rationale: Restraints should be secured to the bedframe, which is more stable than the side rails. A is difficult to release quickly. The restrain should be removed from the side rail before the position of the side rail is changed.

An adult client with schizophrenia begin treatment three days ago with the Antipsychotic risperidone. The client also received prescription for trazodone as needed for sleep and clonazepam as needed for severe anxiety. When the client reports difficulty with swallowing, what action should the nurse take? a. Obtain a prescription for an anticholinergic medication b. Determine how many hours declined slept last night c. Administer the PRN prescription for severe anxiety d. Watch the thyroid cartilage move while the client swallows

Obtain a prescription for an anticholinergic medication Rationale: Antipsychotic medications have an extrapyramidal side effects one of which is difficult to swallowing the nurse should obtain a prescription for an anticholinergic medication which is used for the treatment of extrapyramidal symptoms. Other options are not warranted actions based on the symptoms presented.

The nurse instructs an unlicensed assistive personnel (UAP) to turn an immobilized elderly client with an indwelling urinary catheter every two hours. What additional action should the nurse instruct the UAP to take each time the client is turned? a. Empty the urinary drainage bag b. Feed the client a snack c. Offer the client oral fluids d. Assess the breath sounds

Offer the client oral fluids Rationale: Increasing oral fluid intake reduces the risk of problems associated with immobility, so the UAP should be instructed to offer the client oral fluids every two hours, or whenever turning he client. It is not necessary to empty the urinary bag or feed the client every two hours. Assessment is a nursing function, and UAPs do not have the expertise to perform assessment of breath sounds.

Following a motor vehicle collision, an adult female with a ruptured spleen and a blood pressure of 70/44, had an emergency splenectomy. Twelve hours after the surgery, her urine output is 25 ml/hour for the last two hours. What pathophysiological reason supports the nurse's decision to report this finding to the healthcare provider a. This output is not sufficient to cleat nitrogenous waste b. Oliguria signals tubular necrosis related to hypoperfusion c. Low urine output puts the client at risk for fluid overload d. An increased urine output is expected after splenectomy

Oliguria signals tubular necrosis related to hypoperfusion Rationale: Prolonged low blood pressure leads to renal ischemia, which is the common etiology of acute tubular necrosis(ATN) Decreasing urine output is an early indicator of ATN.

Which needle should the nurse use to administer intravenous fluids (IV) via a client's implanted port? a. The one with the clamp and no needle

One with the clamp

A client with a liver abscess develops septic shock. A sepsis resuscitation bundle protocol is initiated and the client receives a bolus of IV fluids. Which parameter should the nurse monitor to assess effectiveness of the fluid bolus? a. Oxygen saturation

Oxygen saturation

An older male client arrives at the clinic complaining that his bladder always feels full. He complains of weak urine flow, frequent dribbling after voiding, and increasing nocturia with difficulty initiating his urine stream. Which action should the nurse implement? a. Obtain a urine specimen for culture and sensitivity b. Palpate the client's suprapubic area for distention c. Advise the client to maintain a voiding diary for one week d. Instruct in effective technique to cleanse the glans penis

Palpate the client's suprapubic area for distention Rationale: the client is exhibiting classic signs of an enlarge prostate gland, which restricts urine flow and cause bothersome lower urinary tract symptoms (LUTS) and urinary retention, which is characterized by the client's voiding patterns and perception of incomplete bladder emptying.

A 26-year-old female client is admitted to the hospital for treatment of a simple goiter, and levothyroxine sodium (Synthroid) is prescribed. Which symptoms indicate to the nurse that the prescribed dosage is too high for this client? The client experiences a. Palpitations and shortness of breath b. Bradycardia and constipation c. Lethargy and lack of appetite d. Muscle cramping and dry, flushed skin

Palpitations and shortness of breath Rationale: An overdose of thyroid preparation generally manifests symptoms of an agitated state such as tremors, palpitations, shortness of breath, tachycardia, increased appetite, agitation, sweating and diarrhea.

A native-American male client diagnosed with pneumonia, states that in addition to his prescribed medical treatment of IV antibiotics he wishes to have a spiritual cleaning performed. Which outcome statement indicates that the best plan of care was followed? a. Identifies his ethnocentric values and behaviors b. States an understanding of the medical treatment c. Participated actively in all treatments regimens d. Expresses a desire for cultural assimilation

Participated actively in all treatments regimens Rationale: indicates active participation by the client, which is required for treatment to be successful. The best plan of care should incorporate the valued and treatments of both cultures and in this case there is no apparent cultural clash between the two forms of treatment. The client has already identify he's cultural values (A). (B) Only considers one of the two treatment modalities desired by the client the client has already chosen how he wishes to assimilate his cultural values with the prescribed medical treatment (D).

The nurse has received funding to design a health promotion project for AfricanAmerican women who are at risk for developing breast cancer. Which resource is most important in designing this program? a. A listing of African-American women so live in the community b. Participation of community leaders in planning the program c. Morbidity data for breast cancer in women of all races d. Technical assistance to produce a video on breast self-examination.

Participation of community leaders in planning the program

The rapid response team's detects return of spontaneous circulation (ROSC) after 2 min of continuous chest compressions. The client has a weak, fast pulse and no respiratory effort, so the healthcare provider performs a successful oral, intubation. What action should the nurse implement? a. Perform bilateral chest auscultation. b. Resume compression for 2 minutes c. Administer a dose of epinephrine d. Program the monitor for cardioversion.

Perform bilateral chest auscultation Rationale: With the ROSC and no respiratory effort intubation is indicated, and as soon as the procedure is completed, the position of the intubation tube should be assessed for proper placement. Auscultating for breath sounds is the first and quickest method to use to check for proper placement of the intubation tube and can be confirmed by a chest x ray.

Following the vaginal delivery of a 10-pound infant, the nurse assesses a new mother's vaginal bleeding and finds that she has saturated two pads in 30 minutes and has a boggy uterus. What action should the nurse take first?

Perform fundal massage until firm

Following the vaginal delivery of a 10-pound infant, the nurse assesses a new mother's vaginal bleeding and finds that she has saturated two pads in 30 minutes and has a boggy uterus. What action should the nurse take first?

Perform fundal massage until firm

A client is admitted to the hospital after experiencing a brain attack, commonly referred to as a stroke or cerebral vascular accident (CVA). The nurse should request a referral for speech therapy if the client exhibits which finding? a. Abnormal responses for cranial nerves I and II b. Persistent coughing while drinking c. Unilateral facial drooping d. Inappropriate or exaggerated mood swings

Persistent coughing while drinking Rationale: After a stroke, clients may experience dysphagia and an impaired gag reflex that is evaluated by a speech pathology team. Coughing while drinking results from impaired swallowing and gag reflex, so a referral to a speech therapist is indicated to evaluate the coordination of oral movements associated with speech and deglutination. Cranial nerves I and II are sensory nerves for taste and sight and do not require a referral to speech pathology. Unilateral facial drooping is associated with stroke but is not a focus of rehabilitation. D sre not addressed by speech therapy.

A client with rheumatoid arthritis (RA) starts a new prescription of etanercept (Enbrel) subcutaneously once weekly. The nurse should emphasize the importance of reporting problem to the healthcare provider? a. Headache b. Joint stiffness c. Persistent fever d. Increase hunger and thirst

Persistent fever Rationale: Enbrel decrease immune and inflammatory responses, increasing the client's risk of serious infection, so the client should be instructed to report a persistent fever, or other signs of infection to the healthcare provider.

While assisting a male client who has muscular dystrophy (MD) to the bathroom, the nurse observes that he is awkward and clumsy. When he expresses his frustration and complains of hip discomfort, which intervention should the nurse implement? a. Place a portable toilet next to the bed

Place a portable toilet next to the bed

At 40 week gestation, a laboring client who is lying is a supine position tells the nurse that she has finally found a comfortable position. What action should the nurse take? a. Encourage the client to turn on her left side. b. Place a pillow under the client's head and knees. c. Explain to the client that her position is not safe. d. Place a wedge under the client's right hip.

Place a wedge under the client's right hip Rationale: Hypotension from pressure on the vena cava is a risk for the full-term client. Placing a wedge under the right hip will relieve pressure on the vena cava. Other options will either not relieve pressure on the vena cava or would not allow the client the remaining her position of choice.

3. A male Muslim client with pneumonia is scheduled to receive a dose of an intravenous antibiotic but refuses to allow the nurse to begin the medication, stating he cannot allow fluids to enter his body once he is cleansed for prayer. What action should the nurse implement?

Reschedule administration of the antibiotic until after he completes his prayers

A client with cervical cancer is hospitalized for insertion of a sealed internal cervical radiation implant. While providing care, the nurse finds the radiation implant in the bed. What action should the nurse take? a. Call the radiology department b. Reinsert the implant into the vagina c. Apply double gloves to retrieve the implant for disposal. d. Place the implant in a lead container using long-handled forceps

Place the implant in a lead container using long-handled forceps Rationale: Solid or sealed radiation sources, such as Cesium which is removed after treatment, are inserted into an applicator or cervical implant to emit continuous, low energy radiation for adjacent tumor tissues. If the radiation source or the applicator become dislodged long-handled forceps should be used to retrieve the radiation implant to prevent injury due to direct handling. The applicator is then placed in the lead container.

An 11-year-old client is admitted to the mental health unit after trying to run away from home and threatening self-harm. The nurse establishes a goal to promote effective coping, and plans to ask the client to verbalize three ways to deal with stress. Which activity is best to establish rapport and accomplish this therapeutic goal? a. Play a board game with the client and begin taking about stressor

Play a board game with the client and begin taking about stressor

The nurse finds a client at 33 weeks gestation in cardiac arrest. What adaptation to cardiopulmonary resuscitation (CPR) should the nurse implement? a. Apply oxygen by mask after opening the airway b. Position a firm wedge to support pelvis and thorax at 30-degree tilt. c. Give continuous compression with a ventilation ratio at 20:3 d. Apply less compression force to reduce aspiration

Position a firm wedge to support pelvis and thorax at 30 degree tilt. Rationale: To relieve aortocaval compression caused by the gravid uterus, left lateral uterine displacement (LUD) should be maximized using a firm wedge to support the pelvis and thorax at 30- degree tilt to optimize maternal hemodynamic during CPR. Maternal modification should include ventilation with 100% oxygen, not A. Pregnant adults should be resuscitated using a compression-ventilation ration of 30:2 not C without interruption of continuous compressions. Effective chest compression should be forceful rhythmic application of pressure (fast and hard) at 100 compressions/minutes at the depth of 2 inches (5cm) to generate myocardial and cerebral blood flow.

The nurse is teaching a postmenopausal client about osteoporosis prevention. The client reports that she smokes 2 packs of cigarettes a day and takes 750 mg calcium supplements daily. What information should the nurse include when teaching this client about osteoporosis prevention? a. Family history is more important than calcium intake in determining the occurrence of osteoporosis b. Calcium should be taken once a day, preferable at the same time of day c. Smoking cessation is more important than calcium intake in preventing osteoporosis. d. Postmenopausal women need an intake of at least 1,500 mg of calcium daily.

Postmenopausal women need an intake of at least 1,500 mg of calcium daily

During a Woman's Health fair, which assignment is the best for the Practical Nurse (PN) who is working with a register nurse (RN) a. Encourage the woman at risk for cancer to obtain colonoscopy. b. Present a class of breast-self examination c. Prepare a woman for a bone density screening d. Explain the follow-up need it for a client with prehypertension.

Prepare a woman for a bone density screening Rationale: A bone density screening is a fast, noninvasive screening test for osteoporosis that can be explained by the PN. There is no additional preparation needed (A) required a high level of communication skill to provide teaching and address the client's fear. (B) Requires a higher level of client teaching skill than responding to one client. (D) Requires higher level of knowledge and expertise to provide needed teaching regarding this complex topic.

During a cardiopulmonary resuscitation of an intubated client, the nurse detects a palpable pulse throughout the two minutes cycle chest compression and absent breath sounds over the left lung. What action should the nurse implement? a- Instruct the compressor to stop chest compression. b- Advise ventilator to increase bag-mask ventilation rate. c- Plan to suction endotracheal tube at two-minute check. d- Prepare for the endotracheal tube to be repositioned

Prepare for the endotracheal tube to be repositioned

While assessing a radial artery catheter, the client complains of numbness and pain distal to the insertion site. What interventions should the nurse implement? a. Determine of aspirin was given prior to radial artery catheter insertion. b. Promptly remove the arterial catheter from the radial artery. c. Irrigate the arterial line using a syringe with sterile saline d. Administer a PRN analgesic and assess numbness in 30 mints

Promptly remove the arterial catheter from the radial artery. Rationale: The client is manifesting evidence of sensory dysfunction and ischemia distal to the arterial catheter insertion site, so the arterial catheter should promptly be removed to minimize tissue necrosis.

The Nurse recognizes that the primary purpose of recommending a yearly digital rectal examination (DRE) for all men over the age of 40 is to help detect the early stages of which type of cancer?

Prostate cancer.

The nurse is planning preoperative teaching plan of a 12-years old child who is scheduled for surgery. To help reduce the child anxiety, which action is the best for the nurse to implement? a. Give the child syringes or hospital mask to play it at home prior to hospitalization. b. Include the child in pay therapy with children who are hospitalized for similar surgery. c. Provide a family tour of the preoperative unit one week before the surgery is scheduled. d. Provide doll an equipment to re-enact feeling associated with painful procedures

Provide a family tour of the preoperative unit one week before the surgery is scheduled Rationale: School age children gain satisfaction from exploring and manipulating their environment, thinking about objectives, situations and events, and making judgments based on what they reason. A tour of the unit allows the child to see the hospital environment and reinforce explanation and conceptual thinking.

A client with a cervical spinal cord injury (SCI) has Crutchfield tongs and skeletal traction applied as a method of closed reduction. Which intervention is most important for the nurse to include in the client's a plan of care? a. Provide daily care of tong insertion sites using saline and antibiotic ointment b. Modify the client's diet to prevent constipation c. Encourage active range of motion q2 to 4 hours. d. Instruct the client to report any symptoms of upper extremity paresthesia.

Provide daily care of tong insertion sites using saline and antibiotic ointment Rationale: Crutchfield tongs, a skeletal traction device for cervical immobilization, requires daily care of the surgically inserted tongs to minimize the risk of infection of the insertion site and cranial bone. Daily cleansing with normal saline solution and antibiotic applications minimizes bacterial colonization and helps to prevent infection.

A man expresses concern to the nurse about the care his mother is receiving while hospitalized. He believes that her care is not based on any ethical standards and ask what type of care he should expect from a public hospital. What action should the nurse take? a. Provide the man and his mother with a copy of the Patient's Bill of Rights b. Explain that the hospital adheres to all national accreditation standards c. Advise the man to discuss his concerns with his mother's healthcare provider d. Determine if he would like to review the hospital's manual of approved polices.

Provide the man and his mother with a copy of the Patient's Bill of Rights Rationale: The Patient's Bill of Rights is a universally used tool that describes the rights of clients in all healthcare settings and is essential in ensuring that clients care is provided in an ethical manner. (B) may be perceived as defensive and does not provide the man with specific information about expected standards of care. Concern about the quality of care should be addressed by the hospital staff rather than C. All the healthcare agencies are required to maintain policy and procedure manual for the purpose of standardizing delivery of care within the agencies, but the policy manual is unlikely to provide useful information for clients or family members.

Leadership- Interpreter informed consent

Pt should sign the consent

While taking vital signs, a critically ill male client grabs the nurse's hand and ask the nurse not to leave. What action is best for the nurse to take? a. Pull up a chair and sit beside the client's bed

Pull up a chair and sit beside the client's bed

Geriatrics/Emergency Rib fractures fall

Pulmonary embolism

A low-risk primigravida at 28-weeks gestation arrives for her regular antepartal clinic visit. Which assessment finding should the nurse consider within normal limits for this client? a. Pulse increase of 10 beats/minute b. Proteinuria c. Glucosuria d. Fundal height 0f 22 centimeters

Pulse increase of 10 beats/minute Rationale: Blood volume increases 25 to 40 % in pregnancy which increases cardiac output and increases heart rate by approx. 10 to 20 beats/ mints. Proteinuria is for preeclampsia, Glucosuria is for gestational diabetes. A fundal height for 28 weeks should be at 28 cm not 22

While undergoing hemodialysis, a male client suddenly complains of dizziness. He is alert and oriented, but his skin is cool and clammy. His vital signs are: heart rate 128 beats/minute, respirations 18 breaths per minute, and blood pressure 90/60. Which intervention should the nurse implement first? a. Raise the client's legs and feet b. Administer 250 ml saline bolus c. Decrease blood flow from dialyzer d. Stop the hemodialysis procedure.

Raise the client's legs and feet Rationale: To raise the client's blood pressure is the most immediate and easiest intervention for the nurse to implement. B and C should be done asap to add volume to the vascular space by ceasing to pull fluid from the client. If the blood pressure does not increase, then the procedure may be needing to be stopped. (D)

A male client with cirrhosis has ascites and reports feeling short of breath. The client is in semi-Fowler position with his arms at his sides. What action should the nurse implement? a. Reposition the client in a side-lying position and support his abdomen with pillows. b. Elevate the client's feet on a pillow while keeping the head of the bed elevated. c. Raise the head of the bed to a Fowler's position and support his arms with a pillow Place the client in a shock position and monitor his vital signs at frequent intervals

Raise the head of the bed to a Fowler's position and support his arms with a pillow Rationale: The Ascites is the accumulation of fluid in the peritoneal or abdominal cavity, and this fluid pushes on the diaphragm, limiting the client's lung expansion and causing dyspnea. To relieve pressure, the head of the bed should be elevated with the arms supported for comfort.

A client is admitted with an epidural hematoma that resulted from a skateboarding accident. To differentiate the vascular source of the intracranial bleeding, which finding should the nurse monitor? a. Slow increasing intracranial pressure (ICP) b. Decerebrate posturing c. Rapid onset of decreased level of consciousness. d. Coup contrecoup signs

Rapid onset of decreased level of consciousness. Rationale: Epidural hematomas results from arterial bleeds that cause a rapid increase in ICP, which initially manifested by an early and rapid onset of decreased consciousness. Slowly increasing ICP is more likely to occur with a venous subdural hematoma.

A client who had a right hip replacement 3 day ago is pale has diminished breath sound over the left lower lung fields, a temperature of 100.2 F, and an oxygen saturation rate of 90%. The client is scheduled to be transferred to a skilled nursing facility (SNF) tomorrow for rehabilitative critical pathway. Based on the client's symptoms, what recommendation should the nurse give the healthcare provider? a. Reassess readiness for SNF transfer. b. Obtain specimens for culture analysis c. Confer with family about home care plans d. Arrange physical therapy for strengthening.

Reassess readiness for SNF transfer. Rationale: Based on the client's symptoms, reassessing the client's readiness for rehabilitation in the SNF is critical

The nurse is preparing a community education program on osteoporosis. Which instruction is helpful in preventing bone loss and promoting bone formation? a. Recommend weigh bearing physical activity

Recommend weigh bearing physical activity

The nurse is caring for a 17-year-old male who fell 20 feet 5 months ago while climbing the side of a cliff and has been in a sustained vegetative state since the accident. Which intervention should the nurse implement? a. Talk directly to the adolescent while providing care

Talk directly to the adolescent while providing care

When assessing the surgical dressing of a client who had abdominal surgery the previous day, the nurse observes that a small amount of drainage is present on the dressing and the wound's Hemovac suction device is empty with the plug open. How should the nurse respond? a. Replace the dressing and remove the drainage device b. Reposition the drainage device and keep the plug open c. Notify the healthcare provider that the drain is not working d. Recompress the wound suction device and secure to plug

Recompress the wound suction device and secure to plug Rationale: The plug of a wound suction device, such as a Hemovac, should be closed after compressing the device to apply gentle suction in a closed surgical wound to facilitate the evacuation of subcutaneous fluids into the device. Compressing the device and securing the plug should restore function of the closed wound device. A small amount of drainage should be marked on the dressing, but replacing the dressing is not necessary and the nurse should not remove the device. Other options are not indicated.

The nurse identifies a priority diagnosis of, "Altered comfort related to menstrual cramps" for a 25-year-old female client. Which self-care activity should the nurse emphasize in the client's teaching plan?

Regular aerobic exercise.

An 18-year-old female client is seen at the health department for treatment of condylomata acuminate (perineal warts) caused by the human papillomavirus (HPV). Which intervention should the nurse implement? a. Tell the client that the vaccine for HPV is not indicated b. Inform the client that warts do not return following cryotherapy c. Recommended the use of latex condoms to prevent HPV transmission. d. Reinforce the importance of annual papanicolaou (Pap) smears.

Reinforce the importance of annual papanicolaou (Pap) smears.

To evaluate the effectiveness of male client's new prescription for ezetimibe, which action should the clinic nurse implement? a. Remind the client to keep his appointments to have his cholesterol level checked. b. Teach the client to weigh himself weekly and keep a log of the measurements c. Assess the elasticity of the client's skin at the next scheduled clinic appointment d. Encourage the client to keep a diary of his food intake until his next visit to the clinic.

Remind the client to keep his appointments to have his cholesterol level checked. Rationale: Ezetimibe lowers cholesterol and LDL levels, so it is important for the nurse to remind the clients to keep his appointments at the laboratory. D may influence his serum levels, but A provide better indicator.

The leg of a client who is receiving hospice care have become mottled in appearance. When the nurse observes the unlicensed assistive personal (UAP) place a heating pad on the mottled areas, what action should the nurse take? a. Remove the heating pads and place a soft blanket over the client's leg and feet. b. Advise the UAP to observe the client's skin while the heating pads are in place. c. Elevate the client's feet on a pillow and monitor the client's pedal pulses frequently. d. Instruct the UAP to reposition the heating pads to the sides of the legs and feet.

Remove the heating pads and place a soft blanket over the client's leg and feet.

A female client with breast cancer who completed her first chemotherapy treatment today at an out-patient center is preparing for discharge. Which behavior indicates that the client understands her care needs Rented movies and borrowed books to use while passing time at home

Rented movies and borrowed books to use while passing time at home

During an annual physical examination, an older woman's fasting blood sugar (FBS) is determined to be 140 mg/dl or 7.8 mmol/L (SI). Which additional finding obtained during a follow-up visit 2 weeks later is most indicative that the client has diabetes mellitus (DM)? a. An increased thirst with frequent urination b. Blood glucose range during past two weeks was 110 to 125 mg/dl or 6.1 to 7.0 mmol/L(SI) c. Two-hour postprandial glucose tolerance test (GTT) is 160 mg/dL or 8.9 mmol/L (SI) d. Repeated fasting blood sugar (FBS) is 132 mg/dl or 7.4 mmol/L (SI).

Repeated fasting blood sugar (FBS) is 132 mg/dl or 7.4 mmol/L (SI).

A child is admitted to the pediatric unit diagnosed with sickle cell crisis. When the nurse walks into the room, the unlicensed assistive personnel (UAP) is encouraging the child to stay in bed in the supine position. Which action should the nurse implement? a. Reposition the client with the head of the bed elevated. b. Commend the UAP for implementing the proper position c. Tell the UAP that this position is harmful to the client d. Encourage the child to ambulate in the room

Reposition the client with the head of the bed elevated. Rationale: Since children is sickle cell crisis often have shallow breathing due to acute chest syndrome, raising the head of the bed (A) will facilitate chest expansion by decreasing pressure of the diaphragm (B and C) are not be commended, nor should he UAP be corrected in front of the child. D is contraindicated because bed rest is warranted to conserve energy and promote oxygenation.

The nurse is preparing a 4-day-old I infant with a serum bilirubin level of 19 mg/dl (325 micromol/L) for discharge from the hospital. When teaching the parents about home phototherapy, which instruction should the nurse include in the discharge teaching plan? a. Reposition the infant every 2 hours. b. Perform diaper changes under the light. c. Feed the infant every 4 hours. d. Cover with a receiving blanket.

Reposition the infant every 2 hours. Rational: An infant, who is receiving phototherapy for hyperbilirubinemia, should be repositioned every two hours. The position changes ensure that the phototherapy lights reach all of the body surface areas. Bathing, feedings, and diaper changes are ways for the parents to bond with the infant and can occur away from the treatment. Feedings need to occur more frequently than every 4 hours to prevent dehydration. The infant should wear only a diaper so that the skin is exposed to the phototherapy.

A client who had a below the knee amputation is experiencing severe phantom limb pain (PLP) and ask the nurse if mirror therapy will make the pain stop. Which response by the nurse is likely to be most helpful? a. Research indicates that mirror therapy is effective in reducing phantom limb pain b. You can try mirror therapy, but do not expect to complete elimination of the pain c. Transcutaneous electrical nerve stimulators (TENS) have been found to be more effective d. Where did you learn about the use of mirror therapy in treating in treating phantom limb pain?

Research indicates that mirror therapy is effective in reducing phantom limb pain Rationale: pain relief associated with mirror therapy may be due to the activation of neurons in the hemisphere of the brain that is contralateral to the amputated limb when visual input reduces the activity of systems that perceive protopathic pain.

Following an outbreak of measles involving 5 students in an elementary school, which action is most important for the school nurse to take? a. Review the immunization records of all children in the elementary school b. Report the measles outbreak to all community health organizations c. Schedule a mobile public health vehicle to offer measles inoculations to unvaccinated children. d. Restrict unvaccinated children from attending school until measles outbreak is resolved.

Restrict unvaccinated children from attending school until measles outbreak is resolved.

One week after client received above the knee amputation, client experiences phantom pain in left foot, what to do

Review causes and sensations of phantom pain

An unlicensed assistive personnel (UAP) reports that a client's right hand and fingers spasms when taking the blood pressure using the same arm. After confirming the presence of spams what action should the nurse take? a. Ask the UAP to take the blood pressure in the other arm b. Tell the UAP to use a different sphygmomanometer. c. Review the client's serum calcium level d. Administer PRN antianxiety medication.

Review the client's serum calcium level Rationale: Trousseau's sign is indicated by spasms in the distal portion of an extremity that is being used to measure blood pressure and is caused by hypocalcemia (normal level 9.0-10.5 mg/dl, so C should be implemented.

When assessing and adult male who presents as the community health clinic with a history of hypertension, the nurse note that he has 2+ pitting edema in both ankles. He also has a history of gastroesophageal reflex disease (GERD) and depression. Which intervention is the most important for the nurse to implement? a. Arrange to transport the client to the hospital b. Instruct the client to keep a food journal, including portions size. c. Review the client's use of over the counter (OTC) medications. d. Reinforce the importance of keeping the feet elevated.

Review the client's use of over the counter (OTC) medications Rationale: Sodium is used in several types of OTC medications. Including antacids, which the client may be using to treat his GERD. Further evaluation is need it to determine the need for hospitalization (A) A food journal (B) may help over, but dietary modifications are needed now since edema is present. (C) May relieve dependent edema, but not treat the underlying etiology.

To reduce staff nurse role ambiguity, which strategy should the nurse-manager implement? a. Review the staff nurse job description to ensure that it is clear, accurate, and current

Review the staff nurse job description to ensure that it is clear, accurate, and current

Following discharge teaching, a male client with duodenal ulcer tells the nurse the he will drink plenty of dairy products, such as milk, to help coat and protect his ulcer. What is the best follow-up action by the nurse? a. Remind the client that it is also important to switch to decaffeinated coffee and tea. b. Suggest that the client also plan to eat frequent small meals to reduce discomfort c. Review with the client the need to avoid foods that are rich in milk and cream. d. Reinforce this teaching by asking the client to list a dairy food that he might select.

Review with the client the need to avoid foods that are rich in milk and cream Rationale: Diets rich in milk and cream stimulate gastric acid secretion and should be avoided.

When preparing to discharge a male client who has been hospitalized for an adrenal crisis, the client expresses concern about having another crisis. He tells the nurse that he wants to stay in the hospital a few more days. Which intervention should the nurse implement? a. Administer anti-anxiety medication prior to providing discharge instructions b. Schedule an appointment for an out-patient psychosocial assessment. c. Obtain a blood cortisol level after last dose of synthetic ACTH d. Encourage the healthcare provider to delay the client's discharge.

Schedule an appointment for an out-patient psychosocial assessment. Rationale: Emotional stress can precipitate another adrenal crisis and should be monitored with periodic psychosocial assessments. A may be indicated but does not address the problem after discharge. A blood cortisol level is to diagnosis not to monitor the ongoing disease process. Canceling the discharge only delays the resolution of the problem.

An adult client present to the clinic with large draining ulcers on both lower legs that are characteristics of Kaposi's sarcoma lesions. The client is accompanied by two family member. Which action should the nurse take? a. Ask family member to wear gloves when touching the patient b. Send family to the waiting area while the client's history is taking c. Obtain a blood sample to determine is the client is HIV positive d. Complete the head to toes assessment to identify other sign of HIV

Send family to the waiting area while the client's history is taking Rationale: To protect the client privacy, the family member should be asked to wait outside while the client's history is take. Gloves should be worn when touching the client's body fluids if the client is HIV positive and these lesion are actually Kaposi sarcoma lesion. HIV testing cannot legally be done without the client explicit permission. A further assessment can be implemented after the family left the room.

A client at 38-weeks gestation complains of abdominal pain. The nurse notes that her abdomen is rigid. What is the probable cause of these findings?

Unknown

A male client receives a thrombolytic medication following a myocardial infarction. When the client has a bowel movement, what action should the nurse implement? a. Send stool sample to the lab for a guaiac test b. Observe stool for a day-colored appearance. c. Obtain specimen for culture and sensitivity analysis d. Asses for fatty yellow streaks in the client's stool.

Send stool sample to the lab for a guaiac test Rationale: Thrombolytic drugs increase the tendency for bleeding. So, guaiac (occult blood test) test of the stool should be evaluated to detect bleeding in the intestinal tract.

A male client reports the onset of numbness and tingling in his fingers and around his mouth. Which lab is important for the nurse to review before contacting the health care provider? a. capillary glucose b. urine specific gravity c. Serum calcium d. white blood cell count

Serum calcium Rationale: Numbness and tingling of the fingers and around the mouth, along with muscle cramps are signs of hypocalcemia

A male client who is admitted to the mental health unit for treatment of bipolar disorder has a slightly slurred speech pattern and an unsteady gait. Which assessment finding is most important for the nurse to report to the healthcare provider? a. Blood alcohol level of 0.09% b. Serum lithium level of 1.6 mEq/L or mmol/l (SI) c. Six hours of sleep in the past three days. d. Weight loss of 10 pounds (4.5 kg) in past month.

Serum lithium level of 1.6 mEq/L or mmol/l (SI) Rationale: The therapeutic level of Serum lithium is 0.8 to 1.5 mEq/L or mmol/l (SI). Slurred speech and ataxia are sign of lithium toxicity

A client with intestinal obstructions has a nasogastric tube to low intermittent suction and is receiving an IV of lactated ringer's at 100 ml/H. which finding is most important for the nurse to report to the healthcare provider? a. Gastric output of 900 mL in the last 24 hours b. Serum potassium level of 3.1 mEq/L or mmol/L (SI) c. Increased blood urea nitrogen (BUN) d. 24-hour intake at the current infusion rate.

Serum potassium level of 3.1 mEq/L or mmol/L (SI) Rationale: The normal potassium level in the blood is 3.5-5.0 milliEquivalents per liter (mEq/L). Rationale: The normal potassium level in the blood is 3.5-5.0 mill Equivalents per liter (mEq/L).

A young adult client is admitted to the emergency room following a motor vehicle collision. The client's head hit the dashboard. Admission assessment include: Blood pressure 85/45 mm Hg, temperature 98.6 F, pulse 124 beat/minute and respirations 22 breath/minute. Based on these data, the nurse formulates the first portion of nursing diagnosis as " Risk of injury" What term best expresses the "related to" portion of nursing diagnosis? a. Infection b. Increase intracranial pressure c. Shock d. Head Injury.

Shock Rationale: This client has symptoms of shock. Two signs of shock are decreased BP, and increased (often weak and thread) pulse, this client has both symptoms. A temperature of 98.6 F is average normal. An increase of temperature. D is correct but is vague and is not specifically related to the assessment date describe, so it is not the best answer.

A client refuses to ambulate, reporting abdominal discomfort and bloating caused by "too much gas buildup" the client's abdomen is distended. Which prescribed PRN medication should the nurse administer? a. Hydrocodone/Acetaminophen (Lortab) b. Simethicone (Mylicon) c. Promethazine (Phenergan) d. Nalbupine (Nubain)

Simethicone (Mylicon) Rationale: Simethicone is an antiflatulent that is used to increase the client's ability to expel flatus (B), which relieves the clients discomfort (A and D) are analgesic used to manage pain but do not alleviate the causes of the pain (C) is an antiemetic used to treat nauseas and does not relive excess flatus.

A nurse-manager is preparing the curricula for a class for charge nurses. A staffing formula based on what data ensures quality client care and is most cost-effective? a. Client geographic location and age b. Number of staff and number of clients c. Weekend and weekday staff availability d. Skills of staff and client acuity

Skills of staff and client acuity

To reduce the risk of symptoms exacerbation for a client with multiple sclerosis (MS), which instructions should the nurse include in the client's discharge plan? (Select all that apply). a. Space activities to allow for rest periods b. Take warm baths before starting exercise

Space activities to allow for rest periods Take warm baths before starting exercise

The nurse observes a 2cm area of non-blanchable erythema on the sacrum of an immobile client. What documentation of this finding is best for the nurse to enter into the client's record?

Stage 1 pressure ulcer on sacrum.

An older male client with a history of type 1 diabetes has not felt well the past few days and arrives at the clinic with abdominal cramping and vomiting. He is lethargic, moderately, confused, and cannot remember when he took his last dose of insulin or ate last. What action should the nurse implement first? a. obtain a serum potassium level b. administer the client's usual dose of insulin c. assess pupillary response to light d. Start an intravenous (IV) infusion of normal saline

Start an intravenous (IV) infusion of normal saline Rationale: the nurse should first start an intravenous infusion of normal saline to replace the fluids and electrolytes because the client has been vomiting, and it is unclear when he last ate or took insulin. The symptoms of confusion, lethargy, vomiting, and abdominal cramping are all suggestive of hyperglycemia, which also contributes to diuresis and fluid electrolyte imbalance.

A client who was admitted yesterday with severe dehydration is complaining of pain a 24 gauge IV with normal saline is infusing at a rate of 150 ml/hour. Which intervention should the nurse implement first? a. Establish the second IV site b. Asses the IV for blood return c. Stop the normal saline infusion. d. Discontinue the 24-gauge IV

Stop the normal saline infusion. Rationale: If the IV has infiltrated or become dislodges, the fluid is infusing into surrounding tissue and not into the vein. Stopping the infusion C is the priority action. Establishing another IV site is necessary for fluid resuscitation after the infiltrated infusion is discontinuing the IV (D) is necessary due to the pain, and a large gauge needle is preferable.

The mother of an adolescent tells the clinic nurse, "My son has athlete's foot, I have been applying triple antibiotic ointment for two days, but there has been no improvement." What instruction should the nurse provide? a. Antibiotics take two weeks to become effective against infections such as athlete's foot. b. Continue using the ointment for a full week, even after the symptoms disappear. c. Applying too much ointment can deter its effectiveness. Apply a thin layer to prevent maceration. d. Stop using the ointment and encourage complete drying of the feet and wearing clean socks.

Stop using the ointment and encourage complete drying of the feet and wearing clean socks. Rationale: Athlete's foot (tinea pedi) is a fungal infection that afflicts the feet and causes scaliness and cracking of the skin between the toes and on the soles of the feet. The feet should be ventilated, dried well after bathing, and clean socks should be placed on the feet after bathing. Antifungal ointments may be prescribed, but antibiotic ointments are not useful.

A male client reports to the clinic nurse that he has been feeling well and is often "dizzy" his blood pressure is elevated. Based on this findings, this client is at a greatest risk for which pathophysiological condition? a. Pulmonary hypertension b. Left ventricular hypertrophy c. Renal failure d. Stroke

Stroke

While visiting a female client who has heart failure (HF) and osteoarthritis, the home health nurse determines that the client is having more difficulty getting in and out of the bed than she did previously. Which action should the nurse implement first? a. Inquire about an electric bed for the client's home use b. Submit a referral for an evaluation by a physical therapist. c. Explain the usual progression of osteoarthritis and HF d. Request social services to review the client's resources.

Submit a referral for an evaluation by a physical therapist. Rationale: To promote independence and safety in the home, the client's decline in physical mobility and strength should be evaluated first by the physical therapist who is a member if the home health treatment team.

Which assessment finding for a client who is experiencing pontine myelinolysis should the nurse report to the healthcare provider? a. Sudden dysphagia b. Blurred visual field c. Gradual weakness d. Profuse diarrhea

Sudden dysphagia Rationale: Osmotic demyelination, also known as pontine myelinolysis, results in destruction of the myelin sheath that covers nerve cell in the brainstem. This condition can be caused by rapid correction of hyponatremia and is often seen in those with syndrome of inappropriate antidiuretic hormone, Symptoms of pontine myelinolysis are sudden and can include dysphagia, para or quadriparesis and dysarthria. Due to the risk of aspiration the healthcare provider should be notified of the client's sudden onset of difficulty swallowing dysphagia (A). Diplopia not blurred vision (B) may be experienced. Weakness occurs suddenly, rather than gradually (C). Constipation, not diarrhea (D), is common due to decreased motility.

The nursing staff on a medical unit includes a registered nurse (RN), practical nurse (PN), and an unlicensed assistive personnel (UAP). Which task should the charge nurse assign to the RN? a. Supervise a newly hired graduate nurse during an admission assessment. b. Transport a client who is receiving IV fluids to the radiology department. c. Administer PRN oral analgesics to a client with a history of chronic pain d. Complete ongoing focused assessments of a client with wrist restrain.

Supervise a newly hired graduate nurse during an admission assessment. Rationale: The admission assessment of a client should be completed by a professional nurse. A graduate nurse should be supervised by the RN to ensure that the graduate nurse understand and performs within the expected scope of practice. The UAP transport a stable client. (B) The PN can complete C and D

The nurse is performing an intake interview at a prenatal clinic. Which planned activities described by the client who is at 6 weeks gestations will the nurse investigate first?

Supervision of the renovation of an old house the family just purchased due to teratogen defect.

The nurse is assisting a new mother with infant feeding. Which information should the nurse provide that is most likely to result in a decrease milk supply for the mother who is breastfeeding? a. Supplemental feedings with formula b. Maternal diet high in protein c. Maternal intake of increased oral fluid d. Breastfeeding every 2 or 3 hours.

Supplemental feedings with formula Rationale: Infant sucking at the breast increases prolactin release and proceeds a feedback mechanism for the production of milk, the nurse should explain that supplemental bottle formula feeding minimizes the infant's time at the breast and decreases milk supply. B promotes milk production and healing after delivery. C support milk production. C is recommended routine for breast feeding that promote adequate milk supply.

When to use drug for motion sickness drug question

Take Zofran 4 hours before going on trip via sea.

Which nurse's behavior is a breach of clients confidentiality according to the Health Insurance Portable Accountability Act (HIPAA) regulations?

Take home a daily report sheet with the information of the team's client.

A female client receives a prescription for alendronate sodium (Fosamax) to treat her newly diagnose osteoporosis. What instruction should the nurse include in the client's teaching plan? a. Take on an empty stomach with a full glass of water

Take on an empty stomach with a full glass of water

An older client is admitted for repair of a broken hip. To reduce the risk for infection in the postoperative period, which nursing care interventions should the nurse include in the client's plan of care? (Select all that apply) a. Teach client to use incentive spirometer q2 hours while awake. b. Remove urinary catheter as soon as possible and encourage voiding.

Teach client to use incentive spirometer q2 hours while awake. Remove urinary catheter as soon as possible and encourage voiding.

A client with muscular dystrophy is concerned about becoming totally dependent and is reluctant to call the nurse to assist with activities of daily living (ADLs). To achieve maximum mobility and independence, which intervention is most important for the nurse to include in the client's plan of care? a. Elevate lower extremities while out of bed b. Teach family proper range of motion exercises. c. Maintain proper body alignment when in bed d. Encourage diaphragmatic breathing exercises.

Teach family proper range of motion exercises. Rationale: Performing proper range of motion exercised helps maintain maximum mobility by preventing excessive muscle atrophy and joint contractures. Elevating lower extremities decreases the amount of peripheral edema. Proper body alignment reduces strain on joints, tendons, ligaments and muscles and minimizes contractures in an abnormal position. Diaphragmatic breathing exercises may decrease the risk of pulmonary complications.

A client who recently underwent a tracheostomy is being prepared for discharge to home. Which instructions is most important for the nurse to include in the discharge plan? a. Explain how to use communication tools. b. Teach tracheal suctioning techniques c. Encourage self-care and independence. d. Demonstrate how to clean tracheostomy site.

Teach tracheal suctioning techniques Rationale: Suctioning helps to clear secretions and maintain an open airway, which is critical.

A 350-bed acute care hospital declares an internal disaster because the emergency generators malfunctioned during a city-wide power failure. The UAPs working on a general medical unit ask the charge nurse what they should do first. What instruction should the charge nurse provide to these UAPs? a. Go to the emergency department and complete assigned tasks b. Shut all doors to client rooms on the unit in case a fire erupts c. Offer to assist the ICY with ventilator-dependent clients d. Tell all their assigned clients to stay in their rooms.

Tell all their assigned clients to stay in their rooms. Rationale: A power failure leaves a unit in total darkness except for battery operated lighting. The top priority should be ensuring client safety by having clients stay in their rooms, and UAP can implement this. A is a higher priority in external disaster. B would further compound the lighting problems and is not indicated unless file or smoke is visible. C contraindicated until client safety is ensured on the assigned unit.

A client who received multiple antihypertensive medications experiences syncope due to a drop in blood pressure to 70/40. What is the rationale for the nurse's decision to hold the client's scheduled antihypertensive medication? a. Increased urinary clearance of the multiple medications has produced diuresis and lowered the blood pressure b. The antagonistic interaction among the various blood pressure medications has reduced their effectiveness c. The additive effect of multiple medications has caused the blood pressure to drop too low. d. The synergistic effect of the multiple medications has resulted in drug toxicity and resulting hypotension.

The additive effect of multiple medications has caused the blood pressure to drop too low Rationale: When medication with a similar action are administered, an additive effect occurs that is the sum of the effects of each of the medication. In this case, several medications that all lower the blood pressure, when administer together, resulted in hypotension.

You are preparing a plan of care for a client who is a Jehovah's Witness.The client has been told that surgery is necessary. With consideration to the client's religious preferences in development of the plan of care, you would document what?

The administration of blood and blood products is forbidden

A 60 year-old female client takes NPH insulin each morning. What would necessitate holding this clients usual morning NPH insulin dose?

The client is NPO for surgical debridement of a leg ulcer at 11.00am.

A client with a prescription for "do not resuscitate" (DNR) begins to manifest signs of impending death. After notifying the family of the client's status, what priority action should the nurse implement? a. The impending signs of death should be documented b. The client's need for pain medication should be determined. c. The nurse manager should be updated on the client's status d. The client's status should be conveyed to the chaplain

The client's need for pain medication should be determined. Rationale: Palliative care includes nursing interventions that provide relief for the dying client's suffering by assessment and treatment of pain and other problems that are physical, psychosocial and spiritual. After the family is notified for the client's impending death, the client's need for pain medication should be assessed.

The nurse notes that a 2 year-old child recovering from a tonsillectomy has an temperature of 92 degrees Fahrenheit at 8:00 AM. At 10:00 AM the child's mother reports that the child "feels very warm" to touch. The first action by the nurse should be to A) Reassure the mother that this is normal B) Offer the child cold oral fluids C) Reassess the child's temperature D) Administer the prescribed acetaminophen

The correct answer is C: Reassess the child''s temperature

A female client who has been taking the corticosteroid methylprenisolone (Solu-Medrol) for three weeks reports to the nurse that she has gained ten pounds since starting the medication, and she wants to stop taking it. What is the best response by the nurse?

The medication must be disconnected gradually, tapering the dose each day.

A client who is wheelchair-bound demonstrates a positive Thomas test after admission to the rehabilitation unit. The health care provider prescribes positioning the client prone for 30minutes three times each day to prevent further contractures. Based on this finding, what change in the client's plan of care should the nurse expect?

There may be a delay in the rehabilitation process.

An elderly client with degenerative joint disease asks if she should use the rubber jar openers that are available. The nurse's response should be based on which information about assistive devices? a. They can contribute to increased dependency b. They decrease the risk for joint trauma c. They promote muscle strength d. They diminish range of motion ability.

They decrease the risk for joint trauma Rationale: Assistive devices of this kind are very beneficial in reducing joint trauma(B) caused by excessive twisting. These devices promote independence, rather that increasing dependency

A client is admitted to a medical unit with the diagnosis of gastritis and chronic heavy alcohol abuse. What should the nurse administer to prevent the development of Wernicke's syndrome? a. Lorazepam (Ativan) b. Famotidine (Pepcid) c. Thiamine (Vitamin B1) d. Atenolol (Tenormin)

Thiamine (Vitamin B1) Rationale: Thiamine replacement is critical in preventing the onset of Wernickes encephalopathy, an acute triad of confusion, ataxia, and abnormal extraocular movements, such as nystagmus related to excessive alcohol abuse. Other medications are not indicated.

Immediately after extubation, a client who has been mechanically ventilated is placed on a 50% non-rebreather. The client is hoarse and complaining of a sore throat. Which assessment finding should the nurse report to the healthcare provider immediately? a. Upper airway stridor

Upper airway stridor

A 4-year-old with acute lymphocytic leukemia (ALL) is receiving a chemotherapy (CT) protocol that includes methotrexate (Mexate, Trexal, MIX), an antimetabolite. Which information should the nurse provide the parents about caring for their child? A. Use sunblock or protective clothing when outdoors

Use sunblock or protective clothing when outdoors

A client with a history of diabetes and coronary artery disease is admitted with shortness of breath, anxiety, and confusion. The client's blood pressure is 80/60 mmHg, heart rate 120 beats/minute with audible third and fourth heart sounds, and bibasilar crackles. The client's average urinary output is 5 ml/hour. Normal saline is infusing at 124 ml/hour with a secondary infusion of dopamine at mcg/kg/minute per infusion pump. With intervention should the nurse implement? a. Irrigate the indwelling urinary catheter. b. Prepare the client for external pacing. c. Obtain capillary blood glucose measurement. d. Titrate the dopamine infusion to raise the BP.

Titrate the dopamine infusion to raise the BP. Rationale: the client is experiencing cardiogenic shock and requires titration per protocol of the vasoactive secondary infusion, dopamine, to increase the blood pressure. Low hourly urine output is due to shock and does not indicate a need for catheter irrigation. Pacing is not indicated based on the client's capillary blood glucose should be monitored but is not directly indicated at this time.

When caring for a client who has acute respiratory distress syndrome (ARDS), the nurse elevates the head of the bed 30 degrees. What is the reason for this intervention? a. To reduce abdominal pressure on the diaphragm b. to promote retraction of the intercostal accessory muscle of respiration c. to promote bronchodilation and effective airway clearance d. to decrease pressure on the medullary center which stimulates breathing

To reduce abdominal pressure on the diaphragm Rationale: a semi-sitting position is the best position for matching ventilation and perfusion and for decreasing abdominal pressure on the diaphragm, so that the client can maximize breathing

A client who is taking an oral dose of a tetracycline complains of gastrointestinal upset. What snack should the nurse instruct the client to take with the tetracycline? a. Fruit-flavored yogurt. b. Cheese and crackers. c. Cold cereal with skim milk. d. Toasted wheat bread and jelly

Toasted wheat bread and jelly Rationale: Dairy products decrease the effect of tetracycline, so the nurse instructs the client to eat a snack such as toast, which contains no dairy products and may decrease GI symptoms.

A client who is admitted to the intensive care unit with a right chest tube attached to a THORA-SEAL chest drainage unit becomes increasingly anxious and complain of difficulty breathing. The nurse determine the client is tachypneic with absent breath sounds in the client's right lungs fields. Which additional finding indicates that the client has developed a tension pneumothorax? a. Continuous bubbling in the water seal chamber b. Decrease bright red blood drainage c. Tachypnea and difficulty breathing d. Tracheal deviation toward the left lung.

Tracheal deviation toward the left lung. Rationale: Tracheal deviation toward the unaffected left lung with absent breath sounds over the affected right lung are classic late signs of a tension pneumothorax.

A client in the postanesthesia care unit (PACU) has an eight (8) (Normal) on the Aldrete postanesthesia scoring system. What intervention should nurse implement a. The client should be kept in the recovery room b. Assess the client's respiratory status immediately c. Notify the client's surgeon immediately d. Transfer the client to the surgical floor.

Transfer the client to the surgical floor. Rationale: A score of 7 to 8 is normal and indicates that the client can be discharge from PACU. The PACU assessment form includes 5 mints areas of assessment: muscle activity, circulation, consciousness level, and oxygen saturation. Each of these 5 areas receives two points for normal. A, B, C are interventions that are not indicated for a score of 8.

Following and gunshot wound, an adult client a hemoglobin level of 4 grams/dl (40 mmol/L SI). The nurse prepares to administer a unit of blood for an emergency transfusion. The client has AB negative blood type and the blood bank sends a unit of type A Rh negative, reporting that there is not type AB negative blood currently available. Which intervention should the nurse implement? a. Transfuse Type A negative blood until type AB negative is available. b. Recheck the client's hemoglobin, blood type and Rh factor. c. Administer normal saline solution until type AB negative is available d. Obtain additional consent for administration of type A negative blood

Transfuse Type A negative blood until type AB negative is available Rationale: those who have type AB blood are considered universal recipients using A or B blood types that is the same Rh factor. The client's hemoglobin is critically low, and the client should receive a unit of blood that is type A, which must be Rh negative blood. Other options are not indicated in this situation.

The nurse has not finished administering routine oral medication because on e client experienced chest pain and another rectal bleeding. It is now dinner time and two clients must be fed. The unlicensed assistive personnel (UAP) is filling water pitchers and the practical nurse (PN) is charting vital signs. Which change in assignments is best for the team leader to make?

UAP feed the two clients, PN finish oral medications and RN monitor clients with chest pain and rectal bleeding

A client with history of bilateral adrenalectomy is admitted with a week, irregular pulse, and hypotension. Which assessment finding warrants immediate intervention by the nurse? a. Decrease urinary output b. Low blood glucose level c. Profound weight gain d. Ventricular arrhythmias.

Ventricular arrhythmias. Rationale: adrenal crisis, a potential complication of bilateral adrenalectomy, results in the loss of mineralocorticoids and sodium excretions that is characterized by hyponatremia, hyperkalemia, dehydration, and hypotension. Ventricular arrhythmias are life threatening and required immediate intervention to correct critical potassium levels.

The nurse is preparing to mix two medications from two different multidose vials, A and B. In which order should these actions be implemented when drawing the solutions from the vials? (Arrange from first on top to last on the bottom) a. Verify the drug and dose with the label on the vial b. Inject the volume of air to be aspirated from each vial c. Aspirate the desired volume from vial A c. Aspirate the desired volume from vial B

Verify the drug and dose with the label on the vial Inject the volume of air to be aspirated from each vial Aspirate the desired volume from vial A Aspirate the desired volume from vial B

The public nurse health received funding to initiate primary prevention program in the community. Which program the best fits the nurse's proposal? a. Case management and screening for clients with HIV. b. Regional relocation center for earthquake victims c. Vitamin supplements for high-risk pregnant women. d. Lead screening for children in low-income housing.

Vitamin supplements for high-risk pregnant women Rational: Primary prevention activities focus on health promotions and disease preventions, so vitamin for high-risk pregnant women provide adequate vitamin and mineral for fetal developmental.

The nurse is assessing a female client's blood pressure because she reported feeling dizzy. The blood pressure cuff is inflated to 140 mm hg and as soon as the cuff is deflated a korotkoff sound is heard. Which intervention should the nurse implement next? a. Wait 1 minute and palpate the systolic pressure before auscultating again

Wait 1 minute and palpate the systolic pressure before auscultating again

A client is discharged with automated peritoneal dialysis (PD) to be used nightly...which instructions should the nurse include? a. Wash hands before cleaning exit site b. Keep the head of the bed flat at night c. Feel for a thrill and a distal pulse nightly d. Do not get up if fluid is left in the abdomen

Wash hands before cleaning exit site Rationale: meticulous hand hygiene is essential when performing care for a peritoneal dialysis, infections is a common complication of peritoneal dialysis.

A client who is experiencing musculoskeletal pain receives a prescription for ketorolac 15mg IM q6 hours. The medication is depended in a 30mg/ml pre-filled syringe. Which action should the nurse implement when giving the medication? a. Administer the entire pre-filled syringe deep in the dorsogluteal site. b. Use a separate syringe to remove 15mg from the pre-filled syringe and give in the back of the arm. c. Waste 0.5 ml from the pre-filled syringe and inject the medication in the ventrogluteal site. d. Call the healthcare provider to request a prescription change to match the dispensed 30mg dose.

Waste 0.5 ml from the pre-filled syringe and inject the medication in the ventrogluteal site RATIONALE: The pre-filled contain 30mg /1ml, so 0.5ml should be wasted to obtain the correct dosage of 15mg for administration in the preferred IM ventrogluteal site. The nurse is responsible for calculating and preparing the prescribed dose using the available concentration, so other options are not indicated.

In caring for a client who is receiving linezolid IV for nosocomial pneumonia, which assessment finding is most important for the nurse to report to the healthcare provider? a. Watery diarrhea

Watery diarrhea

An adult male who fell from a roof and fractures his left femur is admitted for surgical stabilization after having a soft cast applied in the emergency department. Which assessment finding warrants immediate intervention by the nurse? a. Weak palpable distal pulses

Weak palpable distal pulses

A client with endometrial carcinoma is receiving brachytherapy and has radioactive Cesium (Cs) loaded in a vaginal applicator. What action should the nurse implement?

Wear a dosimeter film badge when in the client's room

When teaching a group of school-age children how to reduce the risk of Lyme disease which instruction should the camp nurse include a. Wash hands frequently b. Avoid drinking lake water c. Wear long sleeves and pants d. Do not share personal products

Wear long sleeves and pants Rationale: Lyme disease is it tick bone disorder and is transmitted to a child via a tick bite. Keeping the skin covered reduces the risk of being bitten by a tick. Other options are not reduce the risk for tick bites.

After multiple attempts to stop drinking, an adult male is admitted to the medical intensive care unit (MICU) with delirium tremens. He is tachycardic, diaphoretic, restless, and disoriented. Which finding indicates a life- threatening condition? a. Widening QRS complexes and flat waves

Widening QRS complexes and flat waves

When planning care for a client with acute pancreatitis, which nursing intervention has the highest priority? a. Withhold food and fluid intake. b. Initiate IV fluid replacement. c. Administer antiemetic as needed. d. Evaluate intake and output ratio.

Withhold food and fluid intake Rational: The pathophysiologic processes in acute pancreatitis result from oral fluid and ingestion that causes secretion of pancreatic enzymes, which destroy ductal tissue and pancreatic cells, resulting in auto digestion and fibrosis of the pancreas. The main focus of the nursing care is reducing pain caused by pancreatic destruction through interventions that decrease GI activity, such as keeping the client NPO. Other choices are also important intervention but are secondary to pain management.

When assessing a male client, the nurse notes that he has unequal lung expansion. What conclusion regarding this finding is most likely to be accurate? The client has: a- A collapsed lung b- A history of COPD c- A chronic lung infection d- Normally functioning lungs

a. A collapsed lung Rationale: Unilateral absence of chest movement (or unequal lung expansion because one lung is not moving at all) may be indicative previous surgical removal of that lung, a bronchial obstruction, or a collapsed lung caused by air or fluid in the pleural space.

When organizing home visits for the day, which older client should the home health nurse plan to visit first? a. A woman who takes naproxen (Naprosyn) and reports a recent onset of dark, tarry stools. b. A man who receives weekly injections of epoetin (Procrit) for a low serum iron level c. A man with emphysema who smokes and is complaining of white patches in his mouth d. A frail woman with heart failure who reported a 2 pounds' weight gain in the last week.

a. A woman who takes naproxen (Naprosyn) and reports a recent onset of dark, tarry stools

An older adult resident of a long-term care facility has a 5-year history of hypertension. The client has a headache and rate the pain 5 on a pain scale 0 to 10. The client's blood pressure is currently 142/89. Which interventions should the nurse implement? (Select all that apply) a. Administer a daily dose of lisinopril as scheduled. b. Assess the client for postural hypotension. c. Notify the healthcare provider immediately d. Provide a PRN dose of acetaminophen for headache e. Withhold the next scheduled daily dose of warfarin.

a. Administer a daily dose of lisinopril as scheduled. d. Provide a PRN dose of acetaminophen for headache Rational: the client' routinely scheduled medication, lisinopril, is an antihypertensive medication and should be administered as scheduled to maintain the client's blood pressure. A PRN dose of acetaminophen should be given for the client's headache. The other options are not indicated for this situation.

The nurse observes a newly hired unlicensed assistive personnel (UAP) performing a fingestick to obtain a client's blood glucose. Prior to sticking the client's finger, the UAP explains the procedure and tell the client that it is painless. What action should the nurse take? a. Allow the UAP to complete the procedure, then discuss the painless comment privately with the UAP. b. Stop the UAP before the procedure and explain to the client that some discomfort may be felt c. Interject that while the procedure is not extremely painful, the client will feel a prick on the finger. d. Report the incident to the education director and request additional instruction for the UAP.

a. Allow the UAP to complete the procedure, then discuss the painless comment privately with the UAP.

In preparing assignments for the shift, which client is best for the charge nurse to assign to a practical nurse (PN)? a- An older client who fell yesterday and is now complaining of diplopia b- An adult newly diagnosed with type 1 diabetes and high cholesterol c- A client with pancreatic cancer who is experience intractable pain. d- An older client post-stroke who is aphasic with right-sided hemiplegia

a. An older client post-stroke who is aphasic with right-sided hemiplegia

While the nurse is conducting a daily assessment of an older woman who resides in a long-term facility, the client begins to cry and tells the nurse that her family has stopped calling and visiting. What action should the nurse take first? a. Ask the client when a family member last visited her. b. Determine the client's orientation to time and space c. Review the client's record regarding social interactions d. Reassure the client of her family's love for her

a. Ask the client when a family member last visited her.

The healthcare provider prescribes oxycodone/ aspirin 1 tab PO every 4h as needed for pain, for a client with polycystic kidney disease. Before administering this medication, which component of the prescription should the nurse question? a- Aspirin content. b- Dose c- Route d- Risk for addiction

a. Aspirin content. Rationale: Aspirin content medication are contraindicated for client with polycystic kidney disease because the risk for bleeding.

The nurse is arranging home care for an older client who has a new colostomy following a large bowel resection three days ago. The client plans to live with a family member. Which action should the nurse implement? Select all that apply A.)Assess the client for self-care ability B.)Provide pain medication instructions C.)Teach care of ostomy to care provider D.)Instruct client to Look up a video E.)Give client a pamphlet

a. Assess the client for self-care ability b. Provide pain medication instructions c. Teach care of ostomy to care provider

An adult client with severe depression was admitted to the psychiatric unit yesterday evening. Although the client ran one year ago, his spouse states that the client no longer runs, bur sits and watches television most of the day. Which is most important for the nurse to include in this client's plan of care for today? a. Assist client in identifying goals for the day. b. Encourage client to participate for one hour in a team sport. c. Schedule client for a group that focuses on self-esteem. d. Help client to develop a list of daily affirmations.

a. Assist client in identifying goals for the day. Rationale: clients with severe depression have low energy and benefit from structured activities because concentration is decreased. The client participates in care by identifying goals for the day is the most important intervention for the client's first day at the unit. Other options can be implemented over time, as the depression decreases.

Which instruction is most important for the nurse to provide a client who is being discharge following treatment for Guillain-Barre syndrome? a. Avoid exposure to respiratory infections. b. Use relaxation exercise when anxious c. Continue physical therapy at home d. Plan short, frequent rest periods.

a. Avoid exposure to respiratory infection

The home care nurse provide self-care instruction for a client with chronic venous insufficiency caused by deep vein thrombosis. Which instructions should the nurse include in the client's discharge teaching plan? Select all that apply a. Avoid prolonged standing or sitting b. Use a recliner for long periods of sitting c. Continue wearing elastic stockings d. Maintain the bed flat while sleeping e. Cross legs at knee but not at ankle

a. Avoid prolonged standing or sitting b. Use a recliner for long periods of sitting c. Continue wearing elastic stockings

A client has a prescription for lorazepam 2mg for alcohol withdrawal symptoms. Which finding... the client? a. Blood pressure 149/101 b. Irregular pulse rate of 80 c. Oral temperature is 98.9 F (37.1 C) d. Pain rated 7 on scale 1-10

a. Blood pressure 149/101

Which information is more important for the nurse to obtain when determining a client's risk for Obstructive Sleep Apnea Syndrome (OSAS)? a- Body mass index b- Level of consciousness c- Self-description of pain d- Breath sounds

a. Body mass index

Which statement is accurate regarding the pathological changes in the pulmonary system associated with acute (adult) respiratory distress syndrome (ARDS)? a. Capillary hydrostatic pressure exceeds colloid osmotic pressure, producing interstitial edema b. A high ventilation-to-perfusion ratio is characteristic of affected lung fields in ARDS c. Functional residual capacity and lung compliance increase as the disease progresses d. Interstitial edema that occurs due to capillary fluid shifts is usually more serious than alveolar edema.

a. Capillary hydrostatic pressure exceeds colloid osmotic pressure, producing interstitial edema

The nurse is administering a 750 ml cleansing enema to an adult client. After approximately 150 ml of enema has informed, the client states, 'stop I can't hold anymore." What action should the nurse take? a. Clamp the tubing and instruct the client to breathe deeply before continuing. b. Discontinue infusing the enema and record the client's response. c. Slow infusion of the enema and instruct the client to use paint breathing d. Place the client on the bedpan and continue infusion of the enema.

a. Clamp the tubing and instruct the client to breathe deeply before continuing. Rationale: Clamping the tube momentarily allows the muscle to relax and prevents expulsion of the solution prematurely. B may be eventually necessary but A should be tried first.

A newly admitted client vomits into an emesis basin as seen in the picture. The nurse should consult with the healthcare provider before administering which of the client's prescribed medications? a. Clopidogrel (Plavix), an antiplatelet agent, given orally b. Methylprednisolone (solu-medrol), a corticosteroid, to be given IV c. Nitroglycerin (Nitro-Dur) an antianginal, to be given transdermally. d. Enoxaparin (lovenox), a low-molecular weight heparin to be given subcutaneous. e. Furosemide (Lasix), a loop diuretic, to be given intravenously.

a. Clopidogrel (Plavix), an antiplatelet agent, given orally Rationale: Because of the emesis is coffee brown appearance, which is an indicator of bleeding in GI tract, the nurse should consult the health care provider because increase the risk of bleeding.

A male client is admitted for the removal of an internal fixation that was inserted for the fracture ankle. During the admission history, he tells the nurse he recently received vancomycin (vancomycin) for a methicillin-resistant Staphylococcus aureus (MRSA) wound infection. Which action should the nurse take? (Select all that apply.) a. Collect multiple site screening culture for MRSA b. Call healthcare provider for a prescription for linezolid (Zyrovix) c. Place the client on contact transmission precautions d. Obtain sputum specimen for culture and sensitivity e. Continue to monitor for client sign of infection.

a. Collect multiple site screening culture for MRSA c. Place the client on contact transmission precautions e. Continue to monitor for client sign of infection. Rationale: Until multi-site screening cultures come back negative (A), the client should be maintained on contact isolation(C) to minimize the risk for nosocomial infection. Linezolid (Zyvox), a broad spectrum anti-infecting, is not indicated, unless the client has an active skin structure infection cause by MRSA or multidrug- resistant strains (MDRSP) of Staphylococcus aureus. A sputum culture is not indicated D) based on the client's history is a wound infection.

A client is admitted with acute pancreatitis. The client admits to drinking a pint of bourbon daily. The nurse medicates the client for pain and monitors vital signs q2 hours. Which finding should the nurse report immediately to the healthcare provider? a. Confusion and tremors b. Yellowing and itching of skin. c. Abdominal pain and vomiting d. Anorexia and abdominal distention

a. Confusion and tremors Rationale: daily alcohol is the likely etiology for the client's pancreatitis. Abrupt cessation of alcohol can result in delirium tremens (DT) causing confusion and tremors, which can precipitate cardiovascular complications and should be reported immediately to avoid life-threatening complications. The other options are expected findings in those with liver dysfunction or pancreatitis, but do not require immediate action.

A 6-year-old child with acute infectious diarrhea is placed on a rehydration therapy... Which action should the nurse instruct the parents to take if the child begins to vomit? a. Continue giving Oral Rehydration Salts frequently in small amounts b. Withhold all oral intake c. Supplement Oral Rehydration Salts with gelatin or chicken broth d. Provide only bottle water.

a. Continue giving Oral Rehydration Salts frequently in small amounts

A client is receiving oxytocin (Pitocin) to augment early labor. Which assessment is most important time the infusion rate is increases? a. Contraction pattern b. Blood pressure c. Infusion site d. Pain level

a. Contraction pattern

The nurse is assessing the thorax and lungs of a client who is having respiratory difficulty. Which finding is most indicative of respiratory distress? a- Contractions of the sternocleidomastoid muscle. b- Respiratory rate of 20 breath/mints c- Downward movement of diaphragm with inspiration d- A pulse oximetry reading of SpO2 95%

a. Contractions of the sternocleidomastoid muscle Rationale: Force inspiration needs to use accessories muscle and rib cage.

A young couple who has been unsuccessful in conceiving a child for over a year is seen in the family planning clinic. During an initial visit, which intervention is most important for the nurse to implement? a. Determine current sexual practices b. Prepare a female client for an ultrasound c. Request a sperm sample for ovulation d. Evaluate hormone levels on both clients

a. Determine current sexual practices Rationale: First a history should be obtained including practices that might be related to the infertility, such as douching, daily ejaculation or the male partner's exposure to heat, such as frequent sauna or work environment which can decrease sperm production (A B or C) may be indicated after a complete assessment is obtained.

A newly hired home health care nurse is planning the initial visit to an adult client who has had multiple sclerosis (MS) for the past 20 years and is currently bed-bound and is lifted by a hoist. An unlicensed caregiver provides care 8 hours/ daily, 5 days/week. During the initial visit to this client, which intervention is most important to the nurse to implement? a. Determine how the client is cared for when caregiver is not present. b. Develop a client needs assessment and review with the caregiver c. Evaluate the caregiver's ability to care for the client's needs. d. Review with the care giver the interventions provided each day.

a. Determine how the client is cared for when caregiver is not present

A male client was transferred yesterday from the emergency department to the telemetry unit because he had ST depression and resolved chest pain. When his EKG monitor alarms for ventricular tachycardia (VT), what action should the nurse take first? a. Determine the client's responsiveness and respirations b. Bring the crash cart to the room to defibrillate the client. c. Immediately initiate chest compressions. d. Notify the emergency response team

a. Determine the client's responsiveness and respirations Rationale: Activities, such as brushing teeth, can mimic the waveform of VI, so first he client should be assessed (A) to determine if the alarm is accurate. The crash cart can be brought to the room by someone else and defibrillation (B) delivered as indicated by the client's rhythm. Based on as assessment of the client, CPR© as summoning the emergency response team (D) may be indicated.

A young adult male was admitted 36 hours ago for a head injury that occurred as the result of a motorcycle accident. In the last 4 hours, his urine output has increased to over 200 ml/H. Before reporting the finding to the healthcare provider, which intervention should the nurse implement? a- Evaluate the urine osmolality and the serum osmolality values. b- Obtain blood pressure and assess for dependent edema c- Measure oral secretions suctioned during last hours d- Obtain capillary blood samples q2 hours for glucose monitoring.

a. Evaluate the urine osmolality and the serum osmolality values. Rationale: With a known head injury, sudden inadequate secretion of antidiuretic hormone (ADH) can cause excessive output of diluted urine. Evaluating laboratory results should de determined to identify findings of neurogenic diabetes insipidus (DI), such as low urine osmolarity and normal serum osmolarity (A) prior to notify the healthcare provider so that these finding can be included in the report. Massive diuresis, dehydration, and thirst manifest hypotension, irregular tachycardia, decrease skin turgor, but B or C are not related to DI.

An adult male was diagnosed with stage IV lung cancer three weeks ago. His wife approaches the nurse and asks how she will know that her husband's death is imminent because their two adult children want to be there when he dies. What is the best response by the nurse? a. Explain that the client will start to lose consciousness and his body system will slow down b. Reassure the spouse that the healthcare provider will let her know when to call the children c. Offer to discuss the client's health status with each of the adult children d. Gather information regarding how long it will take for the children to arrive

a. Explain that the client will start to lose consciousness and his body system will slow down Rationale: Expected signs of approaching death include noticeable changes in the client's level of consciousness and a slowing down of body systems. The nurse should answer the spouse's questions about the signs of imminent death rather than offering reassurance that may or may not be true. Other options listed may be implemented but the nurse should first answer the spouse's question directly.

A neonate with a congenital heart defect (CHD) is demonstrating symptoms of heart failure (HF). Which interventions should the nurse include in the infant's plan of care? a. Give O2 at 6 L/nasal canula for 3 repeated oximetry screens below 90% b. Administer diuretics via secondary infusion in the morning only c. Evaluate heart rate for effectiveness of cardio tonic medications d. Use high energy formula 30 calories/ounce at Q3 hours feeding via soft nipples e. Ensure uninterrupted and frequent rest periods between procedures.

a. Give O2 at 6 L/nasal canula for 3 repeated oximetry screens below 90% c. Evaluate heart rate for effectiveness of cardio tonic medications d. Use high energy formula 30 calories/ounce at Q3 hours feeding via soft nipples e. Ensure uninterrupted and frequent rest periods between procedures. Rationale: Pulse oximetry screening supports prescribed level of O2. HR provides an evaluative criterion for cardiac medications, which reduce heart rate, increase strength contractions (inotropic effects) and consequently affect systemic circulation and tissue oxygenation. Breast milk or basic formula provide 20 calories/ounce, so frequent feedings with high energy formula. D minimize fatigue is necessary.

The healthcare provider prescribes carboprost tromethamine (Hemabate) 250 mcg IM for a multigravida postpartum client who is experiencing heavy, bright red vaginal bleeding. Prior to administering this medication, which interventions should the RN implement? a. Give the prescribed antiemetic.

a. Give the prescribed antiemetic. Rationale: Hemabate side effects are a light-headed feeling, like you might pass out; shortness of breath. severe nausea, vomiting, or diarrhea; or. increased high blood pressure (severe headache, blurred vision, buzzing in your ears, anxiety, confusion, chest pain).

The nurse is assessing a 4-year-old boy admitted to the hospital with the diagnosis of possible nephrotic syndrome. Which statement by the parents indicates a likely correlation to the child's diagnosis? a- I couldn't get my son's socks and shoes on this morning" b- My son has been on amoxicillin/clavulanate for 2 days for an ear infection c- My son has had a red rash over his entire body for the past 4 days. d- I couldn't get my son calm down and sleep last night.

a. I couldn't get my son's socks and shoes on this morning"

The nurse is caring for a client with hypovolemic shock who is receiving two units of packed red blood cells (RBCs) through a large bore peripheral IV. What action promotes maintenance of the client's cardiopulmonary stability during the blood transfusion? a- Increase the oxygen flow via nasal cannula if dyspnea is present. b- Place in a Trendelenburg position to increase cerebral blood flow c- Monitor capillary glucose measurements hourly during transfusion. d- Encourage increased intake of oral fluid to improve skin turgor.

a. Increase the oxygen flow via nasal cannula if dyspnea is present.

A young adult male is admitted to the emergency department with diabetic ketoacidosis (DKA). His pH is 7.25, HCO3 is 12 mEq/L or 12 mmol/L (SI), and blood glucose is 310 mg/dl or 17.2 mmol/L (SI). Which action should the nurse implement? a- Infuse sodium chloride 0.9% (normal saline) b- Prepare an emergency dose of glucagon c- Determine the last time the client ate d- Check urine for ketone bodies with a dipstick

a. Infuse sodium chloride 0.9% (normal saline) Rationale DKA an increase in glucose and ketone bodies, result in hyperosmolar dehydration, so is necessary to restore fluid balance.

The nurse is preparing a client who had a below-the-knee (BKA) amputation for discharge to home. Which recommendations should the nurse provide this client? (Select all that apply) a. Inspect skin for redness b. Use a residual limb shrinker c. Apply alcohol to the stump after bathing d. Wash the stump with soap and water e. Avoid range of motion exercises

a. Inspect skin for redness b. Use a residual limb shrinker d. Wash the stump with soap and water Rationale: Several actions are recommended for home care following an amputation. The skin should be inspected regularly for abnormalities such as redness, blistering, or abrasions. A residual limb shrinker should be applied over the stump to protect it and reduce edema. The stump should be washed daily with a mild soap and carefully rinse and dried. The client should avoid cleansing with alcohol because it can dry and crack the skin. Range of motion should be done daily.

The nurse is assessing a middle-aged adult who is diagnosed with osteoarthritis. Which factor in this client's history is a contributor to the osteoarthritis? a- Long distance runner since high school. b- Lactose intolerant since childhood c- Photosensitive to a drug currently taking d- Recently treated for deep vein thrombosis.

a. Long distance runner since high school. Rationale: Osteoarthritis is a degenerative joint disease of the cause by traumatic or repetitive stress to weight-bearing joint such as high impact sport like running.

A client with cirrhosis of the liver is admitted with complications related to end stage liver disease. Which intervention should the nurse implement? (Select all that apply.) a. Monitor abdominal girth. b. Increase oral fluid intake to 1500 ml daily. c. Report serum albumin and globulin levels. d. Provide diet low in phosphorous. e. Note signs of swelling and edema.

a. Monitor abdominal girth. c. Report serum albumin and globulin levels. e. Note signs of swelling and edema. Rational: monitoring for increasing abdominal girth and generalized tissue edema and swelling are focused assessments that provide data about the progression of disease related complications. In advanced cirrhosis, liver function failure results in low serum albumin and serum protein levels, which caused third spacing that results in generalized fluid retention and ascites. Other options are not indicated in end stage liver disease

The nurse is preparing a client for discharge from the hospital following a liver transplant. Which instruction is most important for the nurse to include in this client's discharge teaching plan? a. Monitor for an elevated temperature b. Measure the abdominal girth daily c. Report the onset of sclera jaundice d. Keep a record of daily urinary output

a. Monitor for an elevated temperature Rationale: The client should be instructed to monitor or elevated temperature because immunosuppressant agents, which are prescribed to reduce rejection after transplantation, place the client at risk for infection. The client should recognize sign of liver rejection, such as sclera jaundice and increasing abdominal girths, but fever may be the only sign of infection. A is not as important and monitoring for signs of infection.

A male client is returned to the surgical unit following a left nephrectomy and is medicated with morphine. His dressing has a small amount of bloody drainage, and a JacksonPratt bulb surgical drainage device is in place. Which intervention is most important for the nurse to include in this client's plan of care? a- Monitor urine output hourly. b- Assess for back muscle aches c- Record drainage from drain d- Obtain body weight daily

a. Monitor urine output hourly. Rationale: When one kidney is removed the remaining kidney must do all the volume filtering, so A is immediate to the postoperative period.

While monitoring a client during a seizure, which interventions should the nurse implement? (Select all that apply) a. Move obstacle away from client b. Monitor physical movements c. Insert an oral padded tongue blade d. Observe for a patent airway e. Record the duration of the seizure f. Restrain extremity to avoid seizures

a. Move obstacle away from client b. Monitor physical movements d. Observe for a patent airway e. Record the duration of the seizure Rationale: Moving this away from the client helps prevent to unnecessary injurie. Observing for the pt airway alert the nurse to provide airway assistance as soon as the seizure stop D and E provide the healthcare provider with an accurate description of the seizure activities. C inserting something on the mouth can obstruct may cause further airway obstruction and is contraindicated even if the client is biting the tongue. F may cause further injury and is contraindicated.

An older female who ambulates with a quad-cane prefers to use a wheel chair because she has a halting and unsteady gait at times. Which interventions should the nurse implement? (Select all that apply) a. Move personal items within client's reach b. Lower bed to the lower possible position c. Raise all bed rails when the client is resting d. Give directions to call for assistance e. Assist client to the bathroom in 2 hours. f. Encourage the use of the wheelchair

a. Move personal items within client's reach b. Lower bed to the lower possible position d. Give directions to call for assistance e. Assist client to the bathroom in 2 hours. Rationale: A client who needs assistive devices, such as quad-cane is at risk for falls. Precautions that should implement include ensuring that personal items are within reach the bed is in the lowest position and directions are given to call assistance to minimize the risk for falls. Frequently assisting the client to the bathroom help ensure this client does not go the bathroom by herself, thereby decreasing the possibility of falling.

A client with urticaria due to an environmental allergies is taking diphenhydramine... Which complaint should the nurse identify to the client as a side effect of the OTC medication? a. Nausea and indigestion. b. Hyper salivation c. Eyelid and facial twitching d. Increased appetite

a. Nausea and indigestion.

The nurse is caring for a toddler with a severe birth anomaly that is dying. The parents... holding the child as death approaches. Which intervention is most important for the nurse? a- Notify nursing supervisor and hospital chaplain of the child's impending death. b- Verify that the no resuscitate forms are in the child's medical record c- Ask the parents if they have made arrangements with a funeral home d- Provide staff coverage to sit with them as the child's death approaches.

a. Notify nursing supervisor and hospital chaplain of the child's impending death.

Two days after admission a male client remembers that he is allergic to eggs, and informs the nurse of the allergy. Which actions should the nurse implement? (Select all that apply) a. Notify the food services department of the allergy. b. Enter the allergy information in the client's record. c. Document the statement in the nurse's notes d. Note the allergy on the diet intake flow sheet e. Add egg allergy to the client's allergy arm band.

a. Notify the food services department of the allergy. b. Enter the allergy information in the client's record. e. Add egg allergy to the client's allergy arm band. Rationale: The dietary department needs to screen menu selections for foods that are prepared with eggs. The client's chart should be clearly marked but the statement does not need to be documented in the nurse's note or included in the intake record. Allergy identification on the arm band is a universal location where allergies are noted while client is hospitalized.

When changing a diaper on a 2-day-old infant, the nurse observes that the baby's legs are... this finding, what action should the nurse take next? a- Notify the healthcare provider b- Continue care since this is a normal finding c- Document the finding in the record d- Perform range of motion to the joint.

a. Notify the healthcare provider

A young adult female college student visits the health clinic in early winter to obtain birth control pills. The clinic nurse asks if the student has received an influenza vaccination. The student stated she did not receive vaccination because she has asthma. How should the nurse respond? a. Offer to provide the influenza vaccination to the student while she is at the clinic b. Encourage the student to obtain a vaccination prior to the next influenza season. c. Confirm that a history of asthma can increase risks associated with the vaccine. d. Advise the student that the nasal spray vaccine reduces side effects for people with asthma.

a. Offer to provide the influenza vaccination to the student while she is at the clinic Rationale: person with asthma are at increased risk related to influenza and should receive the influenza vaccination prior to or during influenza season. Waiting until the start of the next season places the student at risk for the current season. The vaccination does not increase risk for persons with asthma, but the nasal spray may result in increased wheezing after receiving that form of the vaccination.

When conducting diet teaching for a client who is on a postoperative soft diet, which foods should eat? (Select all that apply) a. Pasta, noodles, rice. b. Egg, tofu, ground meat. c. Mashed, potatoes, pudding, milk. d. Brussel sprouts, blackberries, seeds. e. Corn bran, whole wheat bread, whole grains.

a. Pasta, noodles, rice. b. Egg, tofu, ground meat. c. Mashed, potatoes, pudding, milk. Rational: a client's postoperative diet is commonly progressed as tolerated. A soft diet includes foods that are mechanically soft in texture (pasta, egg, ground meat, potatoes, and pudding. High fiber foods that require thorough chewing and gas forming foods, such as cruciferous vegetables and fresh fruits with skin, grains and seeds are omitted.

The nurse is preparing dose # 7 of an IV piggyback infusion of tobramycin for a 73-yearol client with infected pseudomonas aeruginosa. Which assessment data warrants further intervention by the nurse? a- Peak and trough levels have not been drawn since the tobramycin was started b- Today labs report indicates a white blood cell count of 13,000 cell/mm3 or 13 x 10777/L (S1) c- A serum creatinine level of 1.0 mg/dl or 88 mcmol/L (S1) is documented on yesterday flowsheet. d- The culture growth form the burn areas is sensitive to aminoglycosides.

a. Peak and trough levels have not been drawn since the tobramycin was started

When entering a client's room, the nurse discovers that the client is unresponsive and pulseless. The nurse initiate CPR and Calls for assistance. Which action should the nurse take next? a. Prepare to administer atropine 0.4 mg IVP b. Gather emergency tracheostomy equipment c. Prepare to administer lidocaine at 100 mg IVP d. Place cardiac monitor leads on the client's chest.

a. Place cardiac monitor leads on the client's chest. Rationale: Before further interventions can be done, the client's heart rhythm must be determined. This can be done by connecting the client to the monitor. A or C are not a first line drug given for any of the life threatening, pulses dysrhythmias

The nurse is caring for a client immediately after inserting a PICC line. Suddenly, the client becomes anxious and tachycardiac, and loud churning is heard over the pericardium upon auscultation. What action should the nurse take first? a. Place client in Trendelenburg position on the left side. b. Administer precordial thump c. Monitor the client with a 12-lead electrocardiogram d. Request a STAT portable chest x-ray.

a. Place client in Trendelenburg position on the left side

In caring for the body of a client who just died, which tasks can be delegate to the unlicensed assistive personnel (UAP)? (Select all that apply.) a. Place personal religious artifacts on the body. b. Confirm the client's wishes for tissue donation c. Observe consent for autopsy signature by family. d. Attach identifying name tags to the body. e. Follow cultural beliefs in preparing the body.

a. Place personal religious artifacts on the body. d. Attach identifying name tags to the body. e. Follow cultural beliefs in preparing the body.

The nurse is caring for a one week old infant who has a ventriculoperitoneal (VP) shunt that was placed 2 days after birth. Which findings are an indication of a postoperative complication? Select all that Apply a. Poor feeding and vomiting b. Leakage of CSF from the incisional site c. Hyperactive bowel sound d. Abdominal distention e. WBC count of 10000/mm3

a. Poor feeding and vomiting b. Leakage of CSF from the incisional site d. Abdominal distention Rationale: A, B and D are sign of postoperative complications. Shunt malfunction is most often caused by mechanical obstruction, which can result from ventricular exudate, distal end thrombosis or displacement, and/or infection. CNS infection is usually manifested by poor feeding, vomiting, elevated temperature, decreased responsiveness and seizure activity. Incisional leakage should be tested for glucose, an indication of CSF, which place the infant at risk for infection. Abdominal distention is a manifestation of peritonitis or a postoperative ileus from distal catheter placement. C is not a result of a shunt obstruction and E is a normal finding for one-week-old neonate.

Following an esophagogastroduodenoscopy (EGD) a male client is drowsy and difficult to arouse, and his respiration are slow and shallow. Which action should the nurse implement? Select all that apply. a. Prepare medication reversal agent b. Check oxygen saturation level c. Apply oxygen via nasal cannula d. Initiate bag- valve mask ventilation. e. Begin cardiopulmonary resuscitation

a. Prepare medication reversal agent b. Check oxygen saturation level c. Apply oxygen via nasal cannula Rationale: Sedation, given during the procedure may need to be reverse if the client does not easily wake up. Oxygen saturation level should be asses, and oxygen applied to support respiratory effort and oxygenation. The client is still breathing so the bag- valve mask ventilation and CPR are not necessary.

A 56-years-old man shares with the nurse that he is having difficulty making decision about terminating life support for his wife. What is the best initial action by the nurse? a. Provide an opportunity for him to clarify his values related to the decision b. Encourage him to share memories about his life with his wife and family c. Advise him to seek several opinions before making decision d. Offer to contact the hospital chaplain or social worker to offer support.

a. Provide an opportunity for him to clarify his values related to the decision

While assessing a client's chest tube (CT), the nurse discovers bubbling in the water seal chamber of the chest tube collection device. The client's vital signs are: blood pressure of 80/40 mmHg, heart rate 120 beats/minutes, respiratory rate 32 breaths/minutes, oxygen saturation 88%. Which interventions should the nurse implement? a. Provide supplemental oxygen b. Auscultate bilateral lung fields c. Administer a nebulizer treatment d. Reinforce occlusive CT dressing e. Give PRN dose of pain medication

a. Provide supplemental oxygen b. Auscultate bilateral lung fields d. Reinforce occlusive CT dressing Rationale: The air bubbles indicate an air leak from the lungs, the chest tube site, or the chest tube collection system. Providing oxygen improves the oxygen saturation until the leak has been resolved. Auscultating the lung fields helps to identify absent or decrease lung sound due to collapsing lung.

The nurse is assessing a 3-month-old infant who had a pylorotomy yesterday. This child should be medicated for pain based on which findings? Select all that apply: a. Restlessness b. Clenched Fist c. Increased pulse rate d. Increased respiratory rate. e. Increased temperature f. Peripheral pallor of the skin

a. Restlessness b. Clenched Fist c. Increased pulse rate d. Increased respiratory rate.

A client whose wrists are sutured from a recent suicide attempt has been transferred from a medical unit. Which nursing diagnosis is of the highest priority? a- Risk for self-directed violence related to impulsive actions b- Risk for violence related to feeling of guilt and failure c- Low self-esteem related to feeling of loss of control d- Ineffective coping related to violent actions towards self.

a. Risk for self-directed violence related to impulsive actions

The healthcare provider prescribes a low-fiber diet for a client with ulcerative colitis. Which food selection would indicate to the nurse the client understands they prescribed diet? a. Roasted turkey canned vegetables b. Baked potatoes with skin raw carrots c. Pancakes whole-grain cereal's d. Roast pork fresh strawberries

a. Roasted turkey canned vegetables Rationale: Foods allowed on a low-fiber diet includes roasted or baked turkey and canned vegetables the foods in the other options are not low in fiber

When administering ceftriaxone sodium (Rocephin) intravenously to a client before... most immediate intervention by the nurse? a. Stridor b. Nausea c. Headache d. Pruritus

a. Stridor Rationale: Stridor, a crowing respiration, indicates the client is experiencing bronchospasm, as a reaction to Rocephin, and antibiotic. The finding requires immediate action by the nurse. B and C are side effects that are not life-threatening. Pruritus may be the result as... and need nursing intervention but is of less immediacy than stridor.

When attempting to establish risk reduction strategies in a community, the nurse notes that regional studies indicate a high number of persons with growth stunting and irreversible mental deficiencies (cretinism) caused by hypothyroidism. The nurse should seek funding to implement which screening measure? A) T4 levels in newborns. B) TSH levels in women over 45. C) T3 levels in school-aged children D) Iodine levels in all persons over 60.

a. T4 levels in newborns

A client with a large pleural effusion undergoes a thoracentesis. Following the procedure, which assessment finding warrants immediate intervention by the nurse? a. The client has asymmetrical chest wall expansion b. The clients complain of pain at the insertion site c. The client chest's x-ray indicates decreased pleural effusion d. The client's arterial blood gases are pH 7.35, PaO2 85, Pa CO2 35, HCO3 26

a. The client has asymmetrical chest wall expansion Rationale: A potential complication of thoracentesis is a pneumothorax. The symptoms of a pneumothorax are uneven, unequal movement of the chest wall. A is an expected finding after the local anesthetic effects "wear off" B is a desired result of thoracentesis and C is within normal limits.

After the nurse witnesses a preoperative client sign the surgical consent form, the nurse signs the form as a witness. What are the legal implications of the nurse's signature on the client's surgical consent form? (Select all that apply) a. The client voluntarily grants permission for the procedure to be done b. The surgeon has explained to the client why the surgery is necessary. c. The client is competent to sign the consent without impairment of judgment d. The client understands the risks and benefits associated with the procedure e. After considering alternatives to surgery, the client elects to have the procedure.

a. The client voluntarily grants permission for the procedure to be done c. The client is competent to sign the consent without impairment of judgment d. The client understands the risks and benefits associated with the procedure Rationale: Inform consent is required for any invasive procedure. The nurse's signature as a witness to the client's signature on surgical consent indicates that the client voluntary gives consent for the scheduled procedure. C is competent to give consent, and D and understand the risk and benefits of the procedure.

A client with eczema is experiencing severe pruritus. Which PRN prescriptions should the nurse administer? (Select all that apply) a. Topical corticosteroid. b. Topical scabicide. c. Topical alcohol rub. d. Transdermal analgesic. e. Oral antihistamine

a. Topical corticosteroid. e. Oral antihistamine Rationale: anti-inflammatory actions of topical corticosteroids and oral antihistamines provide relief from severe pruritus (itching). Other options are not indicated.

Following a gun shot wound to the abdomen, a young adult male had an emergency bowel...Multiple blood products while in the operating room. His current blood pressure is 78/52...He is being mechanically ventilated, and his oxygen saturation is 87%. His laboratory values...Grams / dl (70 mmol / L SI), platelets 20,000 / mm 3 (20 x 10 9 / L (SI units), and white blood cells. Based on these assessments findings, which intervention, should the nurse implements first? a. Transfuse packed red blood cells b. Obtain blood and sputum cultures. c. Infuse 1000 ml normal saline d. Titrate oxygen to keep o2 saturation 90%

a. Transfuse packed red blood cells Rational: The client is exhibiting signs of multiple organ dysfunction syndrome. Transfusion is the first intervention which provide hemoglobin to carry the oxygen to the tissues, is critical.

A male client is discharged from the intensive care unit following a myocardial infarction, and the healthcare provider low-sodium diet. Which lunch selection indicates to the nurse that this client understands the dietary restrictions? a- Turkey salad sandwich. b- Clam chowder c- Macaroni and cheese d- Bacon, lettuce, and tomato sandwich

a. Turkey salad sandwich.

A woman who takes pyridostigmine for myasthenia gravis (MG) arrives at the emergency department complaining of extreme muscle weakness. Her adult daughter tells the nurse that since yesterday her mother has been unable to smile, which assessment finding warrants immediate intervention by the nurse? a- Uncontrollable drooling b- Inability to raise voice c- Tingling of extremities d- Eyelid drooling

a. Uncontrollable drooling

A client who is at 36 weeks gestation is admitted with severe preeclampsia. After a 6 gram loading dose of magnesium sulfate is administered, an intravenous infusion of magnesium sulfate at a rate of 2 grams/hour is initiated. Which assessment finding warrants immediate intervention by the nurse? A. Urine output 20 ml/hour B. Blood pressure 138/88 C. Respiratory rate 18 breaths/min D. Temperature of 99.8

a. Urine output 20 ml/hour Rationale: urinary output of less than 30 ml/hour indicates that the kidneys are being affected by the high level of magnesium, which is excreted through kidneys.

A client in septic shock has a double lumen central venous catheter with one liter of 0.9% Normal Saline Solution infusing at 1 ml/hour through one lumen and TPN infusing at 50 ml/ hr. through one port. The nurse prepared newly prescribed IV antibiotic that should take 45 mints to infuse. What intervention should the nurse implement? a. Use a secondary port of the Normal Saline solution to administer the antibiotic. b. Add the antibiotic to the TPN solution, and continue the normal saline solution. c. Stop the TPN infusion for the time needed to administer the prescribed antibiotic. d. Add the antibiotic to the Normal Saline solution and continue both infusions.

a. Use a secondary port of the Normal Saline solution to administer the antibiotic.

A client is being discharged home after being treated for heart failure (HF). What instruction should the nurse include in this client's discharge teaching plan? a. Weigh every morning b. Eat a high protein diet c. Perform range of motion exercises d. Limit fluid intake to 1,500 ml daily

a. Weigh every morning should be instructed to weight each morning before breakfast with approximately the same clothing. A is not specifically to HF and fluid retention.

A female client with severe renal impairment is receiving enoxaparin (lovenox) 30 mg SUBQ BID. Which laboratory value due to enoxaparin should the nurse report to the healthcare provider? a. creatinine clearance 25 mL/ minute b. calcium 9 mg/dl c. hemoglobin 12 grams/dl d. partial thromboplastin time (PTT) 30 seconds

a. creatinine clearance 25 mL/ minute

The nurse is conducting health assessments. Which assessment finding increases a 56 year-old woman's risk for developing osteoporosis? a. Body mass index of (BMI) of 31 b. 20 pack-year history of cigarette smoking c. Birth control pill usage until age 45 d. Diabetes mellitus in family history

b. 20 pack-year history of cigarette smoking Rationale: Cigarette smoking (2 packs/day x 10 years = 20 packs-year) increases the risk of osteoporosis. BMI of 30 or greater falls in the category of obesity which increase weight bearing that is protective against osteoporosis. C contain estrogens and are also protective against development of osteoporosis. D is not related to the development of osteoporosis.

A female client comes to the clinic complaining of fatigue and inability to sleep because she is the full-time caretaker for 22-year-old son who was paralyzed by a motor vehicle collision. She adds that her husband left her because he says he can't take her behavior any more since all she does is care for their son. What intervention should the nurse implement? a- Schedule a home visit in the afternoon to assess the son and client role as caregiver. b- Acknowledge the client's stress and suggest that she consider respite care. c- Provide feedback to the client about her atonement for guilt about her son's impairment. d- Teach the client to problem-solve for herself and establish her own priorities.

b. Acknowledge the client's stress and suggest that she consider respite care. Rationale: When this amount of disclosure is offered, the client is usually seeking information focuses on the client's expression of worry, concern and stress and addresses the client's need to initiate a request for assistance with respite care.

After checking the fingerstick glucose at 1630, what action should the nurse implement? a. Notify the healthcare provider b. Administer 8 units of insulin aspart SubQ c. Gives an IV bolus of Dextrose 50% 50 ml d. Perform quality control on the glucometer.

b. Administer 8 units of insulin aspart SubQ

After several hours of non-productive coughing, a client presents to the emergency room complaining of chest tightness and shortness of breath. History includes end stage chronic obstructive pulmonary disease (COPD) and diabetes mellitus. While completing the pulmonary assessment, the nurse hears wheezing and poor air movement bilaterally. Which actions should the nurse implement? (Select all that apply.) a. Apply oxygen via nasal cannula b. Administer PRN nebulizer treatment. c. Obtain 12 lead electrocardiogram. d. Monitor continuous oxygen saturation. e. Give PRN dose of regular insulin

b. Administer PRN nebulizer treatment. c. Obtain 12 lead electrocardiogram. d. Monitor continuous oxygen saturation. Rationale: A nebulizer treatment may improve the wheezing. Chest tightness is most likely to coughing, but a 12-lead electrocardiogram is needed to assess for cardiac ischemia. Oxygen saturation monitors for adequate oxygenation.

A female client with chronic urinary retention explains double voiding technique to the nurse by stating she voids partially, hold the remaining urine in her bladder for three minutes, then voids again to empty her bladder fully. How should the nurse respond? a. Affirm that the client is effectively performing the double voiding. b. Advise the client to empty her bladder fully when she first voids c. Suggest that the client drink water between the two voiding. d. Explain that Kegel exercises help promote full bladder empty.

b. Advise the client to empty her bladder fully when she first voids

The nurse provides feeding tube instructions to the wife of a client with end stage cancer. The client's wife performs a return demonstration correctly, but begins crying and tells the nurse, "I just don't think I can do this every day." The nurse should direct further teaching strategies toward which learning domain? a- Cognitive b- Affective c- Comprehension d- Psychomotor

b. Affective Rationale: The affective domain involves our feelings, emotions, and attitudes, and includes the manner in which we deal with things emotionally (feelings, values, appreciation, enthusiasm, motivations, and attitudes).

A multigravida, full-term, laboring client complains of "back labor". Vaginal examination reveals that the client's 3 cm with 50% effacement and the fetal head is at -1 station. What should the nurse implement? a. Turn the client to a lateral position b. Apply counter-pressure to the sacral area c. Notify the scrub nurse to prepare the OR d. Ambulate the client between contractions

b. Apply counter-pressure to the sacral area Rationale: B provides pain relief during labor.

A male client notifies the nurse that he feels short of breath and has chest pressure radiating down his left arm. A STAT 12-lead electrocardiogram (ECG) is obtained and shows ST segment elevation in leads II, II, aVF and V4R. The nurse collects blood samples and gives a normal saline bolus. What action is most important for the nurse to implement? a- Obtain the results for STAT serum cardiac biomarkers b- Asses for contraindications for thrombolytic therapy c- Measure ST-segment height and waveform changes. d- Transfer for percutaneous coronary intervention (PCI)

b. Asses for contraindications for thrombolytic therapy. Rationale: ST segment elevation myocardial infarction (STEMI) usually occurs with complete occlusion of an epicardial coronary artery which requires early reperfusion therapy. Screening the client for fibrinolytic therapy (B) is most important to determine PCI option for rapid reperfusion. If the client is not a candidate for fibrinolytic therapy, then transfer to a PCI unit or facility is indicated. Reperfusion therapy should be delayed in STEMI (A). (C) is of significant concern in ECG interpretation with ST-segment depression, not STEMI

An adult who is 5 feet 5 inches (165.1 cm) tall and weighs 90 lb. (40.8 Kg) is admitted with a diagnosis of chronic anorexia. The client receives a regular diet for 2 days, and the client's medical records indicates that 100% of the diet provided has been consumed. However the client's weight on the third day morning after admission is 89 lb. (40.4 Kg). What action should the nurse implement? a. Examine the client's room for hidden food. b. Assign staff to monitor what the client eats. c. Ask the client if the food provided is being eaten or discarded. d. Provide the client with a high calorie diet.

b. Assign staff to monitor what the client eats. Rationale: clients with an eating disorder have an unhealthy obsession with food. The client's continued weight loss, despites indication that the client has consumed 100% of the diet, should raise questions about the client's intake of the food provided, so the client should be observed during meals to prevent hiding or throwing away food. Other options may be accurate but ineffective and unnecessary.

The nurse should teach the client to observe which precaution while taking dronedarone? a. Stay out of direct sunlight b. Avoid grapefruits and its juice c. Reduce the use of herbal supplements d. Minimize sodium intake.

b. Avoid grapefruits and its juice Rationale: Grapefruit increase the effect of dronedarone thereby increasing the possibility of serious side effects. A does not cause a serious effect. C may potentiate lethal arrhythmias and should be avoided. D does not directly affect those taking dronedarone.

A client admitted to the emergency center had inspiratory and expiratory wheezing, nasal flaring, and thick, tenacious sputum secretions observed during the physical examination. Based on these assessment findings, what classification of pharmacologic agents should the nurse anticipate administering? a. Beta blockers b. Bronchodilators c. Corticosteroids d. Beta-adrenergic

b. Bronchodilators

A postpartal client complains that she has the urge to urinate every hour but is only able to void a small amount. What interventions provides the nurse with the most useful information? a. Initiate a perineal pad count b. Catheterize for residual urine after next voiding c. Assess for a perineal hematoma d. Determine the client's usual voiding pattern

b. Catheterize for residual urine after next voiding

The nurse is preparing to administer an infusion of amino acid-dextrose total parenteral nutrition (TPN) through a central venous catheter (CVC) line. Which action should the nurse implement first? a. Attached de IV tubing to the central line. b. Check the TPN solution for cloudiness c. Set the infusion PUMP at the prescribed rate. d. Prime the IV tubing with the TPN solution.

b. Check the TPN solution for cloudiness

An adult male who lives alone is brought to the Emergency Department by his daughter who is unresponsive. Initial assessment indicated that the client has minimal respiratory effort, and his pupils are fixed and dilated. At the daughter's request, the client is intubated and... Which nursing intervention has the highest priority? a. Offer to notify the client's minister of his condition. b. Determine if the client has an executed living will c. Provide the family with information about palliative care d. Explore the possibility of organ donation with the family.

b. Determine if the client has an executed living will Rationale: Once the client is intubated and ventilated, emergency intervention should continue until patient t be stable check if the client has an executed living will.

An adolescent's mother calls the clinic because the teen is having recurrent vomiting and...Combative in the last 2 days. The mother states that the teen takes vitamins, calcium, mag...With aspirin. Which nursing intervention has highest priority? a. Advise the mother to withhold all medications by mouth. b. Instruct the mother to take the teen to the emergency room c. Recommend that the teen withhold food and fluids for 2 hours d. Suggest that the adolescent breath slowly and deeply.

b. Instruct the mother to take the teen to the emergency room

A woman just learned that she was infected with Heliobacter pylori. Based on this finding, which health promotion practice should the nurse suggest? a. Schedule a colonoscopy within the next month. b. Encourage screening for a peptic ulcer. c. Screen all family member for hepatitis A d. Eat small, frequent meals thought the day.

b. Encourage screening for a peptic ulcer. Rationale: Helicobacter pylori is a gram- negative organism than can colonize in the stomach and is associated with peptic ulcers formation.

A 2-year-old girl is brought to the clinic for a routine assessment and all findings are within the normal limits. However, the mom expresses concern over her daughter's protruding abdomen and tells the nurse that she is worry that her child is becoming overweight. How should the nurse respond to the mother's comment? a- Tell the mother to keep a twenty- four-hour food diary for the child. b- Explain that a protruding abdomen is typical for toddlers. c- Discuss way to increase the child's daily activity level d- Ask the mother is she has weight problems when she was a child.

b. Explain that a protruding abdomen is typical for toddler

The pathophysiological mechanism are responsible for ascites related to liver failure? (Select all that apply) a. Bleeding that results from a decreased production of the body's clotting factors b. Fluid shifts from intravascular to interstitial area due to decreased serum protein c. Increased hydrostatic pressure in portal circulation increases fluid shifts into abdomen d. Increased circulating aldosterone levels that increase sodium and water retention e. Decreased absorption of fatty acids in the duodenum leading to abdominal distention.

b. Fluid shifts from intravascular to interstitial area due to decreased serum protein c. Increased hydrostatic pressure in portal circulation increases fluid shifts into abdomen d. Increased circulating aldosterone levels that increase sodium and water retention Rationale: When liver fail production of albumin is reduced. Since albumin is the primary serum protein creating intravascular osmotic pressure, decreased serum protein allows a fluids shift into the interstitial space. Pressure increases in the portal circulation © when venous return from the upper GI tract cannot flow freely into sclerosed liver, which cause a pressure gradient to further Increase fluid shifts into the abdomen. A failing liver ineffectively inactivates steroidal hormones, such as aldosterone resulting in sodium and water retention.

During a well-baby, 6-month visit, a mother tells the nurse that her infant has had fewer ear infections than her 10-year-old daughter. The nurse should explain that which vaccine is likely to have made the difference in the siblings' incidence of otitis media? a. Varicella Virus Vaccine Live b. Hemophilic Influenza Type B (HiB) vaccine c. Pneumococcal vaccine d. Palivizumab vaccine for RSV

b. Hemophilic Influenza Type B (HiB) vaccine

A male client who was hit by a car while dodging through traffic is admitted to the emergency department with intracranial pressure (ICP). A computerized tomography (CT) scan reveals an intracranial bleed. After evacuation of hematoma, postoperative prescription include: intubation with controlled mechanical ventilation to PaCO2...what is the pathophysiological basis for this ventilator settings? a- Hypoxemia reduces ICP. b- Hypocapnea reduces ICP. c- Hyperventilation reduces need for temperature control. d- Controlled ventilation reduces need for oxygen to brain.

b. Hypocapnea reduces ICP

Assessment by the home health nurse of an older client who lives alone indicates that client has chronic constipation. Daily medications include furosemide for hypertension and heart failure, and laxatives. To manage the client's constipation, which suggestions should the nurse provide? (Select all that apply) a. Decrease laxative use to every other day and use oil retention enemas as needed. b. Include oatmeal with stewed pruned for breakfast as often as possible. c. Increase fluid intake by keeping water glass next to recliner. d. Recommend seeking help with regular shopping and meal preparation. e. Report constipation to healthcare provider related to cardiac medication side effects.

b. Include oatmeal with stewed pruned for breakfast as often as possible. c. Increase fluid intake by keeping water glass next to recliner. d. Recommend seeking help with regular shopping and meal preparation. Rational: older adult are at higher risk for chronic constipation due to decreased gastrointestinal muscle tone leading to reduce motility. Oatmeal with prunes increases dietary fiber and bowel stimulation, thereby decreasing need for laxatives. Increased fluid intake also decreases constipations. Assistance with food preparation might help the client eat more fresh fruits and vegetables and result on less reliance on microwaved and fast foods, which are usually high in sodium and fat with little fiber. Laxatives can be reduced gradually by improving the diet, without resorting to using enemas.

During a 26-week gestation prenatal exam, a client reports occasional dizziness...What intervention is best for the nurse to recommend to this client? a. Elevate the head with two pillows while sleeping b. Lie on the left or right side when sleeping or resting. c. Increase intake of foods that are high in iron d. Decrease the amount of carbohydrates in the diet.

b. Lie on the left or right side when sleeping or resting

The daughter of an older female client tells the clinic nurse that she is no longer able to care for her mother since her mother has lost the ability to perform activities of daily living (ADLs) due to aging. Which options should the nurse discuss with the daughter? a. Home hospice agency b. Long-term care facility c. Rehabilitation facility d. Independent senior apartment e. Home health agency

b. Long-term care facility e. Home health agency Long term care facilities and home health agencies performs ADLs. Hospice provides empathetic, attentive care for dying. C provide physical therapy to strengthen a part of the body.

A client who had an emergency appendectomy is being mechanically ventilated, and soft wrist restraints are in place to prevent self extubation. Which outcome is most important for the nurse to include in the client's plan of care? a. Understand pain management scale b. Maintain effective breathing patterns c. Absence of ventilator associated pneumonia d. No injuries refer to soft restrains occur

b. Maintain effective breathing patterns Rationale: Basic airway management (B) is the priority. Pain management (A), risk of infection (C), and prevention of injury (D) do not have the same priority as (C)

Which intervention should the nurse include in the plan of care for a client with leukocytosis? a. Avoid intramuscular injections b. Monitor temperature regularly c. Assess skin for petechiae or bruising d. Implement protective isolation measures

b. Monitor temperature regularly

After the risk and benefits of having a cardiac catheterization are reviewed by the healthcare provider, an older adult with unstable angina is scheduled for the procedure. When the nurse presents the consent form for signature, the client asks how the wires will keep a heart beating during the procedure. What action should the nurse take? a. Explain the procedure again in detail and clarify any misconceptions. b. Notify the healthcare provider of the client's lack of understanding. c. Call the client's next of kin and have them provide verbal consent. d. Postpone the procedure until the client understands the risk and benefits.

b. Notify the healthcare provider of the client's lack of understanding. Rational: the nurse is only witnessing the signature and is not responsible for the client's understanding of the procedure. The healthcare provider needs to clarify any questions and misconceptions. Explaining the procedure again is the healthcare provider's legal responsibility. The other options are not indicated.

A male client with a long history of alcoholism is admitted because of mild confusion and fine motor tremors. He reports that he quit drinking alcohol and stopped smoking cigarettes one month ago after his brother died of lung cancer. Which intervention is most important for the nurses to include in the client's plan of care? a. Determine client's level current blood alcohol level. b. Observe for changes in level of consciousness. c. Involve the client's family in healthcare decisions. d. Provide grief counseling for client and his family.

b. Observe for changes in level of consciousness. Rationale: Based on the client's history of drinking, he may be exhibiting sign of hepatic involvement and encephalopathy. Changes in the client's level of consciousness should be monitored to determine if he able to maintain consciousness, so neurological assessment has the highest priority.

The father of 4-year-old has been battling metastatic lung cancer for the past 2 years. After discussing the remaining options with his healthcare provider, the client requests that all treatment stop and that no heroic measures be taken to save his life. When the client is transferred to the palliative care unit, which action is most important for the nurse working on the palliative care unit to take in facilitating continuity of care? a. Reassure the client that his child will be allowed to visit b. Obtain a detailed report from the nurse transferring the client. c. Mark the chart with client's request for no heroic measure d. Provide the client whitening information about end-of-life care

b. Obtain a detailed report from the nurse transferring the client. Rationale: To maintain continuity of care, it is important for the nurse working on the palliative care unit to obtain a detailed "situation, background, assessment, recommendation (SBAR) report, which provide clinical and no clinical information, as well as further information about the client may need. A, C and D are important intervention but not have priority at this time.

A client with pneumonia has an IV of lactated ringer's solution infusing at 30ml/hr current labor....sodium level of 155 mEq/L, a serum potassium level of 4mEq/L.... what nursing intervention is most important? a. Provide a high-potassium snack, such as bananas. b. Obtain a prescription to increase the IV rate c. Administer the next scheduled dose of antibiotic d. Review the report of the most recent chest x-ray.

b. Obtain a prescription to increase the IV rate

A client with a postoperative wound that eviscerated yesterday has an elevated temperature...most important for the nurse to implement? a. Initiate contact isolation b. Obtain a wound swab for culture and sensitivity c. Assess temperature q4 hours d. Use alcohol-based solutions for hand hygiene.

b. Obtain a wound swab for culture and sensitivity

The nurse is assessing a postpartum client who is 36 hours post-delivery. Which finding should the nurse report to the healthcare provider? a. White blood count of 19,000 mm3 b. Oral temperature of 100.6 F c. Fundus deviated to the right side d. Breasts are firm when palpated

b. Oral temperature of 100.6 F Rationale: A temperature greater than 100.4 F (38 C) (B), which is indicative of endometriosis (infection of the lining of the uterus), should be reported to the health care provider. (A and D) are findings that are within normal limits in the postpartum period. Fundal deviation to one side (C) is an expected finding related to a full bladder, so the nurse should encourage the client to void.

In assessing a pressure ulcer on a client's hip, which action should the nurse include? a. Determine the degree of elasticity surrounding the lesion b. Photograph the lesion with a ruler placed next to the lesion c. Stage the depth of the ulcer using the Braden numeric scale d. Use a gloved finger to palpate for tunneling around the lesion

b. Photograph the lesion with a ruler placed next to the lesion Rationale: An ulcer extends into the dermis or subcutaneous tissue and is likely to increase in size and depth, so assessment should include photograph with measuring device to document the size of the lesion.

A health care provider continuously dismisses the nursing care suggestions made by staff nurses. As a result...dealing with the healthcare provider. What action should the nurse manager implement? a- Confront the health care provider about the perceived lack of respect for the staff nurses. b- Plan an interdisciplinary staff meeting to develop strategies to enhance client care c- Request an investigation about the perceived incivility of the healthcare provider interaction. d- Remind the staff that avoidance behavior is not a professional way to handle the problem.

b. Plan an interdisciplinary staff meeting to develop strategies to enhance client care

A client who underwent an uncomplicated gastric bypass surgery is having difficult with diet management. What dietary instruction is most important for the nurse to explain to the client? a. Chew food slowly and thoroughly before attempting to swallow b. Plan volume-controlled evenly-space meal thorough the day c. Sip fluid slowly with each meal and between meals d. Eliminate or reduce intake fatty and gas forming food

b. Plan volume-controlled evenly-space meal thorough the day Rationale: It is most important for the client to learn how to eat without damaging the surgical site and to keep the digestive system from dumping the food instead of digesting it. Eating volume-control and evenly-space meals thorough the day allows the client to fill full, avoid binging, and eliminate the possibility of eating too much one time. Chewing slowly and thoroughly helps prevent over eating by allowing a filling of fullness to occur. Taking sips, rather than large amounts of fluids keeps the stomach from overfilling and allow for adequate calories to be consumed. Gas forming foods and fatty foods should be avoiding decreasing risk of dumping syndrome and flatulence.

The nurse is assessing a client with a small bowel obstruction who was hospitalized 24 hours ago. Which assessment finding should the nurse report immediately to the healthcare provider? a. Hypoactive bowel sounds in the lower quadrant. b. Rebound tenderness in the upper quadrants. c. Tympani with percussion of the abdomen. d. Light colors gastric aspirate via the nasogastric tube.

b. Rebound tenderness in the upper quadrants Rationale: Rebound tenderness in the upper quadrant may be indicative of peritonitis. A is a clinical finding associated with bowel obstruction and does not need to be reported D may be something characteristic of the client's condition.

A client who is newly diagnosed with type 2 diabetes mellitus (DM) receives a prescription for metformin (Glucophage) 500 mg PO twice daily. What information should the nurse include in this client's teaching plan? (Select all that apply.) a. Take an additional dose for signs of hyperglycemia b. Recognize signs and symptoms of hypoglycemia. c. Report persistent polyuria to the healthcare provider. d. Use sliding scale insulin for finger stick glucose elevation. e. Take Glucophage with the morning and evening meal.

b. Recognize signs and symptoms of hypoglycemia. c. Report persistent polyuria to the healthcare provider. e. Take Glucophage with the morning and evening meal. Rationale: Glucophage, an antidiabetic agent, acts by inhibiting hepatic glucose production and increases peripheral tissue sensitivity to insulin. The client and family should be taught to recognize signs and symptoms of hypoglycemia. If the dose of Glucophage is inadequate, signs of hypoglycemia, such as polydipsia and polyuria, should be reported to the healthcare provider. Glucophage should be taken with meals to reduce GI upset and increase absorption (E).

A client with a history of dementia has become increasingly confused at night and is picking at an abdominal surgical dressing and the tape securing the intravenous (IV) line. The abdominal dressing is no longer occlusive, and the IV insertion site is pink. What intervention should the nurse implement? a. Replace the IV site with a smaller gauge. b. Redress the abdominal incision c. Leave the lights on in the room at night. d. Apply soft bilateral wrist restraints.

b. Redress the abdominal incision Rationale: The abdominal incision should be redressed using aseptic-techniques. The IV site should be assessed to ensure that it has not been dislodged and a dressing reapplied, if need it. Leaving the light on at night may interfere with the client's sleep and increase confusion. Restraints are not indicated and should only be used as a last resort to keep client from self-harm.

An older adult male who had an abdominal cholecystectomy has become increasingly confused and disoriented over the past 24 hours. He is found wandering into another client's room and is return to his room by the unlicensed assistive personnel (UAP). What actions should the nurse take? (Select all that apply). a. Apply soft upper limb restrains and raise all four bed rails b. Report mental status change to the healthcare provider c. Assess the client's breath sounds and oxygen saturation d. Assign the UAP to re-assess the client's risk for falls e. Review the client's most recent serum electrolyte values

b. Report mental status change to the healthcare provider c. Assess the client's breath sounds and oxygen saturation e. Review the client's most recent serum electrolyte values Rationale: The healthcare provider should be informed of changes in the client's condition (B) because this behavior may indicate a postoperative complication. Diminished oxygenation (C) and electrolyte imbalance (E) may cause increased confusion in the older adult. Raising all four bed rails (A) may lead to further injury if the client climbs over the rails and falls and restrains should not be applied until other measures such as re-orientation are implemented. The nurse should assess the client's increased risk for falls, rather than assigning this to the UAP (D).

To reduce staff nurse role ambiguity, which strategy should the nurse manager implemented? a. Confirm that all the staff nurses are being assigned to equal number of clients. b. Review the staff nurse job description to ensure that it is clear, accurate, and recurrent. c. Assign each staff nurse a turn unit charge nurse on a regular, rotating basis. d. Analyze the amount of overtime needed by the nursing staff to complete assignments.

b. Review the staff nurse job description to ensure that it is clear, accurate, and recurrent. Rationale: Role ambiguity occurs when there is inadequate explanation of job descriptions and assigned tasks, as well as the rapid technological changes that produce uncertainty and frustration. A and D may be implemented if the nurse manager is concerned about role overload, which is the inability to accomplish the tasks related to one's role. C is not related to ambiguity.

The nurse is making a home visit to a male client who is in the moderate stage of Alzheimer's diseases. The client's wife is exhausted and tells the nurse that the family plans to take turns caring for the client in their home, each keeping him for two weeks at a time. How should the nurse respond? a. Advise the client's spouse to consider inpatient hospice care as an alternative b. Suggest that each rotation last one week, rather than two, to prevent caregiver fatigue c. Use active listening to allow the client and spouse to express their feelings about the plan d. Suggest enrolling the client in adult daycare instead of rotating among family.

b. Suggest enrolling the client in adult daycare instead of rotating among family Rationale: Suggesting a viable alternative, such as adult daycare provides an option to allow the spouse respite the least disruption to routines and environment.

A male client is having abdominal pain after a left femoral angioplasty and stent, and is asking for additional pain medication for right lower quadrant pain (9/10), two hours ago, he received hydrocodone / acetaminophen 7.5/7.50 mg his vital signs are elevated from reading of a previous hour: temperature 97.8 F, heart rate 102 beats / minute, respiration 20 breaths/minutes. His abdomen is swollen, the groin access site is tender, peripheral pulses are present, but left is greater than right. Preoperatively, clopidrogel was prescribed for a history of previous peripheral stents. Another nurse is holding manual pressure on the femoral arterial access site which may be leaking into the abdomen. What data is needed to make this report complete? a. Client's lungs are clear bilaterally and oxygen saturation is 97% b. Surgeon needs to see client immediately to evaluate the situation c. Left peripheral pulses were present only by Doppler pre-procedure d. Client' history includes multiple back surgeries and chronic pain.

b. Surgeon needs to see client immediately to evaluate the situation

The nurse is caring for a group of clients with the help of a licensed practical nurse (LPN) and an experienced unlicensed assistive personnel (UAP). Which procedures can the nurse delegate to the UAP? (Select all that apply) a. Change a saturated surgical dressing for a client who had an abdominal hysterectomy. b. Take postoperative vital signs for a client who has an epidural following knee arthroplasty c. Start a blood transfusion for client who had a below-the knee amputation. d. Collect a sputum specimen for a client with a fever of unknown origin e. Ambulate a client who had a femoral-popliteal bypass graft yesterday

b. Take postoperative vital signs for a client who has an epidural following knee arthroplasty d. Collect a sputum specimen for a client with a fever of unknown origin e. Ambulate a client who had a femoral-popliteal bypass graft yesterday Rationale: Measuring vital signs, collecting specimens, and ambulating a mobile client are within the scope of practice for a UAP

A male client returns to the mental health clinic for assistance with his anxiety reaction that is manifested by a rapid heartbeat, sweating, shaking, and nausea while driving over the bay bridge. What action I the treatment plan should the nurse implement? a. Tell the client to drive over the bridge until fear is manageable b. Teach client to listen to music or audio books while driving c. Encourage client to have spouse drive in stressful places. d. Recommend that the client avoid driving over the bridge.

b. Teach client to listen to music or audio books while driving

A client with polycystic kidney disease (PKD) receiving antibiotics for an infected cyst is experiencing severe pain. What action should the nurse implement? a. Hold the next dose of antibiotic until contacting the healthcare provider b. Teach the client how to use a dry heating pad over the painful area c. Encourage the client to practice pelvic floor exercises every hour d. Assist the client to splint the site by applying an abdominal binder

b. Teach the client how to use a dry heating pad over the painful area

Following breakfast, the nurse is preparing to administer 0900 medications to clients on a medical floor. Which medication should be held until a later time? a. The loop-diuretic furosemide (Lasix) for a client with a serum potassium level of 4.2 mEq/L b. The mucosal barrier, sucralfate (Carafate), for a client diagnosed with peptic ulcer disease. c. The antiplatelet agent aspirin, for a client who is scheduled to be discharged within the hour d. The antifungal nystatin (mycostatin) suspension, for a client who has just brushed his teeth.

b. The mucosal barrier, sucralfate (Carafate), for a client diagnosed with peptic ulcer disease Rationale: Carafate coats the mucosal lining prior to eating a meal, so this medication should be held until prior to the next meal.

A female client with rheumatoid arthritis (RA) comes to the clinic complaining of joint pain and swelling. The client has been taking prednisone (Deltasone) and ibuprofen (Motrin Extra Strength) every day. To assist the client with self-management of her pain, which information should the nurse obtain? a. Presence of bruising, weakness, or fatigue b. Therapeutic exercise included in daily routine. c. Average amount of protein eaten daily d. Existence of gastrointestinal discomfort

b. Therapeutic exercise included in daily routine

The nurse is explaining the need to reduce salt intake to a client with primary hypertension. What explanation should the nurse provide? a. High salt can damage the lining of the blood vessels b. Too much salt can cause the kidneys to retain fluid c. Excessive salt can cause blood vessels to constrict d. Salt can cause information inside the blood vessels

b. Too much salt can cause the kidneys to retain fluid Rationale: Excessive salt intake can contribute to primary hypertension by causing renal salt retention which influence water retention that expands blood volume and pressure (ACD) are not believed to contribute to primary hypertension.

An adult male is brought to the emergency department by ambulance following a motorcycle accident. He was not wearing a helmet and presents with periorbital bruising and bloody drainage from both ears. Which assessment finding warrants immediate intervention by the nurse? a. Rebound abdominal tenderness b. nausea and projectile vomiting c. rib pain with deep inspiration d. diminished bilateral breath sounds

b. nausea and projectile vomiting Rationale: Projective vomiting is indicative of increasing intracranial pressure, which can lead to ischemic brain damage or death, so this finding warrants immediate intervention. Rebound abdominal tenderness may indicate internal bleeding. Diminished breath sound may be related to pain. Rib pain with inspiration may indicate rib fracture.

New RN to Unit

best client to assign is client 4 days post MI

Disseminated intravascular coagulation (DIC)

best test to diagnose, D-Dimer

The nurse is assigned to care for clients on a medical unit. Based on the notes taken during the shift report, which client situation warrants the nurse's immediate attention? a. A young adult with Crohn's disease who reports having diarrheal stools b. An older adult with type 2 diabetes whose breakfast tray arrives 20 minutes late. c. A 10-year-old who is receiving chemotherapy and the infusion pump is beeping. d. A teenager who reports continued pain 30 minutes after receiving an oral analgesic.

c. A 10-year-old who is receiving chemotherapy and the infusion pump is beeping Rationale: The nurse should immediately assess the child whose infusion pump is alarming during chemotherapy administration because infiltration of a caustic agent can cause tissue damage and children are at greater risk for fluid volume imbalance. Diarrhea is a common occurrence for Crohn's disease. Late consumption of food for a diabetic is of concern, but 20 minutes late is usually not life-threatening. Treatment of pain is most important but has been only 30 mints since the client was medicated and this issue can be assessed in 10 mints or delegated to another nurse.

A family member of a frail elderly adult asks the nurse about eligibility requirements for hospice care. What information should the nurse provide? (Select all that apply.) a. All family must agree about the need for hospice care. b. Hospice services are covered under Medicare Part B. c. A client must be willing to accept palliative care, not curative care. d. The healthcare provider must project that the client has 6 months or less to live. e. All medications except pain treatment will be stopped during hospice care.

c. A client must be willing to accept palliative care, not curative care. d. The healthcare provider must project that the client has 6 months or less to live. Rationale: The eligibility criteria for Medicare coverage requires that the client is willing to accept palliative care, not curative care (C). The healthcare provider should provide an expected prognosis of 6 months or less to live (D) which can be extended by the healthcare provider. It is not necessary for all family members to agree with the need for hospice.

Based on principles of asepsis, the nurse should consider which circumstance to be sterile? a. One inch- border around the edge of the sterile field set up in the operating room b. A wrapped unopened, sterile 4x4 gauze placed on a damp table top. c. An open sterile Foley catheter kit set up on a table at the nurse waist level d. Sterile syringe is placed on sterile area as the nurse riches over the sterile field.

c. An open sterile Foley catheter kit set up on a table at the nurse waist level Rationale: A sterile package at or above the waist level is considered sterile. The edge of sterile field is contaminated which include a 1-inch border (A). A sterile object become contaminated by capillary action when sterile objects become in contact with a wet contaminated surface.

When five family members arrive at the hospital, they all begin asking the nurse questions regarding the prognosis of their critically ill mother. What intervention should the nurse implement first? a. Include the family in client's care b. Request the chaplain's presence c. Ask the family to identify a specific spokesperson d. Page the healthcare provider to speak with family.

c. Ask the family to identify a specific spokesperson

On a busy day, one hour after the shift report is completed, the charge nurse learns that a female staff nurse who lives one hour away from the hospital forgot her prescription eye glasses at home. What action should the charge nurse take? a. Encourage the nurse purchase the reading glasses in the hospital gift shop b. Request another nurse to assist the staff nurse with her documentation c. Ask the nurse to return home and get her prescription eyeglasses for work. d. Tell the staff nurse to take a day off and change her weekly work schedule.

c. Ask the nurse to return home and get her prescription eyeglasses for work

A client with gestational diabetes, at 39 weeks of gestation, is in the second stage of labor. After delivering of the fetal head, the nurse recognizes that shoulder dystocia is occurring. What intervention should the nurse implement first? a- Prepare the client for an emergency cesarean birth b- Encourage the client to move to a hands-and-knees position. c- Assist the client to sharply flex her thighs up again the abdomen. d- Lower the head of the bed an apply suprapubic pressure.

c. Assist the client to sharply flex her thighs up again the abdomen.

A client in the intensive care unit is being mechanically ventilated, has an indwelling urinary catheter in place, and is exhibiting signs of restlessness. Which action should the nurse take first? a. Review the heart rhythm on cardiac monitors b. Check urinary catheter for obstruction c. Auscultated bilateral breath sounds d. Give PRN dose of lorazepam (Ativan)

c. Auscultated bilateral breath sounds Rationale: Restlessness often results from decreased oxygenation, so breath sounds should be assessed first. Giving an anxiolytic such as lorazepam, might be indicated but first the client should be assessed for the cause of the restlessness. An obstruction in the urinary drainage system can cause a distended bladder that may result in restlessness, but patent airway is the priority intervention. The client should be assessed before evaluating the cardiac rhythm on the monitor.

A client with a recent colostomy expresses concern about the ability to control flatus. Which intervention is most important for the nurse to include in the client's plan of care? a. Adhere to a bland diet whenever planning to eat out b. Decrease fluid intake at meal times c. Avoid foods that caused gas before the colostomy d. Eliminate foods high in cellulose

c. Avoid foods that caused gas before the colostomy

The nurse is triaging victims of a tornado at an emergency shelter. An adult woman who has been wandering and crying comes to the nurse. What action should the nurse take? a- Check the client's temperature, blood sugar, and urine output. b- Transport the client for laboratory client for laboratory test and electrocardiogram (EKG) c- Delegate care of the crying client to an unlicensed assistant d- Send the client to the shelter's nutrient center to obtain water and food.

c. Delegate care of the crying client to an unlicensed assistant

The nurse is presenting information about fetal development to a group of parents with...when discussing cephalocaudal fetal development, which information should the nurse gives the parents? a- set order in fetal development is expected b- Growth normally occurs within one organ at a time c- Development progress from head to rump d- Organ formation is directed by brain development

c. Development progress from head to rump

The nurse reviews the signs of hypoglycemia with the parents of a child with Type I diabetes mellitus. The parents correctly understand signs of hypoglycemia if they include which symptoms? a- Fruity breath odor b- Polyphagia c- Diaphoresis d- Polydipsia

c. Diaphoresis

During the initial newborn assessment, the nurse finds that a newborn's heart rate is irregular. Which intervention should the nurse implement? a- Notify the pediatrician immediately. b- Teach the parents about congenital heart defects. c- Document the finding in the infant's record. d- Apply oxygen per nasal cannula at 3 L/min.

c. Document the finding in the infant's record

In preparing a diabetes education program, which goal should the nurse identify as the primary emphasis for a class on diabetes self-management? a. Prepare the client to independently treat their disease process b. Reduce healthcare costs related to diabetic complications c. Enable clients to become active participating in controlling the disease process d. Increase client's knowledge of the diabetic disease process and treatment options.

c. Enable clients to become active participating in controlling the disease process Rationale: The primary goal of diabetic self- management education is to enable the client to become an active participant in the care and control of disease process, matching levels of self- management to the abilities of the individual client. The goal is to place the client in a cooperative or collaborative role with healthcare professional rather than (A)

A community health nurse is concerned about the spread of communicable diseases among migrant farm workers in a rural community. What action should the nurse take to promote the success of a healthcare program designed to address this problem? a. Conduct face to face prevention education group session is Spanish b. Offer low literacy material that explain respiratory hygiene and handwashing techniques c. Establish trust with community leaders and respect cultural and family values. d. Provide public services announcements advising those who aril o seek prompt medical attention.

c. Establish trust with community leaders and respect cultural and family values.

The nurse manager is conducting an in-services education program on the fire evacuation of the newborn recovery. What intervention should the nurse manager disseminate to the staff? a. Place infants on a blanket for evacuation via stairwell b. Secure three infants on a stretcher for transport c. Evacuate each infant with mother via wheelchair d. Use the bassinet in evacuate two infants at a time.

c. Evacuate each infant with mother via wheelchair Rationale: Rooming-in and newborn babies are counted with their mothers. To exposure safety and accountability during the evacuations newborns should be evacuated with their mother in a wheelchair while maneuver with fire extinguisher are performed (PASS)...

A client with C-6 spinal cord injury rehabilitation. In the middle of the night the client reports a severe, pounding headache, and has observable piloerection or "goosebumps". The nurse should asses for which trigger? a. Loud hallway noise. b. Fever c. Full bladder d. Frequent cough.

c. Full bladder Rational: a pounding headache is a sign of autonomic hyperreflexia, an acute emergency that occurs because of an exaggerated sympathetic response in a client with a high level spinal cord injury. Any stimulus below the level of injury can trigger autonomic hyperreflexia, but the most common cause is an overly distended bladder. The other options are unlikely to produce the manifestation of autonomic hyperreflexia.

When finding a client sitting on the floor, the nurse calls for help from the unlicensed assistive personnel (UAP). Which task should the nurse ask the UAP to do? a- Check for any abrasions or bruises. b- Help the client to stand. c- Get a blood pressure cuff. d- Report the fall to the nurse-manager.

c. Get a blood pressure cuff.

The nurse is teaching a male adolescent recently diagnosed with type 1 diabetes mellitus (DM) about self-injecting insulin. Which approach is best for the nurse to use to evaluate the effectiveness of the teaching? a. Ask the adolescent to describe his level of comfort with injecting himself with insulin. b. Observe him as he demonstrates self-injection technique in another diabetic adolescent c. Have the adolescent list the procedural steps for safe insulin administration. d. Review his glycosylated hemoglobin level 3 months after the teaching session.

c. Have the adolescent list the procedural steps for safe insulin administration.

A female client who was mechanically ventilated for 7 days is extubated. Two hours later...productive cough, and her respirations are rapids and shallow. Which intervention is most important? a- Review record of recent analgesia b- Provide frequent pulmonary toilet c- Prepare the client for intubation d- Obtain STAT arterial blood gases

c. Prepare the client for intubation

A client with severe full-thickness burns is scheduled for an allografting procedure. Which information should the nurse provide the client? a. The donor site will be painless a few days after the surgery b. Allografts are made from human and nonhuman material sources. c. Human sources graft require monitoring for signs of graft injection d. Something about scarring occurring under the graft?

c. Human source grafts require monitoring for signs of graft rejection Rationale: Allograft is a graft created from the client's own skin, which is called harvest site. All types of grafts, from human and nonhuman sources should be monitor for signs of rejection. Graft site are painful. (A). Allografts are obtained from the client, which is a human source (B). scaring does occur under the graft (D)

The nurse is evaluating the health teaching of a female client with condyloma acuminate. Which statement by the client indicates that teaching has been effective? a. These warts are caused by a fungus b. Early treatment is very effective c. I need to have regular pap smears d. I will clean my hot tub better

c. I need to have regular pap smears

When assessing a multigravida the first postpartum day, the nurse finds a moderate amount of lochia rubra, with the uterus firm, and three fingerbreadths above the umbilicus. What action should the nurse implement first? a. Massage the uterus to decrease atony b. Check for a distended bladder c. Increase intravenous infusion d. Review the hemoglobin to determined hemorrhage

c. Increase intravenous infusion Rationale: a fundus that is dextroverted (up to the right) and elevated above the umbilicus is indicative of bladder distension/urine retention.

A 16-year-old male is admitted to the pediatric intensive care unit after being involved in a house fire. He has full thickness burns to his lower torso and extremities. Before a dressing change to his legs, which intervention is most important for the nurse to implement? a- Encourage the parents to stay at the bedside b- Use distraction techniques to reduce pain. c- Maintain strict aseptic technique d- Place a drape over the pubic area.

c. Maintain strict aseptic technique.

What is the nurse's priority goal when providing care for a 2-year-old child experiencing seizure... a- Stop the seizure activity b- Decrease the temperature c- Manage the airway d- Protect the body from injury

c. Manage the airway

The nurse teaches an adolescent male client how to use a metered dose inhaler. Seen in the picture. What instruction should the nurse provide? a. Secure the mouthpiece under the tongue. b. Press down on the device after breathing in fully c. Move the device one to two inches away from the mouth d. Breathe out slowly and deeply while compressing the device

c. Move the device one to two inches away from the mouth Rationale: Optimal position of a metered dose inhaler includes placing the inhaler one two inches away from the mouth.

After receiving report, the nurse can most safely plan to assess which client last? The client with... a. A rectal tube draining clear, pale red liquid drainage b. A distended abdomen and no drainage from the nasogastric tube c. No postoperative drainage in the Jackson-Pratt drain with the bulb compressed d. Dark red drainage on a postoperative dressing, but no drainage in the Hemovac®.

c. No postoperative drainage in the Jackson-Pratt drain with the bulb compressed Rationale: The most stable client is the one with a functioning drainage device and no drainage. This client can most safely be assessing last. Other clients are either actively bleeding, have an obstruction in the nasogastric tube which may result in vomiting, or may be bleeding and / or may have a malfunction in the Hemovac® drain.

In evaluating the effectiveness of a postoperative client's intermittent pneumatic compression devices, which assessment is most important for the nurse to complete? a. Evaluate the client's ability to use an incentive spirometer b. Monitor the amount of drainage from the client's incision c. Observe both lower extremities for redness and swelling d. Palpate all peripheral pulse points for volume and strength

c. Observe both lower extremities for redness and swelling Rationale: Intermittent compression devices (ICDs) are used to reduce venous stasis and prevent venous thrombosis in mobile and postoperative clients and its effectiveness is best assessed by observing the client's lower extremities for early signs of thrombophlebitis.

In determine the client position for insertion of an indwelling urinary catheter, it is most important for the nurse to recognize which client condition? a- High urinary PH b- Abdominal Ascites c- Orthopnea d- Fever.

c. Orthopnea Rationale: If the client is orthopneic, the nurse needs to adapt the insertion position that does not place the client in a supine position (the head of the bed should be elevated as much as possible).

A client who has been in active labor for 12 hours suddenly tells the nurse that she has a strong urge to have a bowel movement. What action should the nurse take? a- Allow the client to use a bedpan. b- Assist the client to the bathroom c- Perform a sterile vaginal exam d- Explain the fetal head is descending.

c. Perform a sterile vaginal exam Rationale: When a client in active labor suddenly expresses the urge to have a bowel movement, a sterile vaginal exam should be performed to determine if the fetus is descending.

After applying an alcohol-based hand rub to the palms of the hand and rubbing the hand together, what action should the nurse do next? a. Vigorous rub both hands together under running water b. Path both hands dry keeping the fingers lower that the arm c. Place one hand on top of the other and interlace the fingers d. Hold both hand with the fingers pointing upward until dry.

c. Place one hand on top of the other and interlace the fingers

The nurse planning care for a 12 year-old child with sickle cell disease in a vasoocclusive crisis of the elbow should include which one of the following as a priority? A) Limit fluids B) Client controlled analgesia C) Cold compresses to elbow D) Passive range of motion exercise

is B: Client controlled analgesia 6

A client's telemetry monitor indicates ventricular fibrillation (VF). What should the nurse do first? a. Administer epinephrine IV b. Give an IV bolus of amiodarone c. Provide immediate defibrillation d. Prepare for synchronized cardioversion

c. Provide immediate defibrillation

A client with emphysema is being discharged from the hospital. The nurse enters the client's room to complete discharge teaching. The client reports feeling a little short of breath and is anxious about going home. What is the best course of action? a. Postpone discharge instructions at this time and offer to contact the client by phone in a few days b. Invite the client to return to the unit for discharge teaching in a few days, when there is less anxiety c. Provide only necessary information in short, simple explanations with written instructions to take home d. Give detailed instructions speaking slowly and clearly while looking directly at the client when speaking

c. Provide only necessary information in short, simple explanations with written instructions to take home Rationale: Simple, short explanations should be provided. Information is not retained when the recipient is anxious, and too much information can increase worry. Ethically, discharge instructions may not be postponed.

While performing a skin inspection for a female adult client, the nurse observes a rash that is well circumscribed, has silvery scales and plaques, and is located on the elbows and knees. These assessment findings are likely to indicate which condition? a- Tinea corporis b- Herpes zoster c- Psoriasis d- Drug reaction

c. Psoriasis Rationale: Psoriasis is typically located on the elbow and knees

The nurse is preparing a 50 ml dose of 50% dextrose IV for a client with insulin SHOCK... medication? a- Dilute the Dextrose in one liter of 0.9% Normal Saline solution. b- Mix the dextrose in a 50 ml piggyback for a total volume of 100 ml. c- Push the undiluted Dextrose slowly through the currently infusion IV. d- Ask the pharmacist to add the Dextrose to a TPN solution.

c. Push the undiluted Dextrose slowly through the currently infusion IV Rationale: To reverse life-threatening insulin shock, the nurse should administer the 50% Dextrose infusing IV.

A female client is taking alendronate, a bisphosphate, for postmenopausal osteoporosis. The client tells the nurse that she is experiencing jaw pain. How should the nurse respond? a- Determine how the client is administering the medication b- Confirm that this is a common symptom of osteoporosis c- Report the client's jaw pain to the healthcare provider. d- Advise the client to gargle with warm salt water twice daily.

c. Report the client's jaw pain to the healthcare provider. Rationale: Bisphosponates, including alendronate, can cause osteonecrosis of jaw, which should be reported to the healthcare provider © for evaluation. Incorrect administration (A) such as failing to remain upright after taking the medication, can contribute to esophageal reactions, but does not causes haw pain. Jaw pain is not a symptom of osteoporosis and is not relieved with saline throat gargles.

A 35 years old female client has just been admitted to the post anesthesia recovery unit following a partial thyroidectomy. Which statement reflects the nurse's accurate understanding of the expected outcome for the client following this surgery? a- Supplemental hormonal therapy will probably be unnecessary b- The thyroid will regenerate to a normal size within a few years. c- The client will be restricted from eating seafood d- The remainder of the thyroid will be removed at a later date.

c. The client will be restricted from eating seafood

The nurse performs a prescribed neurological check at the beginning of the shift on a client who was admitted to the hospital with a subarachnoid brain attack (stroke). The client's Glasgow Coma Scale (GCS) score is 9. What information is most important for the nurse to determine? a. When the client's stroke symptoms started b. If the client is oriented to time c. The client's previous GCS score. d. The client's blood pressure and respiration rate.

c. The client's previous GCS score. Rationale: The normal GCS is 15, and it is most important for the nurse determine if this abnormal score is a sign of improvement or deterioration in the client's conditions. A is irrelevant. B is part of the GCS. The classic vital signs in late or sudden increasing ICP are Cushing's triad (widening pulse pressure, bradycardia with full, bounding pulse, and irregular respirations) Additional vital signs and trending of values are needed to evaluate the current finding(D) and C is a more sensitive, consistent evaluation

A client with gestational diabetes is undergoing a non-stress test (NST) at 34-week gestation... is 144 beats/minute. The client is instructed to mark the fetal monitor by pressing a button each time the baby moves. After 20 minutes, the nurse evaluates the fetal monitor strip what? a- The mother perceives and marks at least four fetal movements b- Fetal movements must be elicited with vibroacoustic stimulator c- Two FHR accelerations of 15 beats/minute x 15 seconds are recorded. d- No FHR late deceleration occur in response to fetal movement

c. Two FHR accelerations of 15 beats/minute x 15 seconds are recorded

A client admitted with an acute coronary syndrome (ACS) receives eptifibatide, a glycoprotein (GP) IIB IIIA inhibitor, which important finding places the client at greatest risk? a. Blood pressure of 100/60 b. Incontinent with blood in urine c. Unresponsive to painful stimuli d. Presence of hematemesis.

c. Unresponsive to painful stimuli Rationale: Eptifibatide, is an inhibitor of platelet aggregation, is administer IV for ACS, and bleeding is a significant side effect. A sudden onset of unresponsiveness may indicate intracranial bleeding, which is the life threatening finding related to bleeding. Although hypotension may indicate bleeding, it is not as significant as unresponsiveness to pain. This medication has a short half-life, so B and D are not life threatening findings.

The practical nurse (PN) is assigned to work with three registered nurses (RN) who are caring for neurologically compromised clients. The client with which change in status is best to assign to the PN? a- Diabetic ketoacidosis whose Glasgow coma Scale score changed from 10 to 7 b- Myxedema coma whose blood pressure changed from 80/50 to 70/40 c- Viral meningitis whose temperature changed from 101 F to 102 F. d- Subdural hematoma whose blood pressure changed from 150/80 to 170/60.

c. Viral meningitis whose temperature changed from 101 F to 102 F. Rationale: The most stable patient should be assigned to the PN, changes in the Glasgow coma Scale indicated the client's neurological status is worsening. The client decreasing BP is physiologically unstable. An increasing systolic blood pressure and widening pulse pressure is indicative of increasing intracranial pressure.

An African-American man comes into the hypertension screening booth at a community fair. The nurse finds that his blood pressure is 170/94 mmHg. The client tells the nurse that he has never been treated for high blood pressure. What response should the nurse make? a. Your blood pressure indicates that you have hypertension. You need to see a physician at once. b. Your blood pressure is quite high. Go to the closest emergency room for immediate treatment. c. Your blood pressure is a little high. You need to have it rechecked within one week. d. Your blood pressure is little high, but it is within the normal range for your age group.

c. Your blood pressure is a little high. You need to have it rechecked within one week.

The nurse is assessing a client's nailbeds. Witch appearance indicates further follow-up is needed for problems associated with chronic hypoxia? Clubbing FLIP for photo!!!

clubbing

The nurse prepares to insert an oral airway by first measuring for the correct sized airway. Which picture shows the correct approach to airway size measurement?

corner of the mouth to the tip of the ear

(arrange in order) = middle age woman bleeding- woman w/ blanket- child crying- mother and father. (Mine didn't have a middle age woman bleeding, instead it had an old man wandering and

correct order is: wandering old man, woman w/blanket, man holding baby, and mother & father)

The nurse is performing a physical assessment on a toddler. Which of the following should be the first action? A) Perform traumatic procedures B) Use minimal physical contact C) Proceed from head to toe D) Explain the exam in detail

is B: Use minimal physical contact 7

A male client with COPD smokes two packs of cigarettes per day and is admitted to the hospital for a respiratory infection. He complains that he has trouble controlling respiratory distress at home when using his rescue inhaler. Which comment from the client indicates to the nurse that he is not using his inhaler properly? a. "I have a hard time inhaling and holding my breath after I squeeze the inhaler, but I do my best" b. "I never use the inhaler unless I am feeling really short of breath" c. I always shake the inhaler several times before I start" d. "After I squeeze the inhaler and swallow, I always feel a slight wave of nausea, but it goes away"

d. "After I squeeze the inhaler and swallow, I always feel a slight wave of nausea, but it goes away" Rationale: It is vital for the nurse to emphasize to the client that the mist should be inhaled, not swallowed. This assessment should be done for all COPD clients, not just those who complain about their inhalers.

The nurse is teaching a male client with multiple sclerosis how to empty his bladder using the Crede Method. When performing a return demonstration, the client applies pressure to the umbilical areas of his abdomen. What instruction should the nurse provide? a- Stroke the inner thigh below the perineum to initiate urinary flow b- Contract, hold, and then relax the pubococcygeal muscle c- Pour warm water over the external sphincter at the distal glans d- Apply downward manual pressure at the suprapubic regions.

d. Apply downward manual pressure at the suprapubic regions. Rationale: The Crede Method is used for those clients with atonic bladders, which is a concomitant of demyelinating disorders like multiple sclerosis. The client is applying pressure in the wrong region (umbilical Are) and should be instructed to apply pressure at the suprapubic are.

A client present at the clinic with blepharitis. What instructions should the nurse provide for home care? a- Use bilateral eyes patches while sleeping to prevent injury to eyes. b- Wear sunglasses when out of doors to prevent photophobia c- Apply warm moist compresses then gently scrub eyelids with diluted baby shampoo

d. Apply warm moist compresses then gently scrub eyelids with diluted baby shampoo Rationale: This condition is an inflammation of the eyelids edges that occurs in older adults. Is controlled with eyelid care using warm moist compresses followed by gently scrub eyelids.

An older female client tells the nurse that her muscles have gradually been getting weak...what is the best initial response by the nurse? a- Explain that this is an expected occurrence with aging. b- Observe the lower extremity for signs of muscle atrophy c- Review the medical record for recent diagnosis test results. d- Ask the client to describe the changes that have occurred

d. Ask the client to describe the changes that have occurred

An older male client is admitted with the medical diagnosis of possible cerebral vascular accident (CVA). He has facial paralysis and cannot move his left side. When entering the room, the nurse finds the client's wife tearful and trying unsuccessfully to give him a drink of water. What action should the nurse take? a. Give the wife a straw to help facilitate the client's drinking. b. Assist the wife and carefully give the client small sips of water c. Obtain a thickening powder before providing any more fluids. d. Ask the wife to stop and assess the client's swallowing reflex.

d. Ask the wife to stop and assess the client's swallowing reflex. Rationale: Until a swallowing reflex has been established, giving oral fluids can be dangerous, even life-threatening. The nurse should immediately stop the dangerous situation and assess the client. It is most important to determine if the client can swallow before giving him anything by mouth.

A 13 years-old client with non-union of a comminuted fracture of the tibia is admitted with osteomyelitis. The healthcare provider collects home aspirate specimens for culture and sensitivity and applies a cast to the adolescent's lower leg. What action should the nurse implement next? a. Administer antiemetic agents b. Bivalve the cast for distal compromise c. Provide high- calorie, high-protein diet d. Begin parenteral antibiotic therapy

d. Begin parenteral antibiotic therapy Rationale: The standard of treatment for osteomyelitis is antibiotic therapy and immobilization. After bond and blood aspirate specimens are obtained for culture and sensitivity, the nurse should initiate parenteral antibiotics as prescribed.

After a routine physical examination, the healthcare admits a woman with a history of Systemic Lupus Erythematous (SLE) to the hospital because she has 3+ pitting ankle edema and blood in her urine. Which assessment finding warrants immediate intervention by the nurse? a. Dark, rust-colored urine b. Urine output 300 ml/hr c. Joint and muscle aches d. Blood pressure 170/98

d. Blood pressure 170/98 Rationale: SLE can result in renal complication such as glomerulonephritis, which can cause a critically high blood pressure that necessitates immediate intervention. A, B and C are symptoms of glomerulonephritis and should be treated once the blood pressure is under control

The charge nurse in a critical care unit is reviewing clients' conditions to determine who is stable enough to be transferred. Which client status report indicates readiness for transfer from the critical care unit to a medical unit? a. Pulmonary embolus with an intravenous heparin infusion and new onset hematuria b. Myocardial infarction with sinus bradycardia and multiple ectopic beats c. Adult respiratory distress syndrome with pulse oximetry of 85% saturation. d. Chronic liver failure with a hemoglobin of 10.1 and slight bilirubin elevation

d. Chronic liver failure with a hemoglobin of 10.1 and slight bilirubin elevation Rationale: A slight bilirubin elevation and anemia are expected finding in a stable client with chronic liver failure who should be transferred to a less-acute medical unit.

The healthcare provider prescribes the antibiotic cephradine 500mg PO every 6 hours for a client with a postoperative wound infection. Which foods should the nurse encourage this client to eat a. Vanilla-flavored yogurt b. Low fat chocolate milk. c. Calcium fortified juice d. Cinnamon applesauce

d. Cinnamon applesauce RATIONALE: Dairy products and calcium fortified dairy products decrease the absorption of ciprofloxacin. Cinnamon applesauce contains no calcium, so this is the best snack selection. Since other options contains calcium, these snacks should be avoided by a client who is taking ciprofloxacin.

The home health nurse is assessing a male client who has started peritoneal dialysis (PD) 5 days ago. Which assessment finding warrants immediate intervention by the nurse? a. Finger stick blood glucose 120 mg/dL post exchange b. Arteriovenous (AV) graft surgical site pulsations. c. Anorexia and poor intake of adequate dietary protein d. Cloudy dialysate output and rebound abdominal pain

d. Cloudy dialysate output and rebound abdominal pain

A nurse is conducting a physical assessment of a young adult. Which information provides the best indication of the individual nutritional status? a. A 24-hour diet history b. History of a recent weight loss c. Status of current petite d. Condition of hair, nails, and skin

d. Condition of hair, nails, and skin Rationale: The assessment of hair, nails and skin is most indicative of long-term nutritional status, which is important in the healing process.

An adult woman who is seen in the clinic with possible neuropathic pain of the right leg rates her pain as a 7 on a 10 point scale. What action should the nurse take? a. Elevate the foot and leg on two pillows b. Measure the client's capillary glucose c. Ask the client to dorsiflex the right foot. d. Encourage the client to describe the pain.

d. Encourage the client to describe the pain. Rationale: Neuropathic pain is caused by damage within the nervous system. Description of the pain such as burning or numbness helps identify the pain as neuropathic, allowing appropriate treatment to be initiated. Elevation is to unlikely to impact the pain. Persons with diabetes mellitus may develop peripheral neuropathy, nut there is no immediate need to measure this client's capillary glucose. (C) is not a useful intervention in assessing or managing neuropathic pain.

The nurse note a depressed female client has been more withdrawn and noncommunicative during the past two weeks. Which intervention is most important to include in the updated plan of care for this client? a. Encourage the client's family to visit more often b. Schedule a daily conference with the social worker c. Encourage the client to participate in group activities d. Engage the client in a non-threatening conversation.

d. Engage the client in a non-threatening conversation.

Progressive kyphoscoliosis leading to respiratory distress is evident in a client with muscul...Which finding warrants immediate intervention by the nurse? a. Extremity muscle weakness b. Bilateral eyelid drooping c. Inability to swallow pills d. Evidence of hypoventilation

d. Evidence of hypoventilation Rationale: Hypoventilation indicates respiratory muscle weakness, and if the client is unable to breath... respiratory distress and life-threatening.

A clinical trial is recommended for a client with metastatic breast cancer, but she refuses to participate and tells her family that she does not wish to have further treatments. The client's son and daughter ask the nurse to try and convince their mother to reconsider this decision. How should the nurse respond? a. Ask the client with her children present if she fully understands the decision she has made. b. Discuss success of clinical trials and ask the client to consider participating for one month. c. Explain to the family that they must accept their mother's decision. d. Explore the client's decision to refuse treatment and offer support

d. Explore the client's decision to refuse treatment and offer support Rationale: as long as the client is alert, oriented and aware of the disease prognosis, the healthcare team must abide by her decisions. Exploring the decision with the client and offering support provides a therapeutic interaction and allows the client to express her fears and concerns about her quality of life. Other options are essentially arguing with the client's decisions regarding her end of life treatment or diminish the opportunity for the client to discuss her feelings

The nurse caring for a client with dysphagia is attempting to insert an NG tube, but the client will not swallow and is not gagging. What action should the nurse implement to facilitate the NGT passage into the esophagus? a. Push the NGT beyond the oropharynx gently yet swiftly. b. Offer the client sips of water or ice and coax to swallow c. Elevate the bed 90 degree and hyperextend the head. d. Flex the client's head with chin to the chest and insert.

d. Flex the client's head with chin to the chest and insert.

When conducting diet teaching for a client who was diagnosed with nutritional anemia in pregnancy, which foods should the nurse encourage the client to eat? (Select all that apply) a. Seeds, spices, lettuce b. Consomme, celery, carrot c. Oranges, orange juice, bananas d. Fortified whole wheat cereals, whole-grain pasta, brown rice e. Spinach, kale, dried raisins and apricots

d. Fortified whole wheat cereals, whole-grain pasta, brown rice e. Spinach, kale, dried raisins and apricots Rationale: Nutritional anemia in pregnancy should be supplemented with additional iron in the diet. Foods that are high in iron content are often protein based, whole grains (D), green leafy vegetables and dried fruits (E). (A, B, and C) are not iron rich sources

Artificial rupture of the membrane of a laboring reveals meconium-stained fluid, what is... the priority? a- Clean the perineal are to prevent infection b- Assess the mother's blood pressure to check for signs of preeclampsia c- Assess the mother temperature to check for development of sepsis. d- Have a meconium aspirator available at delivery.

d. Have a meconium aspirator available at delivery

The nurse is teaching a mother of a newborn with a cleft lip how to bottle feed her baby using medela haberman feeder, which has a valve to control the release of milk and a slit nipple opening. The nurse discusses placing the nipple's elongated tip in the back of the oral cavity. What instructions should the nurse provide the mother about feedings? a. Squeeze the nipple base to introduce milk into the mouth b. Position the baby in the left lateral position after feeding c. Alternate milk with water during feeding d. Hold the newborn in an upright position

d. Hold the newborn in an upright position Rationale: the mother should be instructed to hold the infant during feedings in a sitting or upright position to prevent aspiration. Impaired sucking is compensated using special feeding appliances and nipples such as the Haberman feeder that prevents aspiration by adjusting the flow of mild according to the effort of the neonate. Squeezing the nipple base may introduce a volume that is greater than the neonate can coordinate swallowing. The preferred position of an infant after feeding is on the right side to facilitate stomach emptying. Sucking difficulty impedes the neonate's intake of adequate nutrient needed for weight gain and water should be provided after the feeding to cleanse the oral cavity and not fill up the neonate's stomach.

The nurse plans to administer a schedule dose of metoprolol (Toprol SR) at 0900 to a client with hypertension. At 0800, the nurse notes that client's telemetry pattern shows a second degree heart block with a ventricular rate of 50. What action should the nurse take? a- Administer the Toprol immediately and monitor the client until the heart rate increases. b- Provide the dose of Toprol as scheduled and assign a UAP to monitor the client's BP q30 minutes. c- Give the Toprol as scheduled if the client's systolic blood pressure reading is greater than 180. d- Hold the scheduled dose of Toprol and notify the healthcare provider of the telemetry pattern.

d. Hold the scheduled dose of Toprol and notify the healthcare provider of the telemetry pattern. Rationale: Beta blockers such as metoprolol (Toprol SR) are contraindicated in clients with second or third-degree heart block because they decrease the heart rate. Therefore, the nurse should hold the medication.

A newly graduated female staff nurse approaches the nurse manager and request reassignment to another client because a male client is asking her for a date and making suggestive comments. Which response is best for the nurse manager to provide? a. I have to call the supervisor to get someone else to transfer to this unit to care for him. b. I know you are good nurse and can handle this client in a professional manner. c. I'll talked to the client about his sexual harassment and I'll insist that he stop it immediately. d. I'll change your assignment, but let's talk about how a nurse should respond to this kind of client.

d. I'll change your assignment, but let's talk about how a nurse should respond to this kind of client.

After receiving lactulose, a client with hepatic encephalopathy has several loose stools. What action should the nurse implement? a- Send stool specimen to the lab b- Measure abdominal girth c- Encourage increased fiber in diet. d- Monitor mental status.

d. Monitor mental status. Rationale: Administer lactulose to a patient with hepatic encephalopathy to lower serum ammonia level, so mental status should be improving.

The nurse is interviewing a client with schizophrenia. Which client behavior requires immediate intervention? a. Lip smacking and frequent eye blinking b. Shuffling gait and stooped posture c. Rocks back and forth in the chair d. Muscle spasms of the back and neck

d. Muscle spasms of the back and neck

The nurse should observe most closely for drug toxicity when a client receives a medication that has which characteristic? a- Low bioavailability b- Rapid onset of action c- Short half life d- Narrow therapeutic index.

d. Narrow therapeutic index. Rationale: Narrow therapeutic index (NTI) drugs are defined as those drugs where small differences in dose or blood concentration may lead to dose and blood concentration dependent, serious therapeutic failures or adverse drug reactions.

While removing an IV infusion from the hand of a client who has AIDS, the nurse is struck with the needle. After washing the puncture site with soap & water, which action should the nurse take? a. Complete a usual incident report b. Start prophylactic treatment c. Seek psychological resources d. Notify the employee health nurse.

d. Notify the employee health nurse.

Which intervention should the nurse implement for a client with a superficial (first degree) burn? a. Spray an anesthetic agent over the burn every 3 to 4 hours b. Position the burn victim in front of a cool fan to decrease discomfort c. Apply ice pack for 30 mints to lower surface temperature d. Place wet clothes on the burned areas for short periods of time.

d. Place wet cloths on the burned areas for short periods of time. Rationale: D provides comfort and helps to relive the pain of a first degree burn, which involves only the epidermal layer of the skin.

Following routine diagnostic test, a client who is symptom-free is diagnosed with Paget's disease. Client teaching should be directed toward what important goal for this client? a- Maintain adequate cardiac output b- Promote adequate tissue perfusion c- Promote rest and sleep d- Reduce the risk for injury

d. Reduce the risk for injury Rationale: Paget's is a metabolic bone disorder which place the client at high risk for injury. Once the client is symptom free the next goal is reducing risk for injury

When gathering for a group therapy session at 1400 hours, a female client complains to the nurse that a smoking break has not been allowed all day. The nurse responds that 15 minute breaks were called over the unit intercom after breakfast and after lunch. The nurse is using what communication technique in responding to the client? a. Doubt b. Observation c. Confrontation d. Reflection

d. Reflection

Which instruction is most important for the nurse to provide a client who receives a new plan of care to treat osteoporosis? a. Begin a weight-bearing exercise plan b. Increase intake of foods rich in calcium c. Schedule a bone density tests every year. d. Remain upright after taking the medication.

d. Remain upright after taking the medication Rationale: Risendronate, causes reflux and esophageal erosion.

During an Insulin infusion for a client with diabetes mellitus who is experiencing hyperglycemic hyperosmolar syndrome in addition to the client's glucose, which laboratory value is most important for the nurse to monitor? a. Urine ketones b. Urine albumin c. Serum protein d. Serum potassium

d. Serum potassium Rationale: Electrolyte shifts are common during correction of hyperosmolar and hyperglycemic states. Monitor electrolyte levels at least every 4 hours, or every 2 hours if needed. Monitor serum sodium and potassium levels closely. If needed, use isotonic and hypotonic saline solutions to adjust the patient's sodium level. Despite major potassium loss during diuresis in early HHS stages, many patients initially present in a hyperkalemic state due to dehydration. When fluid and insulin therapy begin, the serum potassium level may drop dramatically.

The nurse is caring for a 4 year-old admitted after receiving burns to more than 50% of his body. Which laboratory data should be reviewed by the nurse as a priority in the first 24 hours? A) Blood urea nitrogen B) Hematocrit C) Blood glucose D) White blood count

is A: Blood urea nitrogen 13

A client who is recently diagnosed with type 2 diabetes mellitus (DM) ask the nurse how this type of diabetes leads to high blood sugar. What Pathophysiology mechanism should the nurse explain about the occurrence of hyperglycemia in those who have type 2 DM? a- Immune antibodies attack pancreatic beta cells resulting in no insulin b- The body cells develop resistance to the action of insulin. c- Body organs produce less insulin and more glucagon d- The liver produces excess glucose in response to excess glycotrophic hormones

d. The body cells develop resistance to the action of insulin

A client who is scheduled for an elective inguinal hernia repair today in day surgery is seen eating in the waiting area. What action should be taken by the nurse who is preparing to administer the preoperative medications? a- Review the surgical consent with the client b- Explain that vomiting can occur during surgery c- Remove the food from the client d- Withhold the preoperative medication

d. Withhold the preoperative medication

The nurse is triaging clients in an urgent care clinic. The client with which symptoms should be referred to the health care provider immediately? a. headache, photophobia, and nuchal rigidity b. high fever, skin rash, and a productive cough c. nausea, vomiting, and poor skin turgor d. malaise, fever, and stiff, swollen joints

headache, photophobia, and nuchal rigidity Rationale: Headache, photophobia, and nuchal rigidity are classic signs of meningeal infection, so this client should immediately be referred to the health care provider. AC D do not have priority of B

Risk management: pt fell while using some equipment... charge nurse trying to find out what happened- how?

hospital polices

(Select all that apply) Determining pain level for 3mo old infant

infant will clinch fists, increase pulse, restlessness, increased respiratory effort

During orientation, a newly hired nurse demonstrates suctioning of a tracheostomy in a skills class, as seen in the video. After the demonstration, the supervising nurse expresses concern that the demonstrated procedure increased the client's risk for which problem? a. infection

infection

In planning strategies to reduce a client's risk for complications following orthopedic surgery, the nurse recognizes which pathology as the underlying cause of osteomyelitis? a. Infectious process b. Metastatic process c. Autoimmune disorder d. Inflammatory disorder

infectious process

The mother of a child with a neural tube defect asks the nurse what she can do to decrease the chances of having another baby with a neural tube defect. What is the best response by the nurse? A) "Folic acid should be taken before and after conception." B) "Multivitamin supplements are recommended during pregnancy." C) "A well balanced diet promotes normal fetal development." D) "Increased dietary iron improves the health of mother and fetus."

is A: "Folic acid should be taken before and after conception." 5

A 15 year-old client has been placed in a Milwaukee Brace. Which statement from the adolescent indicates the need for additional teaching? A) "I will only have to wear this for 6 months." B) "I should inspect my skin daily." C) "The brace will be worn day and night." D) "I can take it off when I shower."

is A: "I will only have to wear this for 6 months." 3

The parents of a 4 year-old hospitalized child tell the nurse, "We are leaving now and will be back at 6 PM." A few hours later the child asks the nurse when the parents will come again. What is the best response by the nurse? A) "They will be back right after supper." B) "In about 2 hours, you will see them." C) "After you play awhile, they will be here." D) "When the clock hands are on 6 and 1

is A: "They will be back right after supper." 12

A client who was medicated with meperidine hydrochloride (Demerol) 100 mg and hydroxyzine hydrochloride (Vistaril Intramuscular) 50 mg IM for pain related to a fractured lower right leg 1 hour ago reports that the pain is getting worse. The nurse should recognize that the client may be developing which complication? A) Acute compartment syndrome B) Thromboemolitic complications C) Fatty embolism D) Osteomyelitis

is A: Acute compartment syndrome 15

The charge nurse has a health care team that consists of 1 PN, 1 unlicensed assistive personnel (UAP) and 1 PN nursing student. Which assignment should be questioned by the nurse manager? A) An admission at the change of shifts with atrial fibrillation and heart failure - PN B) Client who had a major stroke 6 days ago - PN nursing student C) A child with burns who has packed cells and albumin IV running - charge nurse D) An elderly client who had a myocardial infarction a week ago - UAP

is A: An admission at the change of shifts with atrial fibrillation and heart failure - PN 1

While interviewing a client, the nurse notices that the client is shifting positions, wringing her hands, and avoiding eye contact. It is important for the nurse to A) Ask the client what she is feeling B) Assess the client for auditory hallucinations C) Recognize the behavior as a side effect of medication D) Re-focus the discussion on a less anxiety provoking topic

is A: Ask the client what she is feeling 13

A client has had heart failure. Which intervention is most important for the nurse to implement prior to the initial administration of Digoxin to this client? A) Assess the apical pulse, counting for a full 60 seconds B) Take a radial pulse, counting for a full 60 seconds C) Use the pulse reading from the electronic blood pressure device D) Check for a pulse deficit

is A: Assess the apical pulse, counting for a full 60 seconds 15

The charge nurse on the night shift at an urgent care center has to deal with admitting clients of a higher acuity than usual because of a large fire in the area. Which style of leadership and decision-making would be best in this circumstance? A) Assume a decision-making role B) Seek input from staff C) Use a non-directive approach D) Shared decision-making with others

is A: Assume a decision making role 4

The nurse is teaching a client newly diagnosed with asthma how to use the metereddose inhaler (MDI). The client asks when they will know the canister is empty. The best response is A) Drop the canister in water to observe floating B) Estimate how many doses are usually in the canister C) Count the number of doses as the inhaler is used D) Shake the canister to detect any fluid movement

is A: Drop the canister in water to observe floating 11

The nurse is assessing a 55 year-old female client who is scheduled for abdominal surgery. Which of the following information would indicate that the client is at risk for thrombus formation in the post-operative period? A) Estrogen replacement therapy B) 10% less than ideal body weight C) Hypersensitivity to heparin D) History of hepatitis

is A: Estrogen replacement therapy 8

The nurse is teaching parents about diet for a 4 month-old infant with gastroenteritis and mild dehydration. In addition to oral rehydration fluids, the diet should include A) Formula or breast milk B) Broth and tea C) Rice cereal and apple juice D) Gelatin and ginger ale

is A: Formula or breast milk 10

When interviewing the parents of a child with asthma, it is most important to gather what information about the child's environment? A) Household pets B) New furniture C) Lead based paint D) Plants such as cactus

is A: Household pets 4

A young adult seeks treatment in an outpatient mental health center. The client tells the nurse he is a government official being followed by spies. On further questioning, he reveals that his warnings must be heeded to prevent nuclear war. What is the most therapeutic approach by the nurse? A) Listen quietly without comment B) Ask for further information on the spies C) Confront the client on a delusion D) Contact the government agency

is A: Listen quietly without comment 13

While teaching the family of a child who will take phenytoin (Dilantin) regularly for seizure control, it is most important for the nurse to teach them about which of the following actions? A) Maintain good oral hygiene and dental care B) Omit medication if the child is seizure free C) Administer acetaminophen to promote sleep D) Serve a diet that is high in iron

is A: Maintain good oral hygiene and dental care 11

A client has been admitted to the Coronary Care Unit with a myocardial infarction. Which nursing diagnosis should have priority? A) Pain related to ischemia B) Risk for altered elimination: constipation C) Risk for complication: dysrhythmias D) Anxiety related to pain

is A: Pain related to ischemia 2

The nurse is planning discharge for a 90 year-old client with musculoskeletal weakness. Which intervention should be included in the plan and would be most effective for the prevention of falls? A) Place nightlight in the bedroom B) Wear eyeglasses at all times C) Install grab bars in the bathroom D) Teach muscle strengthening exercises

is A: Place nightlight in the bedroom 9

A client is admitted to the rehabilitation unit following a CVA and mild dysphagia. The most appropriate intervention for this client is A) Position client in upright position while eating B) Place client on a clear liquid diet C) Tilt head back to facilitate swallowing reflex D) Offer finger foods such as crackers or pretzels

is A: Position client in upright position while eating 5

The nurse is preparing a handout on infant feeding to be distributed to families visiting the clinic. Which notation should be included in the teaching materials? A) Solid foods are introduced 1 at a time beginning with cereal B) Finely ground meat should be started early to provide iron C) Egg white is added early to increase protein intake D) Solid foods should be mixed with formula in a bottle

is A: Solid foods are introduced 1 at a time beginning with cereal 7

Handshaking is the preferred form of touch or contact used with clients in a psychiatric setting. The rationale behind this limited touch practice is that A) Some clients misconstrue hugs as an invitation to sexual advances B) Handshaking keeps the gesture on a professional level C) Refusal to touch a client denotes lack of concern D) Inappropriate touch often results in charges of assault and battery

is A: Some clients misconstrue hugs as an invitation to sexual advances 14

A couple experienced the loss of a 7 month-old fetus. In planning for discharge, what should the nurse emphasize? A) To discuss feelings with each other and use support persons B) To focus on the other healthy children and move through the loss C) To seek causes for the fetal death and come to some safe conclusion D) To plan for another pregnancy within 2 years and maintain physical health

is A: To discuss feelings with each other and use support persons 12

The nurse is caring for a client with a colostomy. During a teaching session, the nurse recommends that the pouch be emptied A) When it is 1/3 to 1/2 full B) Prior to meals C) After each fecal elimination D) At the same time each day

is A: When it is 1/3 to 1/2 full 8

A 7 month pregnant woman is admitted with complaints of painless vaginal bleeding over several hours. The nurse should prepare the client for an immediate A) Non stress test B) Abdominal ultrasound C) Pelvic exam D) X-ray of abdomen

is B: Abdominal ultrasound 11

A client with moderate persistent asthma is admitted for a minor surgical procedure. On admission the peak flow meter is measured at 480 liters/minute. Post-operatively the client is complaining of chest tightness. The peak flow has dropped to 200 liters/minute. What should the nurse do first? A) Notify the health care provider B) Administer the PRN dose of Albuterol C) Apply oxygen at 2 liters per nasal cannula D) Repeat the peak flow reading in 30 minutes

is B: Administer the PRN dose of Albuterol 9

A client is scheduled for an IVP (Intravenous Pyelogram). Which of the following data from the client's history indicate a potential hazard for this test? A) Reflex incontinence B) Allergic to shellfish C) Claustrophobia D) Hypertension

is B: Allergic to shellfish 7

The nurse is reinforcing teaching to a 24 year-old woman receiving acyclovir (Zovirax) for a Herpes Simplex Virus type 2 infection. Which of these instructions should the nurse give the client? A) Complete the entire course of the medication for an effective cure B) Begin treatment with acyclovir at the onset of symptoms of recurrence C) Stop treatment if she thinks she may be pregnant to prevent birth defects D) Continue to take prophylactic doses for at least 5 years after the diagnosis

is B: Begin treatment with acyclovir at the onset of symptoms of recurrence 9

The nurse has performed the initial assessments of 4 clients admitted with an acute episode of asthma. Which assessment finding would cause the nurse to call the health care provider immediately? A) Prolonged inspiration with each breath B) Expiratory wheezes that are suddenly absent in 1 lobe C) Expectoration of large amounts of purulent mucous D) Appearance of the use of abdominal muscles for breathing

is B: Expiratory wheezes that are suddenly absent in one lobe 6

A client has been admitted with a fractured femur and has been placed in skeletal traction. Which of the following nursing interventions should receive priority? A) Maintaining proper body alignment B) Frequent neurovascular assessments of the affected leg C) Inspection of pin sites for evidence of drainage or inflammation D) Applying an over-bed trapeze to assist the client with movement in bed

is B: Frequent neurovascular assessments of the affected leg 11

A client with Guillain Barre is in a non responsive state, yet vital signs are stable and breathing is independent. What should the nurse document to most accurately describe the client's condition? A) Comatose, breathing unlabored B) Glascow Coma Scale 8, respirations regular C) Appears to be sleeping, vital signs stable D) Glascow Coma Scale 13, no ventilator required

is B: Glascow Coma Scale 8, respirations regular 1

A client is admitted with infective endocarditis (IE). Which symptom would alert the nurse to a complication of this condition? A) Dyspnea B) Heart murmur C) Macular rash D) Hemorrhage

is B: Heart murmur Large, soft, rapidly developing vegetations attach to the heart valves. 3

The nurse is caring for a newborn with tracheoesophageal fistula. Which nursing diagnosis is a priority? A) Risk for dehydration B) Ineffective airway clearance C) Altered nutrition D) Risk for injury

is B: Ineffective airway clearance 5

The nurse is assessing an 8 month-old infant with a malfunctioning ventriculoperitoneal shunt. Which one of the following manifestations would the infant be most likely to exhibit? A) Lethargy B) Irritability C) Negative Moro D) Depressed fontanel

is B: Irritability 6

A nurse admits a premature infant who has respiratory distress syndrome. In planning care, nursing actions are based on the fact that the most likely cause of this problem stems from the infant's inability to A) Stabilize thermoregulation B) Maintain alveolar surface tension C) Begin normal pulmonary blood flow D) Regulate intra cardiac pressure

is B: Maintain alveolar surface tension 3

The nurse is teaching a newly diagnosed asthma client on how to use a peak flow meter. The nurse explains that this should be used to A) Determine oxygen saturation B) Measure forced expiratory volume C) Monitor atmosphere for presence of allergens D) Provide metered doses for inhaled bronchodilator

is B: Measure forced expiratory volume 12

The nurse admitting a 5 month-old who vomited 9 times in the past 6 hours should observe for signs of which overall imbalance? A) Metabolic acidosis B) Metabolic alkalosis C) Some increase in the serum hemoglobin D) A little decrease in the serum potassium

is B: Metabolic alkalosis 4

A PN is assigned to care for a newborn with a neural tube defect. Which dressing if applied by the PN would need no further intervention by the charge nurse? A) Telfa dressing with antibiotic ointment B) Moist sterile non adherent dressing C) Dry sterile dressing that is occlusive D) Sterile occlusive pressure dressing

is B: Moist sterile non adherent dressing 5

The nurse is providing instructions for a client with asthma. Which of the following should the client monitor on a daily basis? A) Respiratory rate B) Peak air flow volumes C) Pulse oximetry D) Skin color

is B: Peak air flow volumes 12

The provisions of the law for the Americans with Disabilities Act require nurse managers to A) Maintain an environment free from associated hazards B) Provide reasonable accommodations for disabled individuals C) Make all necessary accommodations for disabled individuals D) Consider both mental and physical disabilities

is B: Provide reasonable accommodations for disabled individuals 2

The nurse is caring for a client with a distal tibia fracture. The client has had a closed reduction and application of a toe to groin cast. 36 hours after surgery, the client suddenly becomes confused, short of breath and spikes a temperature of 103 degrees Fahrenheit. The first assessment the nurse should perform is A) Orientation to time, place and person B) Pulse oximetry C) Circulation to casted extremity D) Blood pressure

is B: Pulse oximetry 12

A newly admitted elderly client is severely dehydrated. When planning care for this client, which task is appropriate to assign to an unlicensed assistive personnel (UAP)? A) Converse with the client to determine if the mucous membranes are impaired B) Report hourly outputs of less than 30 ml/hr C) Monitor client's ability for movement in the bed D) Check skin turgor every 4 hours

is B: Report output of less than 30 ml/hr 2

A mother brings her 3 month-old into the clinic, complaining that the child seems to be spitting up all the time and has a lot of gas. The nurse expects to find which of the following on the initial history and physical assessment? A) Increased temperature and lethargy B) Restlessness and increased mucus production C) Increased sleeping and listlessness D) Diarrhea and poor skin turgor

is B: Restlessness and increased mucus production 2

Which of the following times is a depressed client at highest risk for attempting suicide? A) Immediately after admission, during one-to-one observation B) 7 to 14 days after initiation of antidepressant medication and psychotherapy C) Following an angry outburst with family D) When the client is removed from the security room

is B: Seven to 14 days after initiation of antidepressant medication and psychotherapy 15

The nurse has admitted a 4 year-old with the diagnosis of possible rheumatic fever. Which statement by the parent would cause the nurse to suspect an association with this disease? A) Our child had chickenpox 6 months ago. B) Strep throat went through all the children at the day care last month. C) Both ears were infected over 3 months age. D) Last week both feet had a fungal skin infection.

is B: Strep throat went through all the children at the day care last month. 2

A client is admitted to a psychiatric unit with delusions. What findings can the nurse expect? A) Flight of ideas and hyperactivity B) Suspiciousness and resistance to therapy C) Anorexia and hopelessness D) Panic and multiple physical complaints

is B: Suspiciousness and resistance to therapy 13

When parents call the emergency room to report that a toddler has swallowed drain cleaner, the nurse instructs them to call for emergency transport to the hospital. While waiting for an ambulance, the nurse would suggest for the parents to give sips of which substance? A) Tea B) Water C) Milk D) Soda

is B: Water 7

A man diagnosed with epididymitis 2 days ago calls the nurse at a health clinic to discuss the problem. What information is most important for the nurse to ask about at this time? A) What are you taking for pain and does it provide total relief? B) What does the skin on the testicles look and feel like? C) Do you have any questions about your care? D) Did you know a consequence of epididymitis is infertility?

is B: What does the skin on the testicles look and feel like? 15

The charge nurse is planning assignments on a medical unit. Which client should be assigned to the unlicensed assistive personnel (UAP)? A client with A) Difficulty swallowing after a mild stroke B) an order of enemas until clear prior to colonoscopy C) an order for a post-op abdominal dressing change D) transfer orders to a long term facility

is B: an order of enemas until clear prior to colonoscopy 6

The nurse is reviewing with a client how to collect a clean catch urine specimen. Which sequence is appropriate teaching? A) Void a little, clean the meatus, then collect specimen B) clean the meatus, begin voiding, then catch urine stream C) Clean the meatus, then urinate into container D) Void continuously and catch some of the urine

is B: clean the meatus, begin voiding, then catch urine stream 1

The provider orders Lanoxin (digoxin) 125 mg PO and furosomide 40 mg every day. Which of these foods would the nurse reinforce for the client to eat at least daily? A) spaghetti B) watermelon C) chicken D) tomatoes

is B: watermelon 1

As the nurse takes a history of a 3 year-old with neuroblastoma, what comments by the parents require follow-up and are consistent with the diagnosis? A) "The child has been listless and has lost weight." B) "The urine is dark yellow and small in amounts." C) "Clothes are becoming tighter across her abdomen." D) "We notice muscle weakness and some unsteadiness."

is C: "Clothes are becoming tighter across her abdomen." 2

Which statement made by a client indicates to the nurse that he may have a thought disorder? A) "I'm so angry about this. Wait until my partner hears about this." B) "I'm a little confused. What time is it?" C) "I can't find my 'mesmer' shoes. Have you seen them?" D) "I'm fine. It's my daughter who has the problem."

is C: "I can''t find my ''mesmer'' shoes. Have you seen them?" 13

The mother of a 2 year-old hospitalized child asks the nurse's advice about the child's screaming every time the mother gets ready to leave the hospital room. What is the best response by the nurse? A) "I think you or your partner needs to stay with the child while in the hospital." B) "Oh, that behavior will stop in a few days." C) "Keep in mind that for the age this is a normal response to being in the hospital." D) "You might want to "sneak out" of the room once the child falls asleep."

is C: "Keep in mind that for the age this is a normal response to being in the hospital." 10

Following change-of-shift report on an orthopedic unit, which client should the nurse see first? A) 16 year-old who had an open reduction of a fractured wrist 10 hours ago B) 20 year-old in skeletal traction for 2 weeks since a motor cycle accident C) 72 year-old recovering from surgery after a hip replacement 2 hours ago D) 75 year-old who is in skin traction prior to planned hip pinning surgery.

is C: 72 year-old recovering from surgery after a hip replacement 2 hours ago 1

Lactulose (Chronulac) has been prescribed for a client with advanced liver disease. Which of the following assessments would the nurse use to evaluate the effectiveness of this treatment? A) An increase in appetite B) A decrease in fluid retention C) A decrease in lethargy D) A reduction in jaundice

is C: A decrease in lethargy 8

A client enters the emergency department unconscious via ambulance from the client's work place. What document should be given priority to guide the direction of care for this client? A) The statement of client rights and the client self determination act B) Orders written by the health care provider C) A notarized original of advance directives brought in by the partner D) The clinical pathway protocol of the agency and the emergency department

is C: A notarized original of advance directives brought in by the partner 1

A client with a documented pulmonary embolism has the following arterial blood gases: PO2 - 70 mm hg, PCO2 - 32 mm hg, pH - 4.5, SaO2 - 87%, HCO3 - 2 Based on this data, what is the first nursing action? A) Review other lab data B) Notify the health care provider C) Administer oxygen D) Calm the client

is C: Administer oxygen 8

Which order can be associated with the prevention of atelectasis and pneumonia in a client with amyotrophic lateral sclerosis? A) Active and passive range of motion exercises twice a day B) Every 4 hours incentive spirometer C) Chest physiotherapy twice a day D) Repositioning every 2 hours around the clock

is C: Chest physiotherapy twice a day 15

The nurse is caring for a client who is post-op following a thoracotomy. The client has 2 chest tubes in place, connected to 1 chest drain. The nursing assessment reveals bubbling in the water seal chamber when the client coughs. What is the most appropriate nursing action? A) Clamp the chest tube B) Call the surgeon immediately C) Continue to monitor the client to see if the bubbling increases D) Instruct the client to try to avoid coughing

is C: Continue to monitor the client to see if the bubbling increases 9

A 6 year-old child is seen for the first time in the clinic. Upon assessment, the nurse finds that the child has deformities of the joints, limbs, and fingers, thinned upper lip, and small teeth with faulty enamel. The mother states: "My child seems to have problems in learning to count and recognizing basic colors." Based on this data, the nurse suspects that the child is most likely showing the effects of which problem? A) Congenital abnormalities B) Chronic toxoplasmosis C) Fetal alcohol syndrome D) Lead poisoning

is C: Fetal alcohol syndrome 6

When teaching suicide prevention to the parents of a 15 year-old who recently attempted suicide, the nurse describes the following behavioral cue A) Angry outbursts at significant others B) Fear of being left alone C) Giving away valued personal items D) Experiencing the loss of a boyfriend

is C: Giving away valued personal items 13

A nurse is providing care to a 17 year-old client in the post-operative care unit (PACU) after an emergency appendectomy. Which finding is an early indication that the client is experiencing poor oxygenation? A) Abnormal breath sounds B) Cyanosis of the lips C) Increasing pulse rate D) Pulse oximeter reading of 92%

is C: Increasing pulse rate 15

The nurse caring for a 9 year-old child with a fractured femur is told that a medication error occurred. The child received twice the ordered dose of morphine an hour ago. Which nursing diagnosis is a priority at this time? A) Risk for fluid volume deficit related to morphine overdose B) Decreased gastrointestinal mobility related to mucosal irritation C) Ineffective breathing patterns related to central nervous system depression D) Altered nutrition related to inability to control nausea and vomiting

is C: Ineffective breathing patterns related to central nervous system depression 11

A client is admitted with a tentative diagnosis of congestive heart failure. Which of the following assessments would the nurse expect to be consistent with this problem? A) Chest pain B) Pallor C) Inspiratory crackles D) Heart murmur

is C: Inspiratory crackles 15

A nurse is providing a parenting class to individuals living in a community of older homes. In discussing formula preparation, which of the following is most important to prevent lead poisoning? A) Use ready-to-feed commercial infant formula B) Boil the tap water for 10 minutes prior to preparing the formula C) Let tap water run for 2 minutes before adding to concentrate D) Buy bottled water labeled "lead free" to mix the formula

is C: Let tap water run for 2 minutes before adding to concentrate 5

The nurse's primary intervention for a client who is experiencing a panic attack is to A) Develop a trusting relationship B) Assist the client to describe his experience in detail C) Maintain safety for the client D) Teach the client to control his or her own behavior

is C: Maintain safety for the client 14

What finding signifies that children have attained the stage of concrete operations (Piaget)? A) Explores the environment with the use of sight and movement B) Thinks in mental images or word pictures C) Makes the moral judgement that "stealing is wrong" D) Reasons that homework is time-consuming yet necessary

is C: Makes the moral judgment that "stealing is wrong" 10

A client has returned to the unit following a renal biopsy. Which of the following nursing interventions is appropriate? A) Ambulate the client 4 hours after procedure B) Maintain client on NPO status for 24 hours C) Monitor vital signs D) Change dressing every 8 hours

is C: Monitor vital signs 11

A client is admitted with a right upper lobe infiltrate and to rule out tuberculosis. The most appropriate action by the nurse to protect the self would be which of these? A) Negative room ventilation B) Face mask with sheild C) Particulate respirator mask D) Airborne precautions

is C: Particulate respirator mask 8

A 10 year-old client is recovering from a splenectomy following a traumatic injury. The clients laboratory results show a hemoglobin of 9 g/dL and a hematocrit of 28 percent. The best approach for the nurse to use is to A) Limit milk and milk products B) Encourage bed activities and games C) Plan nursing care around lengthy rest periods D) Promote a diet rich in iron

is C: Plan nursing care around lengthy rest periods 6

A 3 year-old child diagnosed as having celiac disease attends a day care center. Which of the following would be an appropriate snack? A) Cheese crackers B) Peanut butter sandwich C) Potato chips D) Vanilla cookies

is C: Potato chips 9

When caring for a client receiving warfarin sodium (Coumadin), which lab test would the nurse monitor to determine therapeutic reponse to the drug? A) Bleeding time B) Coagulation time C) Prothrombin time D) Partial thromboplastin time

is C: Prothrombin time 10

An important goal in the development of a therapeutic inpatient milieu is to A) Provide a businesslike atmosphere where clients can work on individual goals B) Provide a group forum in which clients decide on unit rules, regulations, and policies C) Provide a testing ground for new patterns of behavior while the client takes responsibility for his or her own actions D) Discourage expressions of anger because they can be disruptive to other clients

is C: Provide a testing ground for new patterns of behavior while the client takes responsibility for his or her own actions 14

Which intervention best demonstrates the nurse's sensitivity to a 16 year old's appropriate need for autonomy? A) Alertness for feelings regarding body image B) Allows young siblings to visit C) Provides opportunity to discuss concerns without presence of parents D) Explores his feelings of resentment to identify causes

is C: Provides opportunity to discuss concerns without presence of parents 14

An 18 year-old client is admitted to intensive care from the emergency room following a diving accident. The injury is suspected to be at the level of the 2nd cervical vertebrae. The nurse's priority assessment should be A) Response to stimuli B) Bladder control C) Respiratory function D) Muscle weakness

is C: Respiratory function 4

Which statement best describes time management strategies applied to the role of a nurse manager? A) Schedule staff efficiently to cover the needs on the managed unit B) Assume a fair share of direct client care as a role model C) Set daily goals with a prioritization of the work D) Delegate tasks to reduce work load associated with direct care and meetings

is C: Set daily goals with a prioritization of the work 3

During the check up of a 2 month-old infant at a well baby clinic, the mother expresses concern to the nurse because a flat pink birthmark on the baby's forehead and eyelid has not gone away. What is an appropriate response by the nurse? A) "Mongolian spots are a normal finding in dark-skinned children." B) "Port wine stains are often associated with other malformations." C) "Telangiectatic nevi are normal and will disappear as the baby grows." D) "The child is too young for consideration of surgical removal of these at this time."

is C: Telangiectatic nevi are normal and will disappear as the baby grows 11

A client had 20 mg of Lasix (furosemide) PO at 10 AM. Which would be essential for the nurse to include at the change of shift report? A) The client lost 2 pounds in 24 hours B) The client's potassium level is 4 mEq/liter. C) The client's urine output was 1500 cc in 5 hours D) The client is to receive another dose of Lasix at 10 PM

is C: The client's urine output was 1500 cc in five hours 8

A client with a diagnosis of Methicillin resistant Staphylococcus aureus (MRSA) has died. Which type of precautions is the appropriate type to use when performing postmortem care? A) airborne precautions B) droplet precautions C) contact precautions D) compromised host precautions

is C: contact precautions 1

The nurse explains an autograft to a client scheduled for excision of a skin tumor. The nurse knows the client understands the procedure when the client says, "I will receive tissue from... A) a tissue bank." B) a pig." C) my thigh." D) synthetic skin."

is C: my thigh." 5

When admitting a client to an acute care facility, an identification bracelet is sent up with the admission form. In the event these do not match, the nurse's best action is to A) Change whichever item is incorrect to the correct information B) Use the bracelet and admission form until a replacement is supplied C) Notify the admissions office and wait to apply the bracelet D) Make a corrected identification bracelet for the client

is C: notify the admissions office and wait to apply the bracelet

A female client is admitted for a breast biopsy. She says, tearfully to the nurse, "If this turns out to be cancer and I have to have my breast removed, my partner will never come near me." The nurse's best response would be which of these statements? A) "I hear you saying that you have a fear for the loss of love." B) "You sound concerned that your partner will reject you." C) "Are you wondering about the effects on your sexuality?" D) "Are you worried that the surgery will change you?"

is D: "Are you worried that the surgery will change you?" 13

The nurse knows that which statement by the mother indicates that the mother understands safety precautions with her four month-old infant and her 4 year-old child? A) "I strap the infant car seat on the front seat to face backwards." B) "I place my infant in the middle of the living room floor on a blanket to play with my 4 year old while I make supper in the kitchen." C) "My sleeping baby lies so cute in the crib with the little buttocks stuck up in the air while the four year old naps on the sofa." D) "I have the 4 year-old hold and help feed the four month-old a bottle in the kitchen while I make supper."

is D: "I have the four year-old hold and help feed the four month-old a bottle in the kitchen

A school-aged child has had a long leg (hip to ankle) synthetic cast applied 4 hours ago. Which statement from the mother indicates that teaching has been inadequate? A) "I will keep the cast for the next day uncovered to prevent burning of the skin." B) "I can apply an ice pack over the area to relieve itching inside the cast." C) "The cast should be propped on at least 2 pillows when my child is lying down." D) "I think I remember that standing cannot be done until after 72 hours."

is D: "I think I remember that standing cannot be done until after 72 hours." 3

The pediatric clinic nurse examines a toddler with a tentative diagnosis of neuroblastoma. Findings observed by the nurse that is associated with this problem include which of these? A) Lymphedema and nerve palsy B) Hearing loss and ataxia C) Headaches and vomiting D) Abdominal mass and weakness

is D: Abdominal mass and weakness 3

The nurse is assessing a client with a Stage 2 skin ulcer. Which of the following treatments is most effective to promote healing? A) Covering the wound with a dry dressing B) Using hydrogen peroxide soaks C) Leaving the area open to dry D) Applying a hydrocolloid or foam dressing

is D: Applying a hydrocolloid or foam dressing 12

The nurse is having difficulty reading the health care provider's written order that was written right before the shift change. What action should be taken? A) Leave the order for the oncoming staff to follow-up B) Contact the charge nurse for an interpretation C) Ask the pharmacy for assistance in the interpretation D) Call the provider for clarification

is D: Call the provider for clarification

A client was admitted to the eating disorder unit with bulimia nervosa. The nurse assessing for a history of complications of this disorder expects A) Respiratory distress, dyspnea B) Bacterial gastrointestinal infections, over hydration C) Metabolic acidosis, constricted colon D) Dental erosion, parotid gland enlargement

is D: Dental erosion, parotid gland enlargement 14

A 14 month-old child ingested half a bottle of aspirin tablets. Which of the following would the nurse expect to see in the child? A) Hypothermia B) Edema C) Dyspnea D) Epistaxis

is D: Epistaxis 12

A client has been hospitalized after an automobile accident. A full leg cast was applied in the emergency room. The most important reason for the nurse to elevate the casted leg is to A) Promote the client's comfort B) Reduce the drying time C) Decrease irritation to the skin D) Improve venous return

is D: Improve venous return 5

The nurse is teaching a client with non-insulin dependent diabetes mellitus about the prescribed diet. The nurse should teach the client to A) Maintain previous calorie intake B) Keep a candy bar available at all times C) Reduce carbohydrates intake to 25% of total calories D) Keep a regular schedule of meals and snacks

is D: Keep a regular schedule of meals and snacks 5

The nurse is caring for a 17 month-old with acetaminophen poisoning. Which of the following lab reports should the nurse review first? A) Protime (PT) and partial thromboplastin time (PTT) B) Red blood cell and white blood cell counts C) Blood urea nitrogen and creatinine clearance D) Liver enzymes (AST and ALT)

is D: Liver enzymes (AST and ALT) 10

The nurse is preparing a client with a deep vein thrombosis (DVT) for a Venous Doppler evaluation. Which of the following would be necessary for preparing the client for this test? A) Client should be NPO after midnight B) Client should receive a sedative medication prior to the test C) Discontinue anti-coagulant therapy prior to the test D) No special preparation is necessary

is D: No special preparation is necessary 13

The nurse manager hears a health care provider loudly criticize one of the staff nurses within the hearing of others. The employee does not respond to the health care provider's complaints. The nurse manager's next action should be to A) Walk up to the health care provider and quietly state: "Stop this unacceptable behavior." B) Allow the staff nurse to handle this situation without interference C) Notify the of the other administrative persons of a breech of professional conduct D) Request an immediate private meeting with the health care provider and staff nurse

is D: Request an immediate private meeting with the health care provider and staff nurse 3

A client receiving chlorpromazine HCL (Thorazine) is in psychiatric home care. During a home visit the nurse observes the client smacking her lips alternately with grinding her teeth. The nurse recognizes this assessment finding as what? A) Dystonia B) Akathesia C) Brady dysknesia D) Tardive dyskinesia

is D: Tardive dyskinesia 10

As the nurse provides discharge teaching to the parents of a 15 month-old child with Kawasaki disease. The child has received immunoglobulin therapy. Which instruction would be appropriate? A) High doses of aspirin will be continued for some time B) Complete recovery is expected within several days C) Active range of motion exercises should be done frequently D) The measles, mumps and rubella vaccine should be delayed

is D: The measles, mumps and rubella vaccine should be delayed 6

The nurse is assessing an 8 month-old child with atonic cerebral palsy. Which statement from the mother supports the presence of this problem? A) When I put my finger in the left hand the baby doesn't respond with a grasp. B) My baby doesn't seem to follow when I shake toys in front of the face. C) When it thundered loudly last night the baby didn't even jump. D) When I put the baby in a back lying position that's how I find the baby.

is D: Unable to roll from 15

As the nurse observes the student nurse during the administration of a narcotic analgesic IM injection, the nurse notes that the student begins to give the medication without first aspirating. What should the nurse do? A) Ask the student: "What did you forget to do?" B) Stop. Tell me why aspiration is needed. C) Loudly state: "You forgot to aspirate." D) Walk up and whisper in the student's ear "Stop. Aspirate. Then inject."

is D: Walk up and whisper in the student's ear "Stop. Aspirate. Then inject." 1

The nurse is caring for a client with uncontrolled hypertension. Which findings require priority nursing action? A) Lower extremity pitting edema B) Rales C) Jugular vein distension D) Weakness in left arm

is D: Weakness in left arm 16

Which of the following manifestations observed by the school nurse confirms the presence of pediculosis capitis in students? A) Scratching the head more than usual B) Flakes evident on a student's shoulders C) Oval pattern occipital hair loss D) Whitish oval specks sticking to the hair

is D: Whitish oval specks sticking to the hair 7

An adult client is found to be unresponsive on morning rounds. After checking for responsiveness and calling for help, the next action that should be taken by the nurse is to: A) check the carotid pulse B) deliver 5 abdominal thrusts C) give 2 rescue breaths D) open the client's airway

is D: open the client''s airway

The ear, where do u pull to give a ear drop

it a picture so look it up online (KNOW CHILD & ADULT) (pull the pinnae top side of ear; u can look at a picture to see exactly where pinnae is) (Mine had a picture of ear and put it on the lobule of the pinnae it is the ear lobe). The eye picture asked to find the macula or the fovea centralis (there are 2 prominent spots on the eye - choose the DARK spot to the left not the spot where the light is)

5yr old burn victim has pot of hot liquid fall on him, what should nurse tell mother to do first-

nurse should tell mother to remove clothing and place in cool bath

An Elderly man is having trouble urinating and unsteady he uses a wheelchair to walk to the bathroom what should the nurse implement?

provide a raised toilet seat.

Select all that apply) Mantoux test

select all except 45-degree angle

Pt with DM

she states that she has to eat lunch rite after the gets her insulin to prevent going to hypoglycemia

A male client with diabetes mellitus type 2, who is taking pioglitazone PO daily, reports to the nurse the recent onset of nausea, accompanied by dark-colored urine, and a yellowish cast to his skin. What instructions should the nurse provide? a. you need to seek immediate medical assistance to evaluate the cause of these symptoms"

you need to seek immediate medical assistance to evaluate the cause of these symptoms"


Conjuntos de estudio relacionados

Covenant College | 20th Century World History | Jay Green | Exam 1

View Set

The French Revolution Begins- Instruction

View Set

Advanced Investments Practice Test 1

View Set

BIOL 130 - Ch. 10 Photosynthesis

View Set

ANTH 310 - Module 12 - Chapter 14 - Sex, Gender and Sexuality

View Set

GENERAL CHEMISTRY 103 Chapter 7 Classifying Chemical Reactions Precipitation and Acid-Base Reactions

View Set

APR 271 EXAM 1 (PHASE 1 FORMATIVE RESEARCH: STEPS 1-3: WEEKS 1-4)

View Set